Select Financial Statement Accounts Flashcards

1
Q

Hilltop Co.’s monthly bank statement shows a balance of $54,200. Reconciliation of the statement with company books reveals the following information:

Bank service charge $ 10
Insufficient funds check 650
Checks outstanding 1,500
Deposits in transit 350
Check deposited by Hilltop and cleared by the bank for $125, but improperly recorded by Hilltop as $152
What is the net cash balance after the reconciliation?

A
$52,363
B
$53,023
C
$53,050
D
$53,077
A

Explanation:
The correct answer is (C).

A common format of the bank reconciliation statement is to reconcile both book and bank balances to a common amount known as the true balance or net cash balance. This approach has the advantage of providing the cash figure to be reported in the balance sheet. Furthermore, journal entries necessary to adjust the books can be taken directly from the book balance section of the reconciliation. Net cash is the bank balance adjusted for outstanding checks and deposits in transit ($54,200 + $350 - $1,500 = $53,050).

Net cash is also the book balance adjusted for unrecorded or misrecorded items, such as service charges, insufficient funds, and errors. A normal book to bank reconciliation will compute the unadjusted book balance of $53,737. Net cash from the book side is $53,050 ($53,737 - $10 service charge - $650 insufficient funds - $27 net effect from the error).

Options (A), (B) and (D) are incorrect based on the above explanation.

How well did you know this?
1
Not at all
2
3
4
5
Perfectly
2
Q
Poe, Inc., had the following bank reconciliation at March 31:
Balance per bank statement, 3/31	$ 46,500
Add deposit in transit	10,300
$ 56,800
Less outstanding checks	(12,600)
Balance per books, 3/31	$ 44,200
Data per bank for the month of April:	
Deposits	$ 58,400
Disbursements	49,700All reconciling items at March 31 cleared the bank in April. Outstanding checks at April 30 totaled $7,000. There were no deposits in transit at April 30. What is the amount of cash disbursements per books in April 30?
A
$44,100
B
$49,200
C
$54,300
D
$56,700
A

Answer is A

Explanation:
Disbursements per bank, April $49,700
Less: Outstanding checks at 3/31 (12,600)
Add Outstanding checks at 4/30 7,000
Cash disbursement per books, April $44,100

How well did you know this?
1
Not at all
2
3
4
5
Perfectly
3
Q
At June 30, Almond Co.'s cash balance was $10,012 before adjustments, while its ending bank statement balance was $10,772. Check number 101 was issued June 2 in the amount of $95, but was erroneously recorded in Almond's general ledger balance as $59. The check was correctly listed in the bank statement at $95. The bank statement also included a credit memo for interest earned in the amount of $35, and a debit memo for monthly service charges in the amount of $50. What was Almond's adjusted cash balance at June 30?
A
$9,598
B
$9,961
C
$10,048
D
$10,462
A

Answer is B

Explanation:

When the check was originally recorded the journal entry was:
dr. Some Account……………………………….59

cr. Cash…………………………………………………59

The correcting entry would require an additional $36 credit to cash to get to the correct amount that should have been recorded in the first transaction ($95).

dr. Some Account……………………………….36
cr. Cash………………………………………………..36

Therefore, you should subtract $36 from the Cash balance reported in the general ledger.
=10012-36+35-50
=9,961

How well did you know this?
1
Not at all
2
3
4
5
Perfectly
4
Q
Trans Co. had the following balances at December 31, year 4:
Cash in checking account	$35,000
Cash in money market account	75,000
U. S. Treasury bill, purchased 11/1 year 4, maturing 1/31, year 5	350,000
U. S. Treasury bill, purchased 12/1 year 4, maturing 3/31, year 5	400,000Trans's policy is to treat as cash equivalents all highly-liquid investments with a maturity of three months or less when purchased. What amount should Trans report as cash and cash equivalents in its December 31, year 4, balance sheet?
A
$110,000
B
$385,000
C
$460,000
D
$860,000
A

Answer is C

Explanation:
The $400,000 U.S. Treasury bill purchased 12/1, year 4 and maturing 3/31, year 5 is not included as a cash equivalent because the maturity was more than three months at the time of purchase. Cash and cash equivalents reported at December 31, year 4, are as follows:

Cash in checking account $ 35,000
Cash in money market account 75,000
U. S. Treasury bill, purchased 11/1 year 4, maturing 1/31, year 5 350,000
Total cash and cash equivalents $ 460,000

How well did you know this?
1
Not at all
2
3
4
5
Perfectly
5
Q

________________________ is (are) defined as money or a claim to receive a sum of money, the amount of which is fixed or determinable without reference to future prices of specific goods or services.

A
Monetary assets
B
Monetary liabilities
C
The value in use
D
The recoverable amount
A

Answer is A

Explanation:
Monetary assets are defined as money or a claim to receive a sum of money, the amount of which is fixed or determinable without reference to future prices of specific goods or services.

How well did you know this?
1
Not at all
2
3
4
5
Perfectly
6
Q

Alton Co. had a cash balance of $32,300 recorded in its general ledger at the end of the month, prior to receiving its bank statement. Reconciliation of the bank statement reveals the following information:

Bank service charge $15
Check deposited and returned for insufficient funds check $120
Deposit recorded in the general ledger as $258 but should be $285
Checks outstanding $1,800
After reconciling its bank statement, what amount should Alton report as its cash account balance?

A
$30,338
B
$30,392
C
$32,138
D
$32,192
A

Explanation:
The correct answer is (D).

There are two types of reconciling items.

Type A reconciling items do not require adjustment on books. These include items that have already been recorded on books and require correction from the bank’s side. Outstanding checks, deposits in transit and bank errors are type A items.

Type B adjustment entries are those entries which have been recorded correctly by the bank but have not been correctly recorded in the books of accounts and thus require adjustment on books. The items that are classified as type B reconciling items that require adjustment on books are unrecorded returned insufficient fund checks, unrecorded bank reconciliation of notes receivables, unrecorded bank charges and cash account errors.

Cash Balance      	$32,300
(-)Bank Fees       	($15)
(-) NSF check     	($120)
(+) Deposit Error ($285 -$258)	$27
Corrected cash balance 	$32,192
How well did you know this?
1
Not at all
2
3
4
5
Perfectly
7
Q
he following are held by Smite Co.:
Cash in checking account	$20,000
Cash in bond sinking fund account	30,000
Post-dated check from customer dated one month from balance sheet date	250
Petty cash	200
Commercial paper (matures in two months)	7,000
Certificate of deposit (matures in six months)	5,000What amount should be reported as cash and cash equivalents on Smite's balance sheet?
A
$57,200
B
$32,200
C
$27,450
D
$27,200
A

Answer is D

Explanation:
Items in this question that are not considered cash equivalents are the cash in the bond sinking fund account ($30,000), the postdated check from a customer ($250), and the certificate of deposit that matures in six months ($5,000).

$20,000 Cash in checking account
200 Petty cash
7,000 Commercial paper (matures in two months)
$27,200 Total cash and cash equivalents

How well did you know this?
1
Not at all
2
3
4
5
Perfectly
8
Q
The following information pertains to Grey Co. at December 31 of the previous year:
Checkbook balance	$ 12,000
Bank statement balance	16,000
Check drawn on Grey's account, payable to a vendor, dated and recorded last 12/31 but not mailed until 1/10 this year	1,800
On Grey's December 31, previous year balance sheet, what amount should be reported as cash?
A
$12,000
B
$13,800
C
$14,200
D
$16,000
A

Answer is B

Explanation:
Checkbook balance	$ 12,000
Unmailed checks	1,800
Cash	$ 13,800
Answers (a), (c), and (d) are incorrect because the $1,800 check was not mailed as of 12/31 and, therefore, needs to be added back to the checkbook balance to arrive at the true cash balance as of 12/31. In addition, the starting point is the checkbook, not the bank statement, because the bank statement does not include checks in transit.
How well did you know this?
1
Not at all
2
3
4
5
Perfectly
9
Q

Poe, Inc., had the following bank reconciliation at March 31 of the current year:

Balance per bank statement, 3/31 $ 46,500
Add deposit in transit 10,300
$ 56,800
Less outstanding checks (12,600)
Balance per books, 3/31 $ 44,200
Data per bank for the month of April:
Deposits $ 58,400
Disbursements 49,700
All reconciling items at March 31 cleared the bank in April. Outstanding checks at April 30 totaled $7,000. There were no deposits in transit at April 30.
What is the cash balance per books at April 30?

A
$48,200
B
$52,900
C
$55,200
D
$58,500
A

Explanation:
The correct answer is (A).

The cash balance per books at April 30 is computed by subtracting the outstanding checks at April 30 from the balance per bank. First, adjust the bank balance from March 31 to April 30:

Cash balance per 3/31 bank statement $ 46,500
Add April deposits per bank 58,400
Less April disbursements per bank (49,700)
Cash balance per bank, 4/30 (all reconciling items at 3/31 cleared the bank in April) 55,200
Less outstanding checks, 4/30 (7,000)
Cash balance per books, 4/30 $ 48,200
Option (B) is incorrect because this takes the closing balance as per books as of 3/31 and adds the deposits and deducts disbursements for the month of April without taking into consideration the other impacts ($44,200 + $58,400 - $49,700 = $52,900).

Option (C) is incorrect because this does not take into account the outstanding checks on April 30 of $7,000.

Option (D) is incorrect because of inaccurate calculations

How well did you know this?
1
Not at all
2
3
4
5
Perfectly
10
Q
In preparing its August 31, Year 2 bank reconciliation, Apex Corp. has available the following information:
Balance per bank statement, 8/31	$18,050
Deposit in transit, 8/31	3,250
Return of customer's check for insufficient funds, 8/31	600
Outstanding checks, 8/31	2,750
Bank service charges for August	100At August 31, Year 2, Apex's correct cash balance is
A
$18,550
B
$17,950
C
$17,850
D
$17,550
A

Answer is A

Explanation:
Bank balance ($18,050) + DIT ($3,250) - Outstanding checks ($2,750) = Correct Cash Balance of $18,550
[Editor's note: returned check and service charges are already reflected in the 8/31 bank balance.]
How well did you know this?
1
Not at all
2
3
4
5
Perfectly
11
Q
Smith Co. has a checking account at Small Bank and an interest-bearing savings account at Big Bank. On December 31, year 1, the bank reconciliations for Smith are as follows:
Big Bank	
Bank balance	$150,000
Deposit in transit	5,000
Book balance	$155,000 
Small Bank	
Bank balance	$ 1,500
Outstanding checks	(8,500)
Book balance	$(7,000)What amount should be classified as cash on Smith's balance sheet at December 31, year 1?
A
$148,000
B
$151,000
C
$155,000
D
$156,000
A

Answer is C

Explanation:
Smith would classify the $150,000 bank balance and $5,000 deposit in transit for Big Bank as cash on the balance sheet. The bank balance of $1,500 in Small Bank is negated by the $8,500 in outstanding checks. Overdrafts in accounts with no available cash in another account at the same bank to offset are classified as current liabilities. They are not deducted from the total amount of cash at another bank.

How well did you know this?
1
Not at all
2
3
4
5
Perfectly
12
Q
On March 31, Vale Co. had an unadjusted credit balance of $1,000 in its allowance for uncollectible accounts. An analysis of Vale's trade accounts receivable at that date revealed the following:
Age	Amount	Estimated uncollectible
0 - 30 days	$60,000	5%
31- 60 days	4,000	10%
Over 60 days	2,000	$1,400What amount should Vale report as allowance for uncollectible accounts in its March 31 balance sheet?
A
$4,800
B
$4,000
C
$3,800
D
$3,000
A

Answer is A

Explanation:
While the unadjusted credit balance in the allowance accounts would be used to compute the uncollectible account expense for the period ending 3/31, the amount is not used to compute the balance of the allowance for uncollectible accounts at 3/31.

Age Accounts receivable Estimated % uncollectible Allowance for uncollectible accounts
0 - 30 days $60,000 5% $3,000
31 - 60 days 4,000 10% 400
Over 60 days 2,000 * 1,400
$4,800*The dollar amount of the estimated uncollectible accounts over 60 days is given in the data.

How well did you know this?
1
Not at all
2
3
4
5
Perfectly
13
Q

When the allowance method of recognizing uncollectible accounts is used, the entry to record the write-off of a specific account
A
Decreases both accounts receivable and the allowance for uncollectible accounts.
B
Decreases accounts receivable and increases the allowance for uncollectible accounts.
C
Increases the allowance for uncollectible accounts and decreases net income.
D
Decreases both accounts receivable and net income.

A

Answer is A

Explanation:
The entry to record the write-off of a specific account receivable using the allowance method involves a debit to allowance for uncollectible accounts and a credit to accounts receivable. This affects balance sheet accounts, not net income

How well did you know this?
1
Not at all
2
3
4
5
Perfectly
14
Q
On June 1 of the current year, Pitt Corp. sold merchandise with a list price of $5,000 to Burr on account. Pitt allowed trade discounts of 30% and 20%. Credit terms were 2/15, n/40 and the sale was made FOB shipping point. Pitt prepaid $200 of delivery costs for Burr as an accommodation. On June 12, Pitt received from Burr a remittance in full payment amounting to
A
$2,744
B
$2,940
C
$2,944
D
$3,140
A

Answer is C

Explanation:
List price of merchandise $ 5,000
Less: trade discount—30% (1,500)
Balance $ 3,500
Less: trade discount—20% (700)
Balance $ 2,800
Less: 2% cash discount, remittance received within 15 days of sale ($2,800 × 2%) (56)
Amount received for merchandise $ 2,744
Add: Reimbursement of prepayment of delivery costs 200
Remittance received from Burr in full payment $ 2,944

How well did you know this?
1
Not at all
2
3
4
5
Perfectly
15
Q

Jole Co. lent $10,000 to a major supplier in exchange for a non interest bearing note due in three years and a contract to purchase a fixed amount of merchandise from the supplier at a 10% discount from prevailing market prices over the next three years. The market rate for a note of this type is 10%. On issuing the note, Jole should record

Discount on Note Receivable Deferred Charge
A Yes Yes
B Yes No
C No Yes
D No No

A

Answer is A

Explanation:
Notes receivable are reported net of any discount.If a non-interest bearing (or low) note is exchanged for cash and a promise to provide future goods at lower-than-usual market prices, the issuer values the note at present value. The difference between present value and the cash payments is to be recognized as a part of the future goods’ cost, i.e., a deferred charge.

How well did you know this?
1
Not at all
2
3
4
5
Perfectly
16
Q

According to the percentage-of-outstanding-receivables method, which of the following is (are) true?
A
A percentage of uncollectible accounts in gross accounts receivable is determined based on the entity’s overall experience with uncollectible accounts over a period of time, adjusted for any relevant conditions.
B
The percentage of uncollectible accounts is applied to the ending balance of gross accounts receivable, to obtain the desired ending balance of the allowance for uncollectible accounts.
C
This method is balance-sheet oriented because it attempts to achieve a proper carrying amount for the accounts receivable at the end of a period, at net realizable value.
D
All of the above

A

Answer is D

Explanation:
A percentage of uncollectible accounts in gross accounts receivable is determined based on the entity’s overall experience with uncollectible accounts over a period of time, adjusted for any relevant conditions. The percentage of uncollectible accounts is applied to the ending balance of gross accounts receivable, to obtain the desired ending balance of the allowance for uncollectible accounts. This method is balance-sheet oriented because it attempts to achieve a proper carrying amount for the accounts receivable at the end of a period, at net realizable value. Answer D., all of the above, is the best choice.

How well did you know this?
1
Not at all
2
3
4
5
Perfectly
17
Q
During the year, Hauser Co. wrote off a customer's account receivable. Hauser used the allowance method for uncollectable accounts. What impact would the write-off have on net income and total assets?
Net income	Total assets
A	Decrease	Decrease
B	Decrease	No effect
C	No effect	Decrease
D	No effect	No effect
A

Answer is D

Explanation:
The journal entry to record the write-off of an account is as follows:

Allowance for Uncollectible Accounts XX
Accounts Receivable—Joe Doe XXThis entry would decrease both accounts receivable and allowance for uncollectible accounts. It has no impact on net income or total assets.

How well did you know this?
1
Not at all
2
3
4
5
Perfectly
18
Q

Which of the following describes portfolio segment disclosure in regards to credit losses?

A
The level used by the entity in developing and documenting a systematic method for determining the allowance for credit losses
B
The level based on initial measurement attributes, risk characteristics of the financing receivables, and methods used by reporting entities related to monitoring and assessing credit risk
C
A fully aggregated basis of disclosure
D
None of the above
A

Answer is A

Explanation:
In order to achieve the disclosure objective, reporting entities need to provide disclosures on two levels of disaggregation: portfolio segment and class of financing receivable. A portfolio segment is defined as the level used by the entity in developing and documenting a systematic method for determining the allowance for credit losses. Class of financing receivables generally represents a disaggregation of a portfolio segment, based on initial measurement attributes, risk characteristics of the financing receivables, and methods used by reporting entities related to monitoring and assessing credit risk.
How well did you know this?
1
Not at all
2
3
4
5
Perfectly
19
Q
Based on the aging of its accounts receivable at December 31 Terry Company determined that the net realizable value of the receivables at that date is $190,000. Additional information is as follows:
Accounts receivable at 12/31	$220,000
Allowance for doubtful accounts at 1/1—credit balance	32,000
Accounts written off as uncollectible at 9/30	24,000Terry's doubtful accounts expense for the year ended December 31 is:
A
$38,000
B
$30,000
C
$26,000
D
$22,000
A

Answer is D

Explanation:
The allowance account needs an ending balance of $30,000 [$220,000 - $190,000]. Currently, the allowance balance is $8,000 [$32,000 - $24,000], so an adjustment to bad debt expense and the allowance account for $22,000 is required.

How well did you know this?
1
Not at all
2
3
4
5
Perfectly
20
Q
Frame Co. has an 8% note receivable dated June 30, year 1, in the original amount of $150,000. Payments of $50,000 in principal plus accrued interest are due annually on July 1, year 2, year 3, and year 4. In its June 30, year 3, balance sheet, what amount should Frame report as a current asset for interest on the note receivable?
A
$0
B
$ 4,000
C
$ 8,000
D
$12,000
A

Answer is C

Explanation:
The note can be recorded at its face amount of $150,000 because there is no indication that the rate of interest (8%) stipulated by the parties to the transaction does not represent fair and adequate compensation for the use of the funds. Payments of $50,000 in principal plus accrued interest are due annually on July 1, year 2, year 3, and year 4. Frame should report the interest receivable as a current asset in its 6/30, year 3 balance sheet, because the amount is to be received within one year of the balance sheet date (i.e., it is to be received 7/1, year 3).

Carrying amount of note, 6/30, yr1 $150,000
Less: Principal payment, 7/1, yr2 (50,000)
Carrying amount of note, 7/1, yr2 100,000
Times: Stated interest rate × 8%
Interest receivable, 6/30, year 3 $ 8,000

How well did you know this?
1
Not at all
2
3
4
5
Perfectly
21
Q
Rue Co.'s allowance for uncollectible accounts had a credit balance of $12,000 at December 31, year 2. During year 3, Rue wrote-off uncollectible accounts of $48,000. The aging of accounts receivable indicated that a $50,000 allowance for uncollectible accounts was required at December 31, year 3. What amount of uncollectible accounts expense should Rue report for year 3?
A
$48,000
B
$50,000
C
$60,000
D
$86,000
A

Answer is D

Explanation:
Under the aging of accounts receivable method, after the desired ending balance of the allowance group is determined, the amount of uncollectible accounts (bad debt) expense recognized is the difference between the existing balance in the allowance account and the desired ending balance. The allowance account started the year with a credit balance of $12,000. The write-offs during the year would have been a credit to accounts receivable and a debit to allowance for uncollectible accounts of $48,000 thus bringing the allowance account to a $36,000 debit balance at year-end. To get the allowance account to the desired $50,000 credit balance there would need to be a credit to allowance for uncollectible accounts and debit to uncollectible accounts expense for $86,000.

How well did you know this?
1
Not at all
2
3
4
5
Perfectly
22
Q

Which of the following is true regarding an entity that records a note receivable?

A
When a note is exchanged for cash and a promise to provide merchandise at a discount from market price, the note is presumed to have a present value at issuance equal to the cash proceeds exchanged.
B
Noninterest-bearing notes receivable and those with an unrealistic stated rate of interest are not reported on the balance sheet but disclosed in the notes to the financial statements.
C
Loan origination fees are expensed in full in the period incurred.
D
For interest-bearing notes calling for the prevailing rate of interest at the time of issuance, the present value of the note is the same as the face amount of the note.

A

Answer is D

Explanation:
The correct answer is (D).

For interest-bearing notes calling for the prevailing rate of interest at the time of issuance, the present value of the note is the same as the face amount of the note.

When a note is exchanged for cash and a promise to provide merchandise at a discount from market price, the issuer records the note at present value.

The difference between fair value and cash payments is recognized as interest revenue over the contract life and is recorded as part of the cost of the related merchandise.

For non-interest-bearing notes and those with an unrea­listic stated rate of interest, the receivable must be reported at its present value or the fair value of the property, good, or service exchanged, whichever is more clearly determinable.

How well did you know this?
1
Not at all
2
3
4
5
Perfectly
23
Q

In its December 31 balance sheet, Butler Co. reported trade accounts receivable of $250,000 and related allowance for uncollectible accounts of $20,000.
What is the total amount of risk of accounting loss related to Butler’s trade accounts receivable, and what amount of that risk is off-balance sheet risk?

Risk of accounting loss	Off-balance sheet risk
A	$0	$0
B	$230,000	$0
C	$230,000	$20,000
D	$250,000	$20,000
A

Answer is B

Explanation:
The total risk of accounting loss is the amount of potential loss the entity would suffer if all parties to the financial instruments failed completely to perform and the amounts due proved to be of no value to the entity. Butler Co. had already recorded an allowance for uncollectible accounts of $20,000 on its trade accounts receivable of $250,000, so the net of $230,000 is the risk of accounting loss. The entire amount is shown on the balance sheet, thus there is no off-balance sheet risk involved.

CLOSE

How well did you know this?
1
Not at all
2
3
4
5
Perfectly
24
Q
On July 1 of the previous year, Kay Corp. sold equipment to Mando Co. for $100,000. Kay accepted a 10% note receivable for the entire sales price. This note is payable in two equal installments of $50,000 plus accrued interest on December 31 of the previous and current year. On July 1 of the current year, Kay discounted the note at a bank at an interest rate of 12%. Kay's proceeds from the discounted note were
A
$48,400
B
$49,350
C
$50,350
D
$51,700
A

Answer is D

Explanation:
Face amount of note, 7/1 of previous year $100,000
Less: payment of first installment, 12/31, previous year (50,000)
Face amount of note, 12/31 of previous year (due 12/31 of the current year) $ 50,000
Add interest to maturity ($50,000 x 10% x 12/12) 5,000
Maturity value of remaining portion of note $ 55,000
Less: Bank discount ($55,000 x 12% x 6/12) (3,300)
Proceeds from discounted note $ 51,700

How well did you know this?
1
Not at all
2
3
4
5
Perfectly
25
Q
Rand, Inc. accepted from a customer a $40,000, 90-day, 12% interest-bearing note dated August 31 of the current year. On September 30, Rand discounted the note at the Apex State Bank at 15%. However, the proceeds were not received until October 1. In Rand's September 30 balance sheet, the amount receivable from the bank, based on a 360-day year, includes accrued interest revenue of
A
$170
B
$200
C
$300
D
$400
A

Answer is A

Explanation:
Face amount of note	$ 40,000
Add interest to maturity ($40,000 x 12% × 90/360)	1,200
Maturity value of note	$ 41,200
Less bank discount ($41,200 x 15% x 60/360)	(1,030)
Proceeds from discounted note	$ 40,170
Less face amount of note	(40,000)
Accrued interest revenue, 9/30	$ 170
How well did you know this?
1
Not at all
2
3
4
5
Perfectly
26
Q

On the December 31 balance sheet of Mann Co., the current receivables consisted of the following:
Trade accounts receivable $ 93,000
Allowance for uncollectible accounts (2,000)
Claim against shipper for goods lost in transit (November) 3,000
Selling price of unsold goods sent by Mann on consignment at 130% of cost (not included in Mann’s ending inventory) 26,000
Security deposit on lease of warehouse used for storing some inventories 30,000
Total $150,000At December 31 the correct total of Mann’s current net receivables was
A
$ 94,000
B
$120,000
C
$124,000
D
$150,000

A

Answer is A

Explanation:
The goods out on consignment have not yet been sold and, thus, must be included in Mann’s inventory at their cost of $20,000 ($26,000 / 130%). The security deposit on lease of the warehouse should be classified as a noncurrent asset.

Trade accounts receivable 93,000
Less: Allowance for uncollectible accounts (2,000)
Plus: Claim against shipper for goods lost in transit 3,000
Current net receivables, 12/31 $94,000

How well did you know this?
1
Not at all
2
3
4
5
Perfectly
27
Q
Marr Co. had the following sales and accounts receivable balances, prior to any adjustments at year end:
Credit sales	$10,000,000
Accounts receivable	3,000,000
Allowance for uncollectible accounts (debit balance)	50,000Marr uses 3% of accounts receivable to determine its allowance for uncollectible accounts at year end. By what amount should Marr adjust its allowance for uncollectible accounts at year end?
A
$0
B
$40,000
C
$90,000
D
$140,000
A

Answer is D

Explanation:
The amount of the adjustment to the uncollectible account is the difference between the existing balance and the desired ending balance. The uncollectible account currently has a debit balance of $50,000. To get the allowance account to the desired $90,000 credit balance (3% × $3,000,000) there would need to be a credit to allowance for uncollectible accounts and debit to uncollectible accounts expense for $140,000.

How well did you know this?
1
Not at all
2
3
4
5
Perfectly
28
Q

At the end of year one, Boller Co. had an ending balance in the allowance for uncollectible accounts of $30,000. During year two, Boller wrote-off $40,000 of accounts receivable. At the end of year two, Boller had $300,000 in accounts receivable and determined that 8% of these would be uncollectible. What amount should be reported as uncollectible accounts expense on Boller’s year two income statement?

A
$64,000
B
$34,000
C
$24,000
D
$14,000
A

Answer is B

Explanation:
The correct answer is (B).

Boller Co. should report uncollectible account expense in the income statement for year 2 will be at $34,000.

Allowance for Uncollectible Account

Write Off

$40,000

Opening Balance

$30,000

Allowance for Uncollectible

$24,000

Bad Debt Expense (Plug)

$34,000

Alternative Method:

Summary

Amount

Write-offs in the year 2

$40,000

Bad debt expense

$34,000

Ending balance from year 1

$(30,000)

Allowance of uncollectible in year 2 (i.e. $300,000 x 8%)

How well did you know this?
1
Not at all
2
3
4
5
Perfectly
29
Q

The following information relates to Jay Co.’s accounts receivable for the current year:

Accounts receivable, 1/1 $ 650,000
Credit sales for the year 2,700,000
Sales returns for the year 75,000
Accounts written off during year 40,000
Collections from customers during year 2,150,000
Estimated future sales returns at 12/31 50,000
Estimated uncollectible accounts at 12/31 110,000
What amount should Jay report for accounts receivable, before allowances for sales returns and uncollectible accounts, at December 31?

A
$1,200,000
B
$1,125,000
C
$1,085,000
D
$ 925,000
A

Answer is C

Explanation:
Credit sales increase accounts receivable (A/R). Collections from customers, accounts written-off, and sales returns decrease A/R. Since the estimated future sales returns and the estimated uncollectible accounts are recorded in allowance accounts to A/R, they do not directly decrease the balance of the Accounts Receivable account.

Accounts Receivable

Balance, 1/1 650,000

Credit sales 2,700,000

2,150,000

40,000

75,000

Collections from customers

Accounts written-off

Sales returns

Balance, 12/31 1,085,000

How well did you know this?
1
Not at all
2
3
4
5
Perfectly
30
Q

When the allowance method of recognizing uncollectible accounts is used, how would the collection of an account previously have written off affect accounts receivable and the allowance for uncollectible accounts?

Accounts receivable	Allowance for uncollectible accounts
A	Increase	Decrease
B	Increase	No effect
C	No effect	Decrease
D	No effect	Increase
A

Answer is D

Explanation:
Journal entries to record the collection of an account previously written off as uncollectible are as follows.

Accounts Receivable—Joe Doe XX
Allowance for Uncollectible Accounts XX
To reopen account to the balance it had when written off.

Cash XX
Accounts Receivable—Joe Doe XX
To record receipt of cash in payment of the receivable
These entries would increase cash and allowance for uncollectible accounts. They would have no net effect on net accounts receivable, net income, current assets, or working capital.

How well did you know this?
1
Not at all
2
3
4
5
Perfectly
31
Q

Gar Co. factored its receivables without recourse with Ross Bank. Gar received cash as a result of this transaction, which is best described as a

A
Loan from Ross collateralized by Gar’s accounts receivable.
B
Loan from Ross to be repaid by the proceeds from Gar’s accounts receivable.
C
Sale of Gar’s accounts receivable to Ross, with the risk of uncollectible accounts retained by Gar.
D
Sale of Gar’s accounts receivable to Ross, with the risk of uncollectible accounts transferred to Ross.

A

Answer is D

Explanation:
Factoring of receivables is in substance a sale of receivables when the transfer is without recourse (i.e., the financing institution or “factor” assumes the risk of collection).

How well did you know this?
1
Not at all
2
3
4
5
Perfectly
32
Q

The following information pertains to Oro Corp:
Credit sales for the year ended December 31 $450,000
Credit balance in allowance for uncollectible accounts at January 1 10,800
Bad debts written off during the year 18,000According to past experience 3% of Oro’s credit sales have been uncollectible. After provision is made for bad debt expense for the year ended December 31, the allowance for uncollectible accounts balance would be:
A
$6,300
B
$13,500
C
$24,300
D
$31,500

A

Answer is A

Explanation:
Allowance credit balance of $10,800 - Write offs $18,000 + 3% credit sales of $13,500 (450,000 * 3%) = $6,300

How well did you know this?
1
Not at all
2
3
4
5
Perfectly
33
Q

Delta, Inc. sells to wholesalers on terms of 2/15, net 30. Delta has no cash sales but 50% of Delta’s customers take advantage of the discount. Delta uses the gross method of recording sales and trade receivables. An analysis of Delta’s trade receivables balances at December 31 revealed the following:

Age	Amount	Collectible
0-15 days	$100,000	100%
16-30 days	$60,000	95%
31-60 days	    $5,000	90%
over 60 days	      2,500	$500
 	$          167,500	 
In its December 31 balance sheet, what amounts should Delta report for allowance for discounts?
A
$1,000
B
$1,620
C
$1,675
D
$2,000
A

Answer is A

Explanation:
Only the receivables which have aged 0-15 days are eligible for the discount. The discount is computed using 50% of the dollar amount eligible, not 50% of
the discount. Only 50% of the customers take advantage of the discount.

Amount eligible for discount	$100,000
% of customers that take discount	× 50%
Amount of eligible amount taken	50,000
Discount allowed	× 2%
Allowance for discount	$ 1,000
How well did you know this?
1
Not at all
2
3
4
5
Perfectly
34
Q
Tinsel Co.'s balances in allowance for uncollectible accounts were $70,000 at the beginning of the current year and $55,000 at year end. During the year, receivables of $35,000 were written off as uncollectible. What amount should Tinsel report as uncollectible accounts expense at year end?
A
$15,000
B
$20,000
C
$35,000
D
$50,000
A

Answer is B

Explanation:
The allowance account has an ending balance of $55,000. Prior to the bad debt adjustment, the allowance balance is $35,000 ($70,000 - $35,000 in writeoffs), so an adjustment to bad debt expense and the allowance account for $20,000 is required.

How well did you know this?
1
Not at all
2
3
4
5
Perfectly
35
Q
On December 30 of the current year, Chang Co. sold a machine to Door Co. in exchange for a non-interest-bearing note requiring ten annual payments of $10,000. Door made the first payment on that same date. The market interest rate for similar notes at date of issuance was 8%. Information on present value factors is as follows:
Period	Present value of $1 at 8%	Present value of ordinary annuity of 1$ at 8%
9	0.50	6.25
10	0.46	6.71In its December 31 year-end balance sheet, what amount should Chang report as note receivable?
A
$45,000
B
$46,000
C
$62,500
D
$67,100
A

Answer is C

Explanation:
Notes receivable are generally required to be recorded at their present value. At December 31, Chang is owed 9 more annual payments of $10,000. The appropriate factor to apply is the present value of ordinary annuity of $1 at 8% for 9 periods, which is given as 6.25. $10,000 x 6.25 = $62,500.

How well did you know this?
1
Not at all
2
3
4
5
Perfectly
36
Q

For the current year ended December 31, Beal Co. estimated its allowance for uncollectible accounts using the year-end aging of accounts receivable. The following data are available:
Allowance for uncollectible accounts, 1/1 $42,000
Provision for uncollectible accounts (2% on credit sales of $2,000,000) 40,000
Uncollectible accounts written-off, 11/30 46,000
Estimated uncollectible accounts per aging, 12/31 52,000After the year-end adjustment, the uncollectible accounts expense for the current year should be
A
$46,000
B
$48,000
C
$52,000
D
$56,000

A

Answer is D

Explanation:
Since Beal uses the aging method, the credit sales information is irrelevant. The balance in the allowance account was a debit of $4,000 (42,000 - 46,000) prior to the year-end adjustment. Given that the year-end allowance balance should be $52,000, bad debt expense would be debited for $56,000, and the allowance account would be credited for $56,000.

How well did you know this?
1
Not at all
2
3
4
5
Perfectly
37
Q

In its previous year-end balance sheet, Fleet Co. reported accounts receivable of $100,000 before allowance for uncollectible accounts of $10,000. Credit sales during the current year were $611,000, and collections from customers, excluding recoveries, totaled $591,000. During the year , accounts receivable of $45,000 were written off and $17,000 were recovered. Fleet estimated that $15,000 of the accounts receivable at December 31 were uncollectible.
In its December 31 current year balance sheet, what amount should Fleet report as accounts receivable before allowance for uncollectible accounts?

A
$58,000
B
$67,000
C
$75,000
D
$82,000
A

Answer is C

Explanation:
The A/R balance was increased by credit sales and decreased by collections from customers and by accounts written off. Accounts recovered resulted in an increase and a decrease of equal amount to the A/R balance.

                                                   Accounts Receivable

Balance, 1/1 100,000

Credit sales 611,000

591,000

Customer collections

Accounts reinstated 17,000

45,000

Accounts written-off

17,000

Accounts recovered

Balance, 12/31 75,000

How well did you know this?
1
Not at all
2
3
4
5
Perfectly
38
Q

On December 31, Key Co. received two $10,000 non-interest bearing notes from customers in exchange for services rendered.The note from Alpha Co.,which is due in nine months,was made under customary trade terms, but the note from Omega Co., which is due in two years, was not. The market interest rate for both note sat the date of issuance is 8%. The present value of $1 due in nine months at 8% is .944. The present value of $1 due in two years at 8% is .857. At what amounts should these two notes receiv­able be reported in Key’s December 31 balance sheet?

Alpha	Omega
A	$9,440	$8,570
B	$10,000	$8,570
C	$9,440	$10,000
D	$10,000	$10,000
A

Answer is B

Explanation:
Notes receivable are claims usually not arising from sales in the ordinary course of business.Legally,the claim is evidenced by a note representing an unconditional promise to pay. The recording of notes receiv­able is at their present value. This is not intended to apply to receivables and payables arising from transactions with customers or suppliers in the normal course of business which are due in customary trade terms not exceed­ing approximately one year. Thus, the note from Alpha Co. would be reported at the $10,000 face value. The note from Omega would be discounted at $8,570 ($10,000 × .857).

How well did you know this?
1
Not at all
2
3
4
5
Perfectly
39
Q
Red Co. had $3 million in accounts receivable recorded on its books. Red wanted to convert the $3 million in receivables to cash in a more timely manner than waiting the 45 days for payment as indicated on its invoices. Which of the following would alter the timing of Red's cash flows for the $3 million in receivables already recorded on its books?
A
Change the due date of the invoice
B
Factor the receivables outstanding
C
Discount the receivables outstanding
D
Demand payment from customers before the due date
A

Answer is B

Explanation:
Only factoring the receivables would ensure altering the timing of cash flows for the $3 million in receivables already recorded on its books. Factoring the receivables outstanding is a transfer of the receivables to a factor (transferee) without recourse and is accounted for as any other sale of an asset: debit cash, credit the receivables, and record a gain or loss for the difference. Changing the due date of the invoice, discounting the receivables outstanding, and demanding payment from customers before the due date will not necessarily get the cash for receivables any quicker and alter the timing of cash flows.

How well did you know this?
1
Not at all
2
3
4
5
Perfectly
40
Q

On December 31 of the current year, Jet Co. received two $10,000 notes receivable from customers in exchange for services rendered. On both notes, interest is calculated on the outstanding principal balance at the annual rate of 3% and payable at maturity. The note from Hart Corp., made under customary trade terms, is due in nine months and the note from Maxx, Inc. is due in five years. The market interest rate for similar notes on this date was 8%. The compound interest factors to convert future values into present values at 8% follow:
Present value of $1 due in nine months: .944
Present value of $1 due in five years: .680At what amounts should these two notes receivable be reported in Jet’s December 31 balance sheet?
Hart Maxx
A $9,440 $6,800
B $9,652 $7,820
C $10,000 $6,800
D $10,000 $7,820

A

Answer is D

Explanation:
Both notes were received on the balance sheet date. Since the note from Hart arose from a transaction with a customer in the normal course of business and is due in customary trade terms not exceeding one year, it can be reported at its face amount of $10,000 despite the fact that the 3% stated interest rate of the note differs from the prevailing market interest rate of 8% for similar notes at the transaction date. On the other hand, the note from Maxx is due in more than one year. Therefore, the note from Maxx cannot be reported at its face amount because its 3% stated interest rate differs from the prevailing market interest rate of 8% for similar notes at the transaction date. Because neither the fair value of the services performed by Jet nor the fair value of the note received from Maxx is indicated, the note is reported at its present value, determined by discounting all future cash payments of the note at the prevailing (i.e., market) rate of interest for a note of this type.

Principal amount $ 10,000
Interest on outstanding principal balance due on maturity date of note [($10,000 × 3%) × 5] 1,500
Amount due on maturity date of note $ 11,500
Present value factor of $1 at 8% for 5 periods × 0.680
Present value of note received from Maxx $ 7,820

How well did you know this?
1
Not at all
2
3
4
5
Perfectly
41
Q

Milton Co. pledged some of its accounts receivable to Good Neighbor Financing Corporation in return for a loan. Which of the following statements is correct?
A
Good Neighbor Financing cannot take title to the receivables if Milton does not repay the loan. Title can only be taken if the receivables are factored.
B
Good Neighbor Financing will assume the responsibility of collecting the receivables.
C
Milton will retain control of the receivables.
D
Good Neighbor Financing will take title to the receivables, and will return title to Milton after the loan is paid.

A

Answer is C

Explanation:
Receivables may be pledged as security for loans. Control of the receivables is retained and collections on the receivables are usually required to be applied to a reduction of the loan. Good Neighbor Financing could take title to the receivables if Milton doesn’t pay the loan, does not assume the responsibility of collecting the receivables, and does not take title to the receivables until the loan is repaid.

How well did you know this?
1
Not at all
2
3
4
5
Perfectly
42
Q
On April 1, Aloe, Inc. factored $80,000 of its accounts receivable without recourse. The factor retained 10% of the accounts receivable as an allowance for sales returns and charged a 5% commission on the gross amount of the factored receivables. What amount of cash did Aloe receive from the factored receivables?
A
$68,000
B
$68,400
C
$72,000
D
$76,000
A

Answer is A

Explanation:
Factoring is similar to a sale of receivables because it is generally without recourse and the factor generally handles the billing and collection function. Aloe would receive $68,000 from the factored receivables. The $80,000 factored amount less $8,000 (10% as an allowance for sales returns) and less $4,000 (for the 5% commission charged) equals $68,000.

How well did you know this?
1
Not at all
2
3
4
5
Perfectly
43
Q

Bee Co. uses the direct write-off method to account for uncollectible accounts receivable. During an accounting period, Bee’s cash collections from customers equal sales adjusted for the addition or deduction of the following amounts:
Accounts written-off Increase in accounts receivable balance
A Deduction Deduction
B Addition Deduction
C Deduction Addition
D Addition Addition

A

Answer is A

Explanation:
An increase in A/R indicates that there was an excess of sales over cash collections. A write-off of receivables will offset the gross increase in A/R without affecting cash. Regardless of the accrual method used to account for uncollectible accounts, the cash collections from customers equal sales adjusted by a deduction for an increase in A/R balance and a deduction for accounts written-off during the period.

How well did you know this?
1
Not at all
2
3
4
5
Perfectly
44
Q
Foster Co. adjusted its allowance for uncollectible accounts at year end. The general ledger balances for the accounts receivable and the related allowance account were $1,000,000 and $40,000, respectively. Foster uses the percentage-of-receivables method to estimate its allowance for uncollectible accounts. Accounts receivable were estimated to be 5% uncollectible. What amount should Foster record as an adjustment to its allowance for uncollectible accounts at year end?
A
$10,000 decrease
B
$10,000 increase
C
$50,000 decrease
D
$50,000 increase
A

Answer is B

Explanation:
The allowance account balance would increase by the difference between the required amount and the balance at the end of the year.

$1,000,000		Accounts receivable balance
             5%		Uncollectible
50,000		Balance required
     - 40,000		End of year balance in allowance account
$ 10,000		Adjustment to increase balance
How well did you know this?
1
Not at all
2
3
4
5
Perfectly
45
Q

Clear Co.’s trial balance has the following selected accounts:

Cash (includes $10,000 in a bond sinking fund for long-term bond payable)	$50,000
Accounts receivable	$20,000
Allowance for doubtful accounts	$5,000
Deposits received from customers	$3,000
Merchandise inventory	$7,000
Unearned rent	$1,000
Investment in trading securities	$2,000
 What amount should clear report as total current assets in its balance sheet?
A
$64,000
B
$67,000
C
$72,000
D
$74,000
A

Answer is A

Explanation:
The correct answer is (A).

Ref Summary Amount
A Total Cash $50,000
B Less: Cash used to pay Long-Term Debt $10,000
C Available Cash (A-B) $40,000
D Accounts Receivable $20,000
E Less: Allowance for doubtful debts $5,000
F Net Receivable (D-E) $15,000
G Merchandise Inventory $7,000
H Trading securities $2,000
I Total Current Assets (C + F + G + H) $64,000
The bond sinking fund is not a current asset. The bond sinking fund is reported right after current assets on the balance sheet. The bond sinking fund is part of long-term assets.
Customer deposits and unearned rents represent amounts already received and included in cash in exchange for goods or services that have not yet been delivered. These items are reported in financial statements in liabilities, and not under current assets.

How well did you know this?
1
Not at all
2
3
4
5
Perfectly
46
Q

A note receivable bearing a reasonable interest rate is sold to a bank with recourse. At the date of the discounting transaction, the notes receivable discounted account should be
A
Decreased by the proceeds from the discounting transaction.
B
Increased by the proceeds from the discounting transaction.
C
Increased by the face amount of the note.
D
Decreased by the face amount of the note.

A

Answer is C

Explanation:
A company that discounts a note receivable with recourse is contingently liable to the lender. It must pay the lender the amount due at maturity if the maker of the note fails to pay the obligation. The contingent liability is usually shown in the accounts by recording the note discounted in a Notes Receivable Discounted account at the note’s face amount. The Notes Receivable Discounted account is reported as a contra asset and deducted from Notes Receivable in the balance sheet.

Notes receivable $XXX
Less: Notes receivable discounted (XXX) $XXXExplanation:
A company that discounts a note receivable with recourse is contingently liable to the lender. It must pay the lender the amount due at maturity if the maker of the note fails to pay the obligation. The contingent liability is usually shown in the accounts by recording the note discounted in a Notes Receivable Discounted account at the note’s face amount. The Notes Receivable Discounted account is reported as a contra asset and deducted from Notes Receivable in the balance sheet.

Notes receivable $XXX
Less: Notes receivable discounted (XXX) $XXX

How well did you know this?
1
Not at all
2
3
4
5
Perfectly
47
Q

Inge Co. determined that the net value of its accounts receivable at December 31 of the current year, based on an aging of the receivables, was $325,000.
Additional information is as follows:

Allowance for uncollectible accounts, 1/1 $30,000
Uncollectible accounts written-off during the year 18,000
Uncollectible accounts recovered during the year 2,000
Accounts receivable at 12/31 350,000
For the current year ending December 31, what would be Inge’s uncollectible accounts expense?

A
$5,000
B
$11,000
C
$15,000
D
$21,000
A

ANswer is B

Explanation:
The correct answer is (B).

After the allowance for uncollectible accounts at 12/31 is determined, uncollectible accounts expense can be analyzed.

Current year write-offs 18,000 Balance - 1/1 30,000
Uncollectible accounts recovered 2,000
Uncollectible accounts expense (plug) 11,000
Balance - 12/31 25,000

How well did you know this?
1
Not at all
2
3
4
5
Perfectly
48
Q
A method of estimating uncollectible accounts that emphasizes asset valuation rather than income measurement is the allowance method based on
A
Aging the receivables.
B
Direct write off.
C
Gross sales.
D
Credit sales less returns and allowances.
A

Answer is A

Explanation:
The allowance method based on aging the receivables attempts to value the receivables at their future collectible amounts. Thus, it emphasizes asset valuation rather than income measurement.

How well did you know this?
1
Not at all
2
3
4
5
Perfectly
49
Q

At January 1, Jam Co. had a credit balance of $260,000 in its allowance for uncollectible accounts. Based on past experience, 2% of Jam’s credit sales have been uncollectible. During the year, Jam wrote off $325,000 of uncollectible accounts. Credit sales for the year were $9,000,000. In its December 31 year end balance sheet, what amount should Jam report as allowance for uncollectible accounts?

A
$115,000
B
$180,000
C
$245,000
D
$440,000
A

ANswer is A

Explanation:
Based on the information given, Jamin uses the percentage of sales method, under which bad debt expense is calculated as a percentage of credit sales (credit sales of $9,000,000 × 2% = $180,000), charged to Bad Debt Expense, and credited to the Allowance for Uncollectible Accounts.The allowance account balance is not considered in determining the amount of bad debt expense.

Allowance for Uncollectible Accounts

260,000

Beg. bal. 1/1

Write-offs 325,000

180,000

Bad debt expense

115,000

12/31 balance

How well did you know this?
1
Not at all
2
3
4
5
Perfectly
50
Q

___________ refers to the sale of a note to a third party, usually a bank or other financial institution.

A
Factoring
B
Assigning
C
Discounting
D
None of the above
A

Answer is C

Explanation:
The correct answer is (C).

Discounting refers to the sale of a note to a third party, usually a bank or other financial institution.

The sale of a Note Receivable at a value less than the face value is Discounting.

The sale of Accounts Receivables for a fee is known as Factoring.

Assigning is s a lending agreement whereby the borrower assigns Accounts Receivable to the lending institution in exchange for a loan, where the Accounts Receivable serves as collateral.

How well did you know this?
1
Not at all
2
3
4
5
Perfectly
51
Q
Could current cost financial statements report holding gains for goods sold during the period and holding gains on inventory at the end of the period?
Goods sold	Inventory
A	Yes	Yes
B	Yes	No
C	No	Yes
D	No	No
A

Answer is A

Explanation:
An increase in the current cost of inventory items sold is a realized holding gain. An increase in the current cost of inventory items on hand is an unrealized holding gain. Current cost financial statements will measure and report both realized and unrealized holding gains.

How well did you know this?
1
Not at all
2
3
4
5
Perfectly
52
Q
Wilson Corp. experienced a $50,000 decline in the market value of its inventory in the first quarter of its fiscal year. Wilson had expected this decline to reverse in the third quarter, and in fact, the third quarter recovery exceeded the previous decline by $10,000. Wilson's inventory did not experience any other declines in market value during the fiscal year. What amounts of loss and/or gain should Wilson report in its interim financial statements for the first and third quarters?
First quarter	Third quarter
A	$0	$0
B	$0	$10,000 gain
C	$50,000 loss	$50,000 gain
D	$50,000 loss	$60,000 gain
A

Answer is A

Explanation:
The use of lower of cost or market may result in inventory losses that should not be deferred beyond the interim period in which the decline occurs. Recoveries of these losses in subsequent periods should be recognized as gains, but only to the extent of losses recognized in previous interim periods of the same fiscal year. Temporary market declines, however, need not be recognized at the interim dates since no loss is expected to be incurred in the fiscal year. Because Wilson expected the decline to reverse within the fiscal year, no loss should be recorded for the first quarter. Even though the recovery exceeded the previous decline by $10,000, gains are recognized only to the extent of losses recognized in previous interim periods of the same fiscal year.

How well did you know this?
1
Not at all
2
3
4
5
Perfectly
53
Q

Which of the following statements describes a perpetual inventory system?
A
A perpetual inventory system is characterized by no entries being made to the inventory account during the period.
B
The balance in the inventory account at any time reveals the inventory that should be on hand.
C
Acquisitions of inventory goods are debited to “Purchases” while issuances are not recorded.
D
Cost of goods sold (CGS) is a residual amount obtained by subtracting the ending inventory from the sum of beginning inventory and net purchases.

A

Answer is B

Explanation:
In a periodic inventory system, acquisitions of inventory goods are debited to “Purchases” while issuances are not recorded. At any point in time the balance in the inventory account reflects the amount at the “beginning” of the period. The inventory on hand is “periodically” determined by physical count. In a periodic inventory system, cost of goods sold (CGS) is a residual amount obtained by subtracting the ending inventory from the sum of beginning inventory and net purchases. In a perpetual inventory system, the balance in the inventory account at any time reveals the inventory that should be on hand.

How well did you know this?
1
Not at all
2
3
4
5
Perfectly
54
Q

A company manufactures and distributes replacement parts for various industries. As of December 31, year 1, the following amounts pertain to the company’s inventory:

Item Cost Net
replacement
cost Sale price Cost to sell
or dispose Normal
profit margin
Blades $41,000 $ 38,000 $ 50,000 $ 2,000 $15,000
Towers 52,000 40,000 54,000 4,000 14,000
Generators 20,000 24,000 30,000 2,000 6,000
Gearboxs 80,000 105,000 120,000 12,000 8,000
What is the total carrying value of the company’s inventory as of December 31, year 1, under IFRS?

A
$178,000
B
$191,000
C
$193,000
D
$207,000
A

Answer is B

Explanation:
Under IFRS, inventory is carried at the lower of cost or net realizable value (best estimate of the net amounts inventories are expected to realize).

Item	Cost	Sale price	Cost to sell or
dispose	Lower of
cost/NRV
Blades	$41,000	$ 50,000	$ 2,000	$ 41,000
Towers	52,000	54,000	4,000	50,000
Generators	20,000	30,000	2,000	20,000
Gearboxes	80,000	120,000	12,000	\_\_\_80,000
12/31, yr 1	 	 	 	$191,000
How well did you know this?
1
Not at all
2
3
4
5
Perfectly
55
Q

Assuming constant inventory quantities, which of the following inventory-costing methods will produce a lower inventory turnover ratio in an inflationary economy?

A
FIFO (first in, first out)
B
LIFO (last in, first out)
C
Moving average
D
Weighted average
A

Answer is A

Explanation:
Inventory turnover ratio = COGS/Average inventory. In an inflationary economy, COGS reported using FIFO is low, leading to higher net profits. Inventory would be higher since it is reported at current prices. Therefore, higher Inventories and lower COGS will result in a lower inventory turnover ratio. In turnover ratios, turn-it-over to the denominator. So, higher inventory and lower COGS would lead to lower inventory ratio.

Option (B) is incorrect because the inventory turnover ratio is lower as the COGS reported are higher because the costs of the units purchased are higher when compared to the cost of the units first purchased and inventory reported are lower as it consists of earlier purchases.

Option (C) and (D) are incorrect as per the above explanation.

How well did you know this?
1
Not at all
2
3
4
5
Perfectly
56
Q

Brock Co. adopted the dollar-value LIFO inventory method as of January 1, year 1. A single inventory pool and an internally computed price index are used
to compute Brock’s LIFO inventory layers. Information about Brock’s dollar value inventory follows:

Inventory


Date At base year cost At current year cost At dollar value LIFO
1/1, year 1 $40,000 $40,000 $40,000
Year 1 layer 5,000 14,000 6,000
12/31, year 1 $45,000 $54,000 $46,000
Year 2 layer 15,000 26,000 ?
12/31, year 2 $60,000 $80,000 ?
What was Brock’s dollar value LIFO inventory at December 31, year 2?

A
$80,000
B
$74,000
C
$66,000
D
$60,000
A

Answer is C

Explanation:
The price index is computed by dividing the ending inventory at current year cost by its base year cost.

Date Layers at base year cost Price index Ending inventory at LIFO cost
01/01, year 1 $40,000 1.0000 [1] $40,000
12/31, year 1 5,000 1.2000 [2] 6,000
12/31, year 2 15,000 1.3333 [3] 20,000
$60,000 $66,000
[1] $40,000 / $40,000 [2] $54,000 / $45,000 [3] $80,000 / $60,000

How well did you know this?
1
Not at all
2
3
4
5
Perfectly
57
Q

Trans Co. uses a periodic inventory system. The following are inventory transactions for the month of January:

1/1 Beginning inventory 10,000 units at $3
1/5 Purchase 5,000 units at $4
1/15 Purchase 5,000 units at $5
1/20 Sales at $10 per unit 10,000 units
Trans uses the average pricing method to determine the value of its inventory. What amount should Trans report as cost of goods sold on its income statement for the month of January?

A
$ 30,000
B
$ 37,500
C
$ 40,000
D
$100,000
A

Answer is B

Explanation:
Using the average cost method, Weighted average cost = Cost of goods available for sale/Number of units available for sale = $75,000/20,000 units = 3.75. COGS reported would be, Units sold x Weighted average cost = 10,000 units x $3.75 = 37,500.

Date

Inventory Units

Cost per Unit

Amount

1 – Jan

10,000

$3

$30,000

5 - Jan

5,000

$4

$20,000

15 - Jan

5,000

$5

$25,000

Total

20,000

$75,000

Option (A) is incorrect because the COGS are calculated based on the unit price of the beginning inventory.

Option (C) is incorrect because it is calculated based on the average price of the units [($3 + $4 + $5)/3 = $4]; 10,000 units x $4= $40,000.

Option (D) is incorrect because it represents Sales (10,000 x $10) and not COGS.

How well did you know this?
1
Not at all
2
3
4
5
Perfectly
58
Q
Town Inc. is preparing its financial statements for the current year ended December 31. At December 31, Town had outstanding purchase orders in the ordinary course of business for purchase of a raw material to be used in its manufacturing process. The market price is currently higher than the purchase price and is not anticipated to change within the next year. What is the reporting requirement?
A
Disclosure only
B
Accrual only
C
Both accrual and disclosure
D
Neither accrual nor disclosure
A

Answer is D

Explanation:
Because the outstanding purchase orders occurred in the ordinary course of business and the raw materials have not yet been received, the purchase is not required to be accrued. Since the price difference is only a market price difference occurring in the ordinary course of business, disclosure is not required.

How well did you know this?
1
Not at all
2
3
4
5
Perfectly
59
Q

During periods of rising prices, when the FIFO inventory method is used, a perpetual inventory system results in an ending inventory cost that is
A
The same as in a periodic inventory system.
B
Higher than in a periodic inventory system.
C
Lower than in a periodic inventory system.
D
Higher or lower than in a periodic inventory system, depending on whether physical quantities have increased or decreased.

A

Answer is A

Explanation:
In all cases where FIFO is used, the inventory and cost of goods sold would be the same at the end of a period whether a perpetual or a periodic system is used. This is true because the same costs will always be first in and, therefore, first out, whether cost of goods sold is computed as cost of goods sold throughout the accounting period (the perpetual system) or as a residual at the end of the accounting period (the periodic system).This is true regardless of whether inventory prices rose or fell during the period.

How well did you know this?
1
Not at all
2
3
4
5
Perfectly
60
Q

Hutch Inc. uses the conventional retail inventory method to account for inventory. The following information relates to current year operations:

                 Average                 

  Cost   

       Retail  

Beginning inventory and purchases

$ 600,000 $920,000

Net markups 40,000

Net markdowns 60,000

Sales 780,000

What amount should be reported as cost of sales for the current year?

A
$480,000
B
$487,500
C
$520,000
D
$525,000
A

Explanation:
The correct answer is (D)

Cost of sales is determined by subtracting the estimated ending inventory at cost from the cost of the beginning inventory and purchases (i.e., $600,000 – $75,000 = $525,000). Under the conventional retail inventory method, the beginning inventory and the net markups are included in the cost-to-retail ratio. The net markdowns are not included in the ratio.
=(920K + 40K- 780K +60K) x 62.5%***
= $ 75,000

***Cost-to-retail ratio ($600,000 / $960,000)= 62.5%

How well did you know this?
1
Not at all
2
3
4
5
Perfectly
61
Q

Thread Co. is selecting its inventory system in preparation for its first year of operations. Thread intends to use either the periodic weighted average method or the perpetual moving average method, and to apply the lower of cost or market rule either to individual items or to the total inventory. Inventory prices are expected to generally increase throughout the year, although a few individual prices will decrease. What inventory system should Thread select if it wants to maximize the inventory carrying amount at December 31?
Inventory method Cost or market application
A Perpetual Total inventory
B Perpetual Individual item
C Periodic Total inventory
D Periodic Individual item

A

Answer is A

Explanation:
In a period of rising prices, a moving weighted average method (perpetual system) will give a higher ending inventory figure than the weighted average method with a periodic system because with the moving weighted average method, more of the early (low) costs are released to cost of goods sold with that method than with a periodic system, leaving the higher costs for the ending inventory figure. Also, the item-by-item approach to applying the lower of cost or market rule is the more conservative method. Therefore, a higher inventory figure is obtained by use of the total inventory approach.

How well did you know this?
1
Not at all
2
3
4
5
Perfectly
62
Q

Kauf Co. had the following amounts related to the sale of consignment inventory:

Cost of merchandise shipped to consignee $72,000
Sales value for two-thirds of inventory sold by consignee 80,000
Freight cost for merchandise shipped 7,500
Advertising paid for by consignee, to be reimbursed 4,500
10% commission due the consignee for the sale 8,000
What amount should Kauf report as net profit(loss) from this transaction for the year?

A
$(12,000)
B
$ 8,000
C
$ 14,500
D
$ 32,000
A

Answer is C

Explanation:
Cost of goods sold will be two-thirds of the goods shipped (including two-thirds of the freight), but all of the advertising and commission will be deducted from gross profit to determine net profit.

Revenue $ 80,000
Less: Cost of consigned goods $ 72,000
Plus Freight for consigned goods ____7,500
Total cost of consigned goods $ 79,500
Portion sold ______2/3
Cost of Goods Sold (rounded) _(53,000)
Gross Profit $ 27,000
Less advertising (4,500)
Less commission __(8,000)
Net Profit $ 14,500
Option (A) is incorrect because it is net profit as per above and not loss.

Option (B) is incorrect because $8,000 is net of sales value of the 2/3rd inventory sold by the consignee and the cost of the merchandise shipped to the consignee ($80,000 - $72,000).

Option (D) is incorrect because it does not deduct 2/3rd portion of the freight charges, advertising charges and 10% commission.

How well did you know this?
1
Not at all
2
3
4
5
Perfectly
63
Q

A firm’s ending inventory balance was overstated by $1,000. Which of the following statements is correct according to a periodic inventory system?
A
The retained earnings were overstated by $1,000.
B
The cost of goods sold was overstated by $1,000.
C
The cost of goods available for sale was overstated by $1,000.
D
The gross margin was understated by $1,000.

A

ANswer is A

Explanation:
In a periodic inventory system the inventory on hand is periodically determined by physical count. No entries are made to the inventory account during the period; the account reflects the amount at the beginning of the period until inventory is counted. A firm’s ending inventory balance being overstated by $1,000 would cause the retained earnings to be overstated by $1,000. Cost of goods sold (COGS) is obtained by subtracting the ending inventory from the sum of beginning inventory and net purchases, so COGS would be understated by $1,000. The cost of goods available is the sum of the beginning inventory and net purchases, and thus it is not affected by the ending inventory being overstated. The gross margin is a percentage used as part of the gross margin method to determine COGS. It may be used to verify the accuracy of the year-end physical count but otherwise it is not affected by the periodic inventory system.

How well did you know this?
1
Not at all
2
3
4
5
Perfectly
64
Q

Which of the following is not an indicator that an entity is an agent?

A
The entity does not have inventory risk before or after the goods have been ordered by a customer, during shipping, or on return.
B
The entity does not have discretion in establishing prices for the other party’s goods or services and, thus, the benefit that the entity can receive from those goods or services is limited.
C
The entity is exposed to credit risk for the amount receivable from a customer in exchange for the other party’s goods or services.
D
Another party is primarily responsible for fulfilling the contract.

A

ANswer is C

Explanation:
An agent is not exposed to credit risk for the amount receivable from a customer in exchange for the other party’s goods or services. The following are indicators that an entity is an agent –

Another party is primarily responsible for fulfilling the contract.
The entity does not have inventory risk before or after the goods have been ordered by a customer, during shipping, or on return.
The entity does not have discretion in establishing prices for the other party’s goods or services and, thus, the benefit that the entity can receive from those goods or services is limited.
The entity’s consideration is in the form of a commission.
The entity is not exposed to credit risk for the amount receivable from a customer in exchange for the other party’s goods or services.
Options (A), (B) and (D) are incorrect as they are all indicators that an entity is an agent.

How well did you know this?
1
Not at all
2
3
4
5
Perfectly
65
Q
The original cost of an inventory item is below both replacement cost and net realizable value. The net realizable value less normal profit margin is below the original cost. Under the lower of cost or market method, the inventory item should be valued at
A
Replacement cost.
B
Net realizable value.
C
Net realizable value less normal profit margin.
D
Original cost.
A

Answer is D

Explanation:
According to the lower of cost or market rule, market is defined as replacement cost. Market cannot exceed net realizable value and cannot be less than net realizable value less normal profit margin. In this instance, original cost is between net realizable value and net realizable value less normal profit margin. Since original cost is within the parameters for replacement cost and is less than replacement cost, the inventory should be reported at original cost.

How well did you know this?
1
Not at all
2
3
4
5
Perfectly
66
Q
The following information was derived from the current year accounting records of Clem Co.:
Clem's Central Warehouse	Clem's goods held by consignees
Beginning inventory	$110,000	$12,000
Purchases	$480,000	$60,000
Freight in		$10,000
Transportation to consignees		$ 5,000
Freight out	$ 30,000	$ 8,000
Ending inventory	$145,000	$20,000Clem's cost of sales for the year was:
A
$455,000
B
$485,000
C
$507,000
D
$512,000
A

Answer is D

Explanation:
Goods out on consignment remain the property of the consignor and must be included in the consignor’s inventory at purchase price or production cost, including freight in and other costs incurred to process the goods up to the time of sale.

Beginning inventory ($110,000 + $12,000) $122,000
Add: Purchases ($480,000 + $60,000) $540,000
Freight in 10,000
Transportation to consignees 5,000
Add: Total inventoriable costs 555,000
Goods available for sale 677,000
Less ending inventory ($145,000 + $20,000) 165,000
Cost of sales for the year $512,000

How well did you know this?
1
Not at all
2
3
4
5
Perfectly
67
Q

Estimates of price-level changes for specific inventories are required for which of the following inventory methods?

A
Conventional retail
B
Dollar-value LIFO
C
Weighted average cost
D
Average cost retail
A

Answer is B

Explanation:
The dollar value LIFO method requires estimates of price-level changes for specific inventories. Under this method, goods are combined into pools and are traced by their dollar value, corrected for inflation. The annual layer added to the pool is tracked by calculating the ending inventory @ base year-end price. This is done to remove the effects of inflation from annual LIFO layers to gauge whether the increase or decreases to the inventory are real or due to inflation. The annual layer added @ base is then adjusted @current year price.

Option (A) is incorrect because the conventional retail inventory method is used by retailers to estimate the cost of their ending inventory by using the relationship between the cost of merchandise and its retail price (cost-to-retail ratio) arrived at after considering both net markups and markdowns.

Option (C) is incorrect because the weighted average cost method calculates the value of the ending inventory based upon the weighted average price paid.

Option (D) is incorrect because the average cost retail method does not require estimates of price-level changes for specific inventories.

How well did you know this?
1
Not at all
2
3
4
5
Perfectly
68
Q
On December 28, Kerr Manufacturing Co. purchased goods costing $50,000. The terms were F.O.B. destination. Some of the costs incurred in connection with the sale and delivery of the goods were as follows:
Packaging for shipment	$1,000
Shipping	$1,500
Special handling charges	$2,000These goods were received on December 31. In Kerr's December 31 balance sheet, what amount of cost for these goods should be included in inventory?
A
$54,500
B
$53,500
C
$52,000
D
$50,000
A

Answer is D

Explanation:
The term F.O.B. destination means free on board at destination; that is, the goods are shipped to their destination without charge to the buyer. Thus, the costs incurred in connection with the sale and delivery of the goods (i.e, packaging for shipment, shipping, and special handling charges) are borne by the seller. Thus, Kerr’s cost of the goods purchased is $50,000.

How well did you know this?
1
Not at all
2
3
4
5
Perfectly
69
Q

Fireworks, Inc. had an explosion in its plant that destroyed most of its inventory. Its records show that beginning inventory was $40,000. Fireworks made purchases of $480,000 and sales of $620,000 during the year. Its normal gross profit percentage is 25%. It can sell some of its damaged inventory for $5,000. The insurance company will reimburse Fireworks for 70% of its loss. What amount should Fireworks report as loss from the explosion?

A
$50,000
B
$35,000
C
$18,000
D
$15,000
A

ANswer is D

Explanation:
Beginning inventory + Purchases - COGS - Ending inventory - Available inventory = Inventory damaged in explosion.

Ref

       Summary

 Amount

a

Beginning Inventory

$40,000

b

Purchase, during the year

$480,000

c

Total Inventory

$520,000

d

COGS for Inventory Sold ($620,000 x (1-0.25))

$465,000

e

Proceeds from damaged Inventory

$5,000

f

Inventory Lost in Explosion (c-d-e)

$50,000

g

Proceeds from Insurance Company (f x 70%)

$35,000

h

Loss from Explosion (f-g)

$15,000

Option (A) is incorrect because $50,000 is the loss determined.

Option (B) is incorrect because $35,000 is the proceeds from insurance claim.

Option (C) is incorrect as per the above explanation.

How well did you know this?
1
Not at all
2
3
4
5
Perfectly
70
Q

As of December 31, year 2, a company has an inventory item that was originally purchased for $80 in year 1. The inventory item was written down to its net realizable value of $60 as of December 31, year 1. As of December 31, year 2, the inventory item had a net realizable value of $75 and a replacement cost of $65. Normal profit margins for this company are 20%. Under IFRS, what is the carrying amount of the inventory item as of December 31, year 2?

A
$60
B
$65
C
$75
D
$80
A

Explanation:
The correct answer is (C)

The carrying amount of the inventory item as of December 31st, year 2 is $75.

Under IFRS, inventories are measured at the lower of cost or net realizable value (NRV). Inventories will be written down if needed, and a write-down can only be reversed up to the original cost. On December 31, year 1 the cost of the inventory was $80 and the net realizable value (NRV) was $60. The inventory would be valued at the lower of the two values and would be recorded at $60 in the financial statements at the end of year 1. As of December 31, year 2, the NRV of the inventory was $75. The original cost was $80. The inventory would continue to be stated at the NRV of $75 as this is lower than the original cost of $80.

How well did you know this?
1
Not at all
2
3
4
5
Perfectly
71
Q
Ames Company determined the following values for its inventory as of December 31:
Historical Cost	$200,000
Replacement Cost	$160,000
Sales Value	$190,000
Cost to Complete and Sell	$10,000
Normal Profit Margin	$8,000
Fair Value	$194,000Under IFRS, what amount should Ames report for inventory at December 31?
A
$194,000
B
$180,000
C
$172,000
D
$160,000
A

Answer is B

Explanation:
Compute market (i.e., NRV) and compare to cost

NRV = Selling price - disposal cost = [$190,000 - $10,000] = $180,000

Cost = $200,000

How well did you know this?
1
Not at all
2
3
4
5
Perfectly
72
Q
Bach Co. adopted the dollar value LIFO inventory method as of January 1, year 1. A single inventory pool and an internally computed price index are used to compute Bach''s LIFO inventory layers. Information about Bach''s dollar value inventory follows:
Inventory at
Date	Base year cost	Current year cost
1/1, Year 1	$90,000	$90,000
Year 1 layer	$20,000	$30,000
Year 2 layer	$40,000	$80,000What was the price index used to compute Bach''s Year 2 dollar value LIFO inventory layer?
A
1.09
B
1.25
C
1.33
D
2.00
A

Answer is C

Explanation:
The price index is computed by dividing the ending inventory at current year cost by its base year cost, $200,000 / $150,000 = 1.333.

Date Base year cost Current year cost
1/1, Year 1 $90,000 $90,000
Year 1 layer 20,000 30,000
12/31 year 1 $110,000 $120,000
Year 2 layer 40,000 80,000
12/31 year 2 $150,000 $200,000

How well did you know this?
1
Not at all
2
3
4
5
Perfectly
73
Q

Beck Co.’s inventory of trees is as follows:

Beginning Inventory	 	10 trees at $50
March 4	Purchased	6 trees at $55
March 12	Sold	8 trees at $100
March 20	Purchased	9 trees at $60
March 27	Sold	7 trees at $105
March 30	Purchased	4 trees at $6
What was Beck's cost of goods sold using the last in, first out (LIFO) perpetual method?
A
$910
B
$850
C
$808
D
$775
A

Explanation:
The correct answer is (B).

7 trees sold March 27th are assumed to come entirely from the March 20th purchase at $60/each.
6 of the 8 trees sold March 12th are assumed to come from the March 4th purchase at $55/each and balance, 2 trees will come from beginning inventory at $50/each.
Cost of Goods Sold (COGS) using LIFO is [(7 x $60) + (6 x $55) + (2 x $50)] = $850

How well did you know this?
1
Not at all
2
3
4
5
Perfectly
74
Q
Seafood Trading Co. commenced operations during the year as a large importer and exporter of seafood. The imports were all from one country overseas. The export sales were conducted as drop shipments and were merely transshipped at Seattle. Seafood Trading reported the following data:
Purchases during the year	$12.0 million
Shipping costs from overseas	1.5 million
Shipping costs to export customers	1.0 million
Inventory at year end	3.0 millionWhat amount of shipping costs should be included in Seafood Trading's year-end inventory valuation?
A
$0
B
$250,000
C
$375,000
D
$625,000
A

ANswer is C

Explanation:
Merchandise inventory should include freight-in, taxes, insurance while in transit, warehousing costs, and similar charges paid by the purchaser to bring the merchandise to its existing condition and location. Thus, the $1.5 million in shipping costs from overseas should be included in the inventory valuation. The shipping costs to export customers are a selling expense and should not be included in the cost of inventory. Seafood purchased $12 million in inventory during the year and has $3 million remaining in inventory at year end. The $12 million divided by $3 million means one-quarter of the inventory is still remaining. Thus, one-quarter of the $1.5 million, or $375,000, in shipping costs from overseas should be included in the year-end inventory valuation.

How well did you know this?
1
Not at all
2
3
4
5
Perfectly
75
Q

For a customer to have obtained control of a product in a bill-and-hold arrangement, a substantive reason for the bill-and-hold arrangement must exist. Other criteria to be met for a bill-and-hold arrangement would be:

Product must be identified separately as belonging to the customer.
Product currently must be ready for physical transfer to the customer.
Entity cannot use the product or direct it to another customer.
Substantive reason for the bill-and-hold arrangement (e.g., the customer has requested the arrangement).
A
i, ii, and iv.
B
i and ii.
C
i, iii, and iv.
D
i, ii, iii, and iv.

A

Explanation:
The correct answer is (D).

A bill-and-hold arrangement is a contract under which an entity bills a customer for a product but the entity retains (i.e., holds) physical possession of the product until it is transferred to the customer at a point in time in the future. Revenue should be recognized only when the customer obtains control of the product.

For a customer to have obtained control of a product in a bill-and-hold arrangement, all of the following criteria must be met:

Substantive reason for the bill-and-hold arrangement (e.g., the customer has requested the arrangement).
Product must be identified separately as belonging to the customer.
The product currently must be ready for physical transfer to the customer
The entity cannot use the product or direct it to another customer.

How well did you know this?
1
Not at all
2
3
4
5
Perfectly
76
Q

Herc Co.’s inventory at December 31 of the previous year was $1,500,000, based on a physical count priced at cost, and before any necessary adjustment for the following:
Merchandise costing $90,000, shipped FOB shipping point from a vendor on December 30 of the previous year was received and recorded on January 5 of the current year.
Goods in the shipping area were excluded from inventory although shipment was not made until January 4 of the current year. The goods, billed to the customer FOB shipping point on December 30 had a cost of $120,000.
What amount should Herc report as inventory in its December 31, previous year balance sheet?
A
$1,500,000
B
$1,590,000
C
$1,620,000
D
$1,710,000

A

Answer is D

Explanation:
Goods should be included in the purchaser’s inventory when legal title passes to the purchaser. Therefore, Herc should include the $90,000 cost of goods shipped to it FOB shipping point in inventory at 12/31 of the previous year because title to these goods passed to Herc when the goods were picked up by the common carrier on 12/30. Herc should also include the $120,000 cost of goods in its shipping area in inventory at 12/31 of the previous year. These goods should be included in inventory because shipment of these goods to the customer was not made until the current year.

How well did you know this?
1
Not at all
2
3
4
5
Perfectly
77
Q
During January, Metro Co., which maintains a perpetual inventory system, recorded the following information pertaining to its inventory:
Units	Unit Cost	Total Cost	Units on Hand
Balance on 1/1	1,000	$1	$1,000	1,000
Purchased on 1/7	600	3	1,800	1,600
Sold on 1/20	900			700
Purchased on 1/25	400	5	2,000	1,100Under the LIFO method, what amount should Metro report as inventory at January 31?
A
$1,300
B
$2,700
C
$3,900
D
$4,100
A

ANswer is B

Explanation:
Where the LIFO cost flow method is used in conjunction with a perpetual inventory system, the cost of the last goods purchased are matched against revenue every time a sale is made ($700 + $2,000 = $2,700).

Date	Purchased	Sold	Balance
1/1			(1,000 @ $1) $1,000
1/7	(600 @ $3) $1,800		(1,000 @ $1) $1,000
(600 @ $3) $1,800
1/20		(600 @ $3) $1,800	(700 @ $1) $ 700
(300 @ $1) $ 300	
1/25	(400 @ $5) $2,000		(700 @ $1) $ 700
(400 @ $5) $2,000
How well did you know this?
1
Not at all
2
3
4
5
Perfectly
78
Q

Loft Co. reviewed its inventory values for proper pricing at year end. The following summarizes two inventory items examined for the lower of cost or market:

Inventory Item #1 Inventory Item #2

Original cost $210,000 $400,000
Replacement cost 150,000 370,000
Net realizable value 240,000 410,000
Net realizable value less profit margin 208,000 405,000
What amount should Loft include in inventory at year end, if it uses the total of the inventory to apply the lower of cost or market?

A
$520,000
B
$610,000
C
$613,000
D
$650,000
A

Answer is B

Explanation:
In applying the lower of cost or market, one must first determine the market value. The market refers to the current replacement cost, yet it should not exceed a ceiling (the net realizable value) nor be less than a floor, the net realizable value minus normal profit. The market value is determined by comparing the ceiling, floor and replacement costs. The total inventory ceiling is $650,000 ($240,000 + $410,000), the total inventory floor is $613,000 ($208,000 + $405,000), and the total inventory replacement cost is $520,000 ($150,000 + $370,000). The middle number is selected as the market; here it is the $613,000 floor amount. The total inventory cost of $610,000 is the value of inventory at year end because it is lower than the $613,000 market value.

Option (A) is incorrect because it does not use relevant information.

Option (C) is incorrect because it uses NRV - Normal profit.

Option (D) is incorrect because it uses NRV.

How well did you know this?
1
Not at all
2
3
4
5
Perfectly
79
Q

On December 31 of the previous year, Jason Company adopted the dollar-value LIFO retail inventory method. Inventory data are as follows:

LIFO Cost

 Retail  

Inventory, 12/31 previous year

$360,000

$500,000

Inventory, 12/31 current year

660,000

Increase in price level for current year

10%

Cost to retail ratio for current year

70%

Under the LIFO retail method, Jason’s inventory at December 31 of the current year should be

A
$437,000
B
$462,000
C
$472,000
D
$483,200
A

Answer is A

Explanation:
Inventory at retail, 12/31 current year adjusted ($660,000 / 1.1)

$ 600,000

Beginning inventory at retail, base year price

     (500,000)

New layer added in current year

      100,000

Times: Price level adjustment

  ×         1.1

Current year layer, at LIFO retail

      110,000

Times: Cost to retail ratio

  ×       0.70

Current year layer, at LIFO cost

       77,000

Add: Beg. inventory, at LIFO cost

      360,000

Ending inventory, 12/31 current year

  $  437,000
How well did you know this?
1
Not at all
2
3
4
5
Perfectly
80
Q
On December 30 of the previous year, Astor Corp. sold merchandise for $75,000 to Day Co. The terms of the sale were net 30, FOB shipping point. The merchandise was shipped last December 31 and arrived at Day on January 5 of the current year. Due to a clerical error, the sale was not recorded until January and the merchandise, sold at a 25% markup, was included in Astor's inventory at December 31. As a result, Astor's cost of goods sold for the previous year ended December 31 was
A
Understated by $75,000.
B
Understated by $60,000.
C
Understated by $15,000.
D
Correctly stated.
A

Answer is B

Explanation:
Goods should be removed from the seller’s inventory when legal title passes to the purchaser. Therefore, Astor should exclude the cost of the goods shipped FOB shipping from inventory at 12/31 because title of these goods passed to Day when the goods were picked up by the common carrier on 12/31. The cost of the goods is calculated by dividing the selling price by 125% ($75,000 / 1.25 = $60,000). Since the goods were erroneously in Astor’s inventory at 12/31, Astor’s previous year ending inventory and cost of goods sold were overstated and understated, respectively, by the $60,000 cost of the goods.

How well did you know this?
1
Not at all
2
3
4
5
Perfectly
81
Q
Which of the following would not be included in the cost of the finished goods inventory of a manufacturer?
A
Direct materials
B
Direct labor
C
Fixed and variable manufacturing overhead
D
Freight-out expenses
A

Answer is D

Explanation:
Freight-out expense is a selling expense and should not be included in the cost of inventory.

How well did you know this?
1
Not at all
2
3
4
5
Perfectly
82
Q

Which inventory costing method would a company that wishes to maximize profits in a period of rising prices use?

A
FIFO
B
Dollar-value LIFO
C
Weighted average
D
Moving average
A

Answer is A

Explanation:
The correct answer is (A).

The FIFO inventory costing method assumes that the goods first acquired are the first sold.

This would maximize profits in a period of rising prices due to COGS being comprised of goods purchased at lower prices, which leads to a higher profit.

How well did you know this?
1
Not at all
2
3
4
5
Perfectly
83
Q
The lower-of-cost-or-market rule for inventories may be applied to total inventory, to groups of similar items, or to each item. Which application generally results in the lowest inventory amount?
A
All applications result in the same amount
B
Total inventory
C
Groups of similar items
D
Separately to each item
A

Answer is D

Explanation:
The application of the lower of cost or market (LCM) rule directly to each inventory item generally results in the lowest inventory amount because unrealized losses on inventory items cannot be offset by unrealized gains on other inventory items. Generally, a different inventory amount would be reported when the LCM rule is applied to (1) total inventory, (2) groups of similar inventory items, or (3) each inventory item. The total inventory and groups of similar items methods of applying the LCM rule for inventories would allow unrealized losses on some inventory items to be offset by unrealized gains on others. This would result in a higher inventory amount than if the LCM rules were applied to each inventory item.

How well did you know this?
1
Not at all
2
3
4
5
Perfectly
84
Q

A company manufactured 1,000 units of product during the year and sold 800 units. Costs incurred during the current year are as follows:

Direct materials and direct labor $7,000
Indirect materials and indirect labor $2,000
Insurance on manufacturing equipment $3,000
Advertising $1,000
What amount should be reported as inventory in the company’s year-end balance sheet?

A
$1,400
B
$1,800
C
$2,600
D
$2,400
A

Answer is D

Explanation:
The correct answer is (D).

In order to compute the inventory cost, the following items are considered:

Direct material, labor, and overheads
Freight inwards, insurance and warehousing (up to the point of sale)
Handling expenses, normal spoilage, repacking
Discounts received, if any
In the given case, per unit inventory cost can be computed as under:

Description Amount
Direct materials and direct labor 7,000
Indirect materials and indirect labor 2,000
Insurance on manufacturing equipment 3,000
Total inventory costs 12,000
Total units manufactured = 1,000
Per unit cost = 12,000/1,000 12
Closing stock = Units produced – Units sold = 1,000 – 800 = 200 units

The value of the inventory in the year-end = 200 x 12 = $2,400

How well did you know this?
1
Not at all
2
3
4
5
Perfectly
85
Q
On January 1 of the current year, Card Corp. signed a three-year, noncancelable purchase contract, which allows Card to purchase up to 500,000 units of a computer part annually from Hart Supply Co. at $.10 per unit and guarantees a minimum annual purchase of 100,000 units. During the year, the parts unexpectedly became obsolete. Card had 250,000 units of this inventory at December 31 and believes these parts can be sold as scrap for $.02 per unit. What amount of probable loss from the purchase commitment should Card report in its year-end income statement?
A
$24,000
B
$20,000
C
$16,000
D
$ 8,000
A

Answer is C

Explanation:
A loss on the purchase commitment should be calculated based only on the minimum unit purchase requirement for the remaining years on the contract. Therefore, Card should calculate its loss at 12/31 based on the two years remaining on the purchase contract.

Minimum annual unit purchase requirement 100,000
Years remaining on contract (3 - 1) x 2
Minimum unit purchase requirement for remaining duration on contract 200,000
Expected loss per unit purchased ($0.10 - $0.02) x $0.08
Probable loss on purchase commitment $ 16,000

How well did you know this?
1
Not at all
2
3
4
5
Perfectly
86
Q

Simm Co. has determined its December 31 inventory on a LIFO basis to be $400,000. Information pertaining to the inventory follows:

Estimated selling price	$408,000
Estimated cost of disposal	$20,000
Normal profit margin	$60,000
Current replacement cost	$390,000
At December 31, what should be the amount of Simm's inventory?
A
$400,000
B
$390,000
C
$388,000
D
$328,000
A

Explanation:
The correct answer is (C).

When an entity follows the LIFO method of inventory valuation, the inventory is valued at the lower of cost or market value. Market value is the middle of:

Ceiling = Net Realizable Value = Selling price – costs of disposal
Floor = Net Realizable Value – normal profit margin
Replacement Cost
In the given case, market value shall be the middle of:

Ceiling: 408,000 – 20,000 = $388,000
Floor: 388,000 – 60,000 = $328,000
Current Replacement Cost: $390,000
Market value = $388,000

The inventory should be valued at the lower of $390,000 and $388,000.

The final value of inventory is $388,000

How well did you know this?
1
Not at all
2
3
4
5
Perfectly
87
Q

At the end of the year, Ian Co. determined its inventory to be $258,000 on a FIFO (first in, first out) basis. The current replacement cost of this inventory was $230,000. Ian estimates that it could sell the inventory for $275,000 at a disposal cost of $14,000. If Ian’s normal profit margin for its inventory was $10,000, what would be its net carrying value?

A
$244,000
B
$251,000
C
$258,000
D
$261,000
A

Answer is C

Explanation:
The correct answer is (C).

LIFO or Retail inventories are valued at the Lower of Cost or Market.

FIFO, Weighted Avg, and inventories determined by methods other than LIFO or Retail use the Lower of Cost or Net Realizable Value.

Ian Co. determined its inventory on a FIFO basis and its Inventory should be valued at Lower of Cost or Net Realizable Value (NRV).

Cost = $258,000.
NRV = Net Selling Price - Cost to complete & dispose ($275,000 - $14,000 = $261,000).
As Cost is lower than the NRV, inventory is valued at cost.

Option (A) is incorrect because it reduces the selling cost from the original cost of inventory.

Option (B) is incorrect because it uses market value instead of taking the net realizable value for valuing inventory on a FIFO basis.

Option (D) is incorrect because the cost is lower than the net realizable value.

How well did you know this?
1
Not at all
2
3
4
5
Perfectly
88
Q
Information with respect to Bruno Co.'s cost of goods sold for the current year is as follows:
Historical cost	Units
Inventory, 1/1	$1,060,000	20,000
Production during year	5,580,000	90,000
6,640,000	110,000
Inventory, 12/31	2,520,000	40,000
Costs of goods sold	$4,120,000	70,000Bruno estimates that the current cost per unit of inventory was $58 at January 1, and $72 at December 31. In Bruno's supplementary information restated into average current cost, the cost of goods sold for the current year should be
A
$5,040,000
B
$4,550,000
C
$4,410,000
D
$4,060,000
A

Answer is B

Explanation:
To calculate cost of goods sold on a current cost basis, multiply the number of units sold by the average current cost of the units during the period (sum of the current cost of the units at the beginning and the end of the period, divided by two).

Current cost per unit, 1/17 $ 58
Current cost per unit, 12/31 + 72
Total 130
Divide by two / 2
Average current cost per unit during the year 65
Times units sold in the year × 70,000
Cost of goods sold, average current cost $4,550,000

How well did you know this?
1
Not at all
2
3
4
5
Perfectly
89
Q

On December 31 of the previous year, Jason Company adopted the dollar-value LIFO retail inventory method. Inventory data are as follows:

LIFO Cost Retail

Inventory, 12/31 previous year $360,000 $500,000
Inventory, 12/31 current year — 660,000
Increase in price level for current year 10%
Cost to retail ratio for current year 70%
Under the LIFO retail method, Jason’s inventory at December 31 of the current year should be

A
$437,000
B
$462,000
C
$472,000
D
$483,200
A

Answer is A

Explanation:
Inventory at retail, 12/31 current year adjusted ($660,000 / 1.1) $ 600,000
Beginning inventory at retail, base year price (500,000)
New layer added in current year 100,000
Times: Price level adjustment × 1.1
Current year layer, at LIFO retail 110,000
Times: Cost to retail ratio × 0.70
Current year layer, at LIFO cost 77,000
Add: Beg. inventory, at LIFO cost 360,000
Ending inventory, 12/31 current year $ 437,000

How well did you know this?
1
Not at all
2
3
4
5
Perfectly
90
Q

Which of the following statements are correct when a company applying the lower-of-cost-or-market method reports its inventory at replacement cost?

The original cost is less than replacement cost.
The net realizable value is greater than replacement cost.
A
I only
B
II only
C
Both I and II
D
Neither I nor II
A

Answer is B

Explanation:
The answer to this question assumes that the original cost, replacement cost, and net realizable value of the inventory differ in amount. Statement II is correct. Under lower-of-cost-or-market (LCM) procedures for inventory valuation, market value cannot exceed a ‘ceiling’ of net realizable value and cannot be below a ‘floor’ of net realizable value reduced by a normal profit margin. Since the inventory is reported at its replacement cost, the net realizable of the inventory exceeds its replacement cost. Statement I is incorrect. Under LCM procedures, inventory is reported at the lower of original cost or market value (which is replacement cost in this case). Since the inventory is reported at replacement cost, the original cost of the inventory is greater than its replacement cost.

How well did you know this?
1
Not at all
2
3
4
5
Perfectly
91
Q
Garcel, Inc. held unfinished inventory at a cost of $85,000 with a sales value of $125,000. The inventory will cost $10,500 to complete. The normal profit margin is 30% of sales. The replacement cost of the inventory was $75,000. What amount should Garcel report as inventory on balance sheet?
A
$114,500
B
$85,000
C
$77,000
D
$75,000
A

Answer is C

Explanation:
Garcel should report the unfinished inventory at the lower of cost or market. Market means current replacement cost except that market should not exceed the net realizable value and market should not be less than the net realizable value minus normal profit. The net realizable value is the sales value less reasonably predictable costs of completion ($125,000 – $10,500 = $114,500) and this is the ceiling value. The normal profit is the estimated sales value times the normal profit margin: $125,000 × 30% = $37,500. The net realizable value minus normal profit ($114,500 – $37,500 = $77,000) is the floor value. The replacement cost is $75,000 but the market cannot be lower than the floor value of $77,000 so $77,000 would be the market value. The lower of cost ($85,000) or market ($77,000) is the market value of $77,000.

How well did you know this?
1
Not at all
2
3
4
5
Perfectly
92
Q

A company records inventory at the gross invoice price. Theoretically, how should the following affect the costs in inventory?
Warehousing costs Cash discounts available
A Increase Decrease
B No effect Decrease
C No effect No effect
D Increase No effect

A

Explanation:
Warehousing costs are usually treated as an expense in the period in which they are incurred, although conceptually they comprise part of the total cost of merchandise made ready for sale. Conceptually, the cost of inventory should be reduced for cash discounts available, because the acquisition cost of an asset should not exceed its cash equivalent price.

How well did you know this?
1
Not at all
2
3
4
5
Perfectly
93
Q
Rose Co. sells one product and uses the last-in, first-out method to determine inventory cost. Information for the month of January follows:
Total Units	Unit Cost
Beginning inventory, 1/1	8,000	$8.20
Purchases, 1/5	12,000	7.90
Sales	10,000	Rose has determined that at January 31, the replacement cost of its inventory was $8 per unit and the net realizable value was $8.80 per unit. Rose's normal profit margin is $1 per unit. Rose applies the lower of cost or market rule to total inventory and records any resulting loss. At January 31, what should be the net carrying amount of Rose's inventory?
A
$79,000
B
$79,800
C
$80,000
D
$81,400
A

Answer is C

Explanation:
The market maximum, or ceiling, should not exceed the net realizable value (NRV), which is the estimated selling price in the ordinary course of business less reasonably predictable costs of completion and disposal. Market minimum, or floor should not be less than the net realizable value minus normal profit.

Market:
Ceiling (NRV) $8.80
Replacement cost $8.00
Floor (NRV minus normal profit) $7.80LIFO charges Cost of Goods Sold with the latest acquisition costs, while ending inventories are reported at the older costs of the earliest units. The 10,000 units sold reduce the 1/5 purchases, leaving 8,000 units from the beginning inventory and 2,000 units purchased on 1/5.
Cost [(8,000 x $8.20) + (2,000 x $7.90)] $81,400
Market (10,000 x $8.00) $80,000$80,000 is the lowest of cost or market.

How well did you know this?
1
Not at all
2
3
4
5
Perfectly
94
Q

The following items were included in Opal Co.’s inventory account at December 31:

Merchandise out on consignment, at sales price, including 40% markup on selling price $40,000
Goods purchased, in transit, shipped F.O.B. shipping point $36,000
Goods held on consignment by Opal $27,000
By what amount should Opal’s inventory account at December 31 be reduced?

A
$103,000
B
$ 67,000
C
$ 51,000
D
$ 43,000
A

Explanation:
The correct answer is (D).

Opal’s inventory account at December 31, be reduced by $16,000 + $27,000 = $43,000. Merchandise that is out on consignment will be included in Opal’s inventory till the time it is sold by the consignor.

As it is included in inventory at the sales price, including the margin on the selling price, the same will have to be adjusted for margin to reflect the cost of the inventory on consignment.

The $40,000 will have be reduced by margin of 40% = $16,000 (i.e. 40% of $40,000).

Goods purchased, in transit, shipped f.o.b. shipping point, will not be adjusted as they are already included in the inventory.

Goods held on consignment by Opal will be included in the inventory of the consignor, so inventory will have to be reduced by $27,000.

How well did you know this?
1
Not at all
2
3
4
5
Perfectly
95
Q

The retail inventory method includes which of the following in the calculation of both cost and retail amounts of goods available for sale?

A
Purchase returns
B
Sales returns
C
Net markups
D
Freight in
A

Answer is A

Explanation:
When the retail method is employed, purchase returns is included in the calculation of both cost and retail amounts of goods available for sale (AFS). Sales returns does not appear in the computation of the cost amount of goods AFS. Net markups appears in the retail amount of goods AFS (assuming the retail method is used to approximate a lower of average cost or market figure) but not in the cost amount of goods AFS. Freight in appears in the cost amount of goods available for sale but not in the retail amount of goods AFS.

How well did you know this?
1
Not at all
2
3
4
5
Perfectly
96
Q

The following information was obtained from Smith Co.:

Sales $275,000
Beginning inventory 30,000
Ending inventory 18,000
Smith’s gross margin is 20%. What amount represents Smith purchases?

A
$202,000
B
$208,000
C
$220,000
D
$232,000
A

Explanation:
The correct answer is (B).

Beginning inventory + Net purchases - Ending inventory = COGS

Net Purchases = Ending inventory + COGS - Beginning inventory

Net Purchases = ($18,000 + $220,000 - $30,000) = $208,000. [COGS = Sales x (1- Gross Margin)]

[$275,000 (1-0.2) = $220,000.]

Option (A) and (D) are incorrect as per the above explanation.

Option (C) is incorrect because $220,000 is the COGS, not the net purchases.

Solve for Goods Available for Sale:
(GAFS – COGS = EI)
GAFS – $220,000 = $18,000
GAFS = $238,000

Solve for Purchases:
(BI – Purch = GAFS)
$30,000 – Purch = $238,000
Purchases = $208,000.

Option (A) and (D) are incorrect as per the above explanation.

Option (C) is incorrect because $220,000 is the COGS, not the net purchases

How well did you know this?
1
Not at all
2
3
4
5
Perfectly
97
Q

At the end of year 1, a company reduced its inventory cost from $100 to its net realizable value of $80. As of the end of year 2, the inventory was still on hand and its net realizable value increased to $150. Under IFRS, what journal entry should the company record for year 2 to properly report the inventory value?

A
Debit inventory for $20 and credit expense for $20
B
Debit inventory for $70 and credit expense for $70
C
Debit inventory for $70, credit retained earnings for $50 and credit expense for $20
D
Debit inventory for $20, debit expense for $30 and credit retained earnings for $50

A

Answer is A

Explanation:
The correct answer is (A).

Inventory was written down to its NRV of $80 (from $100). Under IFRS, Net Realizable Value (NRV) is the best approximation of how much inventories are expected to realize moving forward. IFRS only allows inventory recovery up to the point that was written off initially - i.e., $100.Although the NRV has increased to well above the original inventory cost (book value), only $20 of recovery is allowed.

The B/S entry would be:

Inventory $20
Expense $20
Note: Recoveries are recorded as expenses in the I/S because they are a Reduction to COGS.

How well did you know this?
1
Not at all
2
3
4
5
Perfectly
98
Q

Under IFRS, which of the following is not an acceptable method of accounting for inventory?

A
Gross profit method
B
Retail method
C
LIFO
D
Weighted average
A

Answer is C

Explanation:
Under IFRS, specific identification is required when the goods are not interchangeable; otherwise, you can use FIFO, the gross profit method (if a physical count is not possible), the retail method (in certain industries), and weighted average. LIFO is prohibited under IFRS.

How well did you know this?
1
Not at all
2
3
4
5
Perfectly
99
Q
During January, Metro Co., which maintains a perpetual inventory system, recorded the following information pertaining to its inventory:
Units	Unit Cost	Total Cost	Units on Hand
Balance on 1/1	1,000	$1	$1,000	1,000
Purchased on 1/7	600	3	1,800	1,600
Sold on 1/20	900			700
Purchased on 1/25	400	5	2,000	1,100Under the moving-average method, what amount should Metro report as inventory at January 31?
A
$2,640
B
$3,225
C
$3,300
D
$3,900
A

Answer is B

Explanation:
Under the moving-average method, a new average unit price is computed every time a purchase is made. The inventory is then priced on the basis of this “moving average.”

Units	Unit cost	Total cost
Balance on 1/1	1,000	$1.00	$1,000
Purchased on 1/7	600	3.00	1,800
Balance after purchase	1,600	1.75 *	2,800
Sold on 1/20	900	1.75	1,575
Balance after sale	700	1.75	1,225
Purchased on 1/25	400	5.00	2,000
Balance after purchase	1,100		$3,225
* $2,800 / 1,600 units
How well did you know this?
1
Not at all
2
3
4
5
Perfectly
100
Q
During the current year, Kam Co. began offering its goods to selected retailers on a consignment basis. The following information was derived from Kam's current year accounting records:
Beginning Inventory	$122,000
Purchases	540,000
Freight in	10,000
Transportation to consignees	5,000
Freight out	35,000
Ending Inventory--held by Kam	145,000
Ending Inventory--held by consignees	20,000In its current year income statement, what amount should Kam report as cost of goods sold?
A
$507,000
B
$512,000
C
$527,000
D
$547,000
A

Answer is B

Explanation:
Goods out on consignment remain the property of the consignor and must be included in the consignor’s inventory at purchase price or production cost, including freight and other costs incurred to process the goods up to the time of sale.

Beginning inventory $122,000
Add: Purchases $540,000
Freight in shipping 10,000
Transportation to consignees 5,000
Add: Total inventoriable costs 555,000
Goods available for sale 677,000
Less: Ending inventory ($145,000 + $20,000) (165,000)
Cost of goods sold $512,000

How well did you know this?
1
Not at all
2
3
4
5
Perfectly
101
Q
According to the FASB conceptual framework, which of the following attributes would not be used to measure inventory?
A
Historical cost
B
Replacement cost
C
Net realizable value
D
Present value of future cash flows
A

Answer is D

Explanation:
The present value of future cash flows (which would include profits not yet earned) pertains to the time value of money and is not appropriate for measuring inventory. The primary basis for accounting for inventories is cost. The cost of an inventory item is the cash price or fair value of other consideration given in exchange for it. Historical cost, replacement cost, and net realizable value are all appropriate methods for measuring inventory.

How well did you know this?
1
Not at all
2
3
4
5
Perfectly
102
Q
Generally, which inventory costing method approximates most closely the current cost for each of the following?
Cost of goods sold	Ending inventory
A	LIFO	FIFO
B	LIFO	LIFO
C	FIFO	FIFO
D	FIFO	LIFO
A

Answer is A

Explanation:
In using LIFO, the cost of the last goods in are used in pricing the cost of goods sold. Therefore, the LIFO method will result in having cost of goods sold most closely approximate current cost. In using FIFO, the cost of the last goods are used in pricing the ending inventory. Thus, the FIFO method will result in having ending inventory most closely approximate current cost.

How well did you know this?
1
Not at all
2
3
4
5
Perfectly
103
Q

A flash flood swept through Hat, Inc.’s warehouse on May 1. After the flood, Hat’s accounting records showed the following:

Inventory, January 1 $ 35,000
Purchases, January 1 through May 1 200,000
Sales, January 1 through May 1 250,000
Inventory not damaged by flood 30,000
Gross profit percentage on sales 40%
What amount of inventory was lost in the flood?

A
$55,000
B
$85,000
C
$120,000
D
$150,000
A

Answer is A

Explanation:
The amount of inventory lost in the flood is calculated by determining the difference between the estimated ending inventory using the gross margin method
and the actual physical inventory not damaged by the flood.

Beginning inventory, January 1 $35,000
Purchases, January 1 through May 1 200,000
Goods available for sale 235,000
Sales, January 1 through May 1 $250,000
Less: Gross margin (40% x $250,000) (100,000)
Less: Estimated CGS (150,000)
Estimated ending inventory 85,000
Less: Physical ending inventory (30,000)
Estimated flood loss $55,000

How well did you know this?
1
Not at all
2
3
4
5
Perfectly
104
Q

The UNO Company was formed on January 2, year 1, to sell a single product. Over a two-year period, UNO’s acquisition costs have increased steadily. Physical quantities held in inventory were equal to three months’ sales at December 31, year 1, and zero at December 31, year 2. Assuming the periodic inventory system, the inventory cost method which reports the highest amount for each of the following is:

Inventory December 31, Year 1	Cost of Sales Year 2
A	LIFO	FIFO
B	LIFO	LIFO
C	FIFO	FIFO
D	FIFO	LIFO
A

Answer is C

Explanation:
Under the last-in, first-out (LIFO) method of inventory valuation, the units remaining in ending inventory are costed at the oldest unit costs available. Under the first-in, first-out (FIFO) method of inventory valuation, the units remaining in ending inventory are costed at the most recent unit costs available. Therefore, because inventory acquisition costs increased steadily during year 1, the FIFO method of inventory valuation would report a higher amount for ending inventory than the LIFO method. The question indicates that there were no goods in inventory at 12/31, year 2. So, cost of goods sold for year 2 is comprised of the cost of inventory purchases made in year 2 and the cost of ending inventory at 12/31, year 1. Because the cost of the ending inventory at 12/31, year 1, is higher under FIFO, cost of goods sold for year 2 would also be higher under FIFO.

Option (A), (B) and (D) are incorrect as per above explanation.

How well did you know this?
1
Not at all
2
3
4
5
Perfectly
105
Q
West Retailers purchased merchandise with a list price of $20,000, subject to trade discounts of 20% and 10%, with no cash discounts allowable. West should record the cost of this merchandise as
A
$14,000
B
$14,400
C
$15,600
D
$20,000
A

Answer is B

Explanation:
The cost of the merchandise is computed by subtracting the amount of the trade discounts from the list price of the goods.

List price	$20,000
Less: trade discount—20%	4,000
Balance	16,000
Less: trade discount—10%	1,600
Cost of the merchandise	$14,400
How well did you know this?
1
Not at all
2
3
4
5
Perfectly
106
Q

A company decided to change its inventory valuation method from FIFO to LIFO in a period of rising prices. What was the result of the change on ending inventory and net income in the year of the change?

Ending inventory	Net income
A	Increase	Increase
B	Increase	Decrease
C	Decrease	Decrease
D	Decrease	Increase
A

Answer is C

Explanation:
The change in inventory valuation from FIFO to LIFO is a change in accounting principle which should be accounted for retrospectively. However, because it is impracticable to determine the retrospective impact of such a change, the change will impact the current period only. As LIFO results in lower valuation of ending inventory in a period of rising prices it will decrease the net income for the period.

Options (A), (B) and (D) are incorrect based on the above explanation.

How well did you know this?
1
Not at all
2
3
4
5
Perfectly
107
Q

Garcel, Inc. held unfinished inventory at a cost of $85,000 with a sales value of $125,000. The inventory will cost $10,500 to complete. The normal profit margin is 30% of sales. The replacement cost of the inventory was $75,000. What amount should Garcel report as inventory as per retail inventory method on the balance sheet?

A
$114,500
B
$85,000
C
$77,000
D
$75,000
A

Answer is C

Explanation:
The correct answer is (C).

Under US GAAP, inventory is valued at the lower of cost or market (LCM) if LIFO or retail inventory method is used. FASB has issued a recent update replacing the lower of cost or market value criteria with lower of cost or net realizable value (LCNRV) for goods determined by other than LIFO or retail inventory methods.

Market = Middle of the following three numbers.

Ceiling = NRV.
Floor = NRV - normal profit margin.
Replacement cost = Cost to purchase or reproduce
Cost is $85,000.

Market price is $77,000.

Ceiling: NRV = ($125,000 - $10,500) = $114,500.
Floor: NRV- Normal profit = ($114,500 - $37,500) = $77,000. [Normal Profit: ($125,000 x 30%) = $37,500]
Replacement Cost = $75,000
So Inventory should be valued at market price of $77,000

(A) is incorrect because it uses net realizable value.

(B) is incorrect because it uses original cost which is higher than market.

(D) is incorrect because it uses replacement cost.

How well did you know this?
1
Not at all
2
3
4
5
Perfectly
108
Q
Union Corp. uses the first-in, first-out retail method of inventory valuation. The following information is available:
Cost	Retail
Beginning Inventory	$12,000	$ 30,000
Purchases	60,000	110,000
Net additional markups		10,000
Net markdowns		20,000
Sales revenue		90,000If the lower of cost or market rule is disregarded, what would be the estimated cost of the ending inventory?
A
$24,000
B
$20,800
C
$20,000
D
$19,200
A

Answer is A

Explanation:
Under the first-in, first-out (FIFO) retail method of inventory valuation, the goods in beginning inventory are charged to cost of goods sold during the period; therefore, the cost/retail ratio is based only on the purchases for the period. If the lower of cost or market rule is disregarded, both net additional markups and net markdowns are included in the purchases cost-to-retail ratio.

Cost Retail
Purchases $60,000 $110,000
Net additional markups – 10,000
Net markdowns – (20,000)
Purchases cost-to-retail ratio amounts 60,000 100,000
Beginning inventory $12,000 $ 30,000
Goods available for sale $72,000 $130,000
Less: sales (90,000)
Estimated ending inventory at retail $ 40,000
Cost-to-retail ratio ($60,000 / $100,000) x 60%
Estimated ending inventory at cost $ 24,000

How well did you know this?
1
Not at all
2
3
4
5
Perfectly
109
Q
On January 1, year 1, Poe Company adopted the dollar-value LIFO inventory method. Poe's entire inventory constitutes a single pool. Inventory data for year 1 and year 2 are as follows:
Date	Inventory at current year cost	Inventory at base year cost	Relevant price index
1/1/, year 1	$150,000	$150,000	1.00
12/31, year 1	220,000	200,000	1.10
12/31, year 2	276,000	230,000	1.20Poe's LIFO inventory value at December 31, year 2 is
A
$230,000
B
$236,000
C
$241,000
D
$246,000
A

Answer is C

Explanation:
Date	Layers at Base Year Cost	Price Index	Ending Inventory at LIFO Cost
01/01, year1	$150,000	1.00	$150,000
12/31, year1	50,000 [1]	1.10	55,000
12/31, year2	30,000 [2]	1.20	36,000
$230,000		$241,000
[1] $200,000 - $150,000
[2] $230,000 - $200,000
How well did you know this?
1
Not at all
2
3
4
5
Perfectly
110
Q

The following costs pertain to Den Co.’s purchase of inventory:

700 units of product A $3,750
Freight-in 175
Cost of materials and labor incurred to bring product A to saleable condition 900
Insurance cost during transit of purchased goods 100
Total $4,925
What amount should Den record as the cost of inventory as a result of this purchase?

A
$3,925
B
$4,650
C
$4,825
D
$4,925
A

Answer is D

Explanation:
The cost of merchandise inventory is net of any discounts but includes freight-in, taxes, insurance while in transit, warehousing costs, and similar charges paid to bring the article to its existing condition and loca¬tion. The cost of the inventory would be the $3,750 purchase price, plus the $175 freight-in charge, plus the $900 costs of materials and labor to bring it to saleable condition, plus the $100 insurance cost during transit for a total $4,925.All costs necessary to bring an item to its salable condition are capitalized to the inventory account. All four listed costs meet this requirement. Until the goods are ready for sale, the cost of all efforts to achieve that goal are treated as product costs rather than period costs

Option (A) is incorrect because it excludes costs incurred to sell the inventory and insurance charges.

Option (B) is incorrect because it excludes the insurance cost incurred and freight in charges.

Option (C) is incorrect because it excludes the insurance cost incurred.

How well did you know this?
1
Not at all
2
3
4
5
Perfectly
111
Q

In a period of rising general price levels, Pollard Corp. discloses income on a current cost basis in accordance with standards on financial reporting and changing prices. Compared to historical cost income from continuing operations, which of the following conditions increases Pollard’s current cost income from continuing operations?
A
Current cost of equipment is greater than historical cost.
B
Current cost of land is greater than historical cost.
C
Current cost of cost of goods sold is less than historical cost.
D
Ending net monetary assets are less than beginning net monetary assets.

A

Answer is C

Explanation:
When current cost of goods sold is less than historical cost, current cost income from continuing operations will be greater than historical cost income from continuing operations.

How well did you know this?
1
Not at all
2
3
4
5
Perfectly
112
Q
During periods of inflation, a perpetual inventory system would result in the same dollar amount of ending inventory as a periodic inventory system under which of the following inventory valuation methods?
FIFO	LIFO
A	Yes	No
B	Yes	Yes
C	No	Yes
D	No	No
A

Answer is A

Explanation:
Under the FIFO cost-flow method, a perpetual system would result in the same dollar amount of ending inventory as a periodic inventory system. Under the LIFO cost-flow method, however, a perpetual system would generally not result in the same dollar amount of ending inventory as a periodic inventory system.

How well did you know this?
1
Not at all
2
3
4
5
Perfectly
113
Q
Southgate Co. paid the in-transit insurance premium for consignment goods shipped to Hendon Co., the consignee. In addition, Southgate advanced part of the commissions that will be due when Hendon sells the goods. Should Southgate include the in-transit insurance premium and the advanced commissions in inventory costs?
Insurance premium	Advanced commissions
A	Yes	Yes
B	No	No
C	Yes	No
D	No	Yes
A

Answer is C

Explanation:
The in-transit insurance premium should be included in inventory costs because it is a cost necessary to get the goods in the place and condition for their intended sale. The advanced commissions represent a prepaid selling expense and thus should not be included in inventory cost.

How well did you know this?
1
Not at all
2
3
4
5
Perfectly
114
Q

An entity that uses the gross margin method of estimating inventory should do which of the following?
A
Assume that its gross margin percentage is relatively stable
B
Determine cost of goods sold by applying the gross margin ratio to sales and subtracting this amount from the sales figure
C
Must understand that the gross margin method is not considered as GAAP for annual financial reporting purposes
D
All of the above

A

Answer is D

Explanation:
The gross margin method rests on the assumption that the gross margin percentage is relatively stable, computes cost of goods sold by applying the gross margin ratio to sales and subtracting this amount from the sales figure, and is not generally accepted for annual financial reporting purposes. As the answers for A., B., and C. are all correct, the best answer for this question is D., all of the above.

How well did you know this?
1
Not at all
2
3
4
5
Perfectly
115
Q

Ashe Co. recorded the following data pertaining to raw material X during January:

Units


Date Received Cost Issued On Hand
1/01 Inventory $8.00 3,200
1/11 Issue 1,600 1,600
1/22 Purchase 4,800 9.60 6,400
The moving-average unit cost of X inventory at January 31 is

A
$8.80
B
$8.96
C
$9.20
D
$9.60
A

Answer is C

Explanation:
$9.2. Units beginning inventory remaining at year-end (3,200 - 1,600)$8 = $12,800 plus 1/22 purchase: 4,800($9.60) = 46,080. Ending inventory $58,880. Ending unit cost: $58,880/6,400 = $9.20
The moving average method costs issues at the unit cost of goods on hand at that point. Thus, the issue was costed at $8.00 per unit. The cost per unit changes with each purchase.

Option (A), (B) and (D) are incorrect as per above explanation.

How well did you know this?
1
Not at all
2
3
4
5
Perfectly
116
Q
The following information pertains to an inventory item:
Cost	$12.00
Estimated selling price	13.60
Estimated disposal cost	0.20
Normal gross margin	2.20
Replacement	10.90Under the lower-of-cost-or-market rule, this inventory item should be valued at
A
$10.70
B
$10.90
C
$11.20
D
$12.00
A

Answer is C

Explanation:
Under the lower of cost or market rule, replacement cost cannot exceed a ‘ceiling’ of net realizable value (estimated selling price less estimated cost of disposal) and cannot be below a ‘floor’ of net realizable value reduced by a normal profit margin.

Replacement Cost Parameters
Cost	Replacement Cost	Ceiling (NRV)	Floor (NRV - NP)
$12.00	$10.90	$13.40 [1]	$11.20 [2]
[1] $13.60 - $0.20.
[2] $13.40 - $2.20.

The replacement cost is below the prescribed range; therefore, the ‘floor’ of $11.20 is the assigned market value. Because the assigned market value of $11.20 is below the historical cost of $12.00, the inventory item should be valued at $11.20.

How well did you know this?
1
Not at all
2
3
4
5
Perfectly
117
Q

Nomar Co. shipped inventory on consignment to Seabright Co. that cost $20,000. Seabright paid $500 for advertising that was reimbursable from Nomar. At the end of the year, 70% of the inventory was sold for $30,000. The agreement states that a commission of 20% will be provided to Seabright for all sales.
What amount of net inventory on consignment remains on the balance sheet for the first year for Nomar?

A
$0
B
$ 6,000
C
$ 6,500
D
$20,000
A

Answer is B

Explanation:
Nomar Co., is the consignor and the inventory is never owned by consignee. Title remains with consignor until goods are sold by the consignee. Thus, title passes directly to the third-party buyer at point of sale. Consignee acts as an agent with a commission on sales and reimbursable expenses. At the year end, Nomar co., would report the unsold inventory of $6,000 (i.e. 30% x $20,000).

Option (A) is incorrect because the ownership of the goods is with Nomar Co., and unsold inventory at year end should be reported.

Option (C) is incorrect because advertising expenses of $500 are included in the inventory cost [i.e. $6,500 = (30% x $20,000) + $500].

Option (D) is incorrect because total cost of inventory is reported at the year end which includes the inventory that is sold to third party.

How well did you know this?
1
Not at all
2
3
4
5
Perfectly
118
Q

Medus Industries is a manufacturer of pens. Many shops acquire these pens from Medus for their customers. Medus ships 100 pens to a shop on 1/1/Y1 with full payment due immediately. Legal title however, does not transfer and Medus retains the right to require shipment of any unsold pens from this shop to other shops that require these pens. If this right is exercised a full refund would be provided. Similarly, this shop may also return any unsold pens to Medus for a full refund. How should the transaction as of 1/1/Y1 be recognized?

A
Sale.
B
Consignment arrangement.
C
Repurchase agreement put option.
D
Bill and hold arrangement.
A

Answer is B

Explanation:
Medus does not transfer legal title to the shop, continues to hold control as it can demand transfer of the pens to another shop, till the pens are sold to customers by the shop. Also the shop, even though owes full payment for the pens immediately upon transfer from Medus, it is considered more of a deposit, as the Medus would refund the full amount upon return of the pens back to Medus. All the above indicators are satisfied. This is a consignment arrangement.

How well did you know this?
1
Not at all
2
3
4
5
Perfectly
119
Q

Nest Co. recorded the following inventory information during the month of January:

Units Unit cost Total cost Units on hand

Balance on 1/1 2,000 $1 $2,000 2,000
Purchased on 1/8 1,200 3 3,600 3,200
Sold on 1/23 1,800 1,400
Purchased on 1/28 800 5 4,000 2,200
Nest uses the LIFO method to cost inventory. What amount should Nest report as inventory on January 31 under each of the following methods of recording inventory?

Perpetual	Periodic
A	$2,600	$5,400
B	$5,400	$2,600
C	$2,600	$2,600
D	$5,400	$5,400
A

Answer is B

Explanation:
Under perpetual method, inventory is quantified perpetually on an ongoing basis. Unders perpetual system, each time a sale is made, the LIFO method is used to calculate cost of goods sold. When the sale is made on 1/23, the LIFO is applied to associate COGS to 1,200 units at $3 and the remaining 600 units @ $1. The remaining would be 1,400 units at $1. To this add the purchase of 800 units @$5 will be added to calculate the ending inventory. Ending inventory under LIFO perpetual would be:

1,400 units at $1.00 $1,400
800 units at $5.00 $4,000
Total $5,400
Under the periodic inventory system, inventory quantity determined periodically by physical count usually at year / month end. Under the periodic system of LIFO, the COGS of the sale of 1,800 units is considered only at month end when LIFO is applied first to the last purchase made on 1/28. Of the 1,800 units sold, 800 would be associated to $5 per unit and the remaining 1,000 units would be associated to $3 per unit. Ending inventory would consist of the remaining 200 units @ $3 per unit and 2000 units @ $1 per unit. Ending inventory under LIFO periodic would be:

2,000 units at $1.00 $2,000
200 units at $3.00 $600
Total $2,600
Options (A), (C) and (D) are incorrect based on the above explanation.

How well did you know this?
1
Not at all
2
3
4
5
Perfectly
120
Q

A manufacturer whose finished goods inventories are items that are routinely manufactured or otherwise produced in large quantities, on a repetitive basis, should include ______________________ in its cost of finished goods inventory.

A
Manufacturing overhead
B
Interest cost
C
Freight-out expense
D
Trade discounts
A

Answer is A

Explanation:
The finished goods inventory of a manufacturer must include the cost of both variable and fixed manufacturing overhead. Interest cost should not be capitalized for inventories that are routinely manufactured or otherwise produced in large quantities on a repetitive basis. Freight-out is a selling expenses and should not be included in the cost of inventory. A trade discount is a deduction from the list or catalog price of merchandise to arrive at the gross selling price. Trade discounts are not recorded in the seller or purchaser’s accounting records.

Options (B), (C) and (D) are incorrect as per above explanation.

How well did you know this?
1
Not at all
2
3
4
5
Perfectly
121
Q

A corporation entered into a purchase commitment to buy inventory. At the end of the accounting period, the current market value of the inventory was less than the fixed purchase price, by a material amount. Which of the following accounting treatments is most appropriate?

A
Describe the nature of the contract in a note to the financial statements, recognize a loss in the income statement, and recognize a liability for the accrued loss
B
Describe the nature of the contract and the estimated amount of the loss in a note to the financial statements, but do not recognize a loss in the income statement
C
Describe the nature of the contract in a note to the financial statements, recognize a loss in the income statement, and recognize a reduction in inventory equal to the amount of the loss by use of a valuation account
D
Neither describe the purchase obligation, nor recognize a loss on the income statement or balance sheet

A

Answer is A

Explanation:
The correct answer is (A). In a legal non-cancelable agreement for future purchase of Inventory, if Contract price > Market price and it is expected that loss will occur on purchase, recognize loss (Market Price - Contracted Price) at the time of decline in prices.

The journal entry would be:
Dr: Est. loss on purchase commitment.
Cr: Accrued loss on purchase commitment.
Details of the losses to be included in the footnote.

Option (B) is incorrect, because apart from describing the nature of the loss in footnotes, loss needs to be accrued and recognize loss in I/S.

Option (C) is incorrect because loss is accrued as a liability and not as a valuation account to reduce inventory.

Option (D) is incorrect because footnote disclosure, loss recognized in I/S and a liability for accrued loss.

How well did you know this?
1
Not at all
2
3
4
5
Perfectly
122
Q

In its financial statements, Hila Co. discloses supplemental information on the effects of changing prices in accordance with FASB Standards. Hila computed the increase in current cost of inventory as follows:
Increase in current cost (nominal dollars) $15,000
Increase in current cost (constant dollars) 12,000What amount should Hila disclose as the inflation component of the increase in current cost of inventories?
A
$ 3,000
B
$12,000
C
$15,000
D
$27,000

A

Answer is A

Explanation:
The “inflation component” of the increase in the current cost amount is defined as the difference between the nominal dollars and constant dollars measures. $15,000 - $12,000 = $3,000.

How well did you know this?
1
Not at all
2
3
4
5
Perfectly
123
Q

In January, Stitch, Inc. adopted the dollar-value LIFO method of inventory valuation. At adoption, inventory was valued at $50,000. During the year, inventory increased $30,000 using base-year prices, and prices increased 10%. The designated market value of Stitch’s inventory exceeded its cost at year end. What amount of inventory should Stitch report in its year-end balance sheet?

A
$80,000
B
$83,000
C
$85,000
D
$88,000
A

Answer is B

Explanation:
Inventory is reported at the lower of cost or market. As indicated in the problem, market exceeded cost, so inventory should be stated at cost, using dollar value LIFO. The inventory layer added in the current year is computed in terms of base year cost. It then must be converted to current year cost because the layer was added during the current year. The index (1.1) is computed by dividing the ending inventory at current year cost ($88,000) by the ending inventory at base year cost ($80,000). The cost of ending inventory ($83,000) is the $50,000 cost of the beginning amount and the converted Year 1 layer. ($30.000 × 1.1 = $33,000).

Option (A) is incorrect because the calculation does not adjust for inflation 10% on current year inventory.

Option (C) is incorrect because it adjusts the price index @ 10% on prior period inventory of $50,000.

Option (D) is incorrect because it adjusts the price index of 10% on prior period and current year’s inventory

How well did you know this?
1
Not at all
2
3
4
5
Perfectly
124
Q

Jones Wholesalers stocks a changing variety of products. Which inventory costing method will be most likely to give Jones the lowest ending inventory when its product lines are subject to specific price increases?

A
Specific identification.
B
Weighted average.
C
Dollar-value LIFO.
D
FIFO periodic.
A

Answer is C

Explanation:
The dollar value LIFO will give Jones the lowest ending inventory when its product lines are subject to specific price increases. This is because LIFO methods operate under the assumption that the last item of inventory purchased is the first one to be sold. Thus, in the case of rising prices, the cost of the most recently acquired inventory will always be higher than the cost of earlier purchases, so the ending inventory balance will be valued at earlier costs which are lower, while the most recent costs appear in the cost of goods sold. As far as specific identification, weighted average or FIFO periodic methods, none of these would give a higher inventory value than dollar value LIFO method.

Options (A), (B) and (D) are incorrect based on the above explanation.

How well did you know this?
1
Not at all
2
3
4
5
Perfectly
125
Q

Delar Co. completed its year-end physical count of inventory. The inventory was valued at first-in, first-out (FIFO) costs and totaled $500,000.Delar subsequently noted the following two items:

1,000 units of inventory with a FIFO cost of $10 each were shipped and billed to a customer F.O.B.destination. These items were included in the physical count.
6,000 units at a FIFO cost of $5 each were held on consignment for one of its suppliers but were excluded from the physical count.
What amount should Delar report as inventory at year end?

A
$530,000
B
$520,000
C
$500,000
D
$490,000
A

Explanation:
The correct answer is (C).

Delar should report $500,000 as inventory at year-end.

FOB destination means title and risk of loss pass to the buyer when the seller makes a proper tender of delivery of the goods at the destination. This was shipped via FOB destination and should be included in inventory because the inventory had not been delivered to the customer by year-end.

Consigned goods are not sold but rather transferred to an agent for possible sale. Consigned goods are included in the inventory of the consignor (owner). Thus, the inventory that is held on consignment for one of the suppliers should be excluded from the inventory physical count as Delar is a consignee. Therefore, Delar should report $500,000 of inventory at year-end.

How well did you know this?
1
Not at all
2
3
4
5
Perfectly
126
Q
A company determined the following values for its inventory as of the end of its fiscal year:
Historical cost	$100,000
Current replacement cost	70,000
Net realizable value	90,000
Net realizable value less a normal profit margin	85,000
Fair value	95,000Under IFRS, what amount should the company report as inventory on its balance sheet?
A
$70,000
B
$85,000
C
$90,000
D
$95,000
A

Answer is C

Explanation:
Under IFRS, inventory is carried at the lower of cost or net realizable value (best estimate of the net amounts inventories are expected to realize). This amount may or may not equal fair value. The net realizable value of $90,000 is lower than the historical cost of $100,000.

How well did you know this?
1
Not at all
2
3
4
5
Perfectly
127
Q
The following information pertains to each unit of merchandise purchased for resale by Vend Co.:
March 1	December 31
Purchase price	$ 8	
Selling price	12	$ 15
Price level index	110	121
Replacement cost		10Under current cost accounting, what is the amount of Vend's holding gain on each unit of this merchandise?
A
$0
B
$0.80
C
$1.20
D
$2.00
A

ANswer is D

Explanation:
Under current cost accounting, the amount of holding gain on a unit of inventory is the increase in current cost from holding the inventory from period to period. The inventory in question was purchased at $8 per unit on March 1 and has a replacement cost of $10 on December 31. Thus, under current cost accounting, Vend has a holding gain of $2 (i.e., $10 - $8) on each unit of the inventory.

How well did you know this?
1
Not at all
2
3
4
5
Perfectly
128
Q

Manhof Co. prepares supplementary reports on income from continuing operations on a current cost basis in accordance with FASB Standards. How should Manhof compute cost of goods sold on a current cost basis?
A
Number of units sold times average current cost of units during the year.
B
Number of units sold times current cost of units at year end.
C
Number of units sold times current cost of units at the beginning of the year.
D
Beginning inventory at current cost plus cost of goods purchased less ending inventory at current cost.

A

Answer is A

Explanation:
To compute cost of goods sold on a current cost basis, multiply the number of units sold by the average current cost of the units during the year. (Average current cost of the units during the year is the sum of the current cost of the units at the beginning and the end of the year, divided by two.).

How well did you know this?
1
Not at all
2
3
4
5
Perfectly
129
Q

A manufacturer has the following per-unit costs and values for its sole product:

Cost $10.00
Current replacement cost 5.50
Net realizable value 6.00
Net realizable value less normal profit margin 5.20
In accordance with IFRS, what is the per-unit carrying value of inventory in the manufacturer’s statement of financial position?

A
$ 5.20
B
$ 5.50
C
$ 6.00
D
$10.00
A

Answer is C

Explanation:
Under IFRS, inventory is carried at the lower of cost or net realizable value (best estimate of the net amounts inventories are expected to realize). This amount may or may not equal fair value. The net realizable value of $6.00 is lower than the historical cost of $10.00.

How well did you know this?
1
Not at all
2
3
4
5
Perfectly
130
Q

Bren Co.’s beginning inventory at January 1 was understated by $26,000, and its ending inventory was overstated by $52,000. As a result, Bren’s cost of goods sold for the year was

A
Understated by $26,000.
B
Overstated by $26,000.
C
Understated by $78,000.
D
Overstated by $78,000.
A

Answer is C

Explanation:
Cost of goods sold = Beginning inventory + Purchases - Ending inventory
Beginning inventory was understated by $26,000, the beginning inventory would have been beginning inventory - $26,000.
Ending inventory was overstated by $52,000, the ending inventory would have been ending inventory + $52,000.
The incorrect cost of goods sold would have been Beginning inventory - 26,000 + Purchases - (Ending inventory + 52,000).
Thus cost of goods sold would have been Beginning inventory + Purchases - Ending inventory - $78,000 (i.e. - $26,000 - $52,000).
Cost of goods sold (COGS) would have been understated by $78,000. An alternative explanation: understatement of beginning inventory would lead to an understatement of COGS. Overstatement of ending inventory would lead to an understatement of COGS, leading to a net understatement of $78,000.

Options (A), (B) and (D) are incorrect based on the above explanation.

How well did you know this?
1
Not at all
2
3
4
5
Perfectly
131
Q

Based on a physical inventory taken on December 31, Chewy Co. determined its chocolate inventory on a FIFO basis at $26,000 with a replacement cost of $20,000. Chewy estimated that, after further process­ing costs of $12,000, the chocolate could be sold as finished candy bars for $40,000. Chewy’s normal profit margin is 10% of sales. Under the lower of cost or market rule, what amount should Chewy report as chocolate inventory in its December 31 balance sheet?

A
$28,000
B
$26,000
C
$24,000
D
$20,000
A

Explanation:
The correct answer is (B).

Inventory is valued at the Lower of Cost or Market (LCM) if LIFO or retail inventory is used. Exception: FASB has issued an update replacing the LCM valuation with lower of cost or net realizable value (LCNRV) valuation for other than LIFO or retail inventory methods.

Chewy Co. determined its chocolate inventory on a FIFO basis. Inventory should be valued at LCNRV.

Cost = $26,000.
NRV = Net selling price - Costs to complete and dispose = $40,000 - $12,000 = $28,000.
Lower of the two: inventory is $26,000.

How well did you know this?
1
Not at all
2
3
4
5
Perfectly
132
Q

Which of the following statements regarding inventory accounting systems is true?

A
A disadvantage of the perpetual inventory system is that the inventory dollar amounts used for interim reporting purposes are estimated amounts.
B
A disadvantage of the periodic inventory system is that the cost of goods sold amount used for financial reporting purposes includes both the cost of inventory sold and inventory shortages.
C
An advantage of the perpetual inventory system is that the record keeping required to maintain the system is relatively simple.
D
An advantage of the periodic inventory system is that it provides a continuous record of the inventory balance.

A

Explanation:
The correct answer is (B).

A disadvantage of the periodic inventory system is that the cost of goods sold amount used for financial reporting purposes includes both the cost of inventory sold and inventory shortages.

The perpetual inventory system is that the inventory dollar amounts used for interim reporting purposes are actual amounts.

The disadvantage of the perpetual inventory system is that the record keeping required to maintain the system is relatively complex and time-consuming.

The disadvantage of the periodic inventory system is that it provides only a period-end record of the inventory balance

How well did you know this?
1
Not at all
2
3
4
5
Perfectly
133
Q
The replacement cost of an inventory item is below the net realizable value and above the net realizable value less the normal profit margin. The original cost of the inventory item is above the replacement cost and below the net realizable value. As a result, under the lower-of-cost-or-market method, the inventory item should be valued at the
A
Original cost
B
Replacement cost
C
Net realizable value
D
Net realizable value less the normal profit margin
A

Answer is B

Explanation:
Valuation of inventory items is required at the lower of cost or replacement cost (commonly referred to as market). Market cannot exceed the net realizable value (ceiling) of the good (i.e., selling price less expected costs to sell), and market should not be less than this net realizable value reduced by an allowance for a normal profit margin (floor). In this problem, the replacement cost is between the ceiling and floor amounts, so it is used as the market value. Since the original cost is greater than replacement (i.e., market) cost, the item will be carried at the lower market/replacement cost.

How well did you know this?
1
Not at all
2
3
4
5
Perfectly
134
Q

At December 31 of the current year, the following information was available from Huff Co.’s accounting records:

Cost Retail

Inventory, 1/1 $147,000 $ 203,000
Purchases 833,000 1,155,000
Additional markups – 42,000
Available for sale $980,000 $1,400,000
Sales for the year totaled $1,106,000. Markdowns amounted to $14,000. Under the approximate lower of average cost or market retail method, Huff’s inventory at December 31 was

A
$308,000
B
$280,000
C
$215,600
D
$196,000
A

Answer is D

Explanation:
Under the approximate lower of average cost or market retail method, the estimated inventory is computed as follows:

Cost Retail

Inventory, 1/1 $147,000 $203,000
Purchases 833,000 1,155,000
Additional markups – 42,000
Cost-to-retail ratio amounts $980,000 $1,400,000
Less: Sales – (1,106,000)
Markdowns – (14,000)
Estimated ending inventory at retail – $280,000
Times cost-to-retail ratio ($980,000 / $1,400,000)= 0.7 i.e 70% – x 70%
Estimated ending inventory at cost – $196,000

How well did you know this?
1
Not at all
2
3
4
5
Perfectly
135
Q

An entity purchased new machinery from a supplier before the entity’s year end. The entity paid freight charges for the purchased machinery. The entity took out a loan from a bank to finance the purchase. Under IFRS, what is the proper accounting treatment for the freight and interest costs related to the machinery purchase?

A
The freight and interest costs should be immediately expensed.
B
The freight and interest costs should be capitalized as part of property, plant and equipment.
C
The interest cost should be capitalized as part of property, plant and equipment, and the freight cost should be immediately expensed.
D
The freight cost should be capitalized as part of property, plant and equipment, and the interest cost should be immediately expensed.

A

Answer is D

Explanation:
Under both IFRS and US GAAP, freight cost is capitalized as part of property, plant and equipment, and interest is only capitalized on qualifying assets. Assets qualifying for interest capitalization include assets constructed or produced for self-use on a repetitive basis, assets acquired for self-use through arrangements requiring down payments or progress payments, and assets constructed or produced as discrete projects for sale or lease (e.g., ships or real estate developments). Assets not qualifying for interest capitalization include inventories that are routinely manufactured on a repetitive basis, assets in use or ready for use, and assets not in use and not being prepared for use. Without any indication otherwise, it would be determined that this machinery is ready for use and as such the interest costs would not be capitalized but instead would be immediately expensed.

How well did you know this?
1
Not at all
2
3
4
5
Perfectly
136
Q
Walt Co. adopted the dollar-value LIFO inventory method as of January 1 of the current year, when its inventory was valued at $500,000. Walt's entire inventory constitutes a single pool. Using a relevant price index of 1.10, Walt determined that its December 31 inventory was $577,500 at current year cost, and $525,000 at base year cost. What was Walt's dollar-value LIFO inventory at December 31 of the current year?
A
$525,000
B
$527,500
C
$552,500
D
$577,500
A

Answer is B

Explanation:
The current year layer is determined by the difference of the ending inventory at base year cost and the beginning inventory (which is the base year in this case) at base year cost and applying the relevant price index to the difference.

Ending inventory at base year cost $ 525,000
Less: Beginning inventory at base year cost (500,000)
Current year layer at base year cost $ 25,000
Relevant price index 1.10
Current year layer at current year prices $ 27,500
Beginning inventory (1/1) at DV LIFO 500,000
Dollar-value LIFO inventory, 12/31 $ 527,500

How well did you know this?
1
Not at all
2
3
4
5
Perfectly
137
Q

Garson Co. recorded goods in transit purchased F.O.B. shipping point at year end as purchases. The goods were excluded from ending inventory. What effect does the omission have on Garson’s assets and retained earnings at year end?

Assets	Retained earnings
A	No effect	Overstated
B	No effect	Understated
C	Understated	No effect
D	Understated	Understated
A

Answer is D

Explanation:
Goods purchased f.o.b. shipping point should be included in inventory when the goods are shipped. It states the goods are in transit, so they have been shipped. The goods were erroneously excluded from ending inventory, when they should have been included, meaning that inventory is understated. Inventory is an asset, so assets are understated. An understated inventory also would lead to an overstated cost of goods sold (goods available for sale – ending inventory = cost of goods sold). An overstated cost of goods sold would lead to an understated gross profit (revenue – cost of goods sold = gross profit) which in turn results in an understated retained earnings.

Options (A), (B) and (C) are incorrect as per above explanation.

How well did you know this?
1
Not at all
2
3
4
5
Perfectly
138
Q

The following information relates to a company’s year-end inventory:

Inventory cost $910
Selling price of inventory $1,000
Normal profit margin 10% of selling price
Current replacement cost $740
Cost of completion and disposal $100
Under IFRS, what is the company’s year-end inventory balance?

A
$740
B
$800
C
$900
D
$910
A

Answer is C

Explanation:
The correct answer is (C).

Under IFRS, inventories are valued at lower of cost or net realizable value.

Net realizable value = Fair value - Estimated sales cost = $1,000 - $100 = $900.

As the NRV is lower at $900 when compared to the cost of $910, inventory will be valued at $900.

How well did you know this?
1
Not at all
2
3
4
5
Perfectly
139
Q

A firm’s ending inventory balance was overstated by $1,000. Which of the following statements is correct according to a periodic inventory system?

A
The retained earnings were overstated by $1,000.
B
The cost of goods sold was overstated by $1,000.
C
The cost of goods available for sale was overstated by $1,000.
D
The gross margin was understated by $1,000.

A

Explanation:
The correct answer is (A).

Cost of goods sold (COGS) is calculated as beginning inventory plus purchases during the period minus ending inventory.

Therefore, a $1,000 overstatement of the ending inventory balance will result in a $1,000 understatement of cost of goods sold.

The $1,000 understatement of COGS results in a $1,000 overstatement of gross profit as well, which carries over to a $1,000 overstatement of retained earnings.

How well did you know this?
1
Not at all
2
3
4
5
Perfectly
140
Q

The dollar-value LIFO inventory cost flow method involves computations based on
Inventory pools of similar items A specific price index for each year
A No Yes
B No No
C Yes No
D Yes Yes

A

Answer is D

Explanation:
The dollar-value LIFO method is based on the aggregation of similar inventory items into pools. Acquisitions and issuances of similar materials are recorded in the same pool, even if the substitute items are not exactly the same as the replaced items. As a general rule, dollar-value LIFO uses a “double-extension method” to determine: (1) the value of the ending inventory in terms of base year prices, and (2) the value of the ending inventory at current prices. The ratio of (2) over (1), above, provides the specific price index for valuing any layers of inventory added in the period.

How well did you know this?
1
Not at all
2
3
4
5
Perfectly
141
Q

When the double extension approach to the dollar value LIFO inventory method is used, the inventory layer added in the current year is multiplied by an index number. Which of the following correctly states how components are used in the calculation of this index number?
A
In the numerator, the average of the ending inventory at base year cost and at current year cost.
B
In the numerator, the ending inventory at current year cost, and, in the denominator, the ending inventory at base year cost.
C
In the numerator, the ending inventory at base year cost, and, in the denominator, the ending inventory at current year cost.
D
In the denominator, the average of the ending inventory at base year cost and at current year cost.

A

Answer is B

Explanation:
The inventory layer added in the current year is computed in terms of base year cost. It then must be converted to current year cost because the layer was added during the current year. Because we are converting to current year cost from base year cost, the index is computed by dividing the ending inventory at current year cost by the ending inventory at base year cost

How well did you know this?
1
Not at all
2
3
4
5
Perfectly
142
Q

The replacement cost of an inventory item is below the net realizable value and above the net real­izable value less a normal profit margin. The inventory item’s original cost is above the net realizable value. Under the lower of cost or market method, the inventory item should be valued at

A
Original cost
B
Replacement cost
C
Net realizable value
D
Net realizable value less normal profit margin
A

Answer is B

Explanation:
Under U.S. GAAP, inventory is valued at the Lower of Cost or Market (LCM) if LIFO or retail inventory method is used, where Cost = Original cost of inventory and Market = middle for the following three numbers:- Net realizable Value (NRV). NRV - normal profit margin. Replacement cost. Exception: FASB has issued an update replacing the LCM valuation with lower of cost of net realizable value (LCNRV) valuation for other than LIFO or retail inventory methods. As per LCM, in the given case replacement cost is below the net realizable value and above the net realizable value less a normal profit margin, replacement cost is in middle and is the market. Cost is given to be higher than NRV, and would be higher than replacement cost or the market. Thus, the inventory will be valued at lower of cost or market, in this case, market or replacement cost.

Options (A), (C) and (D) are incorrect based on the above explanation.

How well did you know this?
1
Not at all
2
3
4
5
Perfectly
143
Q

Carver Co., a retailer, uses the perpetual inventory method. Carver uses the moving average method to determine the value of its inventory. The following information relates to inventory transactions that took place during the month of March:

3/1 Beginning inventory 30,000 units at $10
3/5 Purchase 10,000 units at $12
3/10 Sales at $20 per unit 20,000 units
3/20 Purchase 20,000 units at $13
What amount should Carver report as cost of goods sold on its income statement at the end of March?

A
$200,000
B
$210,000
C
$240,000
D
$260,000
A

Answer is B

Explanation:
The correct answer is (B)

Under the perpetual inventory method, COGS is updated for every inventory sale according to the valuation method followed by an entity. In the given case, Carver Co. follows a perpetual inventory method and uses the moving average method to determine the inventory’s value. The cost of goods sold on the 10th of March can be calculated as below:

Date Particulars Units and unit cost Inventory (A) Inventory Units (B) Moving Avg. Cost (A/B)
3/1 Beginning 30000@ $10 = $300,000 300,000 30,000 10
3/5 Purchase 10,000@ $12 = $120,000 420,000 40,000 10.5
3/10 Sales 20,000@ $10.5 = $210,000 210,000 20,000 10.5
The 20,000 units sold on 3/10 cost $10.5 per unit and therefore, the cost of goods sold is $210,000. Note: Purchases made on 3/20 is irrelevant to compute the COGS under the perpetual inventory method since it is after the sales date. However, if the entity was using the periodic inventory method, the purchase would have been taken into account.

How well did you know this?
1
Not at all
2
3
4
5
Perfectly
144
Q
In a \_\_\_\_\_\_\_\_\_\_\_\_\_\_\_\_, a customer agrees to purchase the goods, but the seller retains physical possession until the customer requests shipment to designated locations.
A
Side agreement
B
Bill and hold transaction
C
Related-party transaction
D
Trade loading transaction
A

Answer is B

Explanation:
In a “bill and hold” transaction, a customer agrees to purchase the goods, but the seller retains physical possession until the customer requests shipment to designated locations.

How well did you know this?
1
Not at all
2
3
4
5
Perfectly
145
Q
The original cost of an inventory item is below the net realizable value and above the net realizable value less a normal profit margin. The inventory item's replacement cost is below the net realizable value less a normal profit margin. Under the lower of cost or market method, the inventory item should be valued at
A
Original cost.
B
Replacement cost.
C
Net realizable value.
D
Net realizable value less normal profit margin.
A

Answer is D

Explanation:
Under the lower-of-cost-or-market rules, market means current replacement cost with the exception that market value should not exceed the net realizable value (ceiling) and should not be less than the net realizable value minus normal profit (floor). Because in this question replacement cost is below the floor, this would be the minimum amount at which market could be valued. This market value is then compared to the original cost, and the lower amount is used for the value of the inventory item. The question states that the original cost is above the net realizable value less a normal profit margin, thus, the market value is the lower amount.

How well did you know this?
1
Not at all
2
3
4
5
Perfectly
146
Q
Dart Company's accounting records indicated the following information:
Inventory, 1/1	$ 500,000
Purchases during the year	2,500,000
Sales during the year	3,200,000A physical inventory taken on December 31 resulted in an ending inventory of $575,000. Dart's gross profit on sales has remained constant at 25% in recent years. Dart suspects some inventory may have been taken by a new employee. At December 31, what is the estimated cost of missing inventory?
A
$ 25,000
B
$100,000
C
$175,000
D
$225,000
A

Answer is A

Explanation:
The missing inventory is estimated by determining the difference between the estimated ending inventory using the gross margin method and the actual physical inventory on hand at year-end.

Beginning Inventory		$ 500,000
Purchases		2,500,000
Goods available for sale		3,000,000
Less: estimated COGS		
Sales	$3,200,000	
Less: gross margin		
(25% x $3,200,000)	800,000	(2,400,000)
Estimated ending inventory		600,000
Less: actual physical inventory		(575,000)
Estimated cost of missing inventory		$ 25,000
How well did you know this?
1
Not at all
2
3
4
5
Perfectly
147
Q

Drew Co. uses the average cost inventory method for internal reporting purposes and LIFO for financial statement and income tax reporting. At December 31, the inventory was $375,000 using average cost and $320,000 using LIFO. The unadjusted credit balance in the LIFO Reserve account on December 31 was $35,000. What adjusting entry should Drew record to adjust from average cost to LIFO at December 31?

Debit Credit
A Cost of Goods Sold 55,000 Inventory 55,000
B Cost of Goods Sold 55,000 LIFO Reserve 55,000
C Cost of Goods Sold 20,000 Inventory 20,000
D Cost of Goods Sold 20,000 LIFO Reserve 20,000

A

Answer is D

Explanation:
Some companies use LIFO for tax and external reporting purposes, but they maintain a FIFO, average cost, or standard cost system for internal reporting purposes. The difference between the inventory method used for internal reporting purposes and LIFO is often referred to as the LIFO reserve.The LIFO reserve is a contra-inventory account that must be adjusted to its required balance at the financial statement date. At 12/31 Drew’s inventory was $375,000 using average cost and $320,000 using LIFO. The required balance in the LIFO reserve at 12/31 is $55,000 (i.e., $375,000 - $320,000). Since the unadjusted LIFO reserve balance was $35,000, the reserve must be increased by $20,000 (i.e., $55,000 - $35,000).

How well did you know this?
1
Not at all
2
3
4
5
Perfectly
148
Q

The original cost of an inventory item is above the replacement cost. The inventory item’s replacement cost is above the net realizable value. Under the lower of cost or market method, the inventory item should be valued at

A
Original cost
B
Replacement cost
C
Net realizable value
D
Net realizable value less normal profit margin
A

Answer is C

Explanation:
Valuation of inventory items is required at the lower of cost or replacement cost (commonly referred to as market). Market cannot exceed the net realizable value (ceiling) of the good (i.e., selling price less expected costs to sell), and market should not be less than this net realizable value reduced by an allowance for a normal profit margin (floor). In this problem, the replacement cost exceeds net realizable value, so market is defined as NRV. Since the original cost is greater than defined market, the item will be carried at the lower of market/NRV amount.

Option (A) is incorrect because original cost is higher than market.

Option (B) is incorrect because the market price used should be net realizable value when replacement cost is higher.

Option (D) is incorrect because the relevant information is missing related to net realizable value - normal profit. However, given the fact that the replacement clost is higher than net realizable value the middle of the three will be the net realizable value.

How well did you know this?
1
Not at all
2
3
4
5
Perfectly
149
Q
Young Corp. purchased equipment by making a down payment of $4,000 and issuing a note payable for $18,000. A payment of $6,000 is to be made at the end of each year for three years. The applicable rate of interest is 8%. The present value of an ordinary annuity factor for three years at 8% is 2.58, and the present value for the future amount of a single sum of one dollar for three years at 8% is .735. Shipping charges for the equipment were $2,000, and installation charges were $3,500. What is the capitalized cost of the equipment?
A
$19,480
B
$21,480
C
$24,980
D
$27,500
A

Answer is C

Explanation:
Assets are to be recorded at their acquisition cost. Acquisition cost is defined as the cash price, or its equivalent, plus all the costs reasonably necessary to bring it to the location and to make it ready for its intended use. The cash price, or its equivalent, would be the $4,000 down payment plus the $15,480 present value of the $6,000 payments, an ordinary annuity, (2.58 × $6,000). Add to that price of $19,480 the $2,000 in shipping charges to get the equipment to its location and the $3,500 of installation charges to get it ready for its intended use. The total capitalized cost of the equipment is $4,000 + $15,480 + $2,000 + $3,500 = $24,980.

How well did you know this?
1
Not at all
2
3
4
5
Perfectly
150
Q
Weir Co. uses straight-line depreciation for its property, plant, and equipment, which, stated at cost, consisted of the following:
12/31,Yr2	12/31, Yr1
Land	$ 25,000	$ 25,000
Buildings	195,000	195,000
Machinery and equipment	695,000	650,000
915,000	870,000
Less: Accumulated depreciation	(400,000)	(370,000)
$ 515,000	$ 500,000Weir's depreciation expense for year 2 and year 1 was $55,000 and $50,000, respectively. What amount was debited to accumulated depreciation during year 2 because of property, plant, and equipment retirements?
A
$40,000
B
$25,000
C
$20,000
D
$10,000
A

Answer is B

Explanation:
Weir’s Accumulated Depreciation account increased by $30,000 ($400,000 - $370,000) during year 2, despite the fact that $55,000 of depreciation expense was recorded during year 2. Therefore, the debit to accumulated depreciation during year 2 because of plant and equipment retirements was $25,000 (i.e., $55,000 - $30,000).

How well did you know this?
1
Not at all
2
3
4
5
Perfectly
151
Q
Carr, Inc. purchased equipment for $100,000 on January 1, year 2. The equipment had an estimated 10-year useful life and a $15,000 salvage value. Carr uses the 200% declining balance depreciation method. In its year 3 income statement, what amount should Carr report as depreciation expense for the equipment?
A
$13,600
B
$16,000
C
$17,000
D
$20,000
A

ANswer is B

Explanation:
The straight-line (SL) depreciation method is a fixed charge method where an equal amount of depreciable cost is allocated to each period. The SL formula is (historical cost - salvage value) / estimated useful life. The 200% declining balance depreciation method, also known as double-declining balance (DDB), uses a rate of depreciation twice the SL rate applied to the book value (i.e., declining balance) of the asset to obtain the depreciation expense for the period. The salvage value is not used in the calculation except as a lower bound for the asset’s book value. The DDB formula is (2 / estimated useful life) × (historical cost - accumulated depreciation).
year 2: (2/10) × ($100,000 - 0) = $20,000
year 3: (2/10) × ($100,000 - $20,000) = $16,000.

How well did you know this?
1
Not at all
2
3
4
5
Perfectly
152
Q
On January 2, Lem Corp. bought machinery under a contract that required a down payment of $10,000, plus 24 monthly payments of $5,000 each, for total cash payments of $130,000. The cash equivalent price of the machinery was $110,000. The machinery has an estimated useful life of ten years and estimated salvage value of $5,000. Lem uses straight-line depreciation. In its year-end income statement, what amount should Lem report as depreciation for this machinery?
A
$10,500
B
$11,000
C
$12,500
D
$13,000
A

Answer is A

Explanation:
Assets are recorded at their acquisition cost. Acquisition cost is the cash price, or its equivalent.
[($110,000-$5,000) / 10 years = $10,500 / year.].

How well did you know this?
1
Not at all
2
3
4
5
Perfectly
153
Q
Samm Corp. purchased a plot of land for $100,000. The cost to raze a building on the property amounted to $50,000 and Samm received $10,000 from the sale of scrap materials. Samm built a new plant on the site at a total cost of $800,000 including excavation costs of $30,000. What amount should Samm capitalize in its land account?
A
$150,000
B
$140,000
C
$130,000
D
$100,000
A

Answer is B

Explanation:
Land must be recorded at its acquisition cost. Generally, acquisition cost is defined as the cash price, or its equivalent, plus all other costs reasonably necessary to make it ready for its intended use. Such additional costs include demolition of an old building, less any scrap proceeds received.

Purchase price of land $100,000
Add: Cost to raze old building 50,000
Less: Proceeds from sale of scrap materials (10,000)
Capitalized value of land $140,000

How well did you know this?
1
Not at all
2
3
4
5
Perfectly
154
Q
Ichor Co. reported equipment with an original cost of $379,000 and $344,000, and accumulated depreciation of $153,000 and $128,000, respectively, in its comparative financial statements for the years ended December 31, year 5, and year 4. During year 5, Ichor purchased equipment costing $50,000, and sold equipment with a carrying value of $9,000. What amount should Ichor report as depreciation expense for year 5.
A
$19,000
B
$25,000
C
$31,000
D
$34,000
A

Answer is C
Explanation:
Equipment cost, 12/31, year 4 $344,000
Cost of year 5 purchase 50,000
Equipment cost without sale $394,000
Equipment cost, 12/31, year 5 (379,000)
Cost of equipment sold $ 15,000
Carrying value of equipment sold (9,000)
Accumulated depreciation of equipment sold $ 6,000
Accumulated depreciation, 12/31, year 4 $128,000
Accumulated depreciation of equipment sold (6,000)
Balance without year 5 depreciation $122,000
Accumulated depreciation, 12/31, year 5 (153,000)
Depreciation expense year 5 $ 31,000

How well did you know this?
1
Not at all
2
3
4
5
Perfectly
155
Q
\_\_\_\_\_\_\_\_\_\_\_\_\_\_\_\_ are costs that extend the service life of the asset, increase its output rate, or lower production costs.
A
Betterments or improvements
B
Transfers
C
Depreciation
D
Ordinary repairs
A

Explanation:
A
Costs that extend the service life of the asset, increase its output rate, or lower production costs are betterments or improvements and are capitalized.

How well did you know this?
1
Not at all
2
3
4
5
Perfectly
156
Q
A manufacturing firm purchased used equipment for $135,000. The original owners estimated that the residual value of the equipment was $10,000. The carrying amount of the equipment was $120,000 when ownership transferred. The new owners estimate that the expected remaining useful life of the equipment was 10 years, with a salvage value of $15,000. What amount represents the depreciable base used by the new owners?
A
$105,000
B
$110,000
C
$120,000
D
$125,000
A

Explanation:
C
The asset would be recorded at $135,000, its acquisition cost to the new owners. The salvage value would be the $15,000 estimated by the new owners. The carrying amount and residual value of the previous owners does not matter to the new owners. The depreciable base would be $120,000; the $135,000 acquisition cost less the $15,000 estimated salvage value.

How well did you know this?
1
Not at all
2
3
4
5
Perfectly
157
Q
Gold Co. purchased equipment from Marshall Co. on July 1. Gold paid Marshall $10,000 cash and signed a $100,000 noninterest-bearing note payable, due in three years. Gold recorded a $24,868 discount on notes payable related to this transaction. What is the acquired cost of the equipment on July 1?
A
$75,132
B
$85,132
C
$100,000
D
$110,000
A

Explanation:
Equipment purchased by the issuance of securities is recorded at its fair value or the fair value of the securities issued, whichever is more clearly determinable. The fair value of the note would be $75,132 ($100,000 less the $24,868 discount). Gold also paid $10,000 cash that would need to be included in the fair value of the equipment. $75,132 + $10,000 = $85,132.

How well did you know this?
1
Not at all
2
3
4
5
Perfectly
158
Q
On December 1, of the current year, Clark Co. leased office space for five years at a monthly rental of $60,000. On the same date, Clark paid the lessor the following amounts:
First months' rent	$ 60,000
Last months' rent	60,000
Security deposit (refundable at lease expiration)	80,000
Installation of new walls and offices	360,000What should be Clark's current year expense relating to utilization of the office space?
A
$60,000
B
$66,000
C
$120,000
D
$140,000
A

Explanation:
The installation of new walls and offices are leasehold improvements since they are not separable from the leased property and revert to the lessor at the end of the lease term. They are amortized over the lease term. The prepayment of the last month’s rent was made to secure the lease and should be reported as a leasehold within intangible assets. Current year expense is $66,000 ($60,000 rent for December and $6,000 amortization of leasehold improvements [$360,000/60]).

How well did you know this?
1
Not at all
2
3
4
5
Perfectly
159
Q

Net income is understated if, in the first year, estimated salvage value is excluded from the depreciation computation when using the
Straight-line method Production or use method
A Yes No
B Yes Yes
C No No
D No Yes

A

Answer is B

Explanation:
An estimated salvage value is directly incorporated into the formulas for computing annual depreciation expense under the straight-line and productive output methods.

Straight-line depreciation expense = Historical cost - Estimated salvage value
Estimated useful life in time
Productive output depreciation expense = Historical cost - Estimated salvage value x current output
Estimated productive outputTherefore, if estimated salvage value is excluded from either depreciation computation, the depreciable base (i.e., historical cost minus estimated salvage value) would be overstated. Thus, in the first year of the life of the asset, depreciation expense would be overstated and net income would be understated under both methods.

How well did you know this?
1
Not at all
2
3
4
5
Perfectly
160
Q

Which of the following conditions must exist in order for an impairment loss to be recognized?

The carrying amount of the long-lived asset is less than its fair value.
The carrying amount of the long-lived asset is not recoverable.
A
I only
B
II only
C
Both I and II
D
Neither I nor II
A

ANswer is B

Explanation:
Recognition of an impairment loss is required only if a long-term asset’s, or asset group’s, carrying amount is not recoverable and exceeds its fair value.

How well did you know this?
1
Not at all
2
3
4
5
Perfectly
161
Q
Restorations of carrying value for long-lived assets are permitted if an asset's fair value increases subsequent to recording an impairment loss for which of the following?
Held for use	Held for disposal
A	Yes	Yes
B	Yes	No
C	No	Yes
D	No	No
A

Answer is C

Explanation:
Impaired assets are divided into three categories: held for use, held for disposal by sale, and held for disposal other than by sale. For assets held for use, any subsequent reversal of a previously recognized impairment loss is prohibited. For assets held for disposal by sale, the asset is measured at the lower of its book or fair value less cost to sell and its depreciation (or amortization) discontinues. Increases in the fair value, up to but not exceeding book value, would be recognized.

How well did you know this?
1
Not at all
2
3
4
5
Perfectly
162
Q

Last year, Katt Co. reduced the carrying amount of its long-lived assets used in operations from $120,000 to $100,000, in connection with its annual impairment review. During the current year, Katt determined that the fair value of the same assets had increased to $130,000. What amount should Katt record as restoration of a previously recognized impairment loss in the current year’s financial statements?

A
$0
B
$10,000
C
$20,000
D
$30,000
A

Explanation:
The correct answer is (A).

The subsequent reversal or restoration of a previously recognized impairment loss is prohibited under GAAP.

How well did you know this?
1
Not at all
2
3
4
5
Perfectly
163
Q

A company issued a purchase order on December 15, year 1, for a piece of capital equipment that costs $100,000. The capital equipment was shipped from the vendor on December 31, year 1, and received by the company on January 5, year 2. The equipment was installed and placed in service on February 1, year 2. On what date should the depreciation expense begin?

A
December 15, year 1
B
December 31, year 1
C
January 5, year 2
D
February 1, year 2
A

Explanation:
The correct answer is (D).

Depreciation allocates the total cost of an asset over the periods that derive benefit from the asset. This is in line with the matching principle whereby firms recognize revenues and their related expenses in the same accounting period. The equipment was installed and placed in service on February 1, year 2. It is only from this date that the benefit from the asset can be derived and depreciation will also begin from February 1, year 2.

Purchase Order, Shipping and Receiving are irrelevant to depreciation for this piece of equipment.

How well did you know this?
1
Not at all
2
3
4
5
Perfectly
164
Q

On January 1, year 1, a company purchased equipment for $100 million. The equipment consists of four major components, of which two components comprise 80% of the total cost and each has a 20-year useful life. The remaining two components have costs of $10 million each; one of them has a useful life of four years, and the other has a useful life of five years. The company applies the cost model to the equipment and uses the straight-line method of depreciation. Under IFRS, what is the depreciation expense for the year ended December 31, year 1?

A
$4,000,000
B
$5,000,000
C
$8,000,000
D
$8,500,000
A

Explanation:
The correct answer is (D)

Under IFRS, if a tangible asset has significant identifiable components with different values and estimated lives, they should record and depreciated separately.

Hence, depreciation can be calculated as under:

Components Value (in $ million) Useful life (years) Depreciation
Two components comprising 80% of total value 80 20 4
Component 3 10 4 2.5
Component 4 10 5 2
Total Depreciation 8.5

How well did you know this?
1
Not at all
2
3
4
5
Perfectly
165
Q

An entity, upon initial recognition of an asset retirement obligation, should not take which of the following actions?

A
Allocate asset retirement cost to expense over the useful life of the related asset
B
Measure the asset retirement cost at fair value
C
Capitalize the asset retirement cost by increasing the carrying amount of the related asset
D
Capitalize the asset retirement cost at its undiscounted cash flow value

A

Explanation:
The correct answer is (D).

An Asset Retirement Obligation (ARO) involves the retirement of a tangible, long-lived asset that depends on a future event beyond the control of an obligated party. An ARO is an essential part of producing fair and accurate financial statements. So the entity should not capitalize the asset retirement cost at its undiscounted cash flow value. Initial recognition of liability is at fair value (FV) or present value (PV) of expected future restoration cost using adjusted risk-free rate.

(A), (B) and (C) are incorrect because these are the legal obligations associated with the retirement of long-lived assets.

How well did you know this?
1
Not at all
2
3
4
5
Perfectly
166
Q

Which of the following is not a true statement regarding the accounting and reporting requirements related to property, plant, and equipment under both IFRS and US GAAP?

A
Both IFRS and US GAAP require a provision for asset retirement costs to be recorded when there is a legal obligation.
B
Changes in depreciation method, residual value, and useful economic life are treated as a change in accounting estimate requiring prospective treatment under US GAAP and retrospective application under IFRS.
C
Under US GAAP eligible borrowing costs do not include exchange rate differences and under IFRS eligible borrowing costs include exchange rate differences from foreign currency borrowings.
D
Under US GAAP investment property is not separately defined and, therefore, is accounted for as held for use or held for sale whereas under IFRS investment property is separately defined.

A

Answer is B

Explanation:
Changes in depreciation method, residual value, and useful economic life are treated as a change in accounting estimate requiring prospective treatment under both US GAAP and IFRS. Both IFRS and US GAAP require a provision for asset retirement costs to be recorded when there is a legal obligation. Under US GAAP, eligible borrowing costs do not include exchange rate differences and under IFRS eligible borrowing costs include exchange rate differences from foreign currency borrowings. Under US GAAP, investment prop­erty is not separately defined and, therefore, is accounted for as held for use or held for sale whereas under IFRS investment property is separately defined.

How well did you know this?
1
Not at all
2
3
4
5
Perfectly
167
Q

During the year, Bay Co. constructed machinery for its own use and for sale to customers. Bank loans financed these assets both during construction and after construction was complete. How much of the interest incurred should be reported as interest expense in the year-end income statement?
Interest incurred for machinery for own use Interest incurred for machinery held for sale
A All interest incurred All interest incurred
B All interest incurred Interest incurred after completion
C Interest incurred after completion Interest incurred after completion
D Interest incurred after completion All interest incurred

A

Answer is D

Explanation:
Interest cost during construction is to be capitalized on assets constructed for an enterprise’s own use (including assets constructed by others for which progress payments have been made), or assets intended for sale or lease that are constructed as discrete projects. However, interest cost shall not be capitalized for inventories that are routinely manufactured or otherwise produced in large quantities on a repetitive basis. Once the constructed assets are completed, all interest should be expensed as incurred. In the question at hand, it is not clear, but it is assumed that the machinery constructed for sale is routinely manufactured in large quantities on a repetitive basis. Therefore, the interest during construction should be capitalized only for the equipment constructed for Bay’s own use. The interest incurred during construction of the inventory items should be expensed. The interest incurred after completion should be expensed on both groups of equipment.

How well did you know this?
1
Not at all
2
3
4
5
Perfectly
168
Q

The graph below depicts three depreciation expense patterns over time.

Which depreciation expense pattern corresponds to the sum-of-the-years’-digits method and which corresponds to the double-declining-balance method?

Sum-of-the-years'-digits	Double-declining-balance
A	III	II
B	II	I
C	I	III
D	II	III
A

Answer is D

Explanation:
The depreciation pattern corresponding to the sum-of-the-years’-digits (SYD) method is the straight sloping line (II) and the pattern corresponding to the double-declining-balance (DDB) method is the curved line (III). The straight line parallel with the horizontal axis (I) describes the pattern of the straight-line (SL) method of depreciation because the SL method results in a constant amount of depreciation each period. The line for the pattern of the SYD method is a straight sloping line because the method results in a decreasing depreciation charge based on a decreasing fraction of depreciable cost where each fraction has the same denominator (sum of the years). The line for the pattern of the DDB method will be a curved sloping line because that type of method employs a constant percentage multiplied by a decreasing balance to obtain a decreasing charge for depreciation.

How well did you know this?
1
Not at all
2
3
4
5
Perfectly
169
Q

company with a June 30 fiscal year-end entered into a $3,000,000 construction project on April 1 to be completed on September 30. The cumulative construction-in-progress balances at April 30, May 31, and June 30 were $500,000, $800,000, and $1,500,000, respectively. The interest rate on company debt used to finance the construction project was 5% from April 1 through June 30 and 6% from July 1 through September 30. Assuming that the asset is placed into service on October 1 and an additional amount spent at the beginning of the month, what amount of interest should be capitalized to the project on June 30?

A
$11,667
B
$18,750
C
$75,000
D
$90,000
A

Explanation:
The correct answer is (A).

The interest cost incurred during the construction period needs to be capitalized to the asset. The interest to be capitalized is lower of the following:

Weighted Average Accumulated Expenditure * Interest rate.
The actual interest cost incurred.
Interest: ($500,000 x 3/12 (April - June) x 5%) + ($300,000 x 2/12 (May - June) x 5% ) + ($700,000 x 1/12 (June) x 5%) = $11,667.

The company has a June 30th year-end and the question only asks for that, therefore, any information beyond June 30th is irrelevant.

Interest until June 30th is 5%. As of June 30, the interest amount to be capitalized is $11,667.

How well did you know this?
1
Not at all
2
3
4
5
Perfectly
170
Q

Which of the following uses the straight-line depreciation method? Group depreciation Composite depreciation
Group depreciation Composite depreciation
A No No
B Yes No
C Yes Yes
D No Yes

A

Answer is C

Explanation:
The composite depreciation method refers to the depreciation of a collection of assets that are dissimilar. The group depreciation method refers to the depreciation of a collection of assets that are similar in nature. From an accounting standpoint, there is no distinction between the two methods. The same procedures are followed for both, and both can utilize the straight-line depreciation method.

How well did you know this?
1
Not at all
2
3
4
5
Perfectly
171
Q
At the beginning of the year, the carrying value of an asset was $1,000,000 with 20 years of remaining life. The fair value of the liability for the asset retirement obligation was $100,000. At year end, the carrying value of the asset was $950,000. The risk-free interest rate was 5%. The credit-adjusted riskfree interest rate was 10%. What was the amount of accretion expense for the year related to the asset retirement obligation?
A
$ 10,000
B
$ 50,000
C
$ 95,000
D
$ 100,000
A

Answer is A

Explanation:
An asset retirement obligations (ARO) must be recorded at fair value in the accounting period in which it occurs and in which its amount can be reasonably measured. AROs incur depreciation and accretion expenses each year. Accretion expense is offset with an increase to the liability account, and, at the end of the asset’s life, the liability account will have a balance equal to the amount needed to settle the retirement obligation. Accretion expense is calculated by multiplying the balance of the recorded liability by the company’s credit-adjusted discount rate each year, so the amount of accretion expense for the year is $10,000 ($100,000 x 10%).

How well did you know this?
1
Not at all
2
3
4
5
Perfectly
172
Q

A transportation company purchased a passenger bus for $100,000 on January 1, year 1. The company expects the bus to be used for 20 years if it follows a maintenance schedule of replacing the engine after 10 years and replacing the seats every eight years. It estimates that the current cost to replace the engine is $25,000 and the current cost to replace the seats is $10,000. The company uses straight-line depreciation and the bus has no residual value. The company considers any component equal to or greater than 10% of the overall cost to be significant. Under IFRS, how much depreciation expense should the company recognize for the bus for the year ended December 31, Year 1?

A
$5,000
B
$7,000
C
$7,250
D
$8,500
A

Explanation:
The correct answer is (B).

The question is asking to calculate depreciation expense for the bus for the year ended December 31, year 1. Calculate the depreciation separately for each Item. Depreciation for each item:

Bus: $100,000 - $25,000 (engine) - $10,000 (seat) = $65,000 / 20 years = $3,250 depreciation for the bus.

Engine: $25,000 / 10 = $2,500. Engine is replaced every 10 years.

Seats: $10,000 / 8 = $1,250. Seats are replaced every 8 years.

Total Depreciation = $3,250 + $2,500 + $1,250 = $7,000.

How well did you know this?
1
Not at all
2
3
4
5
Perfectly
173
Q
A fixed asset with a five-year estimated useful life and no residual value is sold at the end of the second year of its useful life. How would using the sum-ofthe- years'-digits method of depreciation instead of the double-declining-balance method of depreciation affect a gain or loss on the sale of the fixed asset?
Gain	Loss
A	Decrease	Decrease
B	Decrease	Increase
C	Increase	Decrease
D	Increase	Increase
A

Answer is B

Explanation:
This question requires the determination of how using the sum-of-the-years’ digits method of depreciation instead of the double declining balance method of depreciation affects the gain or loss on the sale of a fixed asset with a five-year estimated useful life and no residual value that is sold at the end of the second year of its useful life. A gain or loss on the sale of a fixed asset is computed by comparing the proceeds received from its sale to its carrying amount at the date of sale. To determine the carrying amount of the fixed asset at the sale date, two years of depreciation must be subtracted from its cost. Because the cost of the fixed asset is not given, there are two options: (1) determine depreciation as a percent or fraction of the unknown cost of the fixed asset, ‘x,’ or (2) plug in any value for the cost of the machine, and simply compute depreciation under both methods based on this amount. We chose the latter approach, assigning to the machine a cost of $300. Now we can compare the carrying amount of the fixed asset at the sale date under both methods. The fixed asset has a greater carrying amount at the date of sale under the sum-of-the-years’-digits method. Therefore, using the sum-of-the-years’-digits method of depreciation instead of the double-declining-balance method of depreciation decreases any gain and increases any loss recognized on the sale of the fixed asset. :

Cost of fixed asset	$300
Depreciation to date of sale:	
Yr. 1: 5/15 x ($300 - $0)	(100)
Yr. 2: 4/15 x ($300 - $0)	     (80)
Carrying amount, date of sale, SYD	120
Cost of fixed asset	$300
Depreciation to date of sale:	
Yr. 1: 2/5 x ($300 - $0)	($120)
Yr. 2: 2/5 x ($300 - $120)	     (72)
Carrying amount, date of sale, DDB	$108
How well did you know this?
1
Not at all
2
3
4
5
Perfectly
174
Q

When equipment is retired, accumulated depreciation is debited for the original cost less any residual recovery under which of the following depreciation methods?
Composite depreciation Group depreciation
A No No
B No Yes
C Yes No
D Yes Yes

A

Answer is D

Explanation:
Under both the group depreciation and the composite depreciation methods, assets are depreciated on the basis of their average lives. New assets are recorded at cost, and retirements are accounted for by crediting Assets for the original cost of the equipment and debiting Accumulated Depreciation for the original cost less proceeds received; thus, no gain or loss is recognized on disposal. The theoretical justification for this is that some assets will be retired early, while some will be used longer than originally estimated. The same approach is used under both group and composite depreciation–the only difference is that “group depreciation” refers to pools of assets that are similar in nature, whereas “composite” refers to essentially dissimilar assets.

How well did you know this?
1
Not at all
2
3
4
5
Perfectly
175
Q

On January 2, Year 1, Ames Corp. signed an eight-year lease for office space. Ames has the option to renew the lease for an additional four-year period on or
before January 2, Year 8. During January of year 1, Ames incurred the following costs:

$120,000 for general improvements to the leased premises with an estimated useful life of ten years.
$50,000 for office furniture and equipment with an estimated useful life of ten years.
At December 31, Year 1, Ames’ intentions as to exercise of the renewal option are uncertain. A full year’s amortization of leasehold improvements is taken for
calendar year 1. In Ames’ December 31, Year 1 balance sheet, accumulated amortization should be

A
$10,000
B
$15,000
C
$17,000
D
$21,250
A

Explanation:
The correct answer is (B).

Leasehold improvements are improvements made by the lessee to the leased property. The improvements are not separable from the leased property and revert to the lessor at the end of the lease term.

Of the items listed, only the general improvements made to the leased premises represent a leasehold improvement.

As such, it should be amortized over the lesser of the lease term (i.e., 8 years) or the estimated useful life of the leasehold improvement (i.e., 10 years).

The likelihood of lease renewal is too uncertain to warrant apportioning the cost over the sum of the remaining term of the lease and the period covered by the renewal option (i.e., 12 years).

Therefore, the accumulated amortization to be reported in the 12/31, year 1 balance sheet with respect to the leasehold improvements is $15,000 ($120,000 / 8 years).

Office furniture and equipment are fixed assets and depreciated (not amortized) and the depreciation of $5,000 ($50,000/10) is not included in accumulated amortization.

How well did you know this?
1
Not at all
2
3
4
5
Perfectly
176
Q

Sea Manufacturing Corp. is constructing a new factory building. During the current calendar year, Sea made the following payments to the construction company:

January 2 $1,000,000
December 31 $1,000,000
Sea has an 8%, three-year construction loan of $3,000,000. What is the amount of interest costs that Sea may capitalize during the current year?

A
$0
B
$80,000
C
$160,000
D
$240,000
A

Explanation:
The correct answer is (B).

Interest cost incurred during the construction period needs to be capitalized to the asset.

The interest to be capitalized is lower of the following:

Weighted average accumulated expenditure x Interest rate
The actual interest cost incurred.
The amount of capitalized interest is based on the weighted average amount of accumulated expenditures.

Weighted average amount of accumulated expenditures is:

$1,000,000 (12/12) + $1,000,000 (0/12) = $1,000,000
$1,000,000 * 8% = $80,000
The full $80,000 should be capitalized because the total interest on the loan is $240,000 ($3,000,000 * 8%).
(A) is incorrect because Interest is capitalized at $80,000, not $0.

(C) is incorrect because it calculated on $2,000,000 x 8% = $160,000.

(D) is incorrect because it is the actual interest cost incurred

How well did you know this?
1
Not at all
2
3
4
5
Perfectly
177
Q

Lano Corp.’s forest land was condemned for use as a national park. Compensation for the condemnation exceeded the forest land’s carrying amount. Lano purchased similar, but larger, replacement forest land for an amount greater than the condemnation award. As a result of the condemnation and replacement, what is the net effect on the carrying amount of forest land reported in Lano’s balance sheet?
A
The amount is increased by the excess of the replacement forest land’s cost over the condemned forest land’s carrying amount.
B
The amount is increased by the excess of the replacement forest land’s cost over the condemnation award.
C
The amount is increased by the excess of the condemnation award over the condemned forest land’s carrying amount.
D
No effect, because the condemned forest land’s carrying amount is used as the replacement forest land’s carrying amount.

A

Answer is A

Explanation:
The condemnation of the forest land represents an example of an involuntary conversion of a nonmonetary asset to a monetary asset. Any gain or loss realized on the property converted be recognized in income even though the enterprise reinvests or is obligated to reinvest the monetary assets in replacement nonmonetary assets. Since any gain or loss realized on the property converted is recognized in income, the replacement nonmonetary asset is recorded at cost. Because the cost of the replacement forest land exceeds the condemnation award which exceeds the condemned forest land’s carrying amount, the debit to Forest Land to record the cost of the replacement property exceeds the credit to Forest Land to remove the carrying amount of the condemned property, and thus the carrying amount of Forest Land in the balance sheet increases by the amount of the excess.

How well did you know this?
1
Not at all
2
3
4
5
Perfectly
178
Q

Land was purchased to be used as the site for the construction of a plant. A building on the property was sold and removed by the buyer so that construction on the plant could begin. The proceeds from the sale of the building should be
A
Netted against the costs to clear the land and expensed as incurred.
B
Netted against the costs to clear the land and amortized over the life of the plant.
C
Deducted from the cost of the land.
D
Classified as other income.c

A

c

Explanation:
Sometimes a structure is constructed on a newly acquired site with an existing building on it. If the existing building is sold and the buyer assumes the costs for its removal, the entire amount of the proceeds from the sale of the building should be deducted from the cost of the land.

How well did you know this?
1
Not at all
2
3
4
5
Perfectly
179
Q

Sun Co. was constructing fixed assets that qualified for interest capitalization. Sun had the following outstanding debt issuances during the entire year of construction: $6,000,000 face value, 8% interest $8,000,000 face value, 9% interest. None of the borrowings were specified for the construction of the qualified fixed asset. Average expenditures for the year were $1,000,000. What interest rate should Sun use to calculate capitalized interest on the construction?

A
8.00%
B
8.50%
C
8.57%
D
9.00%
A

Explanation:
The correct answer is (C).

The cost of assets constructed for the use of the business should include all directly-related costs: cost of direct materials, cost of direct labor, additional overhead incurred, and interest costs incurred during the construction period. If the average accumulated expenditures of an asset exceed the amount of any specific borrowings associated with the asset, the excess should be capitalized at the weighted average of interest rates applicable to other borrowings of the business. There was no specific borrowing mentioned, so the interest rate would simply be the weighted average.

(($6,000,000 × 0.08) + ($8,000,000 × 0.09)) / $14,000,000 = 0.0857.

How well did you know this?
1
Not at all
2
3
4
5
Perfectly
180
Q
Which of the following is not an example of wasting assets?
A
Minerals
B
Gas deposits
C
Land
D
Oil deposits
A

C
Explanation:
Wasting assets are natural resources, such as minerals and gas and oil deposits that are subject to exhaustion through extraction. Land is deemed to have an indefinite life and is not depreciated.

How well did you know this?
1
Not at all
2
3
4
5
Perfectly
181
Q

Derby Co. incurred costs to modify its building and to rearrange its production line. As a result, an overall reduction in production costs is expected. However, the modifications did not increase the building’s market value, and the rearrangement did not extend the production line’s life. Should the building modification costs be capitalized?
Building modification costs Production line rearrangement costs
A Yes No
B Yes Yes
C No No
D No Yes

A

b

Explanation:
Both the building modification costs and the production line rearrangement costs should be capitalized because they have resulted in an overall reduction of production costs, the benefits of which extend beyond the current period.

How well did you know this?
1
Not at all
2
3
4
5
Perfectly
182
Q

Question #1487, Blueprint Area: Property, Plant and Equipment
On January 1, Pax Company acquired for $18,000 a new piece of equipment with an estimated useful life of 10 years. The equipment required the addition of a custom-made component, costing $3,000 that must be replaced in 4 years. Pax uses the straight-line method of depreciation. Under IFRS, what is the depreciation expense for the year ended December 31?

A
$0
B
$1,800
C
$2,100
D
$2,550
A

d

Explanation:
Under IFRS, components with a different estimated useful life must be depreciated separately. The equipment would have $1,800 depreciation ($18,000/10 years) and the component would have $750 depreciation ($3,000/4 years), for total depreciation expense of $2,550 annually.

How well did you know this?
1
Not at all
2
3
4
5
Perfectly
183
Q
A company has a long-lived asset with a carrying value of $120,000, expected future cash flows of $130,000, present value of expected future cash flows of $100,000, and a market value of $105,000. What amount of impairment loss should be reported?
A
$0
B
$5,000
C
$15,000
D
$20,000
A

a

Explanation:
An impairment loss shall be recognized only if the carrying amount of a long-lived asset, or asset group, is not recoverable and exceeds its fair value. The carrying amount (book value) is not recoverable if it exceeds the sum of the undiscounted cash flows expected to result from the use and eventual disposition of the asset. The amount of an impairment loss is the difference between an asset’s book and fair value. The $120,000 carrying value of the company’s longlived does not exceed the $130,000 undiscounted future cash flows expected to result from the asset so there is no impairment loss.

How well did you know this?
1
Not at all
2
3
4
5
Perfectly
184
Q

An asset is being constructed for an enterprise’s own use. The asset has been financed with a specific new borrowing. The interest cost incurred during the construction period as a result of expenditures for the asset is
A
Interest expense in the construction period
B
A prepaid asset to be written off over the estimated useful life of the asset
C
A part of the historical cost of acquiring the asset to be written off over the estimated useful life of the asset
D
A part of the historical cost of acquiring the asset to be written off over the term of the borrowing used to finance the construction of the asset

A

c

Explanation:
The capitalization of interest cost incurred during the construction of most assets having an extended construction period is required. Capitalized interest is included in the cost of the asset in the same manner as any other construction cost; it is then written off over the life of the asset as part of the periodic depreciation charges.

How well did you know this?
1
Not at all
2
3
4
5
Perfectly
185
Q

A depreciable asset has an estimated 15% salvage value. At the end of its estimated useful life, the accumulated depreciation would equal the original cost of the asset under which of the following depreciation methods?
Productive output Sum-of-the-years’-digits
A Yes No
B No No
C No Yes
D Yes Yes

A

b

Explanation:
A fixed asset should never be depreciated below its salvage value under any method of depreciation. The depreciable base of a plant asset is its acquisition cost less any estimated salvage value. The depreciable base is allocated to expense over the life of the asset in a systematic and rational manner. At the end of its estimated useful life, the accumulated depreciation would equal the depreciable base of the asset.

How well did you know this?
1
Not at all
2
3
4
5
Perfectly
186
Q
An expenditure to install an improved electrical system is a
Capital expenditure	Revenue expenditure
A	No	Yes
B	No	No
C	Yes	No
D	Yes	Yes
A

c

Explanation:
The expenditure to install an improved electrical system represents a betterment (or improvement); that is, the substitution of a better asset for an existing asset. The expenditure increases the service potential of the electrical system. Because this increased service potential is a benefit that will be enjoyed throughout the life of the electrical system, the expenditure should be capitalized and depreciated over the life of the system. Revenue expenditures, on the other hand, are recurring expenditures that do not add to the service potential of a plant asset; they serve merely to maintain a given level of services. Revenue expenditures should be expensed when incurred.

How well did you know this?
1
Not at all
2
3
4
5
Perfectly
187
Q
In an exchange with commercial substance, Vey Co. traded equipment with an original cost of $100,000 and accumulated depreciation of $40,000 for similar productive equipment with a fair value of $120,000. In addition, Vey received $30,000 cash in connection with this exchange. What should be Vey's carrying amount for the equipment received on the day of exchange?
A
$ 90,000
B
$ 60,000
C
$ 48,000
D
$120,000
A

d

Explanation:
In an exchange with commercial substance, the transaction is recorded at the fair value of the asset received or the asset given up, whichever is more clearly evident, and a gain or loss is recognized on the exchange. The fair value of the equipment received is given in the scenario, therefore, it is more clearly evident than the fair value of the equipment given up.

The journal entry to be recorded in Vey’s books is:

Cash	30,000	
New Equipment	120,000	
Accumulated Depreciation	40,000	
Old Equipment		100,000
Gain on Exchange		90,000
How well did you know this?
1
Not at all
2
3
4
5
Perfectly
188
Q
On January 1, year 1, Crater, Inc. purchased equipment having an estimated salvage value equal to 20% of its original cost at the end of a 10-year life. The equipment was sold December 31, year 5, for 50% of its original cost. If the equipment's disposition resulted in a reported loss, which of the following depreciation methods did Crater use?
A
Double-declining balance.
B
Sum-of-the-years'-digits.
C
Straight-line.
D
Composite.
A

c

Explanation:
The carrying amount of the equipment at the date of sale is computed by subtracting the accumulated depreciation on the equipment from the cost of the equipment. Under the straight-line method, the accumulated deprecation on the equipment at the date of sale is equal to 40% of the original cost of the equipment [i.e., (100% - 20%) x 1/10 x 5]. Thus, under the straight-line method, the carrying amount of the equipment at the date of sale is 60% (i.e., 100% - 40%) of the original cost of the equipment. Since the equipment was sold for only 50% of its original cost, use of the straight-line method would have resulted in a loss being recognized on the sale equal to 10% (i.e., 60% - 50%) of the original cost of the equipment.

How well did you know this?
1
Not at all
2
3
4
5
Perfectly
189
Q
Cole Co. began constructing a building for its own use in January of the current year. During the year, Cole incurred interest of $50,000 on specific construction debt, and $20,000 on other borrowings. Interest computed on the weighted-average amount of accumulated expenditures for the building during the year was $40,000. What amount of interest cost should Cole capitalize?
A
$20,000
B
$40,000
C
$50,000
D
$70,000
A

b

Explanation:
The amount of interest that may be capitalized is based on the weighted-average amount of accumulated expenditures. The weighted-average amount of accumulated expenditures applies the avoidable interest concept. This concept limits the amount of interest to be capitalized to the lower of the actual interest cost incurred during the period or avoidable interest. Avoidable interest is the amount of interest cost incurred during the period that theoretically could have been avoided if expenditures for the asset had not been made.

How well did you know this?
1
Not at all
2
3
4
5
Perfectly
190
Q

Which of the following statements concerning the acquisition of assets is false?

A
Donated assets should be recorded at book value along with any incidental costs incurred.
B
When an asset is received from a governmental entity, no income is recognized, and the offsetting credit is to an owners’ equity account, “Additional Paid-In Capital: Donated Assets.”
C
Assets donated by entities other than governmental units are included in revenue in the period of receipt.
D
If several dissimilar assets are purchased for a lump sum, the total amount paid should be allocated to each individual asset on the basis of its relative fair value.

A

Explanation:
The correct answer is (A).

The question is asking for which statement is false. Donated assets should be recorded at fair value along with any incidental costs incurred and not at Book Value.

How well did you know this?
1
Not at all
2
3
4
5
Perfectly
191
Q

A company recently moved to a new building. The old building is being actively marketed for sale, and the company expects to complete the sale in four months. Each of the following statements is correct regarding the old building, except:
A
It will be reclassified as an asset held for sale.
B
It will be classified as a current asset.
C
It will no longer be depreciated.
D
It will be valued at historical cost.

A

D

Explanation:
Criteria to determine when long-lived assets are held for sale include requirements that: (a) the asset is available for prompt sale as is, subject only to customary and usual sales terms for such assets; and (b) the asset sale is probable and generally to be completed within 12 months. The asset is reclassified to a current asset, and is measured at the lower of its book or fair value less cost to sell and its depreciation (or amortization) discontinues.

How well did you know this?
1
Not at all
2
3
4
5
Perfectly
192
Q
On January 1, year 2, Victor Company purchased for $85,000 a machine having a useful life of ten years and an estimated salvage value of $5,000. The machine was depreciated by the straight-line method. On July 1, year 7, the machine was sold for $45,000. For the year ended December 31, year 7, how much gain should Victor record on the sale?
A
$0
B
$1,000
C
$4,000
D
$6,750
A

Explanation:
The machine had a depreciable basis of $80,000 ($85,000 cost - $5,000 salvage value) and was depreciated at a rate of $8,000 per year ($80,000 / 10-year useful life). At the time of sale, Victor depreciated the machine for 5.5 years and its carrying amount was equal to $41,000, i.e., [$85,000 - (5.5 × $8,000)]. Thus, a $4,000 gain ($45,000 - $41,000) should be recognized.

How well did you know this?
1
Not at all
2
3
4
5
Perfectly
193
Q
Cart Co. purchased an office building and the land on which it is located for $750,000 cash and an existing $250,000 mortgage. For realty tax purposes, the property is assessed at $960,000, 60% of which is allocated to the building. At what amount should Cart record the building?
A
$500,000
B
$576,000
C
$600,000
D
$960,000
A

Explanation:
Assets are to be recorded at their acquisition cost, which is defined as the cash price or its equivalent. The acquisition cost of the office building and the land together is $1,000,000; the total of the $750,000 cash and $250,000 mortgage. The property is assessed with 60% allocated to the building. Cart would record the building at $600,000 ($1,000,000 x 60%) and the land at the remaining $400,000.

How well did you know this?
1
Not at all
2
3
4
5
Perfectly
194
Q
Rye Co. purchased a machine with a four-year estimated useful life and an estimated 10% salvage value for $80,000 on January 1 of the current year. In its income statement for the third year, what would Rye report as the depreciation expense using the double-declining-balance method?
A
$ 9,000
B
$10,000
C
$18,000
D
$20,000
A

Explanation:
Under the double-declining-balance method of depreciation, the annual depreciation charge is computed by multiplying the carrying amount of the plant asset (i.e., cost minus accumulated depreciation) by a rate equal to 200% of the straight-line rate (e.g., 200% x 25% = 50%). Salvage value is not used in the depreciation formula, but the plant asset cannot be depreciated below its salvage value.

First year expense [50% x ($80,000 - $0)] = $40,000
Second year expense [50% x ($80,000 - $40,000)] = $20,000
Third year expense [50% x ($80,000 - $60,000)] = $10,000

How well did you know this?
1
Not at all
2
3
4
5
Perfectly
195
Q
Talton Co. installed new assembly line production equipment at a cost of $185,000. Talton had to rearrange the assembly line and remove a wall to install the equipment. The rearrangement cost $12,000 and the wall removal cost $3,000. The rearrangement did not increase the life of the assembly line but it did make it more efficient. What amount of these costs should be capitalized by Talton?
A
$185,000
B
$188,000
C
$197,000
D
$200,000
A

D
Explanation:
Assets are to be recorded at their acquisition cost. Acquisition cost is defined as the cash price, or equivalent, plus all costs reasonably necessary to bring it to the location and to make it ready for its intended use. Both the $12,000 rearrangement cost and $3,000 removal cost were necessary to bring the asset to its location and make it ready for its intended use. Talton would capitalized $200,000 ($185,000 + $12,000 + $4,000 = $200,000).

How well did you know this?
1
Not at all
2
3
4
5
Perfectly
196
Q

Ott Co. purchased a machine at an original cost of $90,000 on January 2, year 1. The estimated useful life of the machine is 10 years, and the machine has no salvage value. Ott uses the straight-line method to calculate depreciation. On July 1, year 10, Ott sold the machine for $5,000. What is the amount of gain or loss on the disposal of the machine?

A
$500 loss
B
$500 gain
C
$4,500 loss
D
$4,500 gain
A

Explanation:
The correct answer is (B).

Calculate Depreciation:

Depreciation under straight-line method each year = Cost of the asset / Useful life = $90,000 / 10 years = $9,000 per year.
Calculate Accumulated Depreciation& Carrying Value

So the total accumulated depreciation for 9 ½ years (January 2, year 1 to June 30, year 10) @ $9,000 each year = $9,000 x 9.5 years = $85,500.
The carrying value of the machine on July 1, year 10 = $4,500 [i.e. $90,000 (historical cost) - $85,500 (accumulated depreciation)].
Calculate Gain/Loss on Sale:

The amount of gain on disposal of a machine is equal to the selling price of machine less the carrying value of the machine on the date of sale.
$4,500 Carrying value - Selling price of $5,000 = $500 Gain.

How well did you know this?
1
Not at all
2
3
4
5
Perfectly
197
Q

A company completes construction of a $400 million offshore oil platform and places it into service on January 1. State law requires that the platform bed is mantled and removed at the end of its useful life, which is estimated to be 10 years. The company estimates that the cost of dismantling the platform will be $20 million. The discounted value of the liability is $9 million using the company’s credit-adjusted, risk-free rate. The company has already capitalized the $400 million construction cost of the platform. What amounts should the company record as liability and expense when the asset is placed into service?

A
Liability, $0; expense, $0
B
Liability, $9,000,000; expense, $0
C
Liability, $9,000,000; expense, $9,000,000
D
Liability, $20,000,000; expense, $20,000,000
A

Explanation:
The correct answer is (B).

An Asset Retirement Obligation (ARO) involves the retirement of a tangible, long-lived asset that depends on a future event beyond the control of an obligated party. An ARO is an essential part of producing fair and accurate financial statements so those viewing them can have a better idea of a company’s obligations, as well as its overall value. An ARO liability must be recorded when the asset is placed into service. An accretion expense, which is a periodic expense recognized as the present value of a balance sheet liability increases, will be recognized at the end of each period, gradually increasing the ARO to its full $20,000,000 million amount when due in 10 years. The discounted value of the liability is $9,000,000 million using the company’s credit-adjusted, risk-free rate. The expense will be $0.

How well did you know this?
1
Not at all
2
3
4
5
Perfectly
198
Q

Which of the following would not be an example of an event or change in circumstances indicating that the carrying amount of the asset or asset group may not be recoverable?
A
A significant increase in the market price of a long-lived asset (asset group)
B
A significant adverse change in the extent or manner in which a long-lived asset (asset group) is being used or in the physical condition of the asset
C
An accumulation of costs significantly in excess of the amount originally expected for the acquisition or construction of a long-lived asset (asset group)
D
A current expectation that, more likely than not, a long-lived asset (asset group) will be sold or otherwise disposed of significantly before the end of its previously estimated useful life

A

A
Explanation:
A significant decrease (not increase) in the market price of a long-lived asset (asset group) would be an example of an event or change in circumstances indicating that the carrying amount of the asset or asset group may not be recoverable.

How well did you know this?
1
Not at all
2
3
4
5
Perfectly
199
Q

A company recorded a decommissioning liability and recognized the amount recorded as part of the cost of the related property. After the property was fully depreciated, the decommissioning liability was reviewed and adjusted. How should this change in the decommissioning liability be recognized under IFRS?

A
The change in the liability is recognized in other comprehensive income.
B
The change in the liability is recognized in profit or loss.
C
The change in the liability is recognized as a change in the carrying amount of the property if the liability increases but is otherwise recognized in profit or loss.
D
The change in the decommissioning liability is not recognized until it is settle

A

Explanation:
The correct answer is (B)

The change in the liability is recognized in profit or loss.

A decommissioning liability is the estimated cost to put the property into a usable or saleable condition when it is no longer going to be used for its original purpose. The liability is reviewed and adjusted to reflect changes in the estimated cost with increases or decreases recognized as adjustments to the carrying value of the property. When the property is fully depreciated, any adjustment to the decommissioning liability is recognized as a profit or loss since it is too late to recognize it as an adjustment to depreciation or amortization.

(A), (C) and (D) are incorrect as per the above explanation.

How well did you know this?
1
Not at all
2
3
4
5
Perfectly
200
Q

A company performing its long-lived asset impairment testing is reviewing the fair value of equipment. Each of the following valuation techniques may be appropriate for measuring the fair value of the equipment, except the

A
Market approach.
B
Income approach.
C
Cost approach.
D
Net realizable value approach.
A

Explanation:
The correct answer is (D)

US GAAP permits only the following techniques for measuring the fair value of an asset or liability:

Market approach
Income approach
Cost approach
However, net realizable value approach is not considered to be a valid technique for fair value measurement and is not appropriate.

How well did you know this?
1
Not at all
2
3
4
5
Perfectly
201
Q
On December 31 of the current year a building owned by Carr, Inc., was destroyed by fire. Carr paid $12,000 for removal and clean-up costs. The building had a book value of $250,000 and a fair value of $280,000 on December 31. What amount should Carr use to determine the gain or loss on this involuntary conversion?
A
$250,000
B
$262,000
C
$280,000
D
$292,000
A

B

Explanation:
Building carrying amount, 12/31 $250,000
Removal and clean-up costs 12,000
Amount to determine gain or loss on involuntary conversion $262,000

How well did you know this?
1
Not at all
2
3
4
5
Perfectly
202
Q

A building suffered uninsured water and related damage. The damaged portion of the building was refurbished with upgraded materials. The cost and related accumulated depreciation of the damaged portion are identifiable. To account for these events, the owner should
A
Capitalize the cost of refurbishing and record a loss in the current period equal to the carrying amount of the damaged portion of the building.
B
Capitalize the cost of refurbishing by adding the cost to the carrying amount of the building.
C
Record a loss in the current period equal to the cost of refurbishing and continue to depreciate the original cost of the building.
D
Record a loss in the current period equal to the sum of the cost of refurbishing and the carrying amount of the damaged portion of the building.

A

A
Explanation:
The damaged portion of the building was refurbished with upgraded materials which indicates that a ‘betterment’ is involved. There are benefits to future periods as a result of the refurbishing expenditures; thus, they should be capitalized. The cost and related accumulated depreciation of the damaged portion of the building are identifiable so they should be removed from the books (with a resulting debit to ‘loss’ for the difference) because this portion of the building has been
replaced to a certain extent and upgraded.

How well did you know this?
1
Not at all
2
3
4
5
Perfectly
203
Q
Quick Co. acquired the following assets from a liquidating competitor for a $200,000 lump-sum purchase price:
Competitor's carrying amount	Fair Value
Inventory	$ 70,000	$50,000
Land	40,000	50,000
Building	  110,000  	  150,000  
$220,000	$250,000What amount should Quick report as the cost of the building?
A
$100,000
B
$120,000
C
$150,000
D
$200,000
A

Explanation:
If several dissimilar assets are purchased for a lump sum, the total amount paid should be allocated to each individual asset on the basis of its relative fair value. The allocation formula: Asset Y = Total cost of assets × FV of Y / Total FV. In this case: $200,000 × $150,000 ÷ $250,000 = $120,000.

How well did you know this?
1
Not at all
2
3
4
5
Perfectly
204
Q
During the year, Burr Co. had the following transactions pertaining to its new office building:
Purchase price of land	$ 60,000
Legal fees for contracts to purchase land	2,000
Architects' fees	8,000
Demolition of old building on site	5,000
Sale of scrap from old building	3,000
Construction cost of new building (fully completed)	350,000In Burr's December 31 balance sheet, what amounts should be reported as the cost of land and cost of building?
Land	Building
A	$60,000	$360,000
B	$62,000	$360,000
C	$64,000	$358,000
D	$65,000	$362,000
A

Explanation:
The legal fees for contracts to purchase land are part of the cost of the land. The cost of demolishing the old building (net of any scrap proceeds) is a reasonable and necessary cost to get the land ready for its intended use.

Purchase price of land $ 60,000
Legal fees for contracts to purchase land 2,000
Demolition of old building on site 5,000
Sale of scrap from old building (3,000)
Cost of land $ 64,000
Architect’s fees $ 8,000
Construction cost of new building 350,000
Cost of building $358,000

How well did you know this?
1
Not at all
2
3
4
5
Perfectly
205
Q
A company obtained a $300,000 loan with a 10% interest rate on January 1, year 1, to finance the construction of an office building for its own use. Building construction began on January 1, year 1, and the project was not completed as of December 31, year 1. The following payments were made in year 1 related to the construction project:
January 1	Purchased land for $120,000
September 1	Progress payment to contractor for $150,000What amount of interest should be capitalized for the year ended December 31, year 1?
A
$13,500
B
$15,000
C
$17,000
D
$30,000
A

Explanation:
The amount of interest cost to be capitalized is the interest cost incurred during the acquisition period that could have been avoided if expenditures for the asset had not been made. If a specific interest rate is associated with the asset, that rate should be used to the extent that the average accumulated expenditures on the asset do not exceed the amount borrowed at the specific rate.

Average expenditures during year	
$120,000 × 12/12	$120,000
150,000 × 4/12	50,000
Average expenditures	170,000
Specific borrowing rate	10%
Capitalized interest expense	$ 17,000
How well did you know this?
1
Not at all
2
3
4
5
Perfectly
206
Q

Fountain Co. is constructing an office building for its own use. Fountain started the two-year construction project on April 1, year 1, at which point the interest capitalization period began. Fountain made the following payments in year 1 related to the construction of the building:

April 1 Payment to architect for building plans $ 30,000
July 1 Progress payment to contractor 60,000
October 1 Progress payment to contractor 150,000
For the purpose of capitalizing interest, what is Fountain’s weighted-average accumulated expenditures for the year ended December 31, year 1?

A
$80,000
B
$105,000
C
$120,000
D
$240,000
A

Explanation:
The correct answer is (C).

Interest cost incurred during the construction period needs to be capitalized to the asset. The interest to be capitalized is lower of the following:

  • Weighted-average accumulated expenditure x Interest rate.
  • The actual interest cost incurred.

The weighted-average accumulated expenditures for the purpose of capitalization of borrowing costs can be calculated as under:

Date Particulars Amount Capitalization Period Weighted-average accumulated expenditures
April 1 Payment to architect for building plans $30,000 9/9 $30,000
July 1 Progress payment to contractor $60,000 6/9 $40,000
October 1 Progress payment to contractor $150,000 3/9 $50,000
For the purpose of capitalizing interest, what is Fountain’s weighted-average accumulated expenditures for the year ended December 31, year 1?

How well did you know this?
1
Not at all
2
3
4
5
Perfectly
207
Q
Hudson Corp. operates several factories that manufacture medical equipment. The factories have a historical cost of $200 million. Near the end of the company's fiscal year, a change in business climate related to a competitor's innovative products indicated to Hudson's management that the $170 million carrying amount of the assets of one of Hudson's factories may not be recoverable. Management identified cash flows from this factory and estimated that the undiscounted future cash flows over the remaining useful life of the factory would be $150 million. The fair value of the factory's assets is reliably estimated to be $135 million. The change in business climate requires investigation of possible impairment. Which of the following amounts is the impairment loss?
A
$15 million
B
$20 million
C
$35 million
D
$65 million
A

Explanation:
A long-lived asset shall be tested for recoverability whenever events or changes in circumstances indicate that its carrying amount may not be recoverable. Impairment is the condition that exists when the carrying amount of a long-lived asset, or asset group, exceeds its fair value. An impairment loss shall be recognized only if the carrying amount of a long-lived asset is not recoverable and exceeds its fair value. The carrying amount (book value) is not recoverable if it exceeds the sum of the undiscounted cash flows expected to result from the use and eventual disposition of the asset. The amount of an impairment loss is the difference between an asset’s book and fair value. The new book value is used as a basis for depreciation. Since the carrying amount of $170,000 exceeds the undiscounted cash flows of $150,000 an impairment loss must be recognized. The $35,000 impairment loss is the difference between the book value of $170,000 and the fair value of $135,000.

How well did you know this?
1
Not at all
2
3
4
5
Perfectly
208
Q
On January 3, Quarry Co. purchased a manufacturing machine for $864,000. The machine had an estimated eight-year useful life and a $72,000 estimated salvage value. Quarry expects to manufacture 1,800,000 units over the life of the machine. During the year, Quarry manufactured 300,000 units. Quarry uses the units-of-production depreciation method. In its December 31 balance sheet, what amount of accumulated depreciation should Quarry report for the machine?
A
$ 99,000
B
$108,000
C
$132,000
D
$144,000
A

Explanation:
Units-of-output depreciation takes into account salvage value and the number of units produced by the asset.
(Cost - salvage) / expected output x current output:
$864,000 - $72,000 = $792,000
$792,000 / 1,800,000 = $0.44
$0.44 x 300,000 = $132,000

How well did you know this?
1
Not at all
2
3
4
5
Perfectly
209
Q
If an outlay will provide a service benefit beyond the current period, it is considered which of the following?
A
An expense
B
A capital expenditure
C
A revenue expenditure
D
None of the above
A

B

Explanation:
If an outlay will provide a service benefit beyond the current period, it is a capital expenditure and is recorded as an asset.

How well did you know this?
1
Not at all
2
3
4
5
Perfectly
210
Q

Quick Co. acquired the following assets from a liquidating competitor for a $200,000 lump-sum purchase price:

Competitor’s carrying amount ($) Fair Value ($)

Inventory 70,000 50,000
Land 40,000 50,000
Building 110,000 150,000
Total 220,000 250,000
What amount should Quick report as the cost of the building?

A
$100,000
B
$120,000
C
$150,000
D
$200,000
A

Explanation:
The correct answer is (B).

In the case of a basket purchase, the purchase price is allocated based on the appraised fair value of individual items. The cost of building reported will be calculated as a proportion of total fair value at 60% ($150,000/$250,000).

Proportion of the purchase price allocated to building: $120,000 = 60% x $200,000.

Summary Fair Value ($) Proportion of the FV paid Cost of Acquisition ($)
Inventory 50,000 20% 40,000
Land 50,000 20% 40,000
Building 150,000 60% 120,000
Total 250,000 100% 200,000
(A) is incorrect because it is calculated based on the proportionate share of the carrying value instead of using fair value to determine the proportionate share, $100,000 (50% x 200,000). Proportionate share using carrying value: ($110,000/$220,000 = 50%).

(C) is incorrect because it is the actual fair value of the building. Quick should report at proportionate share derived from the fair values of the assets purchased.

(D) is incorrect because it is the actual cost paid by the quick to acquire inventory, land, and building.

How well did you know this?
1
Not at all
2
3
4
5
Perfectly
211
Q

Isle Co. owned a copy machine that cost $5,000 and had accumulated depreciation of $2,000. Isle exchanged the copy machine for a computer that cost $4,000. Isle’s future cash flows are not expected to change significantly as a result of the exchange. What amount of gain or loss should Isle report and at what amount should it record the asset?

A
No gain or loss in the income statement; $3,000 asset in the balance sheet.
B
No gain or loss in the income statement; $4,000 asset in the balance sheet.
C
$1,000 gain in the income statement; $3,000 asset in the balance sheet.
D
$1,000 gain in the income statement; $4,000 asset in the balance sheet.

A

Explanation:
The correct answer is (A).

An exchange of non-monetary assets that is not expected to change Isle Co’s cash flows significantly lacks commercial substance. Therefore, accounting for a non-monetary exchange is based on the carrying amount of the assets given up. Also, there is no boot, so no gain is recognized. The computer received by Isle Co is recorded at the carrying amount of the copy machine of $3,000 ($5,000 - $2,000) and no gain or loss is recognized in the income statement.

How well did you know this?
1
Not at all
2
3
4
5
Perfectly
212
Q

Murray Co. maintains its records under IFRS. During the year, Murray sold a machine that had been accounted for using the revaluation method. Details are presented below:

Sales Price $150,000
Machine Book Value $ 80,000
Revaluation Surplus $ 9,000
Which of the following is correct regarding the sale?

A
The gain of $70,000 should be credited to Other Comprehensive Income (OCI).
B
The gain of $79,000 should be recorded in profit and loss on the Income Statement.
C
The gain of $70,000 should be recorded in profit and loss, and the $9,000 revaluation surplus should be transferred to retained earnings.
D
No gain or loss should be recorded.

A

Explanation:
Two journal entries are required when an asset accounted for under the revaluation model is sold. First, transfer the related revaluation surplus of $9,000 to retained earnings. Second, record the sale and gain/ loss as usual. Since the machine’s net book value is $80,000, and cash of $150,000 was received, the gain on sale would be $70,000.

How well did you know this?
1
Not at all
2
3
4
5
Perfectly
213
Q
On October 1 of the current year, Shaw Corp. purchased a machine for $126,000 that was placed in service on November 30. Shaw incurred additional costs for this machine as follows:
Shipping	$3,000
Installation	4,000
Testing	5,000In Shaw's December 31 balance sheet, the machine's cost should be reported as
A
$126,000
B
$129,000
C
$133,000
D
$138,000
A

Explanation:
The acquisition cost of the machine includes all of the costs incurred to get it ready for its intended use. Thus, the costs incurred in shipping, installing and testing the machine are capitalized and added to its purchase price, resulting in an acquisition cost for the machine of $138,000 ($126,000 + $3,000 + $4,000 + $5,000).

How well did you know this?
1
Not at all
2
3
4
5
Perfectly
214
Q
During the previous year, Yvo Corp. installed a production assembly line to manufacture furniture. In the current year, Yvo purchased a new machine and rearranged the assembly line to install this machine. The rearrangement did not increase the estimated useful life of the assembly line, but it did result in significantly more efficient production. The following expenditures were incurred in connection with this project:
Machine	$75,000
Labor to install machine	14,000
Parts added in rearranging the assembly line to provide future benefits	40,000
Labor and overhead to rearrange the assembly line	18,000What amount of the above expenditures should be capitalized in the current year?
A
$147,000
B
$107,000
C
$ 89,000
D
$ 75,000
A

Explanation:
All of the expenditures related to the purchase of the new machine ($75,000 + $14,000) and the rearrangement of the assembly line to install this machine ($40,000 + $18,000) should be capitalized. The costs associated with the rearrangement of the assembly line are capitalized because the rearrangement has resulted in significantly more efficient production, the benefits of which extend beyond the current period.

How well did you know this?
1
Not at all
2
3
4
5
Perfectly
215
Q

Dell Printing Co. incurred the following costs for one of its printing presses:
Purchase of collating and stapling attachment $84,000
Installation of attachment 36,000
Replacement parts for overhaul of press 26,000
Labor and overhead in connection with overhaul 14,000The overhaul resulted in a significant increase in production. Neither the attachment nor the overhaul increased the estimated useful life of the press. What amount of the above costs should be capitalized?
A
$0
B
$84,000
C
$120,000
D
$160,000

A

Explanation:
The collating and stapling attachment is an addition to the printing press and so its cost (including installation cost) should be capitalized. The overhaul resulted in a significant increase in the productivity of the printing press; therefore, its cost (replacement parts, labor and overhead) should also be capitalized. The total cost capitalized is $160,000 (i.e., $84,000 + $36,000 + $26,000 + $14,000).

How well did you know this?
1
Not at all
2
3
4
5
Perfectly
216
Q
In January, Vorst Co. purchased a mineral mine for $2,640,000 with removable ore estimated at 1,200,000 tons. After it has extracted all the ore, Vorst will be required by law to restore the land to its original condition at an estimated cost of $180,000. Vorst believes it will be able to sell the property afterwards for $300,000. During the year, Vorst incurred $360,000 of development costs preparing the mine for production and removed and sold 60,000 tons of ore. In its year-end income statement, what amount should Vorst report as depletion?
A
$135,000
B
$144,000
C
$150,000
D
$159,000
A
Explanation:
Purchase price of mine	$2,640,000
Development costs to prepare mine for production	360,000
Estimated restoration costs	180,000
Estimated residual value	  (300,000)
Depletion base of mine	$2,880,000
Estimated tons of removable ore	/ 1,200,000
Depletion charge per ton	$ 2.40
Tons sold in the year	x    60,000
Depletion expense for the year	$ 144,000
How well did you know this?
1
Not at all
2
3
4
5
Perfectly
217
Q
A depreciable asset has an estimated 15% salvage value. Under which of the following methods, properly applied, would the accumulated depreciation equal the original cost at the end of the asset's estimated useful life? Straight-line Double-declining balance
Straight-line	Double-declining balance
A	Yes	Yes
B	Yes	No
C	No	Yes
D	No	No
A

Explanation:
Under neither the straight-line method or the double-declining balance method of depreciation would the accumulated depreciation equal the original cost at the end of the asset’s estimated useful life. The straight-line method depreciates the cost less salvage value evenly over the estimated useful life of the asset. The double-declining balance method uses a rate of depreciation twice that of the straight-line rate, but the asset cannot be depreciated below the salvage value.

How well did you know this?
1
Not at all
2
3
4
5
Perfectly
218
Q

The following information pertains to the transfer of real estate pursuant to a troubled debt restructuring by Knob Co. to Mene Corp. in full liquidation of Knob’s liability to Mene.
Carrying amount of liability liquidated $150,000
Carrying amount of real estate transferred 100,000
Fair value of real estate transferred 90,000At what amount should Mene record the real estate transferred?
A
$60,000
B
$90,000
C
$100,000
D
$150,000

A

Explanation:
In a troubled debt restructuring, the creditor, Mene, records the assets and/or equity securities (real estate transferred) at fair value.

How well did you know this?
1
Not at all
2
3
4
5
Perfectly
219
Q

Sun Ltd. enters into a contract with a customer for equipment with unique specifications. Sun Ltd. and the customer develop the specifications for the equipment, which Sun communicates to Ralph industries, a supplier that the Sun contracts with to manufacture the equipment. Sun Ltd. also arranges to have Ralph industries deliver the equipment directly to the customer. Upon delivery of the equipment to the customer, the terms of the contract require Sun Ltd. to pay Ralph the price agreed to by Sun and Ralph for manufacturing the equipment. Sun and the customer negotiate the selling price, and the entity invoices the customer for the agreed-upon price with 30-day payment terms. Sun’s profit is based on the difference between the sales price negotiated with the customer and the price charged by Ralph. The contract between Sun and the customer requires the customer to seek remedies for defects in the equipment from the Ralph under the Ralph’s warranty. However, Sun is responsible for any corrections to the equipment required resulting from errors in specifications. In this transaction is Sun:

A
A principal
B
An agent
C
A consignor
D
A dealer
A

A

Explanation:
To determine whether Sun’s performance obligation is to provide the specified goods or services itself (i.e. the entity is a principal) or to arrange for another party to provide those goods or services (i.e. the entity is an agent), the following is considered. Sun has promised to provide the customer with specialized equipment; however it has subcontracted the manufacturing of the equipment to a supplier. In determining whether Sun obtains control of the equipment before control transfers to the customer and whether Sun is a principal, the entity considers the below:

Sun is primarily responsible for fulfilling the contract. Although Sun subcontracted the manufacturing, it is ultimately responsible for ensuring that the equipment meets the specifications for which the customer has contracted.
Sun has inventory risk because of its responsibility for corrections to the equipment resulting from errors in specifications, even though Ralph has inventory risk during production and before shipment.
Sun has discretion in establishing the selling price with the customer, and the profit earned by Sun is an amount that is equal to the difference between the selling price negotiated with the customer and the amount to be paid to Ralph.
Sun’s consideration is not in the form of a commission.
Sun would conclude that its promise is to provide the equipment to the customer. On the basis of the above indicators Sun controls the equipment before it is transferred to the customer. Thus, Sun is a principal in the transaction and recognizes revenue in the gross amount of consideration to which it is entitled from the customer in exchange for the equipment

How well did you know this?
1
Not at all
2
3
4
5
Perfectly
220
Q
On July 1, Casa Development Co. purchased a tract of land for $1,200,000. Casa incurred additional costs of $300,000 during the remainder of the year in preparing the land for sale. The tract was subdivided into residential lots as follows:
Lot Class	Number of lots	Sales price per lot
A	100	$24,000
B	100	$16,000
C	200	$10,000Using the relative sales value method, what amount of costs should be allocated to the Class A lots?
A
$300,000
B
$375,000
C
$600,000
D
$720,000
A

Explanation:
Under the relative sales value method, the total cost of the individual units purchased at a single lump-sum price should be allocated to the various units on the basis of their relative sales value. The total cost includes the purchase price of $1,200,000 plus the additional costs of $300,000 to prepare the land for sale.

Lot Class Number of lots Sales price per lot Total sales for class
A 100 $24,000 $2,400,000
B 100 16,000 1,600,000
C 200 10,000 2,000,000
Total $6,000,000Class A lots ($2,400,000) / Total sales value ($6,000,000) = relative Class A sales value (40%). The total cost ($1,500,000) x the Class A lots relative value percentage (40%) = costs allocated to Class A lots ($600,000).

How well did you know this?
1
Not at all
2
3
4
5
Perfectly
221
Q

A state government condemned Cory Co.’s parcel of real estate. Cory will receive $750,000 for this property, which has a carrying amount of $575,000. Cory incurred the following costs as a result of the condemnation:
Appraisal fees to support a $750,000 value $2,500
Attorney fees for the closing with the state 3,500
Attorney fees to review contract to acquire replacement property 3,000
Title insurance on replacement property 4,000What amount of cost should Cory use to determine the gain on the condemnation?
A
$581,000
B
$582,000
C
$584,000
D
$588,000

A

Explanation:
The amount of cost that should be used to determine the gain on condemnation is $581,000 (i.e., the property’’s carrying amount of $575,000 plus the $2,500 appraisal fee plus the $3,500 attorney fees for the closing with the state). The attorney fees to review the contract and the title insurance are costs of acquiring the replacement property.

How well did you know this?
1
Not at all
2
3
4
5
Perfectly
222
Q

Bayberry Co. has an asset with a cost of $200,000 and accumulated depreciation of $120,000. Driftwood Co. has an asset with a cost of $250,000 and accumulated depreciation of $160,000. Both assets have a fair value of $100,000. Bayberry and Driftwood find it mutually advantageous to exchange assets, and the exchange results in improved future cash flow for both companies. What amount, if any, is Bayberry’s gain on the exchange?

A
$0
B
$10,000
C
$20,000
D
$50,000
A

Explanation:
The correct answer is (C).

Non-monetary exchanges with commercial substance, use fair value to measure the transaction and treat as if two unrelated events have occurred. The realized gains / losses are recognized immediately. Bayberry Co. should recognize gain on the exchange at $20,000.

Steps

Exchange with Commercial Substance

FV is determinable

Calculations

Amount

1

Calculate realized gain or loss

FV of asset given up - CV of asset given up.

= $100,000 - $80,000

$20,000

2

Recognize the realized gain or loss

Recognize all the realized gain / loss (Irrespective of whether no boot involved, boot paid / received)

$20,000

3

Record the new asset

FV of the asset given up + cash paid (if any) - cash received (if any)

= $100,000

$100,000

The Journal entry is as follows:

Equipment (Driftwood) $100,000
Accumulated Depreciation $120,000
Equipment (Bayberry) $200,000
Gain on Exchange $20,000

How well did you know this?
1
Not at all
2
3
4
5
Perfectly
223
Q

A company has experienced operating losses from its appliances division for the past five years. The division is the lowest level of identifiable cash flows. Having determined the division is the lowest level of identifiable cash flows, the company’s next step in performing its impairment test is to

A
Perform a recoverability test on the carrying amount of the division’s assets
B
Reduce the carrying amount of the division’s assets to the amount of expected divisional cash flows
C
Adjust the carrying amount of the division’s assets to fair value
D
Adjust the carrying amount of the division’s assets to replacement value

A

Explanation:
The correct answer is (A).

Testing for impairment occurs when events or changes in circumstances indicate that the carrying amount of a long-lived asset or asset groups may not be recoverable. The impairment test is a one-step process:

An indefinite-lived intangible asset is impaired when the fair value is less than its carrying amount.

Goodwill is tested for impairment at least annually, using a one-step process, and the goodwill impairment test may be performed any time during the fiscal year, provided the test is performed at the same time every year.

To identify potential impairment, we compare the reporting unit’s fair value with its carrying amount, including goodwill i.e. performing a recoverability test on the carrying amount of the division’s assets

If the fair value exceeds its carrying amount, the reporting unit’s goodwill is considered not impaired.
If the carrying amount exceeds its fair value, then the Impairment Loss of the reporting unit recognized is calculated as Carrying Value – Fair Value.

How well did you know this?
1
Not at all
2
3
4
5
Perfectly
224
Q

In year 1, a company purchased equipment that cost $70,000. The equipment has a useful life of seven years and no salvage value. The company used the straight-line method to depreciate the equipment and reported $10,000 of depreciation expense in years 1 and 2. At the beginning of year 3, the company determines that the equipment will last for only three more years (five years total) and changes the depreciable life of the asset accordingly. What amount of depreciation expense should the company report in year 3?

A
$10,000
B
$14,000
C
$16,667
D
$22,000
A

Explanation:
The correct answer is (C)

Under the straight-line method of depreciation, the asset is supposed to be depreciated equally over its useful life. The formula for calculating deprecation is as under:

Depreciation every year = (Cost of asset – salvage value)/Estimated useful life

When the useful life of the equipment was decided to be 7 years, the depreciation every year was = (70,000-0)/7 = $10,000 per year.

However, any revision in the useful life of the asset is to be accounted for prospectively. Hence, the depreciation expense is to be recalculated on the revision of useful life.

Depreciation expense = (Cost – salvage value – accumulated depreciation) / Revised remaining useful life. Hence, revised depreciation expense = (70,000 – 0 – 20,000) / (5-2) = $16,667 per year.

Particulars $
Cost of equipment $70,000
Less: Depreciation per year (when useful life was 7 years)
First-year depreciation ($10,000)
Second-year depreciation ($10,000)
Written Down Value of asset at the beginning of 3rd year $50,000
Less: Depreciation per year (when useful life was revised to 5 years)
Third-year depreciation ($16,667)
Fourth-year depreciation ($16,666)
Fifth-year depreciation ($16,666)
Written Down Value of asset at the beginning of 5th year $0

How well did you know this?
1
Not at all
2
3
4
5
Perfectly
225
Q
Four years ago on January 2, Randall Co. purchased a long-lived asset. The purchase price of the asset was $250,000, with no salvage value. The estimated useful life of the asset was 10 years. Randall used the straight-line method to calculate depreciation expense. An impairment loss on the asset of $30,000 was recognized on December 31 of the current year. The estimated useful life of the asset at December 31 of the current year did not change. What amount should Randall report as depreciation expense in its income statement for the next year?
A
$20,000
B
$22,000
C
$25,000
D
$30,000
A

Explanation:
Impairment is the condition that exists when the carrying amount of a long-lived asset exceeds its fair value. The amount of an impairment loss is the difference between an asset’s book and fair value. The new book value is depreciated over the remaining useful life. Subsequent reversal of a previously recognized impairment loss is prohibited. This is classified as a change in accounting estimate and is reported in the period of change. There are no pro forma reports for prior periods, and amounts reported in financial statements of prior periods are not restated. Randall should report $20,000 ($250,000 cost - $100,000 accumulated depreciation - $30,000 impairment loss = $120,000 remaining basis depreciated over 6 years of remaining life) as depreciation expense in its income statement for the next year.

How well did you know this?
1
Not at all
2
3
4
5
Perfectly
226
Q
\_\_\_\_\_\_\_\_\_\_\_\_\_\_\_\_ is(are) allocated to inventory by the application of depletion charges because these resources are subject to exhaustion through extraction.
A
Plant and equipment
B
Land
C
Timber
D
Copyright
A

Explanation:
Natural resources, such as mineral, gas, and oil deposits, and standing timber, are subject to exhaustion through extraction and their costs must be allocated to inventory by the application of depletion charges.

How well did you know this?
1
Not at all
2
3
4
5
Perfectly
227
Q
Cantor Co. purchased a coal mine for $2,000,000. It cost $500,000 to prepare the coal mine for extraction of the coal. It was estimated that 750,000 tons of coal would be extracted from the mine during its useful life. Cantor planned to sell the property for $100,000 at the end of its useful life. During the current year, 15,000 tons of coal were extracted and sold. What would be Cantor's depletion amount per ton for the current year?
A
$2.50
B
$2.60
C
$3.20
D
$3.30
A

Explanation:
Assets are to be recorded at their acquisition cost. Acquisition cost is the cash price, or its equivalent, plus all costs reasonably necessary to bring it to the location and to make it ready for its intended use. Depletion refers to periodic allocation of acquisition costs of natural resources. A per-unit depletion rate is computed by dividing the acquisition cost of the natural resource (i.e., purchase price, and other development costs), less any estimated residual value, by the estimated number of units of the resource available for extraction.

Purchase price of coal mine $2,000,000
Plus: Mine preparation costs 500,000
Mine total acquisition cost $2,500,000
Less: Value at end of useful life (100,000)
Net depletable amount $2,400,000
Divided by: Estimated number of units available for extraction 750,000
Depletion amount per ton $ 3.20

How well did you know this?
1
Not at all
2
3
4
5
Perfectly
228
Q

Which of the following is an example of an asset that qualifies for interest cost capitalization?
A
Asset constructed or produced as a discrete project for sale or lease, such as a ship
B
Item of inventory routinely manufactured on a repetitive basis
C
Asset in use or ready for use
D
Asset not in use and not being prepared for use

A

Explanation:
Assets qualifying for interest capitalization include assets constructed or produced as discrete projects for sale or lease; for example, ships or real estate developments.

How well did you know this?
1
Not at all
2
3
4
5
Perfectly
229
Q

A company is constructing an asset for its own use. Construction began in the previous year. The asset is being financed entirely with a specific new borrowing. Construction expenditures were made in last year and this year at the end of each quarter. The total amount of interest cost capitalized in the current year should be determined by applying the interest rate on the specific new borrowing to the
A
Total accumulated expenditures for the asset in both years.
B
Average accumulated expenditures for the asset in both years.
C
Average expenditures for the asset in the current year.
D
Total expenditures for the asset in the current year.

A

B
Explanation:
The amount of interest cost to be capitalized is that portion of interest cost incurred during the asset’s acquisition periods that theoretically could have been avoided if expenditures for the asset had not been made. In this question, the amount of interest cost to be capitalized in the current year is determined by applying the interest rate on the specific new borrowing to the average accumulated expenditures for the asset in the previous year and current year.

How well did you know this?
1
Not at all
2
3
4
5
Perfectly
230
Q
Town Company purchased for $540,000 a warehouse building and the land on which it is located. The following data were available concerning the property:
Current appraised value	Seller's original cost
Land	$200,000	$140,000
Warehouse building	300,000	280,000
$500,000	$420,000
Town should record the land at
A
$140,000
B
$180,000
C
$200,000
D
$216,000
A

Explanation:
When several dissimilar assets are purchased for a lump sum, the amount paid should be allocated to each asset on the basis of its relative fair value. Land cost = (Total cost of assets) × (FMV of land) / Total FMV. Land = $540,000 × $200,000 / $500,000 = $216,000.

How well did you know this?
1
Not at all
2
3
4
5
Perfectly
231
Q

A corporation issued debt to purchase 10 acres of land for development purposes. Expenditures related to this purchase are as follows:

Description	Amount
Purchase Price	$1,000,000
Real estate taxes in arrears	$15,000
Debt issuance costs	$2,000
Attorney fee title search on land	$5,000
The company should record its acquisition of the land in its financial statements at a value of
A
$1,000,000
B
$1,015,000
C
$1,020,000
D
$1,022,000
A

Explanation:
The correct answer is (C).

The company should record its acquisition of the land in its financial statements at a value of $1,020,000.

All costs necessary to acquire an asset and prepare it for its intended use may be included in the total cost of the asset. The Purchase price of $1,000,000 + Real estate taxes in arrears or $15,000 + Attorney fees of $5,000 were all necessary expenditures to buy the land.

Finance costs, such as debt issuance costs, are generally not capitalizable into the purchase price of an asset

How well did you know this?
1
Not at all
2
3
4
5
Perfectly
232
Q

For an involuntary conversion of a nonmonetary asset, in which event would an entity not be in compliance with the provisions of GAAP revenue recognition criteria?
A
If it recognizes a gain or loss on the involuntary conversion of a nonmonetary asset, even if the insurance or condemnation award proceeds are reinvested in a replacement nonmonetary asset
B
If it takes removal and clean-up costs into account when computing the gain or loss recognized on the involuntary conversion
C
If it takes into account other incidental costs incurred in the acquisition of replacement property when computing the gain or loss recognized on the involuntary conversion
D
None of the above

A

C

Explanation:
Incidental costs incurred in the acquisition of replacement property are capitalized as costs of acquiring the replacement property. The incidental costs do not affect the gain or loss recognized due to the involuntary conversion.

How well did you know this?
1
Not at all
2
3
4
5
Perfectly
233
Q
Finch Co. reported a total asset retirement obligation of $257,000 in last year's financial statements. This year, Finch acquired assets subject to unconditional retirement obligations measured at undiscounted cash flow estimates of $110,000 and discounted cash flow estimates of $68,000. Finch paid $87,000 toward the settlement of previously recorded asset retirement obligations and recorded an accretion expense of $26,000. What amount should Finch report for the asset retirement obligation in this year's balance sheet?
A
$238,000
B
$264,000
C
$280,000
D
$306,000
A

Explanation:
An asset retirement obligations (ARO) must be recorded at fair value in the accounting period in which it occurs and in which its amount can be reasonably measured. If an active market for the ARO doesn’t exist to provide fair value, the expected present value technique using discounted cash flows is the best way to determine the fair value of the asset. AROs incur depreciation and accretion expenses each year. Accretion expense is offset with an increase to the liability account, and, at the end of the asset’s life, the liability account will have a balance equal to the amount needed to settle the retirement obligation. So the amount reported for the asset retirement obligation in this year’s balance sheet would be the starting amount of $257,000 from last year’s financial statements less $87,000 paid towards settlement of previously recorded AROs plus the $68,000 cash flow estimate for newly acquired AROs plus the $26,000 accretion expense. $257,000 - $87,000 + $68,000 + $ 26,000 = $264,000 .

How well did you know this?
1
Not at all
2
3
4
5
Perfectly
234
Q

Which of the following is true regarding depreciation?
A
Depreciation accounting recognizes only the physical decline of an asset.
B
Property, plant, and equipment are reduced by depreciation, and then, written up to reflect appraisal, market, or current values above cost.
C
The depreciation process matches the depreciable cost of the asset with revenues generated from its use.
D
None of the above.

A

C
Explanation:
Depreciation is the process of allocating the depreciable cost of fixed assets over their estimated useful lives in a systematic and rational manner. This process matches the depreciable cost of the asset with revenues generated from its use.

How well did you know this?
1
Not at all
2
3
4
5
Perfectly
235
Q

When should a long-lived asset be tested for recoverability?
A
When external financial statements are being prepared.
B
When events or changes in circumstances indicate that its carrying amount may not be recoverable.
C
When the asset’s carrying amount is less than its fair value.
D
When the asset’s fair value has decreased, and the decrease is judged to be permanent.

A

B
Explanation:
Recoverability is associated with an impairment loss. An impairment loss is recognized only if the carrying amount of a long-lived asset is not recoverable and exceeds its fair value. The carrying amount (book value) is not recoverable if it exceeds the sum of the undiscounted cash flows expected to result from the use and eventual disposition of the asset. That assessment shall be based on the carrying amount of the asset at the date it is tested for recoverability. A long-lived asset (asset group) shall be tested for recoverability whenever events or changes in circumstances indicate that its carrying amount may not be recoverable.

How well did you know this?
1
Not at all
2
3
4
5
Perfectly
236
Q
On July 1, one of Rudd Co.'s delivery vans was destroyed in an accident. On that date, the van's carrying amount was $2,500. On July 15, Rudd received and recorded a $700 invoice for a new engine installed in the van in May, and another $500 invoice for various repairs. In August, Rudd received $3,500 under its insurance policy on the van, which it plans to use to replace the van. What amount should Rudd report as gain (loss) on disposal of the van in its year-end income statement?
A
$1,000
B
$ 300
C
$0
D
$ (200)
A

B

Explanation:
A gain or loss on the involuntary conversion (e.g., casualty, condemnation, theft) of a nonmonetary asset should be recognized in income even if the proceeds received as a result of the involuntary conversion are reinvested in a replacement nonmonetary asset. The normal maintenance performed on the van (i.e., the various repairs) should not be capitalized. Normal maintenance does not enhance the service potential of the van, it serves only to maintain a given level of services from the van. On the other hand, the cost of the new engine should be capitalized because the expenditure for the new engine is nonrecurring in nature, enhances the service potential of the van, and is expected to yield benefits over a number of accounting periods.

Insurance proceeds $3,500
Carrying amount of van at date of conversion:
Carrying amount, 7/1 $2,500
Cost of new engine installed prior to involuntary conversion 700 3,200
Gain recognized on involuntary conversion $ 300

How well did you know this?
1
Not at all
2
3
4
5
Perfectly
237
Q
Sea Manufacturing Corp. is constructing a new factory building. During the current calendar year, Sea made the following payments to the construction company:
January 2	$1,000,000
December 31	1,000,000Sea has an 8%, three-year construction loan of $3,000,000. What is the amount of interest costs that Sea may capitalize during the current year?
A
$0
B
$80,000
C
$160,000
D
$240,000
A

Explanation:
The cost of assets constructed for the use of the business should include all directly related costs, such as direct materials, direct labor, and additional overhead incurred. Interest costs incurred during the construction period must be capitalized. The amount of interest cost to be capitalized is the interest cost incurred during the acquisition period that could have been avoided if expenditures for the asset had not been made. If a specific interest rate is associated with the asset, that rate should be used to the extent that the average accumulated expenditures on the asset do not exceed the amount borrowed at the specific rate. The interest rate is applied to the average amount of accumulated expenditures for the asset during the period. The average accumulated expenditure is computed as [($0 + $2,000,000) /2] = $1,000,000. Then $1,000,000 × 8% interest = $80,000 capitalized interest cost for the current year. The total interest cost capitalized in a period may not exceed the total interest cost incurred during that period. With an 8% three-year construction loan of $3,000,000 Sea incurs $80,000 of interest costs each year.

How well did you know this?
1
Not at all
2
3
4
5
Perfectly
238
Q

What factor must be present to use the units-of-production (activity) method of depreciation?
A
Total units to be produced can be estimated.
B
Production is constant over the life of the asset.
C
Repair costs increase with use.
D
Obsolescence is expected.

A

A

Explanation:
The units-of-production depreciation method allocates the cost of plant assets on the basis of units produced (i.e., activity). Depreciation for a period is computed by multiplying the number of units produced during the period by the amount of depreciation per unit. The amount of depreciation per unit is determined by dividing the depreciable base of the plant asset (i.e., cost minus estimated salvage value) by the estimated number of total units to be produced. Thus, an estimate of total units to be produced must be available to use this method.

How well did you know this?
1
Not at all
2
3
4
5
Perfectly
239
Q
On January 2 of the current year, Cruises, Inc. borrowed $3 million at a rate of 10% for three years and began construction of a cruise ship. The note states that annual payments of principal and interest in the amount of $1.3 million are due every December 31. Cruises used all proceeds as a down payment for construction of a new cruise ship that is to be delivered two years after start of construction. What should Cruise report as interest expense related to the note in its income statement for the second year?
A
$0
B
$300,000
C
$600,000
D
$900,000
A

A

Explanation:
None of the interest is expensed; it is all capitalized. Assets qualifying for interest capitalization include assets constructed or produced for self-use on a repetitive basis, assets acquired for self-use through arrangements requiring down payments or progress payments, and assets constructed or produced as discrete projects for sale or lease (e.g., ships or real estate developments).

How well did you know this?
1
Not at all
2
3
4
5
Perfectly
240
Q

On January 1 ten years ago, Andrew Co. created a subsidiary for the purpose of buying an oil tanker depot at a cost of $1,500,000. Andrew expected to operate the depot for 10 years, at which time it is legally required to dismantle the depot and remove underground storage tanks. It was estimated that it would cost $150,000 to dismantle the depot and remove the tanks at the end of the depot’s useful life. However, the actual cost to demolish and dismantle the depot and remove the tanks in the tenth year is $155,000. What amount of expense should Andrew recognize in its financial statements in year 10?

A
None, recognized in prior years
B
$5,000 expense
C
$150,000 expense
D
$155,000 expense
A

B
Explanation:
Obligations for dismantlement, restoration, and abandonment costs are accounted for as asset retirement obligations. Upon initial recognition of a liability for an asset retirement obligation, an entity capitalizes an asset retirement cost by increasing the carrying amount of the related long-lived asset by the same amount as the liability. An entity subsequently allocates that asset retirement cost to expense using a systematic and rational method over its useful life. Application of a systematic and rational allocation does not preclude an entity from capitalizing an amount of asset retirement cost and allocating an equal amount to expense in the same accounting period. The company increased the carrying amount of the depot the $150,000 cost to dismantle and amortized it over the 10 years. In this case, only the extra $5,000 ($155,000 actual cost - $150,000 estimate) would be expensed in year 10.

Editor’s Note: The AICPA provided item (b) as the unofficial solution. The editors believe the company could have, and perhaps should have, chosen to expense the $150,000 evenly over 10 years in what is termed an accretion expense. If it had done so, the expense in year 10 would have been $20,000 (the $15,000 original allocation plus the additional $5,000 in actual cost). Given that $20,000 was not an answer option, one must assume that the company did not do so. Candidates need to prepare to answer questions with the best solution choice possible.

How well did you know this?
1
Not at all
2
3
4
5
Perfectly
241
Q
Turtle Co. purchased equipment on January 2, year 1, for $50,000. The equipment had an estimated five-year service life. Turtle's policy for five-year assets is to use the 200% double-declining depreciation method for the first two years of the asset's life, and then switch to the straight-line depreciation method. In its December 31, year 3, balance sheet, what amount should Turtle report as accumulated depreciation for equipment?
A
$30,000
B
$38,000
C
$39,200
D
$42,000
A

B
Explanation:
On December 31, year 3 Turtle reports $32,000 + $6,000 = $38,000 as accumulated depreciation. Double-declining-balance is computed using twice the straight-line rate, which in this case is 40% (1/5 = 0.20; 0.20 x 2 = .40). In year 3, depreciation is computed by dividing the remaining life of 3 years = $6,000.

Book value beginning of year Rate Depreciation Expense Accumulated Depreciation Book Value End of Year
Year 1 $50,000 40% $20,000 $20,000 $30,000
Year 2 30,000 40% 12,000 32,000 18,000

How well did you know this?
1
Not at all
2
3
4
5
Perfectly
242
Q

Oak Co., a newly formed corporation, incurred the following expenditures related to land and building:
County assessment for sewer lines $ 2,500
Title search fees 625
Cash paid for land with a building to be demolished 135,000
Excavation for construction of basement 21,000
Removal of old building $21,000 less salvage of $5,000 16,000At what amount should Oak record the land?
A
$138,125
B
$153,500
C
$154,125
D
$175,625

A

C
Explanation:
Assets are to be recorded at their acquisition cost. Acquisition cost is defined as the cash price, or equivalent, plus all other costs reasonably necessary to make it ready for it’s intended use. The land is recorded at the $135,000 cash paid for the land plus the additional costs: the $2,500 for county assessment for sewer lines, the $625 for title search fees, and the $16,000 for removal of old building less salvage value ($21,000 - $5,000) = $154,125. The $21,000 excavation cost for construction of a basement is part of the cost of the new building, not the land.

How well did you know this?
1
Not at all
2
3
4
5
Perfectly
243
Q

Newt Co. sold a warehouse and used the proceeds to acquire a new warehouse. The excess of the proceeds over the carrying amount of the warehouse sold should be reported as a(an):

A
Reduction of the cost of the new warehouse.
B
Gain from discontinued operations, net of income taxes.
C
Part of continuing operations.
D
Gains from discontinued operations, gross of income taxes.

A

Explanation:
The correct answer is (C).

The sale of the warehouse by the company is in the ordinary course of business and its proceeds are used to acquire a new warehouse.

Therefore, the gain from the proceeds over the carrying value of the warehouse will be reported in the income from continuing operations under non-operating Items.

Option (A) is incorrect because the gain from the sale of the warehouse cannot be used to reduce the cost basis of the new warehouse.

Option (B) and (D) is incorrect because the sale was part of continuing operation, not from discontinued operations.

How well did you know this?
1
Not at all
2
3
4
5
Perfectly
244
Q
On January 1, Year 1, Bay Co. acquired a land lease for a 21-year period with no option to renew. The lease required Bay to construct a building in lieu of rent. The building, completed on January 1, Year 2, at a cost of $840,000, will be depreciated using the straight-line method. At the end of the lease, the building's estimated market value will be $420,000. What is the building's carrying amount in Bay's December 31, Year 2, balance sheet?
A
$798,000
B
$800,000
C
$819,000
D
$820,000
A

A
Explanation:
The building constructed on the leased property in lieu of rent represents a leasehold improvement because the building will revert to the lessor at the end of the lease term. The $840,000 cost of the building is allocated equally over the remaining 20-year period of the lease, resulting in annual amortization of $42,000. Thus, the building’s carrying amount at the end of the building’s first year is $798,000 ($840,000 cost minus $42,000 amortization to date).

How well did you know this?
1
Not at all
2
3
4
5
Perfectly
245
Q

A company using the composite depreciation method for its fleet of trucks, cars and campers, retired one of its trucks and received cash from a salvage company. The net carrying amount of these composite asset accounts would be decreased by the
A
Cash proceeds received and original cost of the truck.
B
Cash proceeds received.
C
Original cost of the truck less the cash proceeds.
D
Original cost of the truck.

A

B

Explanation:
Under the composite or group method of depreciation, no gain or loss is recognized upon the retirement of a plant asset. This practice is justified because some assets will be retired before the average service life and others after the average service life. Accumulated depreciation is debited for the difference between original cost and the cash received; no gain or loss is recorded on the disposition. The net carrying amount of these composite asset accounts is decreased by the cash proceeds received of $3,000 (cost removed of $18,000 minus accumulated depreciation removed of $15,000).

Cash 3,000
Accumulated Depreciation 15,000
Truck 18,000To record sale of truck

How well did you know this?
1
Not at all
2
3
4
5
Perfectly
246
Q
Spiro Corp. uses the sum-of-the-years'-digits method to depreciate equipment purchased in January of the current year for $20,000. The estimated salvage value of the equipment is $2,000 and the estimated useful life is four years. What should Spiro report as the asset's carrying amount as of December 31 in the third year?
A
$1,800
B
$2,000
C
$3,800
D
$4,500
A

C

Explanation:
Asset Cost		$20,000
Year 1 [(4/10)($20,000 - 2,000)]	$7,200	
Year 2 [(3/10)($20,000 - 2,000)]	5,400	
Year 3 [(2/10)($20,000 - 2,000)]	3,600	(16,200)
Carrying amount 12/31, year 3		$ 3,800
How well did you know this?
1
Not at all
2
3
4
5
Perfectly
247
Q

Under IFRS, when an entity chooses the revaluation model as its accounting policy for measuring property, plant and equipment, which of the following is correct?
A
When an asset is revalued, the entire class of property, plant and equipment to which the asset belongs must be revalued.
B
When an asset is revalued, individual assets within a class of property, plant and equipment to which that asset belongs can be revalued.
C
Revaluations of property, plant and equipment must be made at least every three years.
D
Increases in an asset’s carrying value as a result of the first revaluation must be recognized as a component of profit or loss.

A

A

Explanation:
Under IFRS, revaluation is a permitted accounting policy if fair value can be reliably measured (i.e., the revaluation model).

How well did you know this?
1
Not at all
2
3
4
5
Perfectly
248
Q
At the beginning of the year, Cann Co. started construction on a new $2 million addition to its plant. Total construction expenditures made during the year were $200,000 on January 2, $600,000 on May 1, and $300,000 on December 1. On January 2, the company borrowed $500,000 for the construction at 12%. The only other outstanding debt the company had was a 10% interest rate, long-term mortgage of $800,000, which had been outstanding the entire year. What amount of interest should Cann capitalize as part of the cost of the plant addition?
A
$140,000
B
$132,000
C
$ 72,500
D
$ 60,000
A

C

Explanation:
The cost of assets constructed for the use of the business should include all directly related costs; cost of direct materials, cost of direct labor, additional overhead incurred, and interest costs incurred during the construction period. If the average accumulated expenditures of an asset exceed the amount of any specific borrowings associated with the asset, the excess should be capitalized at the weighted average of interest rates applicable to other borrowings of the business.

Average expenditure during year	
$200,000 x 12/12	$ 200,000
600,000 x 8/12	400,000
300,000 x 1/12	25,000
Average expenditures	$ 625,000
Specific borrowings	500,000 x 12%	$ 60,000
Excess expenditures	125,000 x 10%	12,500
Total capitalized interest		$ 72,500
How well did you know this?
1
Not at all
2
3
4
5
Perfectly
249
Q
Birk Co. purchased 30% of Sled Co.'s outstanding common stock on December 31 of the current year for $200,000. On that date, Sled's stockholders' equity was $500,000, and the fair value of its identifiable net assets was $600,000. On December 31, what amount of goodwill should Birk attribute to this acquisition?
A
$0
B
$20,000
C
$30,000
D
$50,000
A

B

Explanation:
Goodwill is recognized and recorded at an amount equal to the excess of the cost of the enterprise acquired over the fair value of the identifiable net assets (INA).

Purchase price of 30% of Sled Co’’s O/S common stock $200,000
Fair value of Sled’s INA $600,000
Times: Percentage acquired by Birk x 30%
Less: Sled’s INA acquired by Birk (180,000)
Goodwill $ 20,000

How well did you know this?
1
Not at all
2
3
4
5
Perfectly
250
Q

Lee Corp. reported the following marketable debt security on its December 31 previous year balance sheet, classified as an available-for-sale debt security:

Neu Corp. Bond, at cost $100,000
Market adjustment to reflect decline in fair value (20,000)
Balance $ 80,000
On December 31 of the current year, the fair value of Lee’s investment in the Neu Corp. stock was $85,000. Assuming that no credit losses are expected, as a result of the current year increase in the debt security’s fair value, Lee’s current year income statement should report

A
An unrealized gain of $5,000
B
A realized gain of $5,000
C
An unrealized loss of $15,000
D
No gain or loss
A

Explanation:
The correct answer is (D).

On 12/31 of the previous year, the required balance of the Market Adjustment–AFS account was a $20,000 credit ($100,000 - $80,000). On 12/31 of the current year, the required balance of the Market Adjustment–AFS account is a $15,000 credit ($100,000 - $85,000). Therefore, in the current year, the required balance of the Market Adjustment–AFS account decreased by $5,000. Changes in the Market Adjustment account related to the available-for-sale category are included in other comprehensive income. Lee does not recognize any gain or loss from a change in the Market Adjustment–AFS account on its income statement.

How well did you know this?
1
Not at all
2
3
4
5
Perfectly
251
Q

On January 1, Year 1, Fring Chicken Co. purchased a bond with a face value of $1000, maturing after 7 years, and classified as an available-for-sale debt security on its balance sheet. The coupon rate on the bond was stated to be 8%, payable semi-annually on June 30 and December 31. On January 1, Year 5, the fair value of the said bond was estimated to be $650. What is the impact of the loss on the financial statements of Fring? Assume a discount rate of 10%.

Given:

Present Value of $1 at 5% and n=14: 0.505
Ordinary Annuity of $1 at 5% and n=14: 9.8986
Present Value of $1 at 5% and n=6: 0.746
Ordinary Annuity of $1 at 5% and n=6: 5.0757
Income Statement Other Comprehensive Income
A 0 350
B 350 0
C 96 203.14
D 203.14 96

A

Explanation:
The correct answer is (D).

Impairment for Available-for-Sale Securities is calculated as the difference between Amortized Cost (i.e. Carrying Value) and the Fair Value of the security. However, Credit Losses on the Income Statement are limited to Amortized Cost – Present Value (calculated the same way as a Held-to-Maturity investment) because if the unrealized loss at any given time is more than the expected credit loss till maturity, the Investor can minimize the loss by holding the security.

Excess losses are charged to OCI.

Carrying Value of the Bond on January 1, Year 1:

⇒ $1,000 x Present Value of $1 at 5% for 14 periods + $40 x Present Value of Ordinary Annuity $1 at 5% for 14 periods
⇒ $1,000 x 0.505 + $40 x 9.8986
⇒ $505 + $395.94
⇒ $900.94

Carrying Value of the Bond on January 1, Year 5 (after 8 Periods):

Period	Interest Income (5%)	Interest Received (4%)	Interest Received (4%)	Carrying Value
0	 	 	 	$900.94
1	$45.05	40	$5.05	$905.99
2 	$45.30 	40 	$5.30 	$911.29 
3 	$45.56 	40 	$5.56 	$916.86 
4 	$45.84  	40 	$5.84  	$922.70  
5 	$46.13  	40 	$6.13  	$928.83  
6 	$46.44  	40 	$6.44  	$935.27  
7 	$46.76  	40 	$6.76  	$942.04  
8 	$47.10  	40 	$7.10  	$949.14 
In the given case, Present value of the bond on January 1, Year 5 (after 8 Periods):
⇒ $1,000 x Present Value of $1 at 5% for 6 years
⇒ 1,000 x 0.746
⇒ $746

Expected Credit Loss = Carrying Value – Fair Value = $949.14 - $650 = $299.14

Expected Credit loss = $949.14 - $746 = $203.14.

Loss Charged to Income Statement = $299.14 - $203.14 = $203.14

Loss Charged to OCI = $96

How well did you know this?
1
Not at all
2
3
4
5
Perfectly
252
Q

Pal Corp.’s current year dividend income included only part of the dividend received from its Ima Corp. investment. The balance of the dividend reduced Pal’s carrying amount for its Ima investment. This reflects that Pal accounts for its Ima investment by the

A
Fair method, and only a portion of Ima’s current year dividends represent earnings after Pal’s acqui-sition
B
Fair value method, and its carrying amount exceeded the proportionate share of Ima’s market value
C
Equity method, and its carrying amount exceeded the proportionate share of Ima’s market value
D
Equity method, and Ima incurred a loss in the current year

A

A
Explanation:
Dividend income is not recognized under the equity method of accounting for investments in com­mon stock. Under this method, the investor’s share of dividends declared by the investee reduce the carrying amount of the investment. Pal recognized dividend income from the investment in the current year. Hence it must account for the investment by the fair method. Under this method, investment income reported for the year is usually the investor’s share of dividends declared by the investee during the year. The exception is where the investor’s share of dividends declared by the investee exceeds the investor’s share of investee earnings subse­quent to the date of the investment. In this case, the excess amount represents a liquidating dividend that is recorded as a reduction of the carrying amount of the investment and not as dividend income. Since only part of the dividends that Pal received in the current year was recorded as dividend income, only a portion of the current year dividends represent earnings subsequent to the date of Pal’s investment (i.e., the balance of the current year dividends represent liquidating dividends).

Options (B), (C) and (D) are incorrect as per above explanation.

How well did you know this?
1
Not at all
2
3
4
5
Perfectly
253
Q

A marketable debt security is transferred from the trading portfolio to the available-for-sale portfolio. At the transfer date, the security’s cost exceeds its market value. What amount is used at the transfer date to record the security in the available-for-sale debt portfolio?

A
Market value, regardless of whether the decline in market value below cost is considered permanent or temporary.
B
Market value, only if the decline in market value below cost is considered permanent.
C
Cost, if the decline in market value below cost is considered temporary.
D
Cost, regardless of whether the decline in market value below cost is considered permanent or temporary.

A

A

Explanation:
If there is a change in the classification of a marketable debt security between trading and available-for-sale, the security is transferred between the corresponding portfolios at the fair value at the date of transfer. If fair value is less than cost, the fair value becomes the new basis, and the difference is accounted for as if it were a realized loss and included in the determination of net income.

How well did you know this?
1
Not at all
2
3
4
5
Perfectly
254
Q

Moss Corp. owns 20% of Dubro Corp.’s preferred stock and 40% of its common stock. Dubro’s stock outstanding on December 31, 20X1, is as follows:

10% cumulative preferred stock $100,000
Common stock $700,000
Dubro reported net income of $60,000 for the year ending December 31, 20X1. What amount should Moss record as equity in earnings of Dubro for the year ending December 31, 20X1?

A
$22,000
B
$24,000
C
$24,200
D
$25,000
A

A

Explanation:
When calculating an investor’s income share, deduct the cumulative preferred dividends from net income first..

Moss’ share of preferred dividends
= 20% x 10% x $100,000
= $2,000

Earnings available to common shareholders
= $60,000 - 10% x $100,000
= $50,000

Moss’ share of common earnings
= 40% x $50,000
= $20,000

Moss’ total equity in Dubro earnings
= $2,000 + $20,000
= $22,000

FASB ASC 323-10-35-16 states: “If an investee has outstanding cumulative preferred stock, an investor shall compute its share of earnings (losses) after deducting the investee’s preferred dividends, whether or not such dividends are declared.”

Option (B), (C) and (D) are incorrect as per above explanation.

How well did you know this?
1
Not at all
2
3
4
5
Perfectly
255
Q

Which of the following factors would not be an indicator of an investor’s ability to exercise significant influence over the operating and financial policies of an investee?

A
Investor recommendation for the investee to hire a specific executive
B
Interchange of managerial personnel between investor and investee
C
Investor representation on the investee board of directors
D
Dependence by the investee on the investor’s proprietary technology

A

Explanation:
The correct answer is (A).

Investee has a significant technological or transactional dependency on the investor.
Investor has representation on the board of directors of the investee company.
Investor is a major customer or supplier of the investee.
Material inter-company transactions between the investor and investee company.
Investor company’s participation in the operating or financial policy-making decisions of the investee company.
Interchange of managerial personnel between the investor and investee.
Investor recommendation for the investee to hire a specific executive does not imply exercising significant influence.

How well did you know this?
1
Not at all
2
3
4
5
Perfectly
256
Q

At year-end, Rim Co. held several investments with the intent of selling them in the near term. The investments consisted of $100,000, 8%, five-year bonds, purchased for $92,000, and equity securities purchased for $35,000. At year-end, the bonds were selling on the open market for $105,000 and the equity securities had a market value of $50,000. What amount should Rim report as trading securities in its year-end balance sheet?

A
$ 50,000
B
$105,000
C
$92,000
D
$155,000
A

Explanation:
The correct answer is (B).

Trading securities are the debt securities bought and held principally for the purpose of selling them in the near term. Trading debt securities are carried at market value. Rim Co. investments in bonds will be classified as trading debt securities and reported on the balance sheet at $105,000.

(A) is incorrect because investment in equity securities should be classified as “Fair value through net income” if the market value of securities is readily determinable. There is no longer a trading or available for sale classification for equity securities.

(C) is incorrect because it uses cost instead of market value to report the debt securities purchased.

(D) is incorrect because it includes the market value of both debt and equity securities.

How well did you know this?
1
Not at all
2
3
4
5
Perfectly
257
Q
Cobb Co. purchased 10,000 shares (2% ownership) of Roe Co. on February 12 of the current year. Cobb received a stock dividend of 2,000 shares on March 31, when the carrying amount per share on Roe's books was $35 and the market value per share was $40. Roe paid a cash dividend of $1.50 per share on September 15. In Cobb's income statement for the year ended October 31, what amount should Cobb report as dividend income?
A
$98,000
B
$88,000
C
$18,000
D
$15,000
A

Explanation:
Cobb owns only a 2% interest in Roe. Thus, Cobb does not have the ability to exercise significant influence over Roe by virtue of the investment, and the investment should be accounted for under the cost method. Therefore, Cobb should report the cash dividend received from Roe as dividend income. No income is recognized from the receipt of the stock dividend from Roe, since Cobb’s proportionate interest in Roe has not changed and Roe’s underlying assets and liabilities have also not changed.

Shares of Roe purchased 2/12 10,000
Add: Shares of Roe received from stock dividend, 3/31 2,000
Shares of Roe held, 9/15 12,000
Times: Cash dividend per share x 1.50
Dividend income, year ended 10/31 $18,000

How well did you know this?
1
Not at all
2
3
4
5
Perfectly
258
Q

At the beginning of the fiscal year, End Corp. purchased 25% of Turf Co. for $550,000. At the end of the fiscal year, Turf reported net income of $65,000 and declared and paid cash dividends of $30,000. End uses the equity method of accounting. At year end, what amount should End report in its balance sheet for the investment in Turf?

A
$550,000
B
$558,750
C
$566,250
D
$573,750
A

B

Explanation:
End Corp. has a holding of 25% in Turf Co. and uses the equity method to report its investment in Turf Co.

Initial purchase amount $ 550,000
Share of income ($65,000 × 25%) 16,250
Share of dividends ($30,000 × 25%) __(7,500)
Escrow liability end of year $ 558,750
Option (A) is incorrect because it is the cost of the initial investment, which should be adjusted for proportionate share in net income which would increase the investment account and dividends received at proportionate share of 25% which would reduce the investment account.

Option (C) is incorrect because it should also adjust for dividends received at 25% proportionate share.

Option (D) is incorrect because dividends received is added to the investment; instead of deducting it.

How well did you know this?
1
Not at all
2
3
4
5
Perfectly
259
Q
Puff Co. acquired 40% of Straw, Inc.'s voting common stock on January 2 of the current year for $400,000. The carrying amount of Straw's net assets at the purchase date totaled $900,000. Fair values equaled carrying amounts for all items except equipment, for which fair values exceeded carrying amounts by $100,000. The equipment has a five year life. During the year, Straw reported net income of $150,000. What amount of income from this investment should Puff report in its year-end income statement?
A
$40,000
B
$52,000
C
$56,000
D
$60,000
A

xplanation:
If fair value exceeds the net assets’ carrying value, the excess is allocated among most undervalued assets. Then consideration is given to the existence of goodwill. As the related undervalued asset is depreciated or sold, the investment account is adjusted. Straw’s fair value is $900,000 + $100,000 = $1,000,000. Puff’s investment in Straw is 40% x $1,000,000 = $400,000, the same as the purchase price. Puff offsets its share of Straw’s income with the depreciation of its share of the difference in the carrying and fair value of equipment. [$150,000 - ($100,000 / 5 years)] x 40% = $52,000.

How well did you know this?
1
Not at all
2
3
4
5
Perfectly
260
Q
On January 1 of the current year, Lean Co. made an investment of $10,000. The following is the present value of $1.00 discounted at a 10% interest rate:
Periods	Present value of $1.00 Discounted at 10%
1	0.909
2	0.826
3	0.751What amount of cash will Lean accumulate in two years?
A
$12,000
B
$12,107
C
$16,250
D
$27,002
A

B
Explanation:
The future value factor is equal to 1 divided by the present value factor. An investment in two years would accumulate to the principal multiplied by the future value factor. In this case the $10,000 x 1/0.826 = $12,107.

Editors Note: This question appears exactly as it was received from the AICPA. The AICPA provided $12,107 (derived as explained) as the correct answer. The question stated that Lean Co. made an investment on $10,000 on Jan 1 of the current year and provided no further details. Also provided is a three year table for the present value of $1.00 discounted at a 10% interest rate. In lieu of other information, you have to assume the prevailing rate of interest is 10% and you are to use the table provided.

How well did you know this?
1
Not at all
2
3
4
5
Perfectly
261
Q

Recording a credit loss on a debt security will result in which of the following?

A
Lower retained earnings
B
Increase in retained earnings
C
Higher working capital
D
Increased current assets
A

Explanation:
The correct answer is (A).

Recording expected credit losses cause a decrease in the earnings of an entity since the credit loss expense is reflected in the Income Statement.

Consequently, the indirect effect is lower retained earnings due to a lower net income.

How well did you know this?
1
Not at all
2
3
4
5
Perfectly
262
Q
When the equity method is used to account for investments in common stock, which of the following affect(s) the investor's reported investment income?
A change in market value
of investee's common stock	Cash dividends
from investee
A	Yes	Yes
B	Yes	No
C	No	Yes
D	No	No
A

D

Explanation:
Under the equity method of accounting for investments in common stock, the investment is recorded at cost. Changes in the market value of the investee’s common stock do not affect the Investment account or the Investment Income account. The investor recognizes as income its share of the investee’s earnings or losses in the periods in which they are reported by the investee. Dividends declared by the investee represent a distribution of earnings previously recognized and, thus, do not affect the Investment Income account.

How well did you know this?
1
Not at all
2
3
4
5
Perfectly
263
Q

On March 1, Year 1, Max Inc. bought $200,000 face value 12% US treasury bonds for $200,000 which includes accrued interest. The interest on these is payable semi-annually on June 30 and December 31. These bonds are held to maturity, which is June 30, Year 2. What will be the carrying value of the bond as on December 31, year 1 if the straight-line method of amortization is used?

A
$198,500
B
$196,000
C
$193,500
D
$200,000
A

Explanation:
The correct answer is (A).

The bonds purchased included accrued interest.

The interest component in the purchase price of the bond will be the interest for two months (January and February, year 1), which will be $4,000 (i.e. $200,000 x 12% x 2 /12).

This means that the purchase price of the bond is $196,000 (i.e. $200,000 - $4,000).

The discount of $4,000 will be amortized for 16 months (from March 1, year 1 to June 30, year 2).

The discount amortized for 10 months, which will be added to the carrying value of the bond for year 1, will be $2,500 (i.e. $4,000 x 10 /16).

The carrying value of the bond at the end of year 1 will be $198,500 (i.e. $196,000 + $2,500).

(b) is incorrect because this will be the result when the discount is not amortized and the bond is reported at the purchase price, which is $196,000.
(c) is incorrect because this will be the result when discount amortized is reduced from the purchase price of the bond, which will be $193,500 (i.e. $196,000 - $2,500).
(d) is incorrect because this will be the result when accrued interest is completely ignored and the discount is not considered.

How well did you know this?
1
Not at all
2
3
4
5
Perfectly
264
Q

Janson traded bonds in Flax Co. held as trading debt securities during year 1 as follows:

       Number of Bonds purchased (sold)

          Price per Bond

February 3, year 1

1,100

$11

April 15, year 1

2,500

9

May 28, year 1

(750)

13

July 5, year 1

1,400

12

September 30, year 1

(4,000)

15

No other transactions took place for Flax during the remainder of the year. At December 31, year 1, Flax is trading at $10 per bond. Janson trades debt securities on a last-in, first-out basis. What amount is the net value of the investment in Flax at year-end?

A
$250
B
$2,500
C
$2,750
D
$3,750
A

Explanation:
The correct answer is (B).

Trading securities are the debt securities bought and held principally for the purpose of selling them in the near term. The trading securities are carried at market value. Flax Co. investments which are held as trading securities and reported on the balance sheet at $2,500 (i.e. 250 per bond x $10).

(A) is incorrect because 250 bonds are valued at $1 instead of valuing at a market price of $10.

(C) is incorrect because 250 bonds are valued at $11 instead of valuing at market price of $10 (i.e. $2,750 = 250 x $11).

(D) is incorrect because 250 bonds are valued at $15 instead of valuing at market price of $10 (i.e. $3,750 = 250 x $15).

How well did you know this?
1
Not at all
2
3
4
5
Perfectly
265
Q

On January 2 of the current year, Emme Co. sold equipment with a carrying amount of $480,000 in exchange for a $600,000 noninterest bearing note due on January 2 in three years. There was no established exchange price for the equipment. The prevailing rate of interest for a note of this type at January 2 of the current year was 10%. The present value of 1 at 10% for three periods is 0.75.

In Emme’s current year income statement, what amount should be reported as gain (loss) on sale of machinery?

A
($ 30,000) loss.
B
$ 30,000 gain.
C
$120,000 gain.
D
$270,000 gain.
A

A
Explanation:
A noninterest-bearing note exchanged for property, goods, or services should not be recorded at its face amount. Since there is not an established exchange price for the equipment and the question does not indicate the fair value of the note, the note should be recorded at its present value, which is computed by discounting all future payments of the note at the prevailing rate of interest for a note of this type.

Present value of note ($600,000 x .75) $ 450,000
Carrying amount of machinery (480,000)
Loss on sale of machinery $ (30,000)

How well did you know this?
1
Not at all
2
3
4
5
Perfectly
266
Q
On July 1 of the current year, Denver Corp. purchased 3,000 shares of Eagle Co.'s 10,000 outstanding shares of common stock for $20 per share. On December 15, Eagle paid $40,000 in dividends to its common stockholders. Eagle's net income for the year ended December 31 was $120,000, earned evenly throughout the year. In its year-end income statement, what amount of income from this investment should Denver report?
A
$36,000
B
$18,000
C
$12,000
D
$ 6,000
A

Explanation:
This investment should be accounted for under the equity method because Denver’s purchase of 30% (i.e., 3,000 / 10,000) of Eagle’s common stock on 7/1 gives Denver the ability to exercise significant influence over the operating and financial policies of Eagle by virtue of the investment. Denver should recognize investment income only for its share of Eagle’s net income subsequent to the date of the investment. While dividends declared by Eagle reduce the carrying amount of the investment, they do not affect the amount of investment income that Denver recognizes. Therefore, in the year, Denver should report income from the equity method investment of $18,000 (i.e., $120,000 x 6/12 x 30%).

How well did you know this?
1
Not at all
2
3
4
5
Perfectly
267
Q

In its financial statements, Pare, Inc. uses the cost method of accounting for its 15% ownership of Sabe Co. At December 31, Pare has a receivable from Sabe. How should the receivable be reported in Pare’s December 31 balance sheet?
A
The total receivable should be reported separately.
B
The total receivable should be included as part of the investment in Sabe, without separate disclosure.
C
Eighty-five percent of the receivable should be reported separately, with the balance offset against Sabe’s payable to Pare.
D
The total receivable should be offset against Sabe’s payable to Pare, without separate disclosure.

A

A
Explanation:
The total receivable from Sabe should be separately reported in Pare’s financial statements since Sabe is not a subsidiary of Pare (i.e., Pare has only a 15% interest in Sabe). Pare would have to own a majority voting interest (i.e., > 50%) in Sabe in order for Sabe to be considered a subsidiary.

How well did you know this?
1
Not at all
2
3
4
5
Perfectly
268
Q

On January 1 of the current year, Point, Inc. purchased 10% of Iona Co.’s common stock. Point purchased additional shares bringing its ownership up to 40%
of Iona’s common stock outstanding on August 1. During October, Iona declared and paid a cash dividend on all of its outstanding common stock. How much
income from the Iona investment should Point’s year-end income statement report?

A
10% of Iona’s income for January 1 to July 31, plus 40% of Iona’s income for August 1 to December 31
B
40% of Iona’s income for August 1 to December 31 only
C
40% of Iona’s total year income
D
Amount equal to dividends received from Iona

A

A
Explanation:
On 1/1, Point purchased 10% of Iona’s common stock. On 8/1, when Point increased its investment in Iona’s common stock from 10 percent to 40 percent,
Point gained the ability to exercise significant influence over the financial and operating policies of Iona and accordingly should report its investment using
the equity method. The change from the cost method of reporting the investment to the equity method should be made by prospectively.

How well did you know this?
1
Not at all
2
3
4
5
Perfectly
269
Q

In year 1, Flynn estimated its credit loss on an available-for-sale debt security to be $900. In the subsequent year, the estimated loss was found to be only $600. How should this change be accounted for?

A
Increasing the value of the debt security by $300
B
Decreasing the loss allowance by $300
C
Writing off the original allowance and creating a new allowance of $600
D
Accounting it as a prior period event
A

Explanation:
The correct answer is (B).

Any decrease in expected loss shall call for a decrease in the allowance, thereby causing a reversal of the credit loss expense on the income statement.

The decrease in the loss of $300 should be accounted for by decreasing the pre-existing loss allowance.

How well did you know this?
1
Not at all
2
3
4
5
Perfectly
270
Q

Band Co. uses the equity method to account for its investment in Guard, Inc. common stock. How should Band record a 2% stock dividend received from Guard?
A
As dividend revenue at Guard’s carrying value of the stock.
B
As dividend revenue at the market value of the stock.
C
As a reduction in the total cost of Guard stock owned.
D
As a memorandum entry reducing the unit cost of all Guard stock owned.

A

Explanation:
Stock dividends are recorded as memorandum entries only, reducing the unit cost of the stock owned. No dividend revenue is recorded and the total cost of the stock owned remains the same.

How well did you know this?
1
Not at all
2
3
4
5
Perfectly
271
Q

A company has a 22% investment in another company that it accounts for using the equity method. Which of the following disclosures should be included in the company’s annual financial statements?

A
The names and ownership percentages of the other stockholders in the investee company.
B
The reason for the company’s decision to invest in the investee company.
C
The company’s accounting policy for the investment.
D
Whether the investee company is involved in any litigation.

A

Explanation:
When an entity accounts for an investment in common stock under the equity method, it is required to disclose the names of each of its investee with its ownership percentage in the investee companies, the company’s accounting policies for investment in common stock, and the difference between the carrying value of the investment and the entity’s underlying equity in the investee’s net assets and the accounting treatment of the difference. Entity is not required to disclose the reason for having made the investment, the names and ownership percentage of the other stockholders in the investee company, or whether the investee company is involved in any litigation.

Options (A), (B) and (D) are incorrect based on the above explanation.

How well did you know this?
1
Not at all
2
3
4
5
Perfectly
272
Q
On January 2 of the current year, Well Co. purchased 10% of Rea, Inc.'s outstanding common shares for $400,000. Well is the largest single shareholder in Rea, and Well's officers are a majority on Rea's board of directors. Rea reported net income of $500,000 for the year and paid dividends of $150,000. In its December 31 balance sheet, what amount should Well report as investment in Rea?
A
$450,000
B
$435,000
C
$400,000
D
$385,000
A

Explanation:
Well should use the equity method to account for its investment in Rea because it can exercise significant influence over the operating and financial policies of Rea.

Purchase price, 1/1 $400,000
Add: Share of Rea’’s income ($500,000 x 10%) 50,000
Less: Share of dividends paid ($150,000 x 10%) (15,000)
Investment carrying amount, 12/31 $435,000

How well did you know this?
1
Not at all
2
3
4
5
Perfectly
273
Q
Leaf Co. purchased from Oak Co. a $20,000, 8%, 5-year note that required five equal annual year-end payments of $5,009. The note was discounted to yield a 9% rate to Leaf. At the date of purchase, Leaf recorded the note at its present value of $19,485. What should be the total interest revenue earned by Leaf over the life of this note?
A
$5,045
B
$5,560
C
$8,000
D
$9,000
A

Explanation:
Total interest revenue earned over the life of the note is determined as the excess of the summation of the required annual year-end payments over the present value of the note.

Summation of required annual year-end payments ($5,009 x 5) $25,045
Less: Present value of note (19,485)
Interest revenue earned over life of note $ 5,560 q

How well did you know this?
1
Not at all
2
3
4
5
Perfectly
274
Q
Park Co. uses the equity method to account for its January 1 current year purchase of Tun, Inc.'s common stock. On this date, the fair values of Tun's FIFO inventory and land exceeded their carrying amounts. How do these excesses of fair values over carrying amounts affect Park's reported equity in Tun's current year earnings?
Inventory excess	Land excess
A	Decrease	Decrease
B	Decrease	No effect
C	Increase	Increase
D	Increase	No effect
A

B
Explanation:
The excess of the fair value of Tun’s FIFO inventory over its carrying amount would decrease Park’’s reported equity in Tun’s earnings and the excess of the fair value of Tun’’s land over its carrying amount would have no effect on Park’s reported equity in Tun’s earnings.

How well did you know this?
1
Not at all
2
3
4
5
Perfectly
275
Q

At the beginning of year 2, a company invested $40,000 in a marketable equity security. At that time the security was appropriately classified security with readily determinable market value. At the end of year 2, the security had a fair value of $28,500. The change in fair value is deemed temporary. How should this change in fair value be reported in the financial statements?

A
As a realized loss of $11,500 as part of net income.
B
As a realized loss of $11,500 as part of other comprehensive income.
C
As an unrealized loss of $11,500 as part of net income.
D
As an unrealized loss of $11,500 as part of other comprehensive income.

A

Explanation:
The correct answer is (C)

As per FASB issued accounting standards update 2016-01, investment in marketable equity securities should be measured at fair value through net income (FVTNI).

All type of changes in fair value of equity securities whether temporary or permanent needs to be routed through net income only. Also, there is no longer classification of equity investments as trading or Available For Sale (AFS), and there is no longer a requirement to recognize unrealized holding gains and losses on equity securities in other comprehensive income as previously required.

(A) is incorrect because loss is not realized and hence should not be reported as realized loss as part of net income.

(B) & (D) are incorrect because unrealized and realized are both types of losses that should be reported as part of net income, not as a part of other comprehensive income.

How well did you know this?
1
Not at all
2
3
4
5
Perfectly
276
Q

At the end of year 1, Lane Co. held trading debt securities that cost $86,000 and which had a year-end market value of $92,000. During year 2, all of these securities were sold for $104,500. At the end of year 2, Lane had acquired additional trading debt securities that cost $73,000 and which had a year-end market value of $71,000. What is the impact of these stock activities on Lane’s year 2 income statement?

A
Loss of $2,000
B
Gain of $10,500
C
Gain of $16,500
D
Gain of $18,500
A

Explanation:
The correct answer is (B).

Trading debt securities that are bought and held principally for the purpose of selling them in the near term.

They are reported at fair value (market value) and any unrealized holding gains and losses are included in current earnings.

At the end of year 1, the trading debt securities would have been valued at $92,000 and a gain of $6,000 ($92,000 year-end market value - $86,000 cost) would have been reported on the income statement.

In year 2, there would be a gain of $12,500 ($104,500 selling price - $92,000 beginning value) from the initial set of securities and a $2,000 loss ($71,000 year-end market value - $73,000 cost) from the newly acquired trading debt securities.

The net impact of these stock activities would be a $10,500 gain on the income statement in year 2.

How well did you know this?
1
Not at all
2
3
4
5
Perfectly
277
Q

On January 2 of the current year, Otto Co. purchased 40% of Penn Co.’s outstanding common stock. The carrying amount of Penn’s depreciable assets was $1,000,000 on January 2. Penn’s depreciable assets had an original useful life of 10 years, and a remaining useful life of five years. Otto recognized $8,000 amortization for the current year ending December 31 related to its investment in Penn due to the excess of fair value over book value on these assets. What was the fair value of Penn’s depreciable assets on January 2 of the current year?

A
$100,000
B
$900,000
C
$1,000,000
D
$1,100,000
A

Explanation:
Otto recognized $8,000 amortization for the current year due to the excess of fair value over book value of these assets. The depreciable assets had a remaining useful live of 5 years, which indicates Otto would recognize a total of $40,000 ($8,000 × 5) amortization over the 5 years. Otto was only responsible for 40% of the total of the excess in fair value over book value, so the total fair value over book value must have been $100,000 ($40,000 ÷ 40%). The total fair value would have been $1,100,000 ($1,000,000 book value + $100,000 excess) on January 2 of the current year.

How well did you know this?
1
Not at all
2
3
4
5
Perfectly
278
Q

Bang Inc. acquired 40% stake in Boom Inc. for $120,000 in the beginning of year 1. None of the other investors have more than 20% stake in Boom Inc. The book value and fair value of Boom Inc. is $200,000 and $250,000 respectively. The difference in the book value and fair value is attributable to higher fair value of equipment by $30,000 and land by $20,000. The equipment is depreciated over next 10 years using straight line method. Goodwill is also impaired by 10% during the year and the land is also sold by Boom Inc. If Boom Inc. declares a dividend of $10,000 out of the total earnings of $80,000, what would be the investment income (or charge) recorded in the statement of income of Bang Inc. for the year concerning the investment in Boom Inc?

A
$28,800
B
$32,800
C
($11,200)
D
$20,800
A

D

Explanation:
Bang Inc. holds more than 40% stake in Boom Inc. and also further no other group of shareholders has any majority ownership and exercise total control, the investment will be accounted using equity method in the books of Bang Inc. Under equity method following journal entries will be passed:

At the time of purchase of investment (Recording investment at cost):

Investment in Boom Inc. $120,000
Cash $120,000
Recording dividend income:

Cash (i.e. 40% of $10,000) $4,000
Investment in Boom Inc. $4,000
Recording percentage of earnings:

Investment in Boom Inc. $32,000
Equity in earnings $32,000
Write off excess purchase price paid over book value of Boom Inc:

Excess of purchase price over book value = $120,000 – (40% x $200,000) = $40,000.

Of $40,000, $20,000 [i.e. 40% of (30,000 + 20,000)] is attributable to higher fair values of land and equipment. Thus, balance $20,000 will be the goodwill. Impairment of goodwill = 10% of $20,000 = $2,000.

Depreciation on increased fair value of equipment (Bang Inc’s share) = 40% x $30,000 / 10 years = $1,200.

No depreciation is charged on land. Land is sold during the year. Bang Inc. will write off its share of increased fair value of land, i.e. $8,000 (i.e. $20,000 x 40%).

The journal entry would be:

Equity in earnings $11,200 (i.e. $2,000 + $1,200 + $8,000)
Investment in Boom Inc. $11,200
Thus, investment income recorded in the statement of income of Bang Inc. concerning investment in Boom Inc. for the year 1 would be $20,800 (i.e. $32,000 - $11,200).

How well did you know this?
1
Not at all
2
3
4
5
Perfectly
279
Q

During the previous year, Wall Co. purchased 2,000 shares of Hemp Corp. common stock for $31,500 as an equity investment. The market value of this investment was $29,500 at December 31 of the previous year. Wall sold all of the Hemp common stock for $14 per share on December 15 of the current year, incurring $1,400 in brokerage commissions and taxes. On the sale, Wall should report a realized loss of

A
$3,500
B
$4,900
C
$2,900
D
$1,500
A

Explanation:
The correct answer is (C).

The realized loss reported from the sale of the equity securities is determined as the difference between the proceeds received (i.e., the gross selling price of the shares less any brokerage commissions and taxes incurred in the sale) and the carrying value of the securities. Equity securities are recorded at fair value with unrealized gains and losses included in earnings.

Gross selling price of 2,000 share @ $14 $28,000
Less: brokerage commissions and taxes incurred (1,400)
Proceeds received from sale of securities 26,600
Carrying Value of Securities Sold ($31,500 - $2,000) (29,500)
Realized Loss in current year ($ 2,900)

How well did you know this?
1
Not at all
2
3
4
5
Perfectly
280
Q

Grant, Inc. acquired 30% of South Co.’s voting stock for $200,000 on January 2, year 3. Grant’s 30% interest in South gave Grant the ability to exercise significant influence over South’s operating and financial policies. During year 3, South earned $80,000 and paid dividends of $50,000. South reported earnings of $100,000 for the six months ended June 30, year 4, and $200,000 for the year ended December 31, year 4. On July 1, year 4, Grant sold half of its stock in South for $150,000 cash. South paid dividends of $60,000 on October 1, year 4.

Before income taxes, what amount should Grant include in its year 3 income statement as a result of the investment?

A
$15,000
B
$24,000
C
$50,000
D
$80,000
A

B
Explanation:
Grant acquired 30% of South Co. and exercises significant influence, it will account for this investment under equity method. The cost of investment on January 2, Year 3 is $200,000. This is increased by Grant’s share in South’s earnings for Year 3, in this case it is 30% of $80,000 income. This amounts to $24,000 equity in earnings of South Co, which is recognized as a revenue in the statement of income and increase in the investment value by that amount.

Option (A) is incorrect because $15,000 is Grant’s 30% share in $50,000 dividend declared by South. Grant accounts for this investment under equity method. This dividend would reduce the investment value and not be recognized as dividend income in the statement of income.

Option (C) is incorrect because $50,000 is the total dividend declared by South in Year 3.

Option (D) is incorrect because $80,000 is the total income earned by South in Year 3.

How well did you know this?
1
Not at all
2
3
4
5
Perfectly
281
Q

On January 1, Year 1, White Car Wash Co. purchased a bond with a face value of $1000, maturing after 7 years. The coupon rate on the bond was stated to be 8%, payable semi-annually on June 30 and December 31. On January 1, Year 5, it was estimated that Tuco, the bond issuer, will be unable to service the rest of the interest payments. However, Tuco promises to pay the principal due on maturity. How much credit loss should be recorded in the books of White? Assume a discount rate of 10%.

Given:

Present Value of $1 at 5% and n=14: 0.505
Ordinary Annuity of $1 at 5% and n=14: 9.8986
Present Value of $1 at 5% and n=6: 0.746
Ordinary Annuity of $1 at 5% and n=6: 5.0757
A
$0
B
$746
C
$203
D
$254
A

Explanation:
The correct answer is (C)

If it is estimated that all of the investments in Held-to-Maturity Securities (principal or/and interest) will not be collected. The difference between the Amortized Cost and Present Value of Interest and Principal that will be collected is recognized as a credit loss and the investment will now be reported at present value of the principal and interest that is expected to be collected. Amortized Cost of the Bond is the Carrying Value of the Bond.

Carrying Value of the Bond on January 1, Year 1:

⇒ $1,000 x Present Value of $1 at 5% for 14 periods + $40 x Present Value of Ordinary Annuity $1 at 5% for 14 periods
⇒ $1,000 x 0.505 + $40 x 9.8986
⇒ $505 + $395.94
⇒ $900.944

Carrying Value of the Bond on January 1, Year 5 (after 8 Periods):

Period	Interest Income (5%)	Interest Received (4%)	Interest Received (4%)	Carrying Value
0	 	 	 	$900.94
1	$45.05	40	$5.05	$905.99
2 	$45.30 	40 	$5.30 	$911.29 
3 	$45.56 	40 	$5.56 	$916.86 
4 	$45.84  	40 	$5.84  	$922.70  
5 	$46.13  	40 	$6.13  	$928.83  
6 	$46.44  	40 	$6.44  	$935.27  
7 	$46.76  	40 	$6.76  	$942.04  
8 	$47.10  	40 	$7.10  	$949.14 
In the given case, Present value of the bond on January 1, Year 5 (after 8 Periods):
⇒ $1,000 x Present Value of $1 at 5% for 6 years
⇒ 1,000 x 0.746
⇒ $746

White’s credit loss = $949.14 – $746 = $203.14

How well did you know this?
1
Not at all
2
3
4
5
Perfectly
282
Q

In the current year, Neil Co. held the following investments in common stock:

25,000 shares of B&K, Inc.’s 100,000 outstanding shares. Neil’s level of ownership gives it the ability to exercise significant influence over the financial and operating policies of B&K.
6,000 shares of Amal Corp.’s 309,000 outstanding shares.
During the year, Neil received the following distributions from its common stock investments:

Nov. 6 $30,000 cash dividend from B&K.
Nov. 11 $1,500 cash dividend from Amal.
Dec. 26 3% common stock dividend from Amal. The closing price of this stock on a national exchange was $15 per share.
What amount of dividend revenue should Neil report for the year?

A
$ 1,500
B
$ 4,200
C
$ 31,500
D
$ 34,200
A

A

Explanation:
Of the dividends received, only the $1,500 cash dividend from Amal should be reported as dividend revenue. Neil owns less than a 2% (i.e., 6,000 / 309,000) interest in Amal; thus, Neil does not have the ability to exercise significant influence over Amal by virtue of the investment, and the investment should be accounted for under the cost method. The investment in B&K should be accounted for under the equity method since Neil’s level of ownership gives it the ability to exercise significant influence over the financial and operating policies of B&K. Thus, the cash dividend from B&K should not be reported as dividend revenue; instead, it reduces the carrying amount of the investment in B&K that is reported in Neil’s balance sheet. No income is recognized from the receipt of the stock dividend from Amal since Neil’s proportionate interest in Amal has not changed and Amal’s underlying assets and liabilities have not changed.

How well did you know this?
1
Not at all
2
3
4
5
Perfectly
283
Q

If it is not practicable for an entity to estimate the fair value of a financial instrument, which of the following should be disclosed?

Information pertinent to estimating the fair value of the financial instrument
The reasons it is not practicable to estimate fair value
A
I only
B
II only
C
Both I and II
D
Neither I nor II
A

C

Explanation:
If it is not practicable to estimate the fair value of a financial instrument, disclosures are required that include: (1) the information pertinent to estimating the fair value of that financial instrument, such as the carrying amount, effective interest rate, and maturity; and (2) the reasons why it is not practicable to estimate fair value.

How well did you know this?
1
Not at all
2
3
4
5
Perfectly
284
Q

At the beginning of year 2, a company invested $40,000 in a marketable debt security. At that time the security was appropriately classified as an available-for-sale debt security. At the end of year 2, the security had a fair value of $28,500. The change in fair value is deemed temporary. Assuming no expected credit losses, how should this change in fair value be reported in the financial statements?

A
As a realized loss of $11,500 as part of Net Income
B
As a realized loss of $11,500 as part of Other Comprehensive Income
C
As an unrealized loss of $11,500 as part of Net Income
D
As an unrealized loss of $11,500 as part of Other Comprehensive Income
Explanation:

A

D

Explanation:
Available-For-Sale (AFS) debt securities are accounted for at fair value, determined on the balance sheet date.

The excess of cost over fair value or fair value over cost for AFS debt securities is recorded as a credit or debit in a Market Adjustment account.

The offsetting unrealized gain or loss is excluded from earnings and should be reported in Other Comprehensive Income until realized.

The company would report the change in fair value as an unrealized loss of $11,500 as part of Other Comprehensive Income.

How well did you know this?
1
Not at all
2
3
4
5
Perfectly
285
Q

During the current year, Scott Corp. purchased marketable debt securities and classified them as available-for-sale. Pertinent data follow:

Security	Cost	Fair value at 12/31
D	$ 36,000	$ 40,000
E	80,000	60,000
F	180,000	186,000
 	$296,000	$286,000
Scott appropriately carries these debt securities at fair value and has no expected credit losses. The amount of unrealized loss on these securities in Scott's current year income statement should be
A
$20,000
B
$14,000
C
$10,000
D
$0
A

Explanation:
The correct answer is (D).

Marketable debt securities classified as available-for-sale (AFS) are to be reported at fair value. The amount by which the aggregate cost of an AFS debt portfolio exceeds its fair value should be accounted for as a credit to the Market Adjustment-AFS account.

In addition, an amount equal to the balance of the Market Adjustment-AFS account should be reported separately within equity in accumulated other comprehensive income.

The balance in the Market Adjustment-AFS account at year-end is a $10,000 credit ($296,000 aggregate cost - $286,000 aggregate market value).

Any changes in the Market Adjustment-AFS account is recognized in other comprehensive income, not in net income.

How well did you know this?
1
Not at all
2
3
4
5
Perfectly
286
Q

January 1, Year 5, Elia Company sold a building which had a carrying amount of $350,000, receiving a $125,000 down payment and, as additional consideration, a $400,000 non interest-bearing note due on January 1, Year 8. There was no established exchange price for the building and the note had no ready market. The prevailing rate of interest for a note of this type at January 1, Year 5, was 10%. The present value of 1 at 10% for three periods is 0.75.What amount of interest income should be included in Elia’s Year 5 income statement?

A
$0
B
$30,000
C
$35,000
D
$40,000
A

B

Explanation:
The recorded value of the Note Receivable is $300,000 [400,000 x .75]. Interest income is $30,000 [300,000 x .10]

How well did you know this?
1
Not at all
2
3
4
5
Perfectly
287
Q

Alpha company should report the marketable debt securities that it has classified as trading at
A
Lower of cost or market, with holding gains and losses included in earnings.
B
Lower of cost or market, with holding gains included in earnings only to the extent of previously recognized holding losses.
C
Fair value, with holding gains included in earnings only to the extent of previously recognized holding losses.
D
Fair value, with holding gains and losses included in earnings.

A

D

Explanation:
When an investment is made in debt securities that are publicly traded, and the investment is not large enough to provide the investor with any significant influence over the investee, the accounting for the investment will depend on the below classifications: - Trading or Held-for-trading (HFT) securities. Available-for-sale (AFS) securities. Held-to-maturity (HTM) securities. Trading or Held-for-trading (HFT) securities are debt securities bought and held principally for the purpose of selling them in the near term. They are initially recorded at cost and then carried at Fair Market Value (FMV), as trading securities are marketable and there is intent to sell in the near term.Alpha company should report the marketable debt securities that it has classified as trading at Fair Market value and all holding or unrealized gains and losses are included in earnings.

How well did you know this?
1
Not at all
2
3
4
5
Perfectly
288
Q

Anchor Co. owns 40% of Main Co.’s common stock outstanding and 75% of Main’s noncumulative pre­ferred stock outstanding. Anchor exercises significant influence over Main’s operations. During the current period, Main declared dividends of $200,000 on its common stock and $100,000 on its noncumulative pre­ferred stock. What amount of dividend income should Anchor report on its income statement for the cur­rent period related to its investment in Main?

A
$ 75,000
B
$ 80,000
C
$ 120,000
D
$ 225,000
A

Explanation:
Dividend income on preferred shares is $75,000 ($100,000 x 75%). When the ownership of the common stock is between 20-50%, it implies that the investor has a significant voting influence over the investee and would apply the equity method. Under the equity method, dividends are recognized as a reduction of the investment and not as dividend income.

Option (B) is incorrect because $80,000 is shown as a reduction in the investment under the equity method.

Options (C) and (D) are incorrect as per the above explanation.

How well did you know this?
1
Not at all
2
3
4
5
Perfectly
289
Q

Sork Inc. has a portfolio of equity securities which it does not intend to sell in the near future. At the end of the year, the fair values of the securities exceeded their costs. How should Sork classify these securities and where should the unrealized gain for these securities be recognized?

Classification Recognition of Gain
A Trading Income Statement
B Available-for-Sale Other Comprehensive Income
C Held to Maturity Stockholder’s Equity
D Fair Value through Net Income Income Statement

A

D

Explanation:
Investments in equity securities with a readily determinable market value are to be classified as Fair Value Through Net Income (FVTNI) - i.e., measured at fair value with changes in the fair value recognized through net income (the AFS classification is no longer applicable where unrealized gains/losses were recognized as OCI).

Options (A) and (B) are incorrect because only debt securities are classified as trading or available-for-sale.

Option (C) is incorrect because only debt securities are classified as held-to-maturity.

How well did you know this?
1
Not at all
2
3
4
5
Perfectly
290
Q

Information pertaining to dividends from Wray Corp.’s common stock investments for the current year ended December 31 follows:
On September 8, Wray received a $50,000 cash dividend from Seco, Inc., in which Wray owns a 30% interest. A majority of Wray’s directors are also directors of Seco.
On October 15, Wray received a $6,000 liquidating dividend from King Co. Wray owns a 5% interest in King Co.
Wray owns a 2% interest in Bow Corp., which declared a $200,000 cash dividend on November 27, to stockholders of record on December 15, payable on January 5 of next year.
What amount should Wray report as dividend income in its income statement for the current year ended December 31?
A
$60,000
B
$56,000
C
$10,000
D
$4,000

A

D

Explanation:
Of the dividends listed, only the $4,000 (i.e., $200,000 × 2%) cash dividend receivable from Bow should be reported as dividend income. Wray owns only a 2% interest in Bow; thus, Wray does not have the ability to exercise significant influence over Bow by virtue of the investment. Therefore, the investment should be accounted for under the cost method. The cash dividend received from Seco should be recorded as a reduction of the carrying amount of the investment in Seco reported in Wray’s balance sheet because the investment should be accounted for under the equity method (i.e., Wray owns a 30% interest in Seco and a majority of Wray’s directors are also directors of Seco). The liquidating dividend received from King should be recorded as a reduction of the carrying amount of the investment in King reported in Wray’s balance sheet.

How well did you know this?
1
Not at all
2
3
4
5
Perfectly
291
Q

For an available-for-sale security transferred into the trading category, the portion of the unrealized holding gain or loss at the date of the transfer that has not been previously recognized in earnings shall be

A
Recognized in earnings immediately.
B
Amortized over the period to date of sale.
C
Transferred to other comprehensive earnings.
D
Deferred and recognized when the security is sold.

A

Explanation:
The correct answer is (A)

As long as a security is held for sale, all the unrealized holding gain/loss resulting from changes in the value of such a security shall be classified under Other Comprehensive Income. However, transferring the security into trading category requires the entire portion of the unrealized gain or loss previously recognized as OCI to be recognized in earnings.

How well did you know this?
1
Not at all
2
3
4
5
Perfectly
292
Q

Grant, Inc. acquired 30% of South Co.’s voting stock for $200,000 on January 2, year 3. Grant’s 30% interest in South gave Grant the ability to exercise significant influence over South’s operating and financial policies. During year 3, South earned $80,000 and paid dividends of $50,000. South reported earnings of $100,000 for the six months ended June 30, year 4, and $200,000 for the year ended December 31, year 4. On July 1, year 4, Grant sold half of its stock in South for $150,000 cash. South paid dividends of $60,000 on October 1, year 4.

In Grant’s December 31, year 3, balance sheet, what should be the carrying amount of this investment?

A
$200,000
B
$209,000
C
$224,000
D
$230,000
A

B

Explanation:
Grant acquired 30% of South Co. and exercises significant influence. It will account for this investment under equity method. The investment account for Year 3 will be as follows:

Cost of 30% investment as on January 2, Year 3

$200,000

Add: 30% share of profit for the year (i.e. $80,000 x 30%)

$24,000

Less: 30% share of dividends paid (i.e. $50,000 x 30%)

$15,000

Carrying amount as at December 31, Year 3

$209,000

Option (A) is incorrect because $200,000 is only the cost of Grant’s investment in South Co.

Option (C) is incorrect because $224,000 is the cost of Grant’s investment $200,000 and its proportionate share in South’s earnings for Year 3 $24,000. It does not account for a reduction in value for proportionate share in dividend distributed $15,000.

Option (D) is incorrect because $230,000 is arrived as cost of Grant’s investment $200,000 plus its 30% share in South’s earnings of $100,000 for the first 6 months of Year 4.

How well did you know this?
1
Not at all
2
3
4
5
Perfectly
293
Q

When the market value of an investment in securities exceeds its carrying amount, how should each of the following assets be reported at the end of the year?
Trading Marketable Securities Available-For-Sale Marketable Securities
A Market value Market value
B Carrying amount Carrying amount
C Carrying amount Market value
D Market value Carrying amount

A

A

Explanation:
Marketable securities classified as either trading or available-for-sale are to be accounted for at market.

How well did you know this?
1
Not at all
2
3
4
5
Perfectly
294
Q
After being held for 40 days, a 120-day 12% interest-bearing note receivable was discounted at a bank at 15%. The proceeds received from the bank equal
A
Maturity value less the discount at 12%.
B
Maturity value less the discount at 15%.
C
Face value less the discount at 12%.
D
Face value less the discount at 15%.
A

B

Explanation:
Determining the proceeds received from discounting a note receivable consists of three steps:

Determine the maturity value of the note. This amount is based on the face amount, the stated rate of interest, and the time to maturity of the note.
Apply the bank’s discount rate to the maturity value of the note to obtain the amount of the discount charged by the bank.
Subtract the discount charged by the bank from the maturity value of the note to obtain the proceeds received from the bank.

How well did you know this?
1
Not at all
2
3
4
5
Perfectly
295
Q

Sun Corp. had investments classified as trading debt securities costing $650,000. On June 30 of the current year, Sun decided to hold the investments indefinitely and accordingly reclassified them from trading to available-for-sale on that date. The investment’s fair value was $575,000 on December 31 of the previous year; $530,000 this June 30; and $490,000 on December 31 of the current year. Assuming no expect credit losses, what amount of loss from investments should Sun report in its current year income statement?

A
$ 45,000
B
$ 85,000
C
$120,000
D
$160,000
A

Explanation:
The correct answer is (A).

The transfer of a debt security from the trading category of investments should be accounted for at fair value.

The unrealized holding loss resulting from the decrease in market value while classified as trading debt securities is included in earnings.

As of the date of transfer, June 30 of the current year, the securities had experienced a decline in fair value of $45,000 ($575,000 - $530,000) from the end of the prior year.

Upon transfer to a different category of investment, this unrealized loss is required to be included in earnings. (The decline from $650,000 to $575,000 would have been included as an unrealized loss on the previous year’s income statement.)

CLOSE

How well did you know this?
1
Not at all
2
3
4
5
Perfectly
296
Q

Pare, Inc. purchased 10% of Tot Co.’s 100,000 outstanding shares of common stock on January 2 of the current year for $50,000. On December 31, Pare purchased an additional 20,000 shares of Tot for $150,000. There was no goodwill as a result of either acquisition, and Tot had not issued any additional stock during the year. Tot reported earnings of $300,000 for the year. What amount should Pare report in its December 31 balance sheet as investment in Tot?

A
$170,000
B
$230,000
C
$200,000
D
$290,000
A

Explanation:
The correct answer is (C).

When Pare purchased an additional 20,000 shares of Tot at 12/31, it increased its investment in Tot’s common stock from 10% to 30% [i.e., (10,000 shares + 20,000 shares) ÷ 100,000 shares = 30%].

On 12/31, Pare gained the ability to exercise significant influence over the financial and operating policies of Tot and accordingly should report its investment in Tot using the equity method in its 12/31 balance sheet.

The change from the cost method of reporting the investment in Tot to the equity method should be made by prospectively as of the date significant influence is acquired and going forward.

On the December 31 Balance Sheet would be the original investment of $50,000 plus the price paid for additional 20,000 shares - i.e. $150,000 - which will be equal to $200,000.

Pare will begin using equity method from January 1, Year 2 and proportionate earnings will be included from year 2 and not year 1.

How well did you know this?
1
Not at all
2
3
4
5
Perfectly
297
Q
Sage, Inc. bought 40% of Adams Corp.'s outstanding common stock on January 2 of the current year for $400,000. The carrying amount of Adams' net assets at the purchase date totaled $900,000. Fair values and carrying amounts were the same for all items except for plant and inventory, for which fair values exceeded their carrying amounts by $90,000 and $10,000, respectively. The plant has an 18-year life. All inventory was sold during the year. Goodwill, if any, will be tested for impairment each year. During the year, Adams reported net income of $120,000 and paid a $20,000 cash dividend. What amount should Sage report in its income statement from its investment in Adams for the year ended December 31?
A
$48,000
B
$42,000
C
$36,000
D
$32,000
A

B

Explanation:
The common stock investment should be accounted for under the equity method since Sage has the ability to exercise significant influence over the operating and financial policies of Adams by virtue of the investment. To determine Sage’s reported amount of equity in income in Adams, the cost of the investment must first be allocated as follows:

Purchase price $ 400,000
Percentage of carrying amount acquired ($900,000 x 40%) (360,000)
Payment in excess of purchase price 40,000
Excess payment associated with specific assets:
Plant assets ($90,000 x 40%) 36,000
Inventory ($10,000 x 40%) (4,000)
Goodwill $ 0The amount that Sage should report in its income statement from its investment in Adams can now be determined as follows:
Interest in Adams net income ($120,000 x 40%) $ 48,000
Less: Amortization of excess payment associated with plant assets ($36,000 / 18) (2,000)
Excess payment associated with inventory sold (4,000)
Equity in income of Adams $ 42,000The cash dividends paid by Adams to Sage reduce the carrying amount of the investment in Sage’s balance sheet, they do not effect Sage’s reported amount of equity in income in Adams

How well did you know this?
1
Not at all
2
3
4
5
Perfectly
298
Q

On December 29, year 9, BJ Co. sold a marketable debt security that had been purchased on January 4, year 8. BJ owned no other marketable debt security. An unrealized loss was reported in other comprehensive income in year 8. A realized gain was reported in the year 9 income statement. How was the marketable debt security classified, and did its year 8 market price decline exceed its year 9 market price recovery?

Classification	Year 8 market price decline exceeded the year 9 market price recovery
A	Trading	Yes
B	Trading	No
C	Available-For-Sale	Yes
D	Available-For-Sale	No
A

Explanation:
The correct answer is (D).

An unrealized loss in year 8 on investment in marketable debt securities classified as available-for-sale would be shown as an other comprehensive income item rather than as an income statement item.

The market price decline in the year 8 was less than the increase in market price during the year 9 because there was a realized gain reported on the year 9 income statement which means that the security was sold for a price exceeding its cost.

Thus, in year 9, the market price increased enough to recover the unrealized loss experienced in year 8 and to produce a gain on the sale of the investment.

An unrealized loss in year 8 on investment in marketable debt securities classified as trading debt securities would be reported in the income statement rather than in other comprehensive income.

Trading debt securities would be written to fair value at the end of year 8.

If a trading debt security did report a realized gain in the year 9 it would be indeterminable as to whether any market price declined or not in year 8.

How well did you know this?
1
Not at all
2
3
4
5
Perfectly
299
Q

A company should report the marketable debt securities that it has classified as trading at

A
Lower of cost or market, with holding gains and losses included in earnings.
B
Lower of cost or market, with holding gains included in earnings only to the extent of previously recognized holding losses.
C
Fair value, with holding gains included in earnings only to the extent of previously recognized holding losses
D
Fair value, with holding gains and losses included in earnings

A

D

Explanation:
Trading debt securities are reported at fair value and unrealized holding gains and losses are included in current earnings on the Income Statement.

How well did you know this?
1
Not at all
2
3
4
5
Perfectly
300
Q

How is a held-to-maturity security reported on the balance sheet when there are expected credit losses?

A
Face Value
B
Amortized Cost
C
Present Value of Principal
D
Present Value of Principal and Interest
A

Explanation:
The correct answer is (D).

If it is estimated that not all of the investments in held-to-maturity securities (principal or/and interest) will be collected, the difference between the amortized cost and present value of Interest and principal that will be collected is recognized as a credit loss and the investment will be reported at present value of the principal and interest that is expected to be collected.

How well did you know this?
1
Not at all
2
3
4
5
Perfectly
301
Q

Which of the following securities is subject to the credit loss model of impairment?

A
Bonds payable
B
Investment in common stock
C
Investment in preferred stock
D
Bonds receivable
A

Explanation:
The correct answer is (D).

With the CECL (Current Expected Credit Loss) model, Credit losses expected over the lifetime of a Held-to-Maturity and Available-for-Sale debt Security are to be recognized upon the initial recognition of the asset.

Bonds payable is a liability and not an asset. The CECL model is not applicable to it.

Investments in common and preferred stock are not debt securities.

Bonds receivable is a debt security that is generally held till maturity by the bondholder and can be impaired using the CECL model.

How well did you know this?
1
Not at all
2
3
4
5
Perfectly
302
Q

Grant, Inc. acquired 30% of South Co.’s voting stock for $200,000 on January 2, year 3. Grant’s 30% interest in South gave Grant the ability to exercise significant influence over South’s operating and financial policies. During year 3, South earned $80,000 and paid dividends of $50,000. South reported earnings of $100,000 for the six months ended June 30, year 4, and $200,000 for the year ended December 31, year 4. On July 1, year 4, Grant sold half of its stock in South for $150,000 cash. South paid dividends of $60,000 on October 1, year 4.

In its year 4 income statement, what amount should Grant report as gain from the sale of half of its investment?

A
$24,500
B
$30,500
C
$35,000
D
$45,500
A

Explanation:
The correct answer is (B).

Grant acquired 30% of South Co. and exercises significant influence. It will account for this investment under the equity method.

The investment account will be as follows for years Year 3 and Year 4:

Cost of 30% investment as on January 2, Year 3

$200,000

Add: 30% share of profit for the year (i.e. $80,000 x 30%)

$24,000

Less: 30% share of dividends paid (i.e. $50,000 x 30%)

$15,000

Carrying amount as at December 31, Year 3

$209,000

Add: 30% share of profit for six months ended June 30, Year 4 (i.e. $100,000 x 30%)

$30,000

Carrying value as of June 30, Year 4

$239,000

Carrying amount of 50% of investment (i.e. $239,000 x 50%)

$119,500

Proceeds from the sale of 50% of investment

$150,000

Profit on sale of investment

$30,500

How well did you know this?
1
Not at all
2
3
4
5
Perfectly
303
Q

Larkin Co. has owned 25% of the common stock of Devon Co. for a number of years, and has the ability to exercise significant influence over Devon. The following information relates to Larkin’s investment in Devon during the most recent year:

Carrying amount of Larkin’s investment in Devon at the beginning of the year $ 200,000
Net income of Devon for the year 600,000
Total dividends paid to Devon’s stockholders during the year 400,000
What is the carrying amount of Larkin’s investment in Devon at year end?

A
$100,000
B
$200,000
C
$250,000
D
$350,000
A

C

Explanation:
Larkin Co would apply equity method to account for its investments. The earnings from Devon Co., will increase the amount of its investment and dividend will reduce the investment account.

Ref

Equity Method

Amount ($)

a

Investment in Devon Co.

200,000

b

Net Income ($600,000 x 25%)

150,000

c

Dividend ($400,000 x 25%)

(100,000)

d

Investment reported at year end (a+b-c)

250,000

Option (A) is incorrect because net income’s proportionate share at 25% is not included, which would increase the investment account.

Option (B) is incorrect because it is the cost of the initial investment, which should be adjusted for proportionate share in net income which would increase the investment account and dividends received at proportionate share of 25% which would reduce the investment account.

Option (D) is incorrect because it should adjust for dividends received at 25% proportionate share which would reduce the investment account.

How well did you know this?
1
Not at all
2
3
4
5
Perfectly
304
Q

Long Co. invested in marketable debt securities. At year-end, fair-value changes in this investment were included in Long’s other comprehensive income. How would Long classify this investment?

A
Held-to-maturity securities
B
Trading debt securities
C
Contra securities
D
Available-for-sale debt securities
A

Explanation:
The correct answer is (D).

Available-for-sale debt securities are initially recorded at cost and carried at the fair market value on the balance sheet and any unrealized gain/loss is reported in the statement of comprehensive income and cumulatively accumulates in the balance sheet.

(A) is incorrect because the debt securities that the entity has the positive intent and ability to hold to the maturity date are classified as Held-to-Maturity (HTM). The HTM securities are initially recorded at cost and carried at amortized cost.

(B) is incorrect because trading debt securities are the debt bought and held principally for the purpose of selling them in the near term. They are carried at market value and any unrealized gain/loss at year end are reported in the profit and loss statement.

(C) is incorrect because this is not a classification type.

Note: The updated standard doesn’t change the guidance for classifying and measuring investments in debt securities or loans.

How well did you know this?
1
Not at all
2
3
4
5
Perfectly
305
Q
During year 3, Gilman Co. purchased 5,000 shares of the 500,000 outstanding shares of Meteor Corp.’s common stock for $35,000. During year 3, Gilman received $1,800 of dividends from its investment in Meteor’s stock. The fair value of Gilman’s investment on December 31, year 3, is $32,000. Gilman has elected the fair value option for this investment. What amount of income or loss that is attributable to the Meteor stock investment should be reflected in Gilman’s earnings for year 3?
A
Income of $4,800
B
Income of $1,800
C
Loss of $1,200
D
Loss of $3,000
A

c

Explanation:
Gilman’s earnings would reflect a loss of $1,200 attributable to the Meteor stock. In electing fair value option, the stock would be valued at $32,000 at December 31, year 3, which denotes a loss of $3,000 from the $35,000 purchase price of the stock during year 3. Gilman also received $1,800 in of dividends from the stock which would be income for year 3. The net between the two transactions is a loss of $1,200.

How well did you know this?
1
Not at all
2
3
4
5
Perfectly
306
Q

Wood Co. owns 2,000 shares of Arlo, Inc.’s 20,000 shares of $100 par, 6% cumulative, nonparticipating preferred stock, and 1,000 shares (2%) of Arlo’s common stock. During the current year, Arlo declared and paid dividends of $240,000 on preferred stock. No dividends had been declared or paid the previous year. Also, Wood received a 5% common stock dividend from Arlo when the quoted market price of Arlo’s com­mon stock was $10 per share. What amount should Wood report as dividend income in its year-end income statement?

A
$12,000
B
$12,500
C
$24,000
D
$24,500
A

C

Explanation:
Wood owns 10% of Arlo’s preferred stock and would have received $24,000 ($240,000 dividends × 10%) in dividends in the current year on this preferred stock. Although this amount represents the dividend pre­ference for the previous year, due to the 6% cumulative feature of the stock (2,000 shares x $100 x .06 = $12,000/year), as well as for the current year, the revenue is not recognized until the dividends are actually declared. In the current year the dividends for the current and prior year are both recognized. The 5% stock dividends received are not recognized as income because they are not a distribution, division, or severance of the corporate assets.

How well did you know this?
1
Not at all
2
3
4
5
Perfectly
307
Q
Kale Co. purchased bonds at a discount on the open market as an investment and intends to hold these bonds to maturity. Kale should account for these bonds at
A
Cost.
B
Amortized cost.
C
Fair value.
D
Lower of cost or market.
A

B

Explanation:
The debt securities that the entity has the positive intent and ability to hold to maturity date are classified as Held-to-Maturity (HTM). The HTM securities are initially recorded at cost and carried at amortized cost. As HTM are not going to be sold, fluctuations in market prices are ignored. The difference between cost and maturity value is amortized over the life of the security. Thus, Kale should account for these bonds at amortized cost. Option (a) is incorrect because HTM are initially recorded at cost, and later carried at amortized cost. Option (c) is incorrect because the fluctuations in market prices are ignored and accounted based on amortized cost not fair value. Option (d) is incorrect because inventory is reported at lower of cost or market not HTM securities which is at amortized cost.

How well did you know this?
1
Not at all
2
3
4
5
Perfectly
308
Q

Sun Corp. had investments in debt securities classified as trading debt securities costing $650,000. On June 30 of the current year, Sun decided to hold the investments indefinitely and accordingly reclassified them from trading debt securities to available-for-sale debt securities on that date. The investment’s fair value was $575,000 on December 31 of the previous year; $530,000 this June 30; and $490,000 on December 31 of the current year. Assuming no expected credit losses, what amount should Sun report as a net unrealized loss on investments in debt securities in other comprehensive income at the end of the current year?

A
$40,000
B
$45,000
C
$85,000
D
$160,000
A

Explanation:
The correct answer is (A).

Unrealized holding losses for available-for-sale (AFS) debt securities are excluded from earnings and reported in other comprehensive income until realized.

The decrease in fair value over the time period in which the securities were classified as trading debt securities would have been reported in earnings.

However, the loss occurring after the transfer to the AFS category, $40,000 ($530,000 - $490,000), would be reflected in OCI.

How well did you know this?
1
Not at all
2
3
4
5
Perfectly
309
Q

Ace Co. sold to King Co. a $20,000, 8%, 5-year note that required five equal annual year-end payments. This note was discounted to yield a 9% rate to King.
The present value factors of an ordinary annuity of $1 for five periods are as follows:

8% 3.992
9% 3.890
What should be the total interest revenue earned by King on this note?

A
$9,000
B
$8,000
C
$5,560
D
$5,050
A

C

Explanation:
The total amount of interest revenue to be earned by King over the life of the note is determined as the excess of the summation of the required annual year-end payments over the present value of the note dis­counted to yield 9% to King.

Summation of required annual year-end payments [5 × $5,010 (as determined below)] $ 25,050
Face amount of note $20,000
Divide by: PV factor of an ordinary annuity of $1 for 5 periods at 8% / 3.992
Required equal annual year-end payments under note $ 5,010
Times: PV factor of an ordinary annuity of $1 for 5 periods at 9% × 3.890
Less: present value of note (19,490)
Interest revenue to be earned over the life of the note’s life $ 5,560

How well did you know this?
1
Not at all
2
3
4
5
Perfectly
310
Q
On January 2, Kean Co. purchased a 30% interest in Pod Co. for $250,000. On this date, Pod's stockholders' equity was $500,000. The fair value of plant and equipment exceeded its carrying amount by $200,000. Plant and assets were determined to have a remaining 10 year life at December 31. There were no other under valued assets. Pod reported net income of $100,000 for the year, and paid no dividends. Kean accounts for this investment using the equity method. In its December 31, year-end balance sheet, what amount should Kean report as investment in subsidiary?
A
$210,000
B
$220,000
C
$270,000
D
$280,000
A

D

Explanation:
Since Kean owns a 30% interest in Pod, the investment should be accounted for under the equity method. Under the equity method, the carrying amount of the investment is increased for Kean’s share of Pod’s income, decreased for Kean’s share of any dividends paid by Pod and decreased by the amortization of the excess payment associated with plant and equipment. To determine the carrying amount of Kean’s investment in Pod at 12/31, the original cost of the investment must first be allocated. The carrying amount of the investment at 12/31 is determined as follows:

How well did you know this?
1
Not at all
2
3
4
5
Perfectly
311
Q

On January 1, Year 1, Astor Co. purchased a bond with a face value of $1000, maturing after 7 years, and classified as an available-for-sale debt security on its balance sheet. The coupon rate on the bond was stated to be 8%, payable semi-annually on June 30 and December 31. On January 1, Year 5, the fair value of the said bond was estimated to be $800. What is the impact of the loss on the financial statements of Astor? Assume a discount rate of 10%.

Given:

Present Value of $1 at 5% and n=14: 0.505
Ordinary Annuity of $1 at 5% and n=14: 9.8986
Present Value of $1 at 5% and n=6: 0.746
Ordinary Annuity of $1 at 5% and n=6: 5.0757
Income Statement Other Comprehensive Income
A 0 200
B 100 100
C 149.14 0
D 203.14 0

A

Explanation:
The correct answer is (C)

Impairment for Available-for-Sale Securities is calculated as the difference between Amortized Cost (i.e. Carrying Value) and the Fair Value of the security. However, Credit Losses on the Income Statement are limited to Amortized Cost – Present Value (calculated the same way as a Held-to-Maturity investment) because if the unrealized loss at any given time is more than the expected credit loss till maturity, the Investor can minimize the loss by holding the security.

Excess losses will be charged to OCI.

Carrying Value of the Bond on January 1, Year 1:

⇒ $1,000 x Present Value of $1 at 5% for 14 periods + $40 x Present Value of Ordinary Annuity $1 at 5% for 14 periods
⇒ $1,000 x 0.505 + $40 x 9.8986
⇒ $505 + $395.94
⇒ $900.94

Carrying Value of the Bond on January 1, Year 5 (after 8 Periods):

Period	Interest Income (5%)	Interest Received (4%)	Interest Received (4%)	Carrying Value
0	 	 	 	$900.94
1	$45.05	40	$5.05	$905.99
2 	$45.30 	40 	$5.30 	$911.29 
3 	$45.56 	40 	$5.56 	$916.86 
4 	$45.84  	40 	$5.84  	$922.70  
5 	$46.13  	40 	$6.13  	$928.83  
6 	$46.44  	40 	$6.44  	$935.27  
7 	$46.76  	40 	$6.76  	$942.04  
8 	$47.10  	40 	$7.10  	$949.14 
In the given case, Present value of the bond on January 1, Year 5 (after 8 Periods):
⇒ $1,000 x Present Value of $1 at 5% for 6 years
⇒ 1,000 x 0.746
⇒ $746

Expected Credit Loss = Carrying Value – Fair Value = $949.14 - $800 = $149.14

Expected Credit loss = $949.14 - $746 = $203.14.

All of the loss is recognized on the Income Statement.

How well did you know this?
1
Not at all
2
3
4
5
Perfectly
312
Q

Under IFRS, what valuation methods are used for intangible assets?

A
The cost model or the fair value model.
B
The cost model or the revaluation model.
C
The cost model or the fair value through profit or loss model.
D
The revaluation model or the fair value model.
A

B

Explanation:
Under IFRS, the cost model or the revaluation model is used to value intangible assets. Revaluation model may be used only for intangibles that are traded with active market prices (as revaluation model requires fair value determination from active market):

Cost Model CV = Cost - Accumulated Amortization (if finite life) -Accumulated Impairment.
Revaluation Model CV = Fair Value from active market @revaluation date -Subsequent Accumulated Amortization (if finite life intangibles) – Subsequent Accumulated Impairment.
Options (a), (c) and (d) are incorrect because the fair value model is not used

How well did you know this?
1
Not at all
2
3
4
5
Perfectly
313
Q
Ott Company acquired rights to a patent from Grey under a licensing agreement that required an advanced royalty payment when the agreement was signed. Ott remits royalties earned and due, under the agreement, on October 31 each year. Additionally, on the same date, Ott pays, in advance, estimated royalties for the next year. Ott adjusts prepaid royalties at year-end. Information for the current year ended December 31 is as follows:
Date	Amount
01/01 Prepaid royalties	$ 65,000
10/31 Royalty payment (charged to royalty expense)	$ 110,000
12/31 Year-end credit adjustment to royalty expense	$ 25,000In its December 31 balance sheet, Ott should report prepaid royalties of
A
$25,000
B
$40,000
C
$85,000
D
$90,000
A

D

Explanation:
The balance in the Prepaid Royalties account is $65,000 + $25,000 = $90,000.

How well did you know this?
1
Not at all
2
3
4
5
Perfectly
314
Q
On January 1, year 3, Vick Company purchased a trademark for $400,000, having an estimated useful life of 16 years. In January of year 7, Vick paid $60,000 for legal fees in a successful defense of the trademark. Trademark amortization expense for the year ended December 31, year 7, should be
A
$0
B
$25,000
C
$28,750
D
$30,000
A

D

Explanation:
Trademarks are amortized over their estimated useful life. Legal fees in a successful defense are capitalized because they offer probable future benefits. They are amortized over the remaining useful life of the trademark.

Amortization of original cost ($400,000 /16 years) $25,000
Amortization of legal defense cost, ($60,000 / 12 remaining years) 5,000
Total amortization expense, year 7 $30,000

How well did you know this?
1
Not at all
2
3
4
5
Perfectly
315
Q
During the year, Jase Co. incurred research and development costs of $136,000 in its laboratories relating to a patent that was granted on July 1. Costs of registering the patent equaled $34,000. The patent's legal life is 17 years, and its estimated economic life is 10 years. In its December 31 balance sheet, what amount should Jase report as patent, net of accumulated amortization?
A
$ 32,300
B
$ 33,000
C
$161,500
D
$165,000
A

A

Explanation:
Research and development costs are expensed as incurred. Only the costs of acquiring a patent should be capitalized. Thus, only the cost of registering the patent, $34,000, is capitalized. The capitalized cost of an intangible asset, is amortized over the asset’s economic life. One-half year of amortization is $1,700 ($34,000 / 10 years x 1/2 year). ($34,000 - $1,700) = $32,300.

How well did you know this?
1
Not at all
2
3
4
5
Perfectly
316
Q
On January 2, Paye Co. purchased Shef Co. at a cost that resulted in recognition of goodwill of $200,000. During the first quarter of the year, Paye spent an additional $80,000 on expenditures designed to maintain goodwill. In its December 31 balance sheet, what amount should Paye report as goodwill?
A
$80,000
B
$195,000
C
$200,000
D
$280,000
A

C

Explanation:
The $200,000 of goodwill acquired in connection with the purchase of Shef should be capitalized. The $80,000 of expenditures to maintain the goodwill should be expensed when incurred because costs of developing, maintaining, or restoring goodwill should not be capitalized.

How well did you know this?
1
Not at all
2
3
4
5
Perfectly
317
Q
Under a royalty agreement with another company, Wand Co. will pay royalties for the assignment of a patent for three years. The royalties paid should be reported as expense
A
In the period paid.
B
In the period incurred.
C
At the date the royalty agreement began.
D
At the date the royalty agreement expired.
A

B

Explanation:
Accrual accounting recognizes expenses in the period they are incurred. Accrual accounting does not recognize expenses merely in the period they are paid. Royalties paid should not be recognized as an expense at the date the royalty agreement began or the date the royalty agreement expires.

How well did you know this?
1
Not at all
2
3
4
5
Perfectly
318
Q

After an impairment loss is recognized, the adjusted carrying amount of the intangible asset shall be its new accounting basis. Which of the following statements about subsequent reversal of a previously recognized impairment loss is correct?

A
It is prohibited.
B
It is required when the reversal is considered permanent.
C
It must be disclosed in the notes to the financial statements.
D
It is encouraged, but not required.
A

A

Explanation:
An impairment loss is recognized if the carrying amount of an intangible asset is not recoverable and its carrying amount exceeds its fair value.After an impairment loss is recognized,the adjusted carrying amount of the asset is its new accounting basis.Subsequent reversal of a previously recognized impairment loss is prohibited under US GAAP. Option (b), (c) and (d) are incorrect because US GAAP does not allow reversal of impairment losses.

How well did you know this?
1
Not at all
2
3
4
5
Perfectly
319
Q
To be considered intangible, an asset must be which of the following?
A
Classified as identifiable
B
Without physical substance
C
Internally developed
D
All of the above
A

B

Explanation:
Intangible assets are assets without physical substance that provide economic benefits through the rights and privileges associated with their possession. Intangibles may be classified as identifiable or unidentifiable and externally acquired or internally developed.

How well did you know this?
1
Not at all
2
3
4
5
Perfectly
320
Q

A company is working on a direct response advertising campaign that will likely provide the company future benefits in the form of increased sales over the next two years. The company identified the following costs associated with the advertising campaign:

Catalogs on hand to be mailed to potential customers $100,000
Coupons printed to be mailed to existing customers 50,000
Employee salaries for call center support 45,000
Postage to be paid to mail the catalogs and coupons 25,000
What cost, if any, should be capitalized under IFRS?

A
$220,000
B
$150,000
C
$100,000
D
$0
A

Explanation:
The correct answer is (D)

An expenditure which is expected to yield revenue for more than one accounting year is an example of deferred revenue expenditure. IFRS does not permit capitalization of such expenditures. Hence, the entire expenditure of $220,000 needs to be posted on the Income Statement instead of being capitalized.

How well did you know this?
1
Not at all
2
3
4
5
Perfectly
321
Q

Ott Company acquired rights to a patent from Grey under a licensing agreement that required an advance royalty payment when the agreement was signed. Ott remits royalties earned and due, under the agree­ment, on October 31 each year. additionally, on the same date, Ott pays, in advance, estimated royalties for the next year. Ott adjusts prepaid royalties at year-end. Information for the current year ended Decem­ber 31 is as follows:

Date

Amount

01/01

Prepaid royalties

$65,000

10/31

Royalty payment (charged to royalty expense)

$110,000

12/31

Year-end credit adjustment to royalty expense

$25,000

In its December 31 balance sheet, Ott should report prepaid royalties of

A
$25,000
B
$40,000
C
$85,000
D
$90,000
A

Explanation:
The correct answer is (D).

This is an Accounting Error question.

Ott follows an incorrect method of accounting.

On 1/1 The balance on Prepaid Royalties Account: $65,000.

Ott makes no adjustments to this account until the year-end.

The 12/31 balance on this account would still be $65,000. On 10/31, the following journal entry was recorded. This included advance for next year’s royalties.

However, under Ott’s system, all such payments are debited to royalty expense when paid, and any necessary adjustments to prepaid royalties are made at year-end.

Royalty Expense 110,000
Cash 110,000
On 12/31, the following journal entry was recorded:

Prepaid Royalty Expense 25,000
Royalty Expense 25,000
The balance in Prepaid Royalties would be $90,000 ($65,000 Opening Balance + $25,000 Current Year Prepaid).

How well did you know this?
1
Not at all
2
3
4
5
Perfectly
322
Q
On January 2, Year 1, Lava, Inc. purchased a patent for a new consumer product for $90,000. At the time of purchase, the patent was valid for 15 years; however, the patent's useful life was estimated to be only 10 years due to the competitive nature of the product. On December 31, Year 4, the product was permanently withdrawn from sale under governmental order because of a potential health hazard in the product. What amount should Lava charge against income during year 4, assuming amortization is recorded at the end of each year?
A
$9,000
B
$54,000
C
$63,000
D
$72,000
A

c

Explanation:
The patented product was withdrawn from sale under governmental order. Therefore, the unamortized cost of the patent at December 31, year 4, should be charged to income.

Purchase price of patent $ 90,000
Less amortization recorded prior to year 4 ($90,000 x 3/10) (27,000)
Unamortized cost of patent charged to income in year 4 $ 63,000

How well did you know this?
1
Not at all
2
3
4
5
Perfectly
323
Q

On July 1, Year 1, Hart signed an agreement to operate as a franchise of Ace Printers for an initial franchise fee of $120,000. The same date, Hart paid $40,000 and agreed to pay the balance in four equal annual payments of $20,000 beginning July 1, Year 2. The down payment is not refundable and no future services are required of the franchiser. Hart can borrow at 14% for a loan of this type. Present and future value factors are as follows:
Present value of $1 at 14% for 4 periods 0.59
Future amount of $1 at 14% for 4 periods 1.69
Present value of an ordinary annuity of $1 at 14% for 4 periods 2.91Hart should record the acquisition cost of the franchise on July 1, Year 1, at
A
$135,200
B
$120,000
C
$ 98,200
D
$ 87,200

A

c

Explanation:
The acquisition cost of the franchise is determined as follows:

Cash payment, July 1, Year 1 $40,000
Add: Present value of periodic annual payments:
Periodic annual payments $20,000
Times: Present value of an ordinary annuity of 1 at 14% for 4 periods x 2.91 58,200
Franchise acquisition cost $98,200

How well did you know this?
1
Not at all
2
3
4
5
Perfectly
324
Q

Alta Co. spent $400,000 during the current year developing a new idea for a product that was patented during the year. The legal cost of applying for a patent license was $40,000. Also, $50,000 was spent to successfully defend the rights of the patent against a competitor. The patent has a life of 20 years. What related patent costs should Alta capitalize?

A
$40,000
B
$50,000
C
$90,000
D
$490,000
A

Explanation:
The correct answer is (C).

The cost of developing a new idea for a product should be expensed as R&D.

The legal cost for applying for the patent license and the amount spent to defend the patent from the competitor are capitalized $90,000 ($40,000 + $50,000)

(a) is incorrect because it does not capitalize the amount spent to successfully defend the rights of the patent.
(b) is incorrect because it does not capitalize the expenses incurred in applying for the patent license.
(d) is incorrect because it capitalizes the cost of developing a new idea for a product, which should be expensed

How well did you know this?
1
Not at all
2
3
4
5
Perfectly
325
Q

An entity purchases a trademark and incurs the following costs in connection with the trademark:

One-time trademark purchase price $100,000
Nonrefundable VAT taxes 5,000
Training sales personnel on the use of the new trademark 7,000
Research expenditures associated with the purchase of the new trademark 24,000
Legal costs incurred to register the trademark 10,500
Salaries of the administrative personnel 12,000
Applying IFRS and assuming that the trademark meets all of the applicable initial asset recognition cri­teria, the entity should recognize an asset in the amount of

A
$100,000.
B
$115,500.
C
$146,500.
D
$158,500.
A

B

Explanation:
A trademark is an intangible asset. The cost of an intangible asset is the cash equivalent price and includes purchase price, nonrefundable taxes, and direct costs of preparing the intangible asset and bringing it to an appropriate condition for its intended use. In this case, the cash equivalent price would include the one-time trademark purchase price, the nonrefundable VAT taxes, and the legal costs incurred to register the trade­mark ($100,000 + $5,000 + $10,500 = $115,500). The training, research, and salary expenditures would be expensed as incurred.

How well did you know this?
1
Not at all
2
3
4
5
Perfectly
326
Q
Gray Co. was granted a patent on January 2, year 1, and appropriately capitalized $45,000 of related costs. Gray was amortizing the patent over its estimated useful life of fifteen years. During year 4, Gray paid $15,000 in legal costs in successfully defending an attempted infringement of the patent. After the legal action was completed, Gray sold the patent to the plaintiff for $75,000. Gray's policy is to take no amortization in the year of disposal. In its year 4 income statement, what amount should Gray report as gain from sale of patent?
A
$15,000
B
$24,000
C
$27,000
D
$39,000
A

b

Explanation:
The cost of a successful legal defense of an existing patent is capitalized because it offers probable future benefits.

Original cost of patent (year 1) $ 45,000
Amortization (3 yrs. x $45,000/15) (9,000)
Carrying value 12/31, year 3 $ 36,000
Capitalization of legal costs 15,000
Carrying value of patent (year 4) $ 51,000
Proceeds from sale (year 1) $ 75,000
Carrying value of patent (51,000)
Gain from sale $ 24,000

How well did you know this?
1
Not at all
2
3
4
5
Perfectly
327
Q

Under IFRS, which of the following is a criterion that must be met in order for an item to be recognized as an intangible asset other than goodwill?
A
The item’s fair value can be measured reliably.
B
The item is part of the entity’s activities aimed at gaining new scientific or technical knowledge.
C
The item is expected to be used in the production or supply of goods or services.
D
The item is identifiable and lacks physical substance.

A

d

Explanation:
Under both IFRS and U.S. GAAP, an intangible asset is defined as identifiable nonmonetary assets without physical substance. The recognition criteria for both accounting models require that there be probable future economic benefits and its cost, not its fair value, can be reliably measured. The item need not be expected to be used in the production or supply of goods or services nor be part of the entity’s activities aimed at gaining new scientific or technical knowledge.

How well did you know this?
1
Not at all
2
3
4
5
Perfectly
328
Q

Birk Co. purchased 30% of Sled Co.’s outstanding common stock on December 31, 20X0, for $200,000. On that date, Sled’s stockholders’ equity was $500,000, and the fair value of its identifiable net assets was $600,000. On December 31, 20X0, what amount of goodwill should Birk attribute to this acquisition?

A
$0
B
$20,000
C
$30,000
D
$50,000
A

b

Explanation:
The difference between the purchase price paid to the investee and the fair value of its net assets is accounted as goodwill by the investor. On the date of purchase, the fair value of net assets is $600,000. The proportionate amount of the fair value of investment is $180,000 (i.e. 30% of $600,000). Hence, the goodwill amount will be $20,000 (i.e. $200,000 - $180,000). Option (a) is incorrect because goodwill is recorded at $20,000 and not $0. Option (c) is incorrect because it is taking the difference in the proportionate share of stockholders’ equity $150,000 (i.e. 30% of $500,000) with the fair value of investment is $180,000 (i.e. 30% of $600,000). Option (d) is incorrect because it is taking the difference in the proportionate share of stockholders’ equity $150,000 (i.e. 30% of $500,000) instead of using the proportionate share of fair value of the net assets $180,000 (i.e. 30% of $600,000).

How well did you know this?
1
Not at all
2
3
4
5
Perfectly
329
Q

Goodwill should be tested for value impairment at which of the following levels?

A
Entire business as a whole
B
Each identifiable long-term asset
C
Each acquisition unit
D
Each reporting unit
A

D

Explanation:
Accounting for goodwill is based on reporting units as an aggregate view of goodwill. Goodwill is tested for impairment at a reporting unit level. A reporting unit is an operating segment or one level below an operating segment, referred to as a component. A component of an operating segment is a reporting unit if the component constitutes a business for which discrete financial information is available and segment management regularly reviews the operating results of that component.Option (a), (b) and (c) are incorrect as per the above explanation

How well did you know this?
1
Not at all
2
3
4
5
Perfectly
330
Q

Which of the following would cause an indefinite-lived intangible asset to have an impairment loss?

A
Carrying value is greater than fair value
B
Fair value is greater than carrying value
C
Carrying value plus salvage value is greater than fair value
D
Fair value plus salvage value is greater than carrying value

A

Explanation:
The correct answer is (A).

Testing for impairment occurs when events or changes in circumstances indicate that the carrying amount of a long-lived asset or asset groups may not be recoverable.

The impairment test is a one-step process:

An indefinite-lived intangible asset is impaired when the fair value is less than its carrying amount.

Goodwill is tested for impairment at least annually, using a one-step process, and the goodwill impairment test may be performed any time during the fiscal year, provided the test is performed at the same time every year.

To identify potential impairment, we compare the reporting unit’s fair value with its carrying amount, including goodwill i.e. performing a recoverability test on the carrying amount of the division’s assets

If the fair value exceeds its carrying amount, the reporting unit’s goodwill is considered not impaired.
If the carrying amount exceeds its fair value, then the Impairment Loss of the reporting unit recognized is calculated as Carrying Value – Fair Value i.e. $100,000,000 - $80,000,000 = $20,000,000.
Options C and D are incorrect because Carrying (and Fair) value plus salvage value is greater than fair (and carrying) value is a nonsense distractor.

How well did you know this?
1
Not at all
2
3
4
5
Perfectly
331
Q

During the current year, Orr Co. incurred the following costs:
Research and development services performed by Key Corp. for Orr $150,000
Design, construction, and testing of preproduction prototypes and models 200,000
Testing in search for new products or process alternatives 175,000In its current year income statement, what should Orr report as research and development expense?
A
$150,000
B
$200,000
C
$350,000
D
$525,000

A

d

Explanation:
All three activities are examples of activities that typically are included in research and development and should be expensed. $150,000 + $200,000 + $175,000 = $525,000.

How well did you know this?
1
Not at all
2
3
4
5
Perfectly
332
Q

Which of the following expenditures qualifies for asset capitalization?

A
Cost of materials used in prototype testing
B
Costs of testing a prototype and modifying its design
C
Salaries of engineering staff developing a new product
D
Legal costs associated with obtaining a patent on a new product

A

D

Explanation:
The external acquisition costs of a patent,which includes the legal costs associated with obtaining a patent on a new product,qualifies for asset capitalization. Cost of materials used in prototype testing, costs of testing a prototype and modifying its design,and salaries of engineering staff developing a new product are all examples of research and development costs.These research and development costs are not capitalized, but instead expensed in the year in which incurred.Costs incurred to legally protect product and process ideas resulting from R&D. Following costs are capitalized:

Costs include costs of patent application.
Costs of purchase if the patent is purchased from another party.
Costs incurred in successful defense of a patent if infringed during its economic life
Option (a), (b) and (c) are incorrect because these are R&D costs which are expensed when incurred

How well did you know this?
1
Not at all
2
3
4
5
Perfectly
333
Q

Under IFRS, which of the following statements about intangible assets is correct?

A
Internally generated goodwill cannot be recognized as an asset.
B
Intangible assets within a class may be measured differently using either the cost model or the revaluation model.
C
Research and development costs are capitalized as incurred.
D
Intangible assets with indefinite lives must be amortized annually.

A

Explanation:
The correct answer is (A)

According to IFRS, internally generated goodwill cannot be recognized as an asset. Goodwill is recognized only if it is externally acquired. Internally developed intangible assets under IFRS are normally recognized only if:

It is probable that the expected future economic benefits that are attributable to the asset will flow to the entity.
It is easy and clear to measure the cost of an asset.
(B), (C) and (D) are incorrect as per IFRS Accounting Standards

How well did you know this?
1
Not at all
2
3
4
5
Perfectly
334
Q

Grayson Co. incurred significant costs in defending its patent rights. Which of the following is the appropriate treatment of the related litigation costs?
A
Litigation costs would be capitalized regardless of the outcome of the litigation.
B
Litigation costs would be expensed regardless of the outcome of the litigation.
C
Litigation costs would be capitalized if the patent right is successfully defended.
D
Litigation costs would be capitalized only if the patent was purchased rather than internally developed.

A

C

Explanation:
Legal fees in a successful defense are capitalized because they offer probable future benefits. They are amortized over the remaining useful life of the patent.

How well did you know this?
1
Not at all
2
3
4
5
Perfectly
335
Q

The following information relates to two projects performed by Miley Co. during the year for laboratory research aimed at discovering new knowledge:

Project Costs Likelihood that effort will result in future benefits
$100,000 Probable
$50,000 Reasonably possible
What should Miley report as research and development expenses in its income statement for the year?

A
$0
B
$50,000
C
$100,000
D
$150,000
A

Explanation:
The correct answer is (D).

The R&D costs include:

New knowledge or new technology.
Model or prototype.
Application of new research findings.
Costs incurred for the two projects are expensed as incurred, regardless of the likelihood that efforts will result in future benefits. Miley should report R&D costs of $150,000 ($100,000 + $50,000).

(A) is incorrect because all laboratory research aimed at discovering new knowledge will be reported as R&D costs.

(B) is incorrect because Project I costs of $100,000 should also be expensed regardless of the high probability.

(C) is incorrect because Project II costs of $50,000 should be expensed as R&D costs.

How well did you know this?
1
Not at all
2
3
4
5
Perfectly
336
Q

A company recently acquired a copyright that now has a remaining legal life of 30 years. The copyright initially had a 38-year useful life assigned to it. An analysis of market trends and consumer habits indicated that the copyrighted material will generate positive cash flows for approximately 25 years. What is the remaining useful life, if any, over which the company can amortize the copyright for accounting purposes?

A
0 years.
B
25 years.
C
30 years.
D
38 years
A

Explanation:
Copyrights are amortized over the shorter of useful or legal life. The market trends indicate that the copyrighted materials will generate positive cash flows for approximately 25 years which is shorter than the remaining legal life of 30 years.Thus, Company will amortize copyright for 25 years.Option (a) is incorrect because copyrights are amortized.Option (c) is incorrect because copyrighted material is amortized over 30 years of remaining legal life, while it should be amortized at 25 years useful life.Option (d) is incorrect because copyrighted material was initially amortized over 38years of legal life, now the useful life is reassessed at 25 years which is shorter than the remaining useful legal life, and amortized over 25years.

How well did you know this?
1
Not at all
2
3
4
5
Perfectly
337
Q
On January 2, Judd Co. bought a trademark from Krug Co. for $500,000. Judd retained an independent consultant, who estimated the trademark's remaining life to be 50 years. Its unamortized cost on Krug's accounting records was $380,000. In Judd's December 31 balance sheet, what amount should be reported as accumulated amortization?
A
$7,600
B
$9,500
C
$10,000
D
$12,500
A

c

Explanation:
The $500,000 acquisition cost of the trademark is amortized over the useful life, resulting in accumulated amortization of $10,000 at 12/31. The unamortized cost on Krug’s books is irrelevant in determining Judd’s acquisition cost.

How well did you know this?
1
Not at all
2
3
4
5
Perfectly
338
Q

Which of the following types of assets would typically be reported on a company’s balance sheet as an intangible asset?

A
Derivative securities
B
Cost of research and development
C
Leasehold improvements made by the lessor
D
Cost of patent registrations
A

Explanation:
The correct answer is (D).

Intangible assets are long-lived legal rights and competitive advantages developed or acquired by a business. Intangible assets do not have physical substance. Examples of intangible assets include patents, copyright, goodwill, trademark etc. Patents are costs incurred to legally protect product and process ideas resulting from research and development. Cost of patent registration is capitalized as patent and is reported in the balance sheet as an intangible asset as a patent does not have physical substance.

How well did you know this?
1
Not at all
2
3
4
5
Perfectly
339
Q

Which of the following is a true statement regarding the accounting and reporting requirements related to intangible costs under both IFRS and U.S. GAAP?
A
Amortization of intangible assets over their estimated useful lives is required under both U.S. GAAP and IFRS, with one minor exception in U.S. GAAP.
B
Under U.S. GAAP, development costs are capitalized unless addressed by a separate standard.
C
Under IFRS, advertising and promotional costs are capitalized.
D
Under both IFRS and U.S. GAAP, revaluation is not permitted.

A

A

Explanation:
Amortization of intangible assets over their estimated useful lives is required under both U.S. GAAP and IFRS, with one minor exception in U.S. GAAP. Under U.S. GAAP, development costs are expensed as incurred, not capitalized, unless addressed by a separate standard. Under IFRS, advertising and promotional costs are expensed as incurred. Under U.S. GAAP, revaluation is not permitted. Under IFRS, revaluation to fair value of intangible assets other than goodwill is a permitted accounting policy election for a class of intangible assets.
How well did you know this?
1
Not at all
2
3
4
5
Perfectly
340
Q

Which of the following types of assets would typically be reported on a company’s balance sheet as an intangible asset?

A
Derivative securities.
B
Cost of research and development.
C
Leasehold improvements made by the lessor.
D
Cost of patent registrations.
A

D

Explanation:
Intangible assets are long-lived legal rights and competitive advantages developed or acquired by a business. Intangible assets do not have physical substance.Examples of intangible assets includes patents, copyright, goodwill, trademark etc.Patents are costs incurred to legally protect product and process ideas resulting from research and development. Cost of patent registration is capitalized as patent and is reported in the balance sheet as an intangible asset as a patent does not have physical substance.

Option (a) is incorrect because derivative securities is not an intangible asset.Derivative securities usually takes the form of an agreement to buy or sell an asset or item (commodity, property, security) at a fixed price on or before a certain date.Derivative securities are traded on exchanges like other financial instruments, and their value varies with the value of the underlying assets. These have physical substance and are not intangibles.

Option (b) is incorrect because even though research and development (R&D) is an intangible, the cost of R&Dis typically expensed as a period cost and is not reported as an asset on the balance sheet. Research is a discovery of new knowledge for anew or significantly improved product or process while development is conversion of the knowledge into plan / design for application.Option (c) is incorrect because a leasehold improvement made by the lessor is capitalized as an asset and depreciated. Only a leasehold improvement made by the lessee is treated as an intangible asset in the lessee’s books.

How well did you know this?
1
Not at all
2
3
4
5
Perfectly
341
Q
On June 30, Union, Inc., purchased goodwill of $125,000 when it acquired the net assets of Apex Corp. During the year Union incurred additional costs of developing goodwill, by training Apex employees ($50,000) and hiring additional Apex employees ($25,000). Union's December 31 balance sheet should report goodwill of
A
$200,000.
B
$175,000.
C
$150,000.
D
$125,000.
A

D

Explanation:
Only the purchased goodwill of $125,000 from the acquisition of the net assets of Apex should be reported. Costs of developing, maintaining, or restoring goodwill should be expensed when incurred

How well did you know this?
1
Not at all
2
3
4
5
Perfectly
342
Q

Under IFRS, which of the following statements about intangible assets is correct?
A
Internally generated goodwill cannot be recognized as an asset.
B
Intangible assets within a class may be measured differently using either the cost model or the revaluation model.
C
Research and development costs are capitalized as incurred.
D
Intangible assets with indefinite lives must be amortized annually

A

A

Explanation:
Goodwill is recognized only in a business combination under both US GAAP and IFRS. With the exception of development costs, internally developed intangibles are not recognized as an asset under either US GAAP or IFRS. Under US GAAP, revaluation is not permitted. Under IFRS, revaluation to fair value of intangible assets other than goodwill is a permitted accounting policy election for a class of intangible assets. Under IFRS, development costs are capitalized when technical and economic feasibility of a project can be demonstrated in accordance with specific criteria. Amortization of intangible assets over their estimated useful lives is generally required under both US GAAP and IFRS. If there is no foreseeable limit to the period over which an intangible asset is expected to generate net cash inflows, the useful life is considered indefinite and the asset is not amortized.
How well did you know this?
1
Not at all
2
3
4
5
Perfectly
343
Q

In year 3, a company incurred $500,000 of legal costs defending several patents. Included in that amount was $400,000 of legal costs associated with successful outcomes and $100,000 of legal costs associated with unsuccessful outcomes. What amount of legal costs, if any, should the company expense for year 3?

A
$500,000
B
$400,000
C
$100,000
D
$0
A

c

Explanation:
The costs incurred in successful defense of patents are capitalized. Costs on failure to defend patent are expensed as no legal benefit would exist. Thus, legal costs associated with unsuccessful outcomes is expensed at $100,000.Option (a) is incorrect because it includes legal costs associated with successful defense of the patent which should be capitalized. ($500,000 = $400,000 + $100,000).Option (b) is incorrect because legal costs associated with successful defense of the patent are expensed, instead of capitalizing it. Option (d) is incorrect because the legal costs for unsuccessful out comes are expensed

How well did you know this?
1
Not at all
2
3
4
5
Perfectly
344
Q
Which of the following is an adverse qualitative factor related to goodwill?
A
Unanticipated competition
B
Loss of key personnel
C
Adverse action or assessment by a regulator
D
All of the above
A

d

Explanation:
All of the factors listed are considered adverse qualitative factors related to testing goodwill impairment.

How well did you know this?
1
Not at all
2
3
4
5
Perfectly
345
Q

A company is completing its annual impairment analysis of the goodwill included in one of its cash generating units (CGUs). The recoverable amount of the CGU is $32,000. The company noted the following related to the CGU:

Goodwill Patents Other assets Total

Historical Cost $15,000 $10,000 $35,000 $60,000
Depreciation and amortization ______0 __3,333 __11,667 __15,000
Carrying amount, December 31 $15,000 $ 6,667 $23,333 $45,000
Under IFRS, which of the following adjustments should be recognized in the company’s consolidated financial statements?

A
Decrease goodwill by $13,000
B
Decrease goodwill by $15,000
C
Decrease goodwill by $3,250; patents by $2,167; and other assets by $7,583
D
Decrease goodwill by $4,333; patents by $1,926; and other assets by $6,741
A

a

Explanation:
Under IFRS, a one-step approach requires that an impairment test be done at the cash-generating unit (CGU) level by comparing the CGU’s carrying amount, including goodwill, with its recoverable amount. Any impairment loss on the CGU (amount by which the CGU’s carrying amount, including goodwill, exceeds its recoverable amount) is allocated first to reduce goodwill to zero, then, subject to certain limitations, the carrying amount of other assets in the CGU are reduced pro rata, based on the carrying amount of each asset. In this case, the entire impairment loss of $13,000 ($45,000 – $32,000) would be allocated to goodwill (leaving a bal­ance of $2,000).

How well did you know this?
1
Not at all
2
3
4
5
Perfectly
346
Q

Johan Co. has an intangible asset, which it estimates will have a useful life of 10 years, while Abco Co. has goodwill, which has an indefinite life. Which company should report amortization in its financial statements?

Johan	Abco
A	Yes	Yes
B	Yes	No
C	No	Yes
D	No	No
A

B

Explanation:
For identifiable intangibles with definite life straight line method (SLM) is used to amortize the intangible assets over the lower of useful life or legal life. The intangibles with indefinite life cannot be amortized and must be tested for impairment. Therefore only Johan Co. will report amortization in its financial statements.Option (a), (c) and (d) are incorrect because Abco Co., should test goodwill for impairment since it is an Intangible asset with indefinite life.

How well did you know this?
1
Not at all
2
3
4
5
Perfectly
347
Q

Under IFRS, an entity that acquires an intangible asset may use the revaluation model for subsequent measurement only if

A
The useful life of the intangible asset can be readily determined.
B
An active market exists for the intangible asset.
C
The cost of the intangible asset can be measured reliably.
D
The intangible asset is a monetary asset.

A

B

Explanation:
Under IFRS, revaluation to fair value of intangible assets other than goodwill is a permitted account­ing policy election for a class of intangible assets. Revaluation to fair value requires an active market for the specific type of intangible. Though a reliable useful life and cost are used for amortization, they are not elements required for use of the revaluation model. By definition, intangible assets are nonmonetary assets.
How well did you know this?
1
Not at all
2
3
4
5
Perfectly
348
Q

A company reported $6 million of goodwill in last year’s statement of financial position. How should the company account for the reported goodwill in the current year?

A
Determine the current year’s amortizable amount and report the current-year’s amortization expense.
B
Determine whether the fair value of the reporting unit is greater than the carrying amount and report a gain on goodwill in the income statement.
C
Determine whether the fair value of the reporting unit is less than the carrying amount and report an impairment loss on goodwill in the income statement.
D
Determine whether the fair value of the reporting unit is greater than the carrying amount and report the recovery of any previous impairment in the income statement.

A

c

Explanation:
The intangibles with indefinite life cannot be amortized and must be tested for impairment. Goodwill is an unidentifiable intangible asset that represents capitalized excess earning power. Goodwill is not amortized but tested for impairment annually.If the carrying value of reporting unit is greater than the fair value, an impairment loss for the excess of carrying value of goodwill is reported in the I/S.Option (a) is incorrect because goodwill is not amortized.Option (b) is incorrect because gain on goodwill is not reported.Option (d) is incorrect because restoration of previously recognized impairment loss is not allowed, unless the asset is held for disposal.

How well did you know this?
1
Not at all
2
3
4
5
Perfectly
349
Q

Northstar Co. acquired a registered trademark for $600,000. The trademark has a remaining legal life of five years, but can be renewed every 10 years for a nominal fee. Northstar expects to renew the trademark indefinitely. What amount of amortization expense should Northstar record for the trademark in the current year?

A
$0
B
$15,000
C
$40,000
D
$120,000
A

a

Explanation:
A trademark is an identifiable tangible asset.With an externally acquired trademark, normally the acquisition costs are capitalized and amortized over the useful life of the trademark.If an intangible asset has an indefinite life, as is the case with the trademark in this question, it is not amortized but rather tested at least annually for impairment until its useful life is determined to be no longer indefinite.If the trademark had been internally developed the costs would have been expensed as incurred. Option (b) is incorrect because trademark is amortized over 40 years. ($15,000 =$600,000/40). Option (c) is incorrect because trademark is amortized over 15 years. ($40,000 =$600,000/15). Option (d) is incorrect because trademark is amortized over 5 years. ($120,000 =$600,000/5)

How well did you know this?
1
Not at all
2
3
4
5
Perfectly
350
Q

West Co. paid $50,000 for an intangible asset other than goodwill. The fair value of the asset is $55,000. West signed a contract to sell the asset for $10,000 in 10 years. What amount of amortization expense should West record each year?

A
$4,000
B
$4,500
C
$5,000
D
$5,500
A

Explanation:
The correct answer is (A).

Amount to be capitalized is equal to the fair value of the consideration given or the fair value of the asset acquired, whichever is more clearly evident. When both are given, the fair value of the consideration given is used i.e. $50,000.

Amortization for an identifiable intangible with a definite life is done using the straight-line method. Amortization per year = (Capitalized amount less value of the asset at the end of its useful life) / Lower of useful life or legal life of the asset.

The amount of an intangible asset to be amortized is the amount initially assigned to that asset ($50,000) less any residual value ($10,000). The amount to be amortized is $40,000 ($50,000 - $10,000) over 10 years, or $4,000 a year.

Note: The residual value is the estimated fair value of the intangible asset at the end of its useful life to the reporting entity less any disposal costs.

How well did you know this?
1
Not at all
2
3
4
5
Perfectly
351
Q

An intangible asset with a finite useful life is amortized. Which of the following factors is not considered in the estimate of the finite useful life?
A
The expected use of the asset by the entity
B
Any legal, regulatory, or contractual provisions that may limit the useful life
C
Goodwill
D
The effects of competition

A

c

Explanation:
Goodwill is not a factor to be considered in the estimate of a finite useful life. Goodwill has an indefinite, not finite, useful life and is not amortized, but rather is tested at least annually for impairment.

How well did you know this?
1
Not at all
2
3
4
5
Perfectly
352
Q

OnJune30,Union Inc. purchased goodwill of $125,000 when it acquired the net assets of Apex Corp.During the year, Union incurred additional costs of developing goodwill, by training Apex employees ($50,000) and hiring additional Apex employees($25,000).Union’sDecember31balance sheet should report goodwill of

A
$200,000
B
$175,000
C
$150,000
D
$125,000
A

d

Explanation:
Costs of goodwill from a business combination accounted for as a purchase should be capitalized.However, costs of developing, maintaining, or restoring goodwill should be expensed when incurred.Thus, the good will of $125,000 from the acquisition of the net assets of Apex should be capitalized,while the additional costs of developing good will should be expensed as incurred. Option (a) is incorrect because it includes both additional costs of developing goodwill, by training Apex employees ($50,000) and hiring additional Apex employees($25,000) which should be expensed off. Option (b) is incorrect because it includes cost of training Apex employees ($50,000) which should be expensed off. Option (c) is incorrect because it includes cost of hiring additional Apex employees($25,000) which should be expensed off.

How well did you know this?
1
Not at all
2
3
4
5
Perfectly
353
Q

On December 31, 20X0, Bit Co. had capitalized costs for a new computer software product with an economic life of five years. Sales for 20X1 were 30 percent of expected total sales of the software. At December 31, 20X1, the software had a net realizable value equal to 90 percent of the capitalized cost. What percentage of the original capitalized cost should be reported as the net amount on Bit’s December 31, 20X1,balance sheet?

A
70%
B
72%
C
80%
D
90%
A

a

Explanation:
The capitalized costs of research and development (R&D) is amortized for the greater of (a) cost x ratio of current year revenue of software to the total revenue expected or (b) straight-line method which is cost / economic life of product. The ratio of current year revenue of software to the total revenue expected is given as 30% and the straight-line rate will be 20% (1 / 5 years). Thus, the amortization would be at the greater of the two percentages = 30%. The percentage of original capitalized cost to be reported in the balance sheet would therefore be 70% (i.e.100% - 30%). Capitalized software cost should be reported at the lower of carrying value (70% of original cost) or net realizable value (90% of original cost). Thus, the software cost would be reported at 70%. Options (b), (c) and (d) are incorrect because of above explanation.

How well did you know this?
1
Not at all
2
3
4
5
Perfectly
354
Q

A company began developing computer software to be sold as a separate product on January 1, year 1. During the planning, coding, and testing phases, the company incurred $1,300,000 of costs. On June 30, year 1, the product was determined to be technologically feasible. The company began producing product masters of the software and incurred an additional $750,000 of costs from July 1, year 1, through September 30, year 1. After the software was available for release on October 1, year 1, the company incurred an additional $275,000 of costs relating to maintenance and customer support. What amount of software-related costs should be capitalized?

A
$275,000
B
$750,000
C
$1,300,000
D
$2,050,000
A

Explanation:
The correct answer is (B).

Costs associated with computer software may be capitalized once technological feasibility has been established. The costs will continue to be capitalized until the date the software is released for sale. The software was determined to be technologically feasible on June 30th and was released for sale on October 1st. Between July 1st and September 30th, costs of $750,000 are capitalized. All costs incurred prior to June 30th and after October 1st are expensed. Examples of expensed items include sales revenue and R&D.

(A) is incorrect because $275,000 costs incurred after software sales begin are inventories and included in COGS.

(C) is incorrect because $1,300,000 should be expensed as these costs incurred prior to technical feasibility was established.

(D) is incorrect because it includes expenses incurred prior to the technical feasibility was established and the costs incurred after the product was available for release ($2,050,000 = $1,300,000 + $750,000).

How well did you know this?
1
Not at all
2
3
4
5
Perfectly
355
Q
Tech Co. bought a trademark on January 2, two years ago. Tech accounted for the copyright as instructed under the appropriate GAAP guidance during the current year. The intangible was being amortized over 40 years. The carrying value at the beginning of the year was $38,000. It was determined that the cash flow will be generated indefinitely at the current level for the trademark. What amount should Tech report as amortization expense for the current year?
A
$0
B
$922
C
$1,000
D
$38,000
A

a

Explanation:
An intangible asset with an indefinite useful life is not amortized. If no legal, regulatory, contractual, competitive, economic, or other factors limit the useful life, it is considered to be indefinite. The term indefinite does not mean infinite. Goodwill and intangible assets with indefinite useful lives are not amortized, but rather are tested at least annually for impairment.

How well did you know this?
1
Not at all
2
3
4
5
Perfectly
356
Q

Which of the following is a pair of values that are compared to determine the amount of a possible impairment loss on an intangible asset, with an indefinite life, other than goodwill?

A
Fair value, present value
B
Carrying value, book value
C
Future value, carrying value
D
Fair value, carrying value
A

Explanation:
The correct answer is (D).

Indefinite life intangibles should be tested for impairment annually or more frequently if needed. If the circumstances indicating impairment exist, then the indefinite life intangible is tested for impairment by comparing its carrying value with its fair value. If the carrying value of the indefinite life intangible exceeds its fair value then it is impaired and the difference between the two values is recorded as an impairment loss and the value of the asset is reduced by that amount. Thus, the carrying value and the fair value of the indefinite life intangible are compared to determine the amount of possible impairment loss

How well did you know this?
1
Not at all
2
3
4
5
Perfectly
357
Q
On January 2, Gant Co. purchased a franchise with a useful life of five years for $60,000 and an annual fee of 1% of franchise revenues. Franchise revenues were $20,000 during the year. Gant projects future revenues of $40,000 next year and $60,000 per year for the following three years. Gant uses the straightline method of amortization. What amount should Gant report as intangible asset-franchise, net of related amortization in its December 31 balance sheet?
A
$48,000
B
$48,160
C
$49,920
D
$56,000
A

a

Explanation:
A franchise represents a special right to operate under the name and guidance of another enterprise over a limited geographic area. A franchise is always externally purchased; it cannot be internally developed. Capitalize all significant costs incurred to acquire the franchise (e.g., purchase price, legal fees, etc.). If the acquisition cost of the franchise requires future cash payments, these payments should be capitalized at their present value using an appropriate interest rate. On the other hand, periodic service fees charged as a percentage of revenues are not capitalized; these costs represent a current operating expense of the franchisee. The $60,000 frinchise purchase divided by the five years of useful life would mean the intangible asset-franchise would be amortized $12,000 per year. $60,000 - $12,000 = $48,000.

How well did you know this?
1
Not at all
2
3
4
5
Perfectly
358
Q

Anchor Co. is experiencing financial difficulties. Anchor negotiated a settlement of $100,000 in debt owed to Bowden, Inc. in exchange for Anchor’s gross receivables of $100,000. The receivables have an allowance for uncollectible accounts of $25,000. The impact of this transaction on Anchor’s net income is a $25,000

A
Increase in bad debt expense.
B
Gain on restructuring of payables.
C
Loss on restructuring of payables.
D
Decrease in bad debt expense
A

Explanation:
The correct answer is (B)

Anchor Co. is experiencing financial difficulties. Anchor negotiated a settlement of $100,000 in debt owed to Bowden Inc. in exchange for Anchor’s gross accounts receivables of $100,000. This is a troubled debt restructuring, and the debt is settled by a transfer of assets (A/R). The debtor recognizes a gain equal to the excess of the carrying amount of the debt over the carrying amount of the assets. The carrying amount of the receivables transferred is $75,000 because $25,000 is deemed uncollectible. Anchor will recognize a $25,000 gain on restructuring of payables.

How well did you know this?
1
Not at all
2
3
4
5
Perfectly
359
Q
On January 1, Stunt Corp. had outstanding convertible bonds with a face value of $1,000,000 and an unamortized discount of $100,000. On that date, the bonds were converted into 100,000 shares of $1 par stock. The market value on the date of conversion was $12 per share. The transaction will be accounted for with the book value method. By what amount will Stunt's stockholders' equity increase as a result of the bond conversion?
A
$ 100,000
B
$ 900,000
C
$1,000,000
D
$1,200,000
A

b

Explanation:
Convertible bonds provide the bondholder the option of converting the bond to capital stock, typically common stock. Using the book value method, the conversion of the bonds into common stock is generally recorded by crediting the paid-in capital accounts for the carrying amount of the debt at the date of the conversion, less any cost associated with the conversion. The carrying amount of the bonds on the date of conversion is the $1,000,000 face value less the $100,000 unamortized discount. The market value is not considered when using the book value method. The journal entry:

Bonds Payable 1,000,000
Bond Discount 100,000
Common Stock (100,000 × $1 par) 100,000
APIC (to balance) 800,000The $100,000 credit to common stock and $800,000 credit to APIC would increase stockholders’ equity $900,000.

How well did you know this?
1
Not at all
2
3
4
5
Perfectly
360
Q
On December 31, Roth Co. issued a $10,000 face value note payable to Wake Co. in exchange for services rendered to Roth. The note, made at usual trade terms, is due in nine months and bears interest, payable at maturity, at the annual rate of 3%. The market interest rate is 8%. The compound interest factor of $1 due in nine months at 8% is .944. At what amount should the note payable be reported in Roth's December 31 balance sheet?
A
$10,300
B
$10,000
C
$ 9,652
D
$ 9,440
A

b

Explanation:
The note payable arose from a transaction with a vendor in the normal course of business and is due in customary trade terms not exceeding one year; therefore, the note can be reported at its face amount of $10,000, despite the fact that the 3% stated interest rate of the note does not approximate the prevailing market interest rate of 8% for similar notes at the date of the transaction.

How well did you know this?
1
Not at all
2
3
4
5
Perfectly
361
Q

Album Co. issued 10-year $200,000 debenture bonds on January 2. The bonds pay interest semiannually. Album uses the effective interest method to amortize bond premiums and discounts. The carrying value of the bonds on January 2 was $185,953. A journal entry was recorded for the first interest payment on June 30, debiting interest expense for $13,016 and crediting cash for $12,000. What is the annual stated interest rate for the debenture bonds?

A
6%
B
7%
C
12%
D
14%
A

c

Explanation:
Interest paid = Face value x Stated interest rate.

On 30th Jun the interest paid is semi-annual, hence $12,000 x 2 = $24,000 represents annual interest paid.

$12,000/$200,000 = 6% which is semi-annual rate. To arrive at the annual stated interest rate for the debenture of 12%, semi-annual rate should be multiplied by 2. 12% = (6% x 2). (OR) using annual amount 24,000/$200,000 = 12%.

Option (A) is incorrect because it represents the semi-annual rate 6% ($12,000/$200,000).
Option (B) is incorrect because 7% is calculated on discounted amount and effective interest expense ($13,016/$185,953) for semi-annual period.
Option (D) is incorrect because 14% is the annualized effective interest rate (7% x 2).

362
Q
On July 1, year 2, Flax Corporation issued 2,000 of its 9%, $1,000 callable bonds for $1,920,000. The bonds are dated July 1, year 2 and mature on July 1, year 12. Interest is payable semiannually on January 1 and July 1. Flax uses the straight-line method of amortizing bond discount. The bonds can be called by the issuer at 101 at any time after June 30, year 7. On July 1, year 8, Flax called in all of the bonds and retired them. Before income taxes, how much loss should Flax report on this early extinguishment of debt for the year ended December 31, year 8?
A
$ 20,000
B
$ 52,000
C
$ 68,000
D
$100,000
A

b

Explanation:
The loss on the early extinguishment of debt is calculated as the difference between the cost of reacquiring the bonds and the carrying amount of the bonds.

Face amount of bonds (2,000 × $1,000) $2,000,000
Less unamortized discount at extinguishment:
Discount at issuance ($2,000,000 - $1,920,000) $80,000
Amortized to date of extinguishment ($80,000 × 6/10) 48,000 32,000
Bond carrying amount at extinguishment 1,968,000
Cost to reacquire ($2,000,000 × 101%) 2,020,000
Loss on early extinguishment of debt, before taxes $ 52,000

363
Q

An issuer of bonds uses a sinking fund for the retirement of the bonds. Cash was transferred to the sinking fund and subsequently used to purchase investments. The sinking fund

Increases by revenue earned on the investments.
Is not affected by revenue earned on the investments.
Decreases when the investments are purchased.
A
I only.
B
I and III.
C
II and III.
D
III only.

A

A

Explanation:
When cash is transferred to a sinking fund account, the sinking fund balance increases for the amount of cash transferred. When this cash is used to purchase investments, there is no effect in the balance of sinking fund, only the composition changes - cash becomes investments or a current asset becomes a non-current asset. Revenue earned on the investments is reported as income in the income statement and is added to the sinking fund account. Therefore, the sinking fund increases with the revenue earned on investments and is not affected when investments are purchased.Option (b) and (d) are incorrect because statement III has no impact on bond sinking fund account. Cash transferred to sinking fund account which was current asset is now reported as non-current investments.Option (c) is incorrect because statement II increases the bond sinking fund investment account and statement III has no impact because cash is converted to investments and classified from current to non-current assets.

364
Q

A company issues bonds at 98, with a maturity value of $50,000. The entry the company uses to record the original issue should include which of the following?

A
A debit to bond discount of $1,000.
B
A credit to bonds payable of $49,000.
C
A credit to bond premium of $1,000.
D
A debit to bonds payable of $50,000.
A

A
Explanation:
When bond is issued for less than face value i.e. $98, the bond is issued at discount.

J/E for bond issued at discount:

DR Cash $49,000
DR Discount on bond issuance $1,000
CR Bonds payable $50,000
Option (B) is incorrect because bonds payable is credited at $50,000 and not at discounted value.

Option (C) is incorrect because the bond is issued at discount, not premium.

Option (D) is incorrect because bonds payable is a liability and should be credited not debited.

365
Q

On December 31, year 2, Paxton Co. had a note payable due on August 1, year 3. On January 20, year 3, Paxton signed a financing agreement to borrow the balance of the note payable from a lending institution to refinance the note. The agreement does not expire within one year, and no violation of any provision in the financing agreement exists. On February 1, year 3, Paxton was informed by its financial advisor that the lender is not expected to be financially capable of honoring the agreement. Paxton’s financial statements were issued on March 31, year 3. How should Paxton classify the note on its balance sheet at December 31, year 2?
A
As a current liability because the financing agreement was signed after the balance sheet date
B
As a current liability because the lender is not expected to be financially capable of honoring the agreement
C
As a long-term liability because the agreement does not expire within one year
D
As a long-term liability because no violation of any provision in the financing agreement exists

A

b

Explanation:
Short-term obligations are those scheduled to mature within one year or operating cycle, whichever is longer. Generally, short-term obligations are classified as current liabilities since they will require use of working capital during the ensuing period. If they are to be refinanced on a long-term basis they will not require the use of working capital and appropriately are classified as noncurrent liabilities. Exclusion from current liabilities requires that two conditions be met: (1) the enterprise must intend to refinance the obligation on a long-term basis, and (2) the enterprise must have the ability to consummate the financing. Evidence of the ability to consummate the refinancing is provided by either a refinancing that occurs after the balance sheet date but before the balance sheet is issued or a financing agreement before the balance sheet is issued that permits the refinancing and extends beyond one year or operating cycle. If any violation of the agreement has occurred, a waiver from the lender must be obtained. Further, the lender must be expected to be financially capable of honoring the agreement. This problem initially involves the refinancing of a short-term note payable (due within a year) on a long-term basis (because the agreement does not expire within a year). The note payable meets the first reclassification requirement to exclude it from current liabilities in that the enterprise intends to refinance the obligation on a long-term basis. The enterprise also provides an ability to consummate the refinancing because the agreement was signed on January 20, year 3, after the balance sheet date of December 31, year 2, but before the balance sheet is issued, March 31, year 3. Because the lender wasn’t expected to be financially capable of honoring the agreement, the note must be classified as a current liability.

366
Q
How would the amortization of premium on bonds payable affect each of the following?
Carrying amount of bond	Net income
A	Increase	Decrease
B	Increase	Increase
C	Decrease	Decrease
D	Decrease	Increase
A

D

Explanation:
The carrying amount of the bonds is the sum of their face amount and the unamortized premium. Therefore, premium amortization will reduce the carrying amount of the bonds. Bond premium amortization will increase income because it will reduce the interest expense associated with the bonds.

367
Q

On December 1, year 1, Tigg Mortgage Co. gave Pod Corp. a $200,050, 12% loan. Pod received proceeds of $193,829 after the deduction of a $6,221 nonrefundable loan origination fee. Principal and interest are due in 60 monthly installments of $4,450, beginning January 1, year 2. The repayments yield an effective interest rate of 12% at a present value of $200,050 and 13.4% at a present value of $193,829. What amount of accrued interest receivable should Tigg include in its December 31, year 1, balance sheet?

A
$4,450
B
$2,164
C
$2,000
D
$0
A

c

Explanation:
The note was issued on 12/1, year 1. Principal and interest on the note are due in monthly installments, beginning 1/1, year 2. Therefore, at 12/31, year 1, Tigg should report one month of accrued interest receivable, the amount of which is determined by multiplying the face amount of the note by 1/12 of the note’s stated annual interest rate, as follows:

Face amount of note, issued 12/1, year 1 $200,050
Times 1/12 stated annual interest rate of note (12% ÷ 12) × 1%
Accrued interest receivable, 12/31, year 1 $ 2,000
A helpful way of looking at this problem is making the journal entries that Tigg Company made at 12/1, year 1 and 12/31, Year 1.

12/1, Year 1
Notes Receivable $200,050
Loan Orig. Fees $6,221
Cash $193,829

12/31, Year 1	 	 
Accrued Int. Rec. (200,050 × 12% × 1/12)	$2,000	 
Amort. Loan Orig. Fees (Plug)	$164	 
Interest Income (193,829 × 13.4% × 1/12)	 	$2,164
1/1, Year 2	 	 
Cash	$4,450	 
Accrued Interest Receivable	 	$2,000
Notes Receivable	 	$2,450
368
Q

Clay Corp. had $600,000 convertible 8% bonds outstanding at June 30. Each $1,000 bond was convertible into 10 shares of Clay’s $50 par value common stock. On July 1, the interest was paid to bondholders, and the bonds were converted into common stock, which had a fair market value of $75 per share. The unamortized premium on these bonds was $12,000 at the date of conversion. Under the book value method, this conversion increased the following elements of the stockholders’ equity section by

Common stock	Additional paid-in capital
A	$300,000	$312,000
B	$306,000	$306,000
C	$450,000	$162,000
D	$600,000	$12,000
A

a

Explanation:
Under the book value method, the paid-in capital accounts are credited for the carrying amount of the debt; no gain or loss is recognized on the conversion.
The market price per common share is irrelevant under this method.

Bonds payable 600,000
Bond premium 12,000
Common stock [($600,000 / 1,000) × 10 shs. × $50 PV] 300,000
Additional paid-in capital (to balance) 312,000

369
Q

On June 1 of the current year, Cross Corp. issued $300,000 of 8% bonds payable at par with interest pay­ment dates of April 1 and October 1. In its income statement for the current year ended December 31, what amount of interest expense should Cross report?

A
$6,000
B
$8,000
C
$12,000
D
$14,000
A

D

Explanation:
Interest expense for the period would be interest to be paid from 1st Jun to 31st Dec, which is for 7 months ($300,000 x 8% x 7/12) = $14,000.

Option (A) is incorrect because $6,000 is the accrued amount for interest from the last payment date of 1st Oct for 3 months ($300,000 x 8% x 3/12) = $6,000.
Option (B) is incorrect because $8,000 is the interest to be paid on 1st Oct for period of 4 months from 1st Jun to 1st Oct ($300,000 x 8% x 4/12) = $8,000.
Option (C) is incorrect because it represents the semi-annual interest payment ($300,000 x 8% x 6/12) = $8,000.

370
Q

On January 2, Vole Co. issued bonds with a face value of $480,000 at a discount to yield 10%. The bonds pay interest semiannually. On June 30, Vole paid bond interest of $14,400. After Vole recorded amortization of the bond discount of $3,600, the bonds had a carrying amount of $363,600. What amount did Vole receive upon issuing the bonds?

A
$360,000
B
$367,200
C
$476,400
D
$480,000
A

a

Explanation:
Carrying value of the bonds Payable = Face value of the bond - Unamortized discount on bond Issue.

Ending carrying value of bond = Beginning balance + Amortization expense. When bond is amortized, Beginning period balance is added to amortized portion to arrive at ending carrying value of the bonds $360,000 = ($363,600 - $3,600). Amortization of $3,600 is reduced from the $363,600 carrying value at the end of the year which is otherwise added to the beginning value.

Option (B) is incorrect because it is adding the semi-annual amortization for period 0.5 to the ending balance.
Option (C) is incorrect because $476,400 is reducing amortizing expense $3,600.
Option (D) is incorrect because $480,000 is the bonds payable at maturity.

371
Q
At December 31 of the prior year, Cain, Inc. owed notes payable of $1,750,000, due on May 15, of the current year. Cain expects to retire this debt with proceeds from the sale of 100,000 shares of its common stock. The stock was sold for $15 per share on March 10 of the current year, prior to the issuance of the year-end financial statements. In Cain's prior year December 31 balance sheet, what amount of the notes payable should be excluded from current liabilities?
A
$0
B
$ 250,000
C
$1,500,000
D
$1,750,000
A

c
Explanation:
Since after the date of the enterprise’s balance sheet but before the balance sheet was issued, $1,500,000 (100,000 x $15) of common stock was issued for the purpose of refinancing the note payable on a long-term basis, $1,500,000 of the note payable should be excluded from current liabilities at the balance sheet date.

372
Q

Which of the following statements is false regarding the amortization of bonds?
A
Straight-line amortization calls for the amortization of an equal amount of premium or discount each period over the life of the bonds.
B
The straight-line method is acceptable in all cases.
C
The effective interest method of amortization calls for recognizing interest expense at the effective interest rate at which the bonds were sold.
D

A

b

Explanation:
The straight-line method is acceptable only when the premium or discount is immaterial, because it fails to determine the periodic interest expense in terms of the effective rate of interest.

373
Q

Webb Co. has outstanding a 7%, 10-year $100,000 face-value bond. The bond was originally sold to yield 6% annual interest. Webb uses the effective interest rate method to amortize bond premium. On June 30, year 2, the carrying amount of the outstanding bond was $105,000. What amount of unamortized premium on bond should Webb report in its June 30, year 3, balance sheet?

A
$1,050
B
$3,950
C
$4,300
D
$4,500
A

C

Explanation:
Unamortized bond premium, 6/30, year 2 ($105,000 - $100,000) $5,000
Bonds payable carrying amount, 6/30, year 2 $105,000
Times: Annual effective interest rate × _ 6%
Interest expense, 6/30, year 2 - 6/30, year 3 $ 6,300
Annual interest payment ($100,000 × 7%) _(7,000)
Bond premium amortization, 6/30, yr 2 - 6/30, yr3 __ (700)
Unamortized bond premium, 6/30, year 3 $4,300
Option (A), (B), and (D) are incorrect due to improper calculations

374
Q

On July 31 of the current year, Dome Co. issued $1,000,000 of 10%, 15-year bonds at par and used a por­tion of the proceeds to call its 600 outstanding 11%, $1,000 face value bonds, due in ten years on July 31, at 102. On that date, unamortized bond premium relating to the 11% bonds was $65,000. In its year-end income statement, what amount should Dome report as gain or loss, before income taxes, from retirement of bonds?

A
$ 53,000 gain
B
$0
C
$(65,000) loss
D
$(77,000) loss
A

A

Explanation:
A gain is recognized because the cost to redeem the bonds is less than the carrying amount of the bonds.

Face amount of bonds retired (600 × $1,000) $ 600,000
Add: Unamortized bond premium _ 65,000
Bond carrying amount at retirement date $ 665,000
Less: Cost to retire ($600,000 × 102%) __(612,000)
Pretax gain on retirement of bonds $ 53,000
Option (B) is incorrect because gain is reported for the difference between carrying value and amount paid.
Option (C) is incorrect because retirement of bonds resulted in a gain and not a loss. It incorrectly treats unamortized premium as loss instead of adding it to the face value of the bonds to be retired to arrive at its carrying value. Once the carrying value is computed, amount paid to retire bonds is deducted from carrying value of the bonds to arrive at gain or loss.
Option (D) is incorrect because this option is adding the redemption premium of $12,000 to the unamortized premium of $65,000 to get the total loss. Refer explanation above.

375
Q

A marketable debt security was purchased on September 1 of the current year between interest dates and classified as a held-to-maturity debt security. The next interest payment date was February 1, next year. Because of a permanent decline in market value, the cost of the debt security substantially exceeded its market value at December 31. On the balance sheet at December 31 of the current year, the debt security should be carried at

A
Market value plus the accrued interest paid
B
Market value
C
Cost plus the accrued interest paid
D
Cost
A

b

Explanation:
Subsequent to purchase, debt securities classified as held-to-maturity are carried in the investment account at cost, net of premium or discount amortization to date, with a separate valuation account for any difference between unamortized cost and fair value. If there is a decline in market value which is deemed permanent, then the security should be written down to fair value by a credit to the investment account, and a realized loss is recognized. Debt securities purchased between interest dates should be recorded separately from accrued interest.

Investment in marketable debt securities XX
Accrued interest receivable XX
Cash (cost of security plus accrued interest) XX

376
Q

Which of the following statements is false with respect to serial bonds?
A
To determine the selling price of serial bonds, compute the present value of the principal and interest payments for each series separately, then total the present value of each series.
B
The amortization of bond premium or discount on serial bonds requires the recognition of a declining debt principal.
C
Successive bond years cannot be charged with equal amounts of premium or discount because of a shrinking debt and successively smaller interest payments.
D
Bond premium or discount, if material, should be amortized using the straight-line method.

A

d

Explanation:
Bond premium or discount, if material, should be amortized using the effective interest method prescribed.

377
Q
How would the amortization of discount on bonds payable affect each of the following?
Carrying amount of bond	Net Income
A	Increase	Decrease
B	Increase	Increase
C	Decrease	Decrease
D	Decrease	Increase
A

a

Explanation:
When bonds are issued at a discount, the Discount on Bonds Payable account is a contra-liability account to Bonds Payable, i.e., it reduces the carrying amount of the bonds. Thus, as the bond discount is amortized, the carrying amount of the bonds increases. The bond discount amortization increases the interest expense on the bonds, and so net income decreases.

378
Q

Which of the following statements concerning debt classification is false?
A
Current liabilities are reported at their face amount.
B
Advanced payments received from customers and others are liabilities until the transaction is completed.
C
The portion of long-term debt due within the next fiscal period is classified as a current liability if payment is expected to require the use of current assets or the creation of other current liabilities.
D
The portion of long-term debt due within the next fiscal period is classified as current if the maturing portion will be paid from the proceeds of a new bond issue or noncurrent assets.

A

d

Explanation:
The portion of long-term debt due within the next fiscal period is not classified as current if the maturing portion will be paid from the proceeds of a new bond issue or noncurrent assets.

379
Q
What type of bonds mature in installments?
A
Debenture
B
Term
C
Variable rate
D
Serial
A

d

Explanation:
Serial bonds are bonds issued at the same time but having different maturity dates. These are also called installment bonds because they provide a series of installments for repayment of principal. Debenture bonds are unsecured bonds; they are not supported by a lien or mortgage on specific assets, but they mature at the same time. Term bonds all mature on a specified date. Variable rate bonds have a fluctuating interest rate, but mature at the same time.

380
Q

The market price of a bond issued at a discount is the present value of its principal amount at the market (effective) rate of interest
A
Less the present value of all future interest payments at the market (effective) rate of interest.
B
Less the present value of all future interest payments at the rate of interest stated on the bond.
C
Plus the present value of all future interest payments at the market (effective) rate of interest.
D
Plus the present value of all future interest payments at the rate of interest stated on the bond.

A

c

Explanation:
The total amount consists of 2 calculations: the present value of the interest annuity and the present value of the principle. Present value is computed using the market/yield/effective interest rate.

381
Q

An investor purchased a bond classified as a long-term investment between interest dates at a discount. At the purchase date, the carrying amount of the bond is more than the

Cash paid to seller	Face amount of bond
A	No	Yes
B	No	No
C	Yes	No
D	Yes	Yes
A

b

Explanation:
If bonds are purchased between interest payment dates, the purchaser/borrower will also include the accrued interest through the purchase date in the total cash paid for the bonds. When a bond is purchased at a discount, the carrying value of the bond will be lower than the face value of the bond. Therefore, a bond purchased between interest dates at a discount has a carrying amount that is lower than both the cash paid to the seller and the face value of the bond.

Options (A), (C) and (D) are incorrect as per above.

382
Q

A note payable was issued in payment for services received. The services had a fair value less than the face amount of the note payable. The note payable has no stated interest rate. How should the note payable be presented in the statement of financial position?
A
At the face amount.
B
At the face amount with a separate deferred asset for the discount calculated at the imputed interest rate.
C
At the face amount with a separate deferred credit for the discount calculated at the imputed interest rate.
D
At the face amount minus a discount calculated at the imputed interest rate.

A

d

Explanation:
Notes payable, like bonds payable, have a principal component and interest component that must be calculated each period. If the interest rate is not stated, the note is recorded at the fair value of the property, goods, or services exchanged or at the amount that approximates the market value of the note, whichever is more clearly determinable. In the absence of established exchange prices for the related services or evidence of the market value of the note, the note is recorded at its present value by discounting all future payments on the note using an imputed interest rate. The note payable is not presented at the just the full face amount and there is no separate deferred asset or credit calculated for the discount.

383
Q

On January 2 of the current year, Emme Co. sold equipment with a carrying amount of $480,000 in exchange for a $600,000 noninterest bearing note due on January 2 in three years. There was no established exchange price for the equipment. The prevailing rate of interest for a note of this type at January 2 of the current year was 10%. The present value of 1 at 10% for three periods is 0.75.

In Emme’s current year income statement, what amount should be reported as interest income?

A
$ 9,000
B
$45,000
C
$50,000
D
$60,000
A

b

Explanation:
A noninterest-bearing note exchanged for property, goods, or services should not be recorded at its face amount. Since there is not an established exchange price for the equipment and the question does not indicate the fair value of the note, the note should be recorded at its present value, which is computed by discounting all future payments of the note at the prevailing rate of interest for a note of this type.

Face amount of note	$ 600,000
Less: Imputed interest [$600,000 × (1 - 0.75)]	(150,000)
Carrying amount of note, 1/2	450,000
Times: Effective interest rate	× 10%
Interest income	$ 45,000
384
Q

If a premium on a bonds payable transaction is not amortized, what are the effects on interest expense and total stockholders’ equity?

Interest expense	Total stockholders’ equity
A	Overstated	Overstated
B	Understated	Overstated
C	Overstated	Understated
D	Understated	Understated
A

c

Explanation:
Amortization of a bond premium decreases interest expense and the carrying amount of the bond for the issuer, while the amortization of a bond discount increases the issuer’s interest expense and the carrying amount of the bond. If a premium on a bonds payable transaction is not amortized, then interest expense would be overstated and total stockholders’ equity would be understated.

Options (A), (B) and (D) are incorrect as per above explanation.

385
Q

Kale Co. purchased bonds at a discount on the open market as an investment and intends to hold these bonds to maturity. Kale should account for these bonds at

A
Cost
B
Amortized cost
C
Fair value
D
Lower of cost or market
A

Explanation:
The correct answer is (B).

Debt securities (e.g., bonds) that the enterprise has the positive intent and ability to hold to maturity are classified as held-to-maturity securities and reported at amortized cost.

Only debt securities classified as either trading debt securities or available-for-sale debt securities are reported at fair value.

Debt securities generally should not be reported at cost or lower-of-cost-or-market if they are accounted for as held-to-maturity securities.

386
Q

The market price of a bond issued at a premium is equal to the present value of its principal amount
A
Only, at the stated interest rate.
B
And the present value of all future interest payments, at the stated interest rate.
C
Only, at the market (effective) interest rate.
D
And the present value of all future interest payments, at the market (effective) interest rate.

A

d

Explanation:
The market price of a bond is equal to the present value of the bond’s interest and principal payments, discounted using the market interest rate for that type of bond.

387
Q

The following is deducted/added from carrying value of bonds and amortized using effective interest method, except:

A
Discount on bonds payable
B
Premium on bonds payable
C
Loss on Extinguishment of debt
D
Bond issue costs
A

c

Explanation:
The correct answer is (C).

A Bond issue cost is a cost directly associated with bond issuance e.g., printing & engraving costs, legal & accounting fees, underwriter commissions, promotion costs, etc.

Per the FASB standard update issued in 2015, bond issue costs should be deducted from the carrying value of bonds and amortized using the effective interest method (i.e., bond issue costs are treated similar to the discount/premium on bonds payable and is reported as an adjustment to bond payable liability).

Gain or loss on extinguishment of debt is the difference between cash paid at retirement and the carrying value of the bond at retirement and is recognized in the year of redemption in income from continuing operations.

388
Q

Question #571, Blueprint Area: Notes and Bonds Payable
Which of the following is reported as interest expense?
A
Pension cost interest
B
Postretirement healthcare benefits interest
C
Imputed interest on non-interest bearing note
D
Interest incurred to finance construction of machinery for own use

A

c

Explanation:
Noninterest-bearing notes must be recorded at the fair value of the property, goods, or services exchanged, or at an amount which approximates the market value of the note, whichever is the more clearly determinable. If neither of these amounts can be determined, the note should be recorded at its present value, computed by discounting all future payments of the note at the prevailing rate of interest for similar notes. The difference between the face amount of the noninterest-bearing note payable and the amount at which it is recorded is amortized as interest expense over the life of the note by the effective interest method.

389
Q

A company issued 10-year term bonds at a discount in year 1. Bond issue costs were incurred at that time. The company uses the effective interest method to amortize bond issue costs. Reporting the bond issue costs as a deferred charge would result in

A
More of a reduction in net income in year 2 than reporting the bond issue costs as a reduction of the related debt liability
B
The same reduction in net income in year 2 as reporting the bond issue costs as a reduction of the related debt liability
C
Less of a reduction in net income in year 2 than reporting the bond issue costs as a reduction of the related debt liability
D
No reduction in net income in year 2

A

Explanation:
The correct answer is (B).

Bond issue cost is the cost directly associated with bond issuance e.g., printing & engraving costs, legal & accounting fees, underwriter commissions, promotion costs, etc. As per FASB standard update issued in 2015, bond issue costs should be deducted from the carrying value of bonds and amortized using effective interest method i.e., bond issue costs is treated similar to discount/premium and is reported as an adjustment to bond payable liability.

Prior to this update, bond issue costs were capitalized and deferred as a separate asset on a company’s balance sheet and amortized using the straight-line method.

In either of the two methods of reporting- either as a deferred asset or as a reduction in the liability, the same amount will be deducted as an expense in the income statement using the effective interest method.

390
Q

On June 30, Huff Corp. issued at 99, one thousand of its 8%, $1,000 bonds. The bonds were issued through an underwriter to whom Huff paid bond issue costs of $35,000. On June 30, Huff should report the bond liability at

A
$955,000
B
$ 990,000
C
$1,000,000
D
$1,035,000
A

Explanation:
The correct answer is (A).

FASB issued Accounting Standards Update 2015-03 requiring Bond Issue Costs to deducted from the CV of bonds and amortized using effective interest method i.e., Bond Issue Costs are treated similar to discount/premium & reported as an adjustment to Bond Payable liability.

The Bonds Payable liability is $1,000,000 - $10,000 - $35,000 = $955,000.

Face amount of bonds ($1,000 x 1,000) $1,000,000
Bond Issue Costs (35,000)
Discount on bonds [$1,000,000 - ($1,000,000 x 99%)] (10,000)
Amount to be reported as bond liability, 6/30 $ 955,000
The journal entry would be as follows:

DR Cash 955,000
DR Bond issue Costs 35,000
DR Discount on Bonds Payable 10,000
CR Bonds Payable 1,000,000

391
Q

Able, Inc. had the following amounts of long-term debt outstanding at December 31 of the current year:
14 1/2% term note, due next year $ 3,000
11 1/8% term note, due in four years 107,000
8% note, due in 11 equal annual principal payments, plus interest beginning December 31 of next year 110,000
7% guaranteed debentures, due in five years 100,000
Total $320,000Able’s annual sinking-fund requirement on the guaranteed debentures is $4,000 per year. What amount should Able report as current maturities of long-term debt in its current year December 31 balance sheet?
A
$ 4,000
B
$ 7,000
C
$10,000
D
$13,000

A

D
Explanation:
The principal payments on the long-term debt that are due in the next year are reported as current maturities of long-term debt at 12/31 of the current year.

14 1/2% term note, due next year $ 3,000
8% note, principal due 12/31 next year ($110,000 / 11) 10,000
Current maturities of long-term debt, 12/31 of current year $13,000

392
Q

Ace Corp. entered into a troubled debt restructuring agreement with National Bank. National agreed to accept land with a carrying amount of $75,000 and a fair value of $100,000 in exchange for a note with a carrying amount of $150,000. Disregarding income taxes, what amount should Ace report as a gain on troubled debt restructuring in its income statement?

A
$25,000
B
$75,000
C
$50,000
D
$0
A

Explanation:
The correct answer is (C).

In a debt restructuring, the debtor follows a two-step process:

  1. Revalues the asset to its fair value and recognizes a gain or loss and then
  2. Determines restructuring gain = Carrying Value of debt - Fair Value of asset given up.

In the given case, Ace first recognizes a gain for revaluing the real estate to its Fair Value

= Fair Value of real estate - Carrying Value of the real estate

Revaluation = $100,000 - $75,000 = $25,000 gain

Ace then would recognize a restructuring gain for the difference between the Carrying Value of the liability and the Fair Value of the real estate transferred.

Restructuring = $150,000 - $100,000 = $50,000 gain

Option (A), (B) and (D) are incorrect as per above explanation.

393
Q

When an entity retires its bonds payable, either through retirement before maturity or through an extinguishment method, the entity would not be in compliance with authoritative guidance if it did which of the following?
A
Wrote-off both the principal and the pro rata portion of the unamortized premium or discount on retired bonds
B
Wrote off a pro rata portion of bond issue costs when the bonds were retired before maturity, if the bond issue costs were initially incurred and recorded as an asset, that is, as a deferred charge
C
Automatically classified an early extinguishment of debt as an extraordinary item
D
None of the above

A

c

Explanation:
An entity must evaluate whether early debt extinguishment is extraordinary using the same criteria as other events

394
Q

A five-year term bond was issued by a company on January 1, year 1, at a premium. The carrying amount of the bond at December 31, year 2, would be
A
The same as the carrying amount at January 1, year 1
B
Higher than the carrying amount at December 31, year 1
C
Lower than the carrying amount at December 31, year 3
D
Lower than the carrying amount at December 31, year 1

A

d

Explanation:
When bonds are issued at a premium, the Premium on Bonds Payable account is added to the Bonds Payable account to determine the initial carrying amount of the bonds. As the bond premium is amortized, the carrying amount of the bond decreases.

395
Q

On January 1, a company issued a $50,000 face value, 8% five-year bond for $46,139 that will yield 10%. Interest is payable on June 30 and December 31. What is the bond carrying amount on December 31 of the current year?

A
$46,139
B
$46,446
C
$46,768
D
$47,106
A

c

Explanation:
In determining the carrying amount of bonds, an adjustment is made for premium or discount amortization to the date of sale. Amortization of a bond premium decreases interest expense and the carrying amount of the bond, while the amortization of a bond discount increases the issuer’s interest expense and carrying amount of the bond.

Date	Interest Paid (4%)	Interest Expense (5%)	Amortization	Carrying Value
1/1	 	 	 	$46,139
6/30	$2,000	$2,307	$307	46,446
12/31	2,000	2,322	322	46,768
$46,139 + $307 + $322 = $46,768 .

Option (A) is incorrect because $46,139 is the discounted value of the bond at issuance on1st Jan.
Option (B) is incorrect because $46,446 is the carrying value of the bond on Jun 30.
Option (D) is incorrect because $47,106 is the carrying value at the end of 1.5 years.

396
Q
On January 1 of the current year, Carr Company purchased Fay Corp., 9% bonds with a face amount of $400,000 for $375,600 to yield 10%. The bonds are dated this January 1, mature on December 31 in ten years, and pay interest annually on December 31. Carr uses the effective interest method of amortizing bond discount. In its income statement for the current year ended December 31, what total amount should Carr report as interest revenue from the long-term bond investment?
A
$40,000
B
$37,560
C
$36,000
D
$34,440
A

b

Explanation:
The amount of interest revenue recognized is determined as follows:

Bond investment carrying amount, January 1 $375,600
Effective interest rate × 10%
Interest revenue recognized in the current year $ 37,560

397
Q
When bonds are issued the \_\_\_\_\_\_\_\_\_\_\_\_ of the bonds is recorded in the "Bonds Payable" account.
A
Face amount
B
Issue amount
C
Bond discount
D
Bond premium
A

a

Explanation:
When bonds are issued, only the face amount of the bonds is recorded in the “Bonds Payable” account

398
Q
The discount resulting from the determination of a note payable's present value should be reported on the balance sheet as a (an)
A
Addition to the face amount of the note.
B
Deferred charge separate from the note.
C
Deferred credit separate from the note.
D
Direct reduction from the face amount of the note.
A

d

Explanation:
If a bond is sold at a discount, it is sold for less than its face value. Therefore, the discount is a direct reduction from the face amount of the note. ‘Addition to the face amount of the note’ describes a premium, not a discount. Discounts and premiums directly determine the carrying amount of the bond; they are not handled as deferred charges or credits.

399
Q

On July 1 of the current year, Eagle Corp. issued 600 of its 10%, $1,000 bonds at 99 plus accrued interest. The bonds are dated April 1 of the current year and mature in ten years. Interest is payable semiannually on April 1 and October 1. What amount did Eagle receive from the bond issuance?

A
$579,000
B
$594,000
C
$600,000
D
$609,000
A

d

Explanation:
When bonds are issued between interest dates (April 1 and October 1), the issue price will also include the accrued interest from the date of the last interest payment (April 1) till the date of issue (July 1). This accrued interest will be a current liability and will serve to reduce the subsequent payment of interest for the time period the bond was not held.

Bond Price ($1,000 × 99% × 600 bonds)	$594,000
Plus: Accrued interest at stated interest rate (10% × $1,000 × 600 bonds × 3/12)	 $ 15,000
Proceeds from bond issuance	$609,000
Option (A) is incorrect because accrued interest should be added to bond issue price received, not subtracted.
Option (B) is incorrect because accrued interest is also received along with the bond issue price.
Option (C) is incorrect because bond is issued at a discount and therefore only $594,000 will be received and accrued interest of $15,000 should be added.
400
Q

In a partnership liquidation proceeding, where a partnership converts its assets into cash and distributes the cash to creditors and partners, the partnership does which of the following?
A
Gives priority to creditors on any distribution
B
May hold each partner personally liable if a partner or other individual prematurely distributes cash to another partner whose capital account later shows a deficit, and the latter partner is unable to repay the premature distribution
C
Is required to distribute the liquidation’s proceeds to partners in a lump sum, after all assets have been sold and all creditors have been satisfied
D
None of the above

A

a

Explanation:
In a partnership liquidation, creditors have priority on any distribution.

401
Q

On January 1, year 1, Boston Group issued $100,000 par value, 5% five-year bonds when the market rate of interest was 8%. Interest is payable annually on December 31. The following present value information is available:

5%

8%

Present value of $1 (n = 5)

  1. 78353
  2. 68058

Present value of an ordinary annuity (n = 5)

  1. 32948
  2. 99271

What amount is the value of net bonds payable at the end of year 1?

A
$88,022
B
$90,064
C
$100,000
D
$110,638
A

Explanation:
The correct answer is (B).

Step 1 : PV of Bond

Market Interest Rate of 8% (PV of $1) x Principle
PV of Bond = $100,000 x .68058 = $68,058
Step 2: PV of Interest

Calculate Bond Interest: Bond Interest Rate of 5% x Principle = $100,000 x 5% = $5,000
PV of Ordinary Annuity Due on 12/31 at 8% x Bond interest = $5,000 * 3.99271 = $19,964
Step 3: PV of bond + PV of interest

$68,058 + $19,964 = $88,022
Step 4: Calculate the amortization for this discount bond.

Effective Interest = $88,022 x 8% = $7,041.76
Interest Paid = $5,000
Amortization of Discount = Effective Interest - Interest Paid = $7,041.76 - $5,000 = $2,041.76
Step 5: Add the amortization back to the Carrying Value

$88,022 + $2,041.76 = $90,064 (rounded)
(A) is incorrect as this represents the value of net bonds payable at the beginning of the year.

(C) is incorrect because this is the face value of the bonds.

(D) is incorrect because of improper calculations.

402
Q
Wilk Co. reported the following liabilities at December 31 of the current year
Accounts payable-trade	$ 750,000
Short-term borrowings	400,000
Bank loan, current portion $100,000	3,500,000
Other bank loan, matures June 30, next year	1,000,000The bank loan of $3,500,000 was in violation of the loan agreement. The creditor had not waived the rights for the loan. What amount should Wilk report as current liabilities at December 31 this year?
A
$1,250,000
B
$2,150,000
C
$2,250,000
D
$5,650,000
A

d

Explanation:
Current liabilities are obligations whose liquidation is expected to require the use of existing current assets or the creation of other current liabilities. Current assets are assets that are reasonably expected to be converted into cash or used during the normal operating cycle of the business or one year, whichever is longer. Therefore, current liabilities are expected to be liquidated within the normal operating cycle of the business or one year whichever is longer. Even though the bank loan appears to be long term, it is in violation of the loan agreement. The creditor has not waived their rights to demand payment of the loan, therefore, Wilk should report the entire balance of the loan as a current liability, not just the amount that would be due within one year if it was not in violation of the loan agreement. The “other bank loan” will mature in less than one year and is classified as a current liability. Of course, accounts payable and short-term borrowings are also current.

$750,000 + $400,000 + $3,500,000 + $1,000,000 = $5,650,000

CLOSE

403
Q

Which of the following is true regarding debt issued with detachable stock warrants?
A
Proceeds must be allocated between the warrants and the debt security based on relative fair values.
B
If the fair value (FV) of one security is not determinable, the proceeds are assigned based on the book value of the other security.
C
The warrants are accounted for as an expense.
D
All of the above

A

a

Explanation:
When bonds are issued with detachable stock warrants, the proceeds must be allocated between the warrants and the debt security based on relative fair values.

404
Q

On March 1, Year 1, a company established a sinking fund in connection with an issue of bonds due in year 8. At December 31, Year 5, the independent trustee held cash in the sinking fund account representing the annual deposits to the fund and the interest earned on those deposits. How should the sinking fund be reported in the company’s balance sheet at December 31, Year 5?
A
The cash in the sinking fund should appear as a current asset.
B
Only the accumulated deposits should appear as a noncurrent asset.
C
The entire balance in the sinking fund account should appear as a current asset.
D
The entire balance in the sinking fund account should appear as a noncurrent asset.

A

d

Explanation:
Because the bond sinking fund is earmarked for the retirement of long-term debt, its entire balance (i.e., all contributions to the fund plus all interest accumulations added to the fund balance to date) should be reported as a noncurrent asset in the investments section of the balance sheet.

405
Q
On January 2 of year 2, Gill Co. issued $2,000,000 of 10-year, 8% bonds at par. The bonds, dated January 1, year 2, pay interest semi-annually on January 1 and July 1. Bond issue costs were $250,000. What amount of bond issue costs are unamortized at June 30, year 3?
A
$237,500
B
$225,000
C
$220,800
D
$212,500
A

D
Explanation:
Three interest dates exist from 1/2, year 2 to 6/30, year 3. Gill Co. should have amortized 15% (3 interest dates ÷ 20) of the bond issue costs at 6/30, year 3.

$250,000 - $37,500 = $212,500

Bond issue costs, 1/2, year 2 $ 250,000
Divide by: Number of semi-annual interest dates (10 x 2) / 20
Semiannual amortization of bond issue costs $ 12,500
Times: Number of interest dates from 1/2, year 2 to 6/30, year 3 x 3
Amortization, 1/2, year 2 to 6/30, year 3 (37,500)

406
Q

A company issued bonds with detachable common stock warrants. The issue price exceeded the sum of the warrants’ fair value and face value of the bonds. The fair value of the bonds cannot be determined. What value, if any, should be assigned to the warrants?

A
The excess of the proceeds over the face value of the bonds.
B
The proportion of the proceeds that the warrants’ fair value bears to the face value of the bonds.
C
The fair value of the warrants.
D
No amount, because the total proceeds should be assigned to the bonds.

A

c

Explanation:
The correct answer is (C)

Detachable Warrant is a security that can be sold / exercised by the bondholder while retaining the bond. Since the warrant is separable from the bond, a value is assigned to both bond and warrant.Value for the detachable warrants is recorded in Additional Paid in capital (APIC). If FMV of both bonds and warrants are known relative fair market value is assigned to bond and warrant. However, If FMV of only warrant is known. The FMV of warrants is assigned to warrants and balance is assigned to Bonds.

407
Q
For a bond issue which sells for less than its face amount, the market rate of interest is
A
Dependent on rate stated on the bond
B
Equal to rate stated on the bond
C
Less than rate stated on the bond
D
Higher than rate stated on the bond
A

d

Explanation:
A bond issue will sell for less than its face amount (i.e., at a discount) when the nominal or stated interest rate is less than the market rate.

408
Q

When the effective interest method of amortization is used for bonds issued at a premium, the amount of interest payable for an interest period is calculated by multiplying the

A
Face value of the bonds at the beginning of the period by the contractual interest rate.
B
Face value of the bonds at the beginning of the period by the effective interest rates.
C
Carrying value of the bonds at the beginning of the period by the contractual interest rate.
D
Carrying value of the bonds at the beginning of the period by the effective interest rates

A

a

Explanation:
The effective interest method of amortization calls for recognizing interest expense at the effective interest rate at which the bonds were sold. Thus, this interest method overcomes the criticism of the straight-line method because it offers a more accurate measurement of interest expense. To amortize a premium using the effective interest method, multiply the carrying amount of the bond issue by the effective yield. This equals interest expense for the period. The difference between the cash interest payment and the interest expense equals the amount of premium amortization for the period. The cash payment is always computed by multiplying the face amount of the bond by the face or stated interest rate.

Option (B) is incorrect because it is the interest expense in the year 1.
Option (C) is incorrect as per the above explanation.
Option (D) is incorrect because it is used to calculate the interest expense at any point of time before bond matures.

409
Q
On July 1, Cody Co. paid $1,198,000 for 10%, 20-year bonds with a face amount of $1,000,000. Interest is paid on December 31 and June 30. The bonds were purchased to yield 8%. Cody uses the effective interest rate method to recognize interest income from this investment. What should be reported as the carrying amount of the bonds in Cody's December 31 balance sheet?
A
$1,207,900
B
$1,198,000
C
$1,195,920
D
$1,193,050
A

c

Explanation:
Bond investment carrying amount, 7/1 $1,198,000
Times: Effective interest rate (8% / 2) x 4%
Interest income, 7/1 - 12/31 $ 47,920
Semiannual interest payment [$1,000,000 x (10% / 2)] (50,000)
Amortization of bond premium, 12/31 $ (2,080)

410
Q
On July 1, year 1, Pell Co. purchased Green Corp. ten-year, 8% bonds with a face amount of $500,000 for $420,000. The bonds mature on June 30, year 9 and pay interest semiannually on June 30 and December 31. Using the interest method, Pell recorded bond discount amortization of $1,800 for the six months ended December 31, year 1. From this long-term investment, Pell should report year 1 revenue of
A
$16,800
B
$18,200
C
$20,000
D
$21,800
A

d

Explanation:
Bonds purchased at a discount are purchased at less than their face amount. The subsequent amortization of the discount increases the carrying amount of the bond investment and the amount of interest income recognized.

Simple interest 7/1 - 12/31, year 1 [$500,000 x (8% / 2)] $20,000
Amortization of discount on bond investment (given) 1,800
Interest revenue recognized in year 1 $21,800

411
Q

On July 1, year 1, Kay Corp. sold equipment to Mando Co. for $100,000. Kay accepted a 10% note receivable for the entire sales price. This note is payable in two equal installments of $50,000 plus accrued interest on December 31, year 1 and year 2. On July 1, year 2, Kay discounted the note at a bank at an interest rate of 12%. Kay’s proceeds from the discounted note were

A
$48,400
B
$49,350
C
$50,350
D
$51,700
A

d

Explanation:
Face amount of note on 7/1, year 1 $100,000
Less: Payment of first installment on 12/31, year 1 (50,000)
Face amount of note on 12/31, year 1 (due 12/31, year 2) 50,000
Add: Interest to maturity ($50,000 × 10% × 12/12) 5,000
Maturity value of remaining portion of note 55,000
Less: Bank discount ($55,000 × 12% × 6/12) (3,300)
Proceeds from discounted note $ 51,700

412
Q

As of December 1, year 2 a company obtained a $1,000,000 line of credit maturing in one year on which it has drawn $250,000, a $750,000 secured note due in five annual installments, and a $300,000 three-year balloon note. The company has no other liabilities. How should the company’s debt be presented in its classified balance sheet on December 31, year 2 if no debt repayments were made in December?
A
Current liabilities of $1,000,000; long-term liabilities of $1,050,000.
B
Current liabilities of $500,000; long-term liabilities of $1,550,000.
C
Current liabilities of $400,000; long-term liabilities of $900,000.
D
Current liabilities of $500,000; long-term liabilities of $800,000.

A

c

Explanation:
A line-of-credit (LOC) is an agreement that provides the borrower with the option to make multiple borrowings up to a specified maximum amount, to repay portions of previous borrowings, and to then reborrow under the same contract. Short-term obligations are those that are scheduled to mature within one year (or the operating cycle, if applicable) after the date of an entity’s balance. Long-term obligations are those scheduled to mature beyond one year (or the operating cycle, if applicable) from the date of an entity’s balance sheet. Total current liabilities would be $400,000 – the amount outstanding on the LOC ($250,000) and the current amount due on the secured note ($750,000/5 = $150,000). Total long-term liabilities would be $900,000 – the remaining $600,000 secured note and the $300,000 balloon note (i.e., no payment is due until the third year).

413
Q

Casey entered into a troubled debt restructuring agreement with First State Bank. First State agreed to accept land with a carrying amount of $85,000 and a fair value of $120,000 in exchange for a note with a carrying amount of $185,000. Casey has restructured debt twice in the last five years. Disregarding income taxes, what amount should Casey report as gain on troubled debt restructuring in its income statement?

A
$ 35,000
B
$ 0
C
$100,000
D
$65,000
A

Explanation:
The correct answer is (D).

In a debt restructuring, the debtor follows a two-step process:-

Revalues the asset to its Fair Value and recognizes a gain or loss and then
Determines Restructuring Gain = Carrying Value of debt - Fair Value of asset given up.
In the given case, Casey first recognizes a gain for revaluing the real estate to its fair value

= Fair Value of real estate - Carrying Value of the real estate

Revaluation Gain = $120,000 - $85,000 = $35,000 gain

Casey then would recognize a restructuring gain for the difference between the carrying value of the liability and the fair value of the real estate transferred

Restructuring Gain = $185,000 - $120,000 = $65,000.

Option (A), (B) and (C) are incorrect as per above explanation.

414
Q
A bond issued on June 1 of the current year, has interest payment dates of April 1 and October 1. Bond interest expense for the current year ended December 31, is for a period of
A
Three months
B
Four months
C
Six months
D
Seven months
A

Explanation:
When the interest date does not coincide with the end of the accounting period, the issuer must accrue interest expense through year-end. Therefore, bond interest expense for the year ended December 31, is for a period of seven months–from the date of issuance of June 1 to year-end of December 31.

415
Q
On January 1, year 13, Hart, Inc., redeemed its 15-year bonds of $500,000 face amount for 102. They were originally issued on January 1, year 1 at 98 with a maturity date of January 1, year 16. The bond issue costs relating to this transaction were $20,000. Hart amortizes discounts, premiums, and bond issue costs using the straight-line method. Before income taxes, what amount of loss should Hart recognize on the redemption of these bonds?
A
$16,000
B
$12,000
C
$10,000
D
$0
A

$0
Explanation:
A loss is recognized on the redemption because the cost to redeem the bonds exceeds the carrying amount of the bonds.

Face amount of bonds $ 500,000
Discount at issuance [$500,000 - ($500,000 x 98%)] $ 10,000
Amortization to extinguishment date ($10,000 × 12/15) (8,000)
Less unamortized discount at extinguishment $ (2,000)
Bond issue costs incurred $ 20,000
Amortization to date of extinguishment ($20,000 × 12/15
) (16,000)
Less unamortized bond issue cost at extinguishment (4,000)
Bond carrying amount at extinguishment 494,000
Cost to redeem ($500,000 × 102%) (510,000)
Pretax loss on early debt extinguishment $ (16,000)* The interest on the 15-year bonds was payable annually. Thus, there were 15 interest dates over the life of the bonds. The discount and bond issue costs must be amortized up to the extinguishment date (i.e., year 13–the twelfth interest date).

416
Q
Kamy Corp. is in liquidation under Chapter 7 of the Federal Bankruptcy Code. The bankruptcy trustee has established a new set of books for the bankruptcy estate. After assuming custody of the estate, the trustee discovered an unrecorded invoice of $1,000 for machinery repairs performed before the bankruptcy filing. In addition, a truck with a carrying amount of $20,000 was sold for $12,000 cash. This truck was bought and paid for in the year before the bankruptcy. What amount should be debited to estate equity as a result of these transactions?
A
$0
B
$1,000
C
$8,000
D
$9,000
A

d

Explanation:
A trustee in a bankruptcy case takes over the assets of the debtor corporation and is accountable for those assets until released by the bankruptcy court. In liquidation cases, the trustee often establishes a new set of books for the bankruptcy estate. The assets are recorded on the trustee’s books at carrying amounts rather than at expected realizable values because of the subjectivity involved in estimating realizable amounts at the time of filing. The trustee records gains and losses and liquidation expenses directly in the Estate Equity account. Any unrecorded assets or liabilities that are discovered by the trustee are also entered in the Estate Equity account. Therefore, $9,000 [i.e., $1,000 + ($20,000 - $12,000)] should be debited to Estate Equity as a result of the discovery of the unrecorded invoice and the sale of the truck.

417
Q

Bonds with detachable stock warrants were issued by Flack Co. Immediately after issue, the aggregate market value of the bonds and the warrants exceeds the proceeds. Is the portion of the proceeds allocated to the warrants less than their market value, and is that amount recorded as contributed capital?

Less than warrants' market value	Contributed capital
A	No	Yes
B	Yes	No
C	Yes	Yes
D	No	No
A

c

Explanation:
When bonds are issued with detachable stock purchase warrants, the proceeds are allocated between the bonds and the warrants with the amount allocated to the warrants is recorded in additional paid-in capital (APIC). Thus, the amount allocated to warrants is recorded as contributed capital. The proceeds are allocated in the ratio of the relative fair market values of the bonds and the warrants. Since the aggregate market value of those securities exceeds the proceeds, the amount allocated to each security, in proportion of the market value, would be lower than its actual market value.

Options (A), (B) and (D) are incorrect based on the above explanation.

418
Q
How would the carrying amount of a bond payable be affected by amortization of the following?
Discount	Premium
A	Increase	Increase
B	Decrease	Decrease
C	Increase	Decrease
D	Decrease	Increase
A

c

Explanation:
The account Discount on Bonds Payable reduces the carrying amount of the bonds by the amount of the discount. Thus, as the bond discount is amortized, the carrying amount of the bond increases until it reaches the face amount of the bond. The account Premium on Bonds Payable increases the carrying amount of the bonds by the amount of the premium. Thus, as the bond premium is amortized, the carrying amount of the bond decreases toward the face amount of the bond.

419
Q

When an investor entity purchases bonds, it must do which of the following?

A
Record the bond investment initially at the purchase price of the bond plus other direct costs of acquisition, such as broker’s fees
B
Pay an additional amount for the interest accrued on the bonds since the last interest date (or the bond date, if no interest payment dates have yet occurred), if the bonds are bought between interest dates
C
Record the additional amount paid, for interest accrued on bonds bought between interest dates, as purchased interest in a separate account, such as interest receivable
D
All of the above

A

d

Explanation:
Although an investor entity initially records bonds held for investment at an amount equal to the pur­chase price of the bond plus other direct costs of acquisition, such as broker’s fees, it is also true that: (1) if it buys bonds between interest dates, it will have to pay an additional amount for the interest accrued on the bond since the last interest date (or the bond date, if before the first interest date); and (2) it must record the additional amount paid due to purchasing the bonds between interest dates as purchased interest, such as interest receiv­able, rather than adding it to the cost of the bond investment.
Since the answers for (A), (B), and (C) are all correct, the best answer for this question is D., all of the above.

420
Q

Which of the following is generally associated with the terms of convertible debt securities?
A
An interest rate that is lower than nonconvertible debt
B
An initial conversion price that is less than the market value of the common stock at time of issuance
C
A noncallable feature
D
A feature to subordinate the security to nonconvertible debt

A

a

Explanation:
Convertible debt securities are debt securities that are convertible into common stock of the issuer or an affiliated company at a specified price at the option of the holder. The terms of such securities generally include (1) an interest rate that is lower than the issuer could establish for nonconvertible debt, (2) an initial conversion price that is greater than the market value of the common stock at time of issuance, and (3) a conversion price that does not decrease except pursuant to antidilution provisions.

421
Q
When the cash proceeds from a bond issued with detachable stock purchase warrants exceeds the sum of the par value of the bonds and the fair value of the warrants, the excess should be credited to
A
Additional paid-in capital
B
Retained earnings
C
Premium on bonds payable
D
Detachable stock warrants outstanding
A

c

Explanation:
The proceeds from the sale of debt with stock purchase warrants should be allocated between the two instruments based on the relative fair values of the debt security without the warrants and the warrants themselves. The portion of the proceeds allocated to the warrants is accounted for as paid-in capital. The remainder is allocated to the face amount of the bond, and to the extent remaining, to premium on bonds payable..

422
Q

When purchasing a bond, the present value of the bond’s expected net future cash inflows discounted at the market rate of interest provides what information about the bond?

A
Price.
B
Par.
C
Yield.
D
Interest.
A

a

Explanation:
The correct answer is (A).

The market value of a bond consists of two parts:

The present value of cash flows from interest (calculated at the stated rate) and discounted at market rate, and
The present value of the principal discounted at the market rate of interest.
These two amounts are added together to get the market price or selling price of the bond.

423
Q

A company has outstanding accounts payable of $30,000 and a short-term construction loan in the amount of $100,000 at year end. The loan was refinanced through issuance of long-term bonds after year end but before issuance of financial statements. How should these liabilities be recorded in the balance sheet?
A
Long-term liabilities of $130,000.
B
Current liabilities of $130,000.
C
Current liabilities of $30,000, long-term liabilities of $100,000.
D
Current liabilities of $130,000, with required footnote disclosure of the refinancing of the loan.

A

c

Explanation:
The term current liabilities is used primarily to designate obligations whose liquidation is reasonably expected to require the use of existing resources classified as current assets, or the creation of other current liabilities. Accounts payable is a type of current liability. Short-term obligations are those scheduled to mature within one year or operating cycle, whichever is longer, and generally are classified as current liabilities. However, if they are to be refinanced on a long-term basis they will be appropriately classified as long-term liabilities. Exclusion from current liabilities requires two conditions be met; 1) the enterprise must intend to refinance the obligation on a long-term basis, and 2) the enterprise must have the ability to consummate the financing. A refinancing that occurs after the balance sheet date but before the issuance of the balance sheet is evidence of intent and ability.

424
Q

In the previous year, Lee Co. acquired, at a premium, Enfield, Inc. 10-year bonds as a long-term investment. At December 31 of the current year, Enfield’s bonds were quoted at a small discount. Which of the following situations is the most likely cause of the decline in the bonds’ market value?

A
Enfield issued a stock dividend.
B
Enfield is expected to call the bonds at a premium, which is less than Lee’s carrying amount.
C
Interest rates have declined since Lee purchased the bonds.
D
Interest rates have increased since Lee purchased the bonds.

A

d

Explanation:
The purchaser of a bond acquires the right to receive two cash flows: a lump sum paid at maturity for the face amount of the bond, and an annuity consisting of periodic interest payments over the life of the bond. The price the market is willing to pay for the bond is equal to the present value of these two cash flows, discounted at the prevailing market interest rate for bonds having the same maturity and perceived degree of risk. When interest rates increase, the present value of the two cash flows decreases, causing the market value of the bonds to decline. The issuance of a stock dividend should not cause a decline in the market value of the bonds. If the bonds currently are quoted at a small discount and the bonds are expected to be called at a premium (i.e., above face amount), this situation would most likely have the effect of causing a rise in the market value of the bonds. A decline in interest rates would cause a rise in the bond’s market value.

Option (A) is incorrect because issuing a stock dividend will not affect the market value of a bond.
Option (B) is incorrect because market value of bond does not depend on what amount it is going to be called at in future.
Option (C) is incorrect because interest rates would have increased not decreased which would lead to a decline in the bond value.

425
Q
In a troubled debt restructuring, the total fair value of the consideration given to discharge the obligation will \_\_\_\_\_\_\_\_\_\_ the recorded amount of the debt.
A
Always be less than
B
Always be more than
C
Always equal
D
Equal or exceed
A

a

Explanation:
In a troubled debt restructuring, the total fair value of the consideration given to discharge the obligation will always be less than the recorded amount of the debt.

426
Q

A company issues $2,000,000 of par bonds at 97 on January 1, year 1, with a maturity date of December 31, year 30. Bond issue costs are $120,000, and the stated interest rate of the bonds is 8%. Interest is paid semiannually on January 1 and July 1. Ten years after the issue date, the entire issue was called at 103 and canceled. The company uses the straight-line method of amortization for bond discounts and issue costs, and the result of this method is not materially different from the effective interest method. The company should classify what amount as the loss on extinguishment of debt at the time the bonds are called?

A
$40,000
B
$80,000
C
$120,000
D
$180,000
A

Explanation:
The correct answer is (D).

The par bonds being sold at 97 indicate that they are being sold at a 3% discount. Total Discount is 3% x $2,000,000 = $60,000.

Discounts on bonds held long-term must be amortized from date of acquisition to maturity date, normally using the interest method to amortize these differences.
Other methods of amortization (straight-line) may be used if the effects are not material. Bond issue costs should be classified as a deferred charge (i.e., asset) and amortized over the life of the bonds as an increase to interest expense. When all or part of a bond issue is retired before maturity, it is necessary to write off both the principal and the pro-rata portion of the unamortized premium or discount on the retired bonds.

If bond issue costs were incurred and recorded as an asset (i.e., as a deferred charge), it is also necessary to write off a pro-rata portion of the bond issue costs (when retired before maturity). The amount of such write-off increases any loss or reduces any gain recognized on retirement.

The sale of bonds results in a gain or loss equal to the difference between the carrying amount of the bonds and the proceeds received on their disposal. In determining the carrying amount of the bonds, an adjustment must be made for the discount amortization to date of sale.

Bonds Payable (Face Value) $2,000,000
Unamortized Discount [($60,000 / 30) × 20 yrs.] ($40,000)
Unamortized Issue costs [($120,000 / 30) × 20 yrs.] ($80,000)
Carrying amount of bonds retired $1,880,000
Purchase price ($2,000,000 × 103%) $2,060,000
Loss on bond retirement, before income taxes ($180,000)

427
Q
Godart Co. issued $4,500,000 notes payable as a scrip dividend that matured in five years. At maturity, each shareholder of Godart's three million shares will receive payment of the note principal plus interest. The annual interest rate was 10%. What amount should be paid to the stockholders at the end of the fifth year?
A
$ 450,000
B
$2,250,000
C
$4,500,000
D
$6,750,000
A

d

Explanation:
The amount paid to the stockholders at the end of the fifth year should be the accumulated annual interest, $4,500,000 × 10% × 5 yrs = $2,250,000 over the five years, plus the notes payable issue amount of $4,500,000.

428
Q
On July 1, year 1, Cobb, Inc., issued 9% bonds in the face amount of $1,000,000, which mature in ten years. The bonds were issued for $939,000 to yield 10%, resulting in a bond discount of $61,000. Cobb uses the interest method of amortizing bond discount. Interest is payable annually on June 30. At June 30, year 3, Cobb's unamortized bond discount should be
A
$52,810
B
$51,000
C
$48,800
D
$43,000
A

a

Explanation:
Unamortized bond discount, 7/1, year 1 $61,000
Amortization, 7/1, year 1 - 6/30, year 2:
Bonds payable carrying amount, 7/1, year 1 $939,000
Effective interest rate x 10%
Interest expense, 7/1, year 1 - 6/30, year 2 $ 93,900
Interest payment ($1,000,000 x 9%) (90,000)
Bond discount amortization, 7/1, year 1 - 6/30, year 2 (3,900)
Bonds payable carrying amount, 7/1, year 2 ($939,000 + $3,900) $942,900
Effective interest rate x 10%
Interest expense, 7/1, year 2 - 6/30, year 3 $ 94,290
Interest payment ($1,000,000 x 9%) (90,000)
Amortization for 7/1, year 2 - 6/30, year 3 (4,290)
Unamortized bond discount, 6/30, year 3 $52,810

429
Q

A corporation recently issued $4 million of 10-year, 3% bonds at 101. There were 200,000 detachable stock warrants included as part of the sale. Each warrant allows the bondholder to purchase one share of no-par common stock for $12 per share. On the date of issuance, the stock warrants had a fair value of $1 per warrant. By what amount did the corporation’s long-term debt increase as a result of this issuance?

A
$3,840,000
B
$4,000,000
C
$4,040,000
D
$4,200,000
A

Explanation:
The correct answer is (A).

When bonds are issued with detachable warrants, the value for such detachable warrants is posted in Additional Paid-in Capital (APIC). A value is given to both the bond and the warrant for the simple reason that warrants are clearly separable from the bond. If the fair market values of both the securities are known, values are determined as per relative FMV approach. However, in case the FMV of only one of the security is known, the other security’s value is derived using a plug.

In the given problem statement, the fair value of only the detachable warrant is known i.e. $1 x 200,000 = $200,000.

Since the bonds were issued at 101, the cash inflow = $4,000,000 x 101 = $4,040,000.

The value of bonds i.e. long-term debt = 4,040,000 – 200,000 = $3,840,000.

430
Q
On September 30, World Co. borrowed $1,000,000 on a 9% note payable. World paid the first of four quarterly payments of $264,200 when due on December 30. In its income statement for the year, what amount should World report as interest expense?
A
$0
B
$14,200
C
$22,500
D
$30,000
A

c

Explanation:
World’s interest rate on the $1,000,000 note payable was 9%. The loan is outstanding for three months interest for the year. The interest expense is $1,000,000 × .09 × 3/12 = $22,500.

431
Q
On May 1, Bolt Corp. issued 11% bonds in the face amount of $1,000,000 that mature in ten years. The bonds were issued to yield 10%, resulting in bond premium of $62,000. Bolt uses the effective interest method of amortizing bond premium. Interest is payable semiannually on November 1 and May 1. In its October 31 balance sheet, what amount should Bolt report as unamortized bond premium?
A
$62,000
B
$60,100
C
$58,900
D
$58,590
A

b

Explanation:
$62,000 - $1,900 = $60,100

Bonds payable carrying amount, 5/1 ($1,000,000 + 62,000) $1,062,000
Semiannual effective interest rate (10% / 2) × 5%
Interest expense, 5/1 - 10/31 $ 53,100
Semiannual interest payment due 11/1 [$1,000,000 × (11% / 2)] (55,000)
Bond premium amortization, 5/1 - 10/31 $ (1,900)

432
Q

Perk, Inc. issued $500,000, 10% bonds to yield 8%. Bond issuance costs were $10,000. How should Perk calculate the net proceeds to be received from the issuance?
A
Discount the bonds at the stated rate of interest.
B
Discount the bonds at the market rate of interest.
C
Discount the bonds at the stated rate of interest and deduct bond issuance costs.
D
Discount the bonds at the market rate of interest and deduct bond issuance costs.

A

d

Explanation:
The net proceeds of a bond issuance is determined by calculating the present value of the projected cash flows of the bonds at the yield rate (market rate) of interest and then deducting bond issuance costs. The stated rate of interest is used to determine the amount of cash to be paid at each payment date, but the market rate is the rate used to discount the cash flows to present values.

433
Q

What type of bonds in a particular bond issuance will not all mature on the same date?

A
Debenture bonds
B
Serial bonds
C
Term bonds
D
Sinking fund bonds
A

b

Explanation:
Serial bonds are a set of bonds issued at the same time but having a different maturity date; thus providing a series of installments for repayment of principal.

Option (A) is incorrect because debenture bonds are unsecured bonds.
Option (C) is incorrect because term bond single maturity date with entire principal maturing on a single date at the end of the term.
Option (D) is incorrect because sinking fund bond may be set up for the retirement of bonds (may even be required as per bond indenture).

Explanation:
Serial bonds are a set of bonds issued at the same time but having a different maturity date; thus providing a series of installments for repayment of principal.

Option (A) is incorrect because debenture bonds are unsecured bonds.
Option (C) is incorrect because term bond single maturity date with entire principal maturing on a single date at the end of the term.
Option (D) is incorrect because sinking fund bond may be set up for the retirement of bonds (may even be required as per bond indenture).

434
Q

Hancock Co.’s December 31, year 1 balance sheet contained the following items in the long-term liabilities section:

Unsecured
9.375% registered bonds ($25,000 maturing annually beginning in year 5) $275,000
11.5% convertible bonds, callable beginning in year 10, due year 21 125,000
Secured
9.875% guaranty security bonds, due 2020 $250,000
10.0% commodity-backed bonds ($50,000 maturing annually beginning year 6) 200,000
What are the total amounts of serial bonds and debenture bonds?

Serial bonds	Debenture bonds
A	$475,000	$400,000
B	$475,000	$125,000
C	$450,000	$400,000
D	$200,000	$650,000
A

Explanation:
The correct answer is (A).

Bond issues maturing on a single date are called term bonds, whereas bond issues maturing in installments are called serial bonds.

Since the registered bonds and the commodity-backed bonds both mature in installments, the total amount of serial bonds is $475,000 (i.e., $275,000 + $200,000).

Debenture bonds are unsecured bonds; they are not supported by a lien or mortgage on specific assets.

Since the registered bonds and the convertible bonds are unsecured, the total amount of debenture bonds is $400,000 (i.e., $275,000 + $125,000).

435
Q

On November 1 of the current year, Mason Corp. issued $800,000 of its 10-year, 8% term bonds dated October 1 of the current year. The bonds were sold to yield 10%, with total proceeds of $700,000 plus accrued interest. Interest is paid every April 1 and October 1. What amount should Mason report for interest payable in its December 31 current year balance sheet?

A
$17,500
B
$16,000
C
$11,667
D
$10,667
A

b

Explanation:
Interest payable is the amount of interest due to the bondholder from the date of last payment of interest (October 1, 20X1) till the year end (December 31, 20X1), irrespective of the date of issue of the bond (November 1, 20X1). Interest payable = Face value x Stated interest rate x period since last payment = $8,00,000 x 8% x 3/12= $16,000.

Option (A) is incorrect because interest payable is calculated using total proceeds received and market interest rate. Interest payable should be calculated based on the face value and stated interest rate.
Option (C) is incorrect because interest payable is calculated using total proceeds received and market interest rate for a period of two months calculated from date of issue (November 1). Interest payable should be calculated based on the face value and stated interest rate for a period of three months from date of last interest payment (October 1).
Option (D) is incorrect because interest payable is calculated for a period of two months from date of issue (November 1). It should be calculated for a period of three months from date of last interest payment (October 1)

436
Q
On June 30, year 1, King Co. had outstanding 9%, $5,000,000 face value bonds maturing on June 30, year 6. Interest was payable semiannually every June 30 and December 31. On June 30, year 1, after amortization was recorded for the period, the unamortized bond premium and bond issue costs were $30,000 and $50,000, respectively. On that date, King acquired all its outstanding bonds on the open market at 98 and retired them. At June 30, year 1, what amount should King recognize as gain before income taxes on redemption of bonds?
A
$ 20,000
B
$ 80,000
C
$120,000
D
$180,000
A

b

Explanation:
A gain is recognized because the cost to redeem the bonds is less than the carrying amount of the bonds.

Face amount of bonds $5,000,000
Add: Unamortized bond premium 30,000
Less: Unamortized bond issue costs (50,000)
Bond carrying amount at redemption date $ 4,980,000
Cost to redeem ($5,000,000 × 98%) (4,900,000)
Pretax gain on redemption of bonds $ 80,000

437
Q
On December 31, year 1, Taylor, Inc. signed a binding agreement with a bank for the refinancing of an existing note payable scheduled to mature in February, year 2. The terms of the refinancing included extending the maturity date of the note by three years. On January 15, year 2, the note was refinanced. How should Taylor report the note payable in its December 31, year 1, balance sheet?
A
A current liability
B
A long-term liability
C
A long-term note receivable
D
A current note receivable
A

b

Explanation:
Long-term liabilities are all obligations not expected to be liquidated by the use of existing current assets or by the creation of current liabilities. Examples include: (1) long-term notes payable, (2) refinancing of short-term obligations, and (3) bonds payable. The binding agreement to refinance the existing note payable, extending the maturity date three years, changed the note payable classification from a current liability to a long-term liability. A note payable would not be reported as a receivable.

438
Q

Nu Corp agreed to give Rand Co. a machine in full settlement of a note payable to Rand. The machine’s original cost was $140,000. The note’s face amount was $110,000. On the date of the agreement:

The note’s carrying amount was $105,000, and its present value was $96,000.
The machine’s carrying amount was $109,000, and its fair value was $96,000.
What amount of gains (losses) should Nu recognize?

A
$4,000
B
($4,000)
C
($9,000)
D
($13,000)
A

Explanation:
The correct answer is (B).

Nu Corp recognizes an ordinary gain on the early extinguishment of debt equal to the excess of the carrying amount of the obligation settled over the fair value of the asset transferred. Nu Corp also recognizes an ordinary loss equal to the excess of the carrying amount over the fair value of the asset transferred, which is reported in income from continuing operations. [($9,000 - $13,000 = (4,000)].

Carrying amount of obligation at extinguishment date $105,000
Fair value of asset transferred (96,000)
Gain on early debt extinguishment $ 9,000

Carrying amount of asset transferred $109,000
Fair value of asset transferred (96,000)
Loss on transfer of asset ($13,000)

Total Net Gain (Loss) ($4,000)

439
Q
On December 31 of the current year, Moss Co. issued $1,000,000 of 11% bonds at 109. Each $1,000 bond was issued with 50 detachable stock warrants, each of which entitled the bondholder to purchase one share of $5 par common stock for $25. Immediately after issuance, the market value of each warrant was $4. On December 31, what amount should Moss record as discount or premium on issuance of bonds?
A
$ 40,000 premium.
B
$ 90,000 premium.
C
$110,000 discount.
D
$200,000 discount.
A

c

Explanation:
Since the fair market value of the bonds is not determinable, the incremental method is used to determine the value of the bonds and warrants. That is, the market value is used for the warrants and the remainder of the purchase price is allocated to the bonds.

Issuance Price (1,000 bonds × $1,000 × 109%) $1,090,000
Fair value of the warrants (1,000 × 50 × $4) (200,000)
Portion allocated to bonds $ 890,000
Face value of bonds $1,000,000
Portion allocated to bonds (890,000)
Discount on bonds $ 110,000Answers (a), (b), and (d) are incorrect because the market value of the warrants is known and, therefore, must be used to determine the portion of the total issuance price that is allocated to the warrants.

440
Q
When debt is issued at a discount, interest expense over the term of debt equals the cash interest paid
A
Minus discount
B
Minus discount minus par value
C
Plus discount
D
Plus discount plus par value
A

c

Explanation:
A bond will sell at a discount (less than par) when the stated interest rate is less than the market rate. The amount of the discount will be equal to the difference between the bond payable amount and the cash received. The bond discount is recorded in a separate account in the balance sheet as a direct deduction from the face amount of the bond. The discount will be amortized over the period from the date of sale to the maturity date as interest expense. Thus, the total interest expense over the term of debt equals the cash interest paid plus the discount amount at issue.

441
Q

On January 1, year 7, Dean Company issued ten-year bonds with a face value of $1,000,000 and a stated interest rate of 8% per year payable semiannually on July 1 and January 1. The bonds were sold to yield 10%. Present value factors are as follows:
Present value of $1 for 10 periods at 10% .386
Present value of $1 for 20 periods at 5% .377
Present value of an annuity of 1 for 10 periods at 10% 6.145
Present value of an annuity of 1 for 20 periods at 5% 12.462What is the total amount of Dean Company’s term bonds?
A
$ 875,480
B
$ 877,600
C
$ 980,000
D
$1,000,000

A

a

Explanation:
The total amount consists of 2 calculations: the present value of the interest annuity and the present value of the principle. Interest is paid semi-annually, so there are 20 compounding periods. The discount rate is always the market/yield rate, which is 10% annually or 5% semi-annually. The semi-annual interest payment is (computed using the face/stated interest rate) $40,000 [$1,000,000 * 8% * 6/12]. The total amount for the bonds is $875,480: the pv of the interest annuity $498,480 [$40,000 * 12.462] + the pv of the principle $377,000 [$1,000,000 * .377].

442
Q

House Publishers offered a contest in which the winner would receive $1,000,000, payable over 20 years. On December 31, year 1, House announced the winner of the contest and signed a note payable to the winner for $1,000,000, payable in $50,000 installments every January 2. Also on December 31, year 1, House purchased an annuity for $418,250 to provide the $950,000 prize money remaining after the first $50,000 installment, which was paid on January 2 year 2.

In its December 31, year 1, balance sheet, what amount should House report as note payable contest winner, net of current portion?

A
$368,250
B
$418,250
C
$900,000
D
$950,000
A

b

Explanation:
The amount that should be reported as the noncurrent portion of Note Payable to Contest Winner at 12/31 of year 1 is $418,250, the cost of the annuity purchased to provide for the $950,000 of prize money to be paid after 12/31 of year 2. This is the most objective available evidence of the present value of the prize money due.

443
Q

A bond issued on June 1, year 1, has interest payment dates of April 1 and October 1. Bond interest expense for the year ended December 31, year 1, is for a period of

A
Three months.
B
Four months.
C
Six months.
D
Seven months.
A

Explanation:
The correct answer is (D).

When the bond is issued on June 1, the bond issuer is liable to pay the interest for a period starting from the date of issue.

In the given case, interest for 4 months (June to September) should be paid on October 1.

Further, interest for 3 months (October to December) is accrued and paid on April 1, year 2.

In total, there are 7 months of interest expense

444
Q

On July 28, Vent Corp. sold $500,000 of 4%, eight-year subordinated debentures for $450,000. The pur­chasers were issued 2,000 detachable warrants, each of which was for one share of $5 par common stock at $12 per share. Shortly after issuance, the warrants sold at a market price of $10 each. What amount of discount on the debentures should Vent record at issuance?

A
$50,000
B
$60,000
C
$70,000
D
$74,000
A

c

Explanation:
When bonds are issued with detachable stock warrants there must be an allocation of the proceeds between the warrants and the debt security based on relative fair values. Vent would debit Cash $450,000, credit Bond Payable $500,000, and credit APIC-Stock Warrants for their fair value of $20,000 (2,000 detachable war­rants at the market price of $10). The entry needs a $70,000 debit to balance and that is the discount on the debentures.

445
Q

On October 1 of the prior year, Fleur Retailers signed a 4-month, 16% note payable to finance the purchase of holiday merchandise. At that date, there was no direct method of pricing the merchandise, and the note’s market rate of interest was 11%. Fleur recorded the purchase at the note’s face amount. All of the merchandise was sold by December 1 of the prior year. Fleur’s prior year financial statements reported interest payable and interest expense on the note for three months at 16%. All amounts due on the note were paid February 1 of the current year. Fleur’s prior year cost of goods sold for the holiday merchandise was
A
Overstated by the difference between the note’s face amount and the note’s October 1 present value.
B
Overstated by the difference between the note’s face amount and the note’s October 1 present value plus 11% interest for two months.
C
Understated by the difference between the note’s face amount and the note’s October 1 present value.
D
Understated by the difference between the note’s face amount and the note’s October 1 present value plus 16% interest for two months.

A

c

Explanation:
The note should not be recorded at its face amount because its stated interest rate of 16% does not approximate the market rate of interest of 11% for similar notes. When a note which does not bear the market rate of interest is exchanged for property, goods, or services, the note is to be recorded at the fair value of the property, goods, or services or the fair value of the note, whichever is more clearly determinable. Since neither of these amounts are determinable for the note in question, the note should be recorded at its present value. The interest rate to be used in the discounting process is the borrower’s incremental borrowing rate (market rate) at the date the note is issued (11% in this case). Because the market rate of 11% is less than the stated rate of the note of 16%, the present value of the note is greater than the face amount of the note. Therefore, the correct journal entry at the date the note was issued would involve a debit to Purchases for the present value of the note, a credit to Note Payable for the face amount of the note, and a credit to Premium on Note Payable for the excess of the note’s present value over its face amount. Fleur recorded the purchase at the note’s face amount which understated the cost of the merchandise which was subsequently sold in the prior year. Therefore, prior year cost of goods sold is understated by the amount of the unrecorded premium on the note (i.e., the excess of the note’s present value over its face amount).

446
Q
Long-term liabilities include which of the following?
A
Dividends payable
B
Accounts payable
C
Refinancing of short-term obligations
D
Loss contingencies concerning unasserted claims
A

c

Explanation:
Long-term liabilities are all obligations not expected to be liquidated by the use of existing current assets or by the creation of current liabilities. Examples include long-term notes payable, refinancing of short-term obligations, and bonds payable.

447
Q

On January 1, year 1, Fox Corp. issued 1,000 of its 10%, $1,000 bonds for $1,040,000. These bonds were to mature on January 1, year 11, but were callable at 101 any time after December 31, year 4. Interest was payable semiannually on July 1 and January 1. On July 1, year 6, Fox called all of the bonds and retired them. Bond premium was amortized on a straight-line basis. Before income taxes, Fox’s gain or loss in year 6 on this early extinguishment of debt was

A
$30,000 gain
B
$12,000 gain
C
$10,000 loss
D
$8,000 gain
A

d

Explanation:
Face amount of bonds (1,000 × $1,000) $1,000,000
Premium at issuance ($1,040,000 - $1,000,000) $40,000
Amortized to extinguishment date ($40,000 × 11/20*) _ (22,000)
Add unamortized premium at extinguishment 18,000
Bond carrying amount at extinguishment 1,018,000
Cost to reacquire ($1,000,000 × 101%) _(1,010,000)
Pretax gain on early extinguishment of debt $ 8,000
*The interest on the 10-year bonds was payable semiannually. Therefore, there were 20 (i.e., 10 × 2) interest dates over the life of the bonds. The premium must be amortized up to the date of retirement (i.e., 7/1, year 6 — the eleventh interest date).

Option (A) is incorrect because this does not take into account of the amortized premium.
Option (B) is incorrect because this calculates the amortized premium based on 4.5 years instead of 5.5 years.
Option (C) is incorrect because this is the difference between the face value and the carrying value.

448
Q

The following information pertains to Camp Corp.’s issuance of bonds on July 1, year 1:

Face amount $800,000
Term 10 years
Stated interest rate 6%
Interest payment dates Annually on July 1
Yield 9%
At 6% At 9%
Present value of 1 for 10 periods 0.558 0.422
Future value of 1 for 10 periods 1.791 2.367
Present value of ordinary annuity of 1 for 10 periods 7.360 6.418
What should be the issue price for each $1,000 bond?

A
$1,000
B
$ 864
C
$ 807
D
$ 700
A

c

Explanation:
The market price of a bond is equal to the present value of the principal amount due at the end of the bond period and the present value of all future interest payments (present value of all cash flows of the bond), irrespective of whether it is issued at a discount or a premium. The present value is always computed using the bond’s market (effective) rate of interest. The principal amount of a $1,000 to be paid at the end of 10 years. Interest payment payable at the end of each period = $1,000 x 6% = $60 per year for 10 years. Present value for the principal and the interest would be the bond issue price, to be calculated using the bond’s effective/ yield rate of interest = 9% as under:

Amount

Present Value factor

Present Value

Principal Amount

$1,000

0.422

$422

Interest Payment

$60

6.418

$385

Issue Price of the Bond
$807

Option (A) is incorrect because it is the face value of the bond at maturity. It should consider the present value of the principal and present value of interest payment using present value factors at market rate of 9%.
Option (B) is incorrect because interest payments are discounted at stated rate of 6% using the present value factor at 7.36, instead of using present value factor 6.418 [($1,000 x 0.422) + ($60 x 7.36) = $864].
Option (D) is incorrect because present value of the principal amount is calculated at stated interest rate of 6% using present value factor 0.558 and the present value of interest payments are calculated using the future value factor of 2.367 instead of using the present value factor at market rate of 9% for both principal and the interest payments [($1,000 x 0.558) + ($60 x 2.367) = $700].

449
Q
Which of the following is reported as interest expense?
A
Pension cost interest.
B
Amortization of discount of a note.
C
Deferred compensation plan interest.
D
Interest incurred to finance a software development for internal use.
A

Explanation:
Amortization of a bond premium decreases interest expense and the carrying amount of the bond for the issuer, while the amortization of a bond discount increases the issuer’s interest expense and the carrying amount of the bond. Pension cost interest is part of net pension expense. Deferred compensation plan interest is part of compensation expense. Interest incurred to finance a software development for internal use is capitalized as part of the cost of the software.

450
Q
Foley Co. is preparing the electronic spreadsheet below, to amortize the discount on its 10-year, 6%, $100,000 bonds payable. Bonds were issued on December 31 to yield 8%. Interest is paid annually. Foley uses the effective interest method to amortize bond discounts.
A	B	C	D	E
1	Year	Cash paid	Interest expense	Discount amortization	Carrying amount
2	1				$86,580
3	2	$6,000			Which formula should Foley use in cell E3 to calculate the bonds' carrying amount at the end of year 2?
A
E2 + D3
B
E2 - D3
C
E2 + C3
D
E2 - C3
A

Explanation:
Amortization of a bond discount increases the carrying amount of the bond. Adding the bonds’ year 1 carrying amount (cell E2) the discount amortization (cell D3) would net the bond’s carrying amount at the end of year 2 (cell E3). Amortization of a bond premium decreases the carrying amount of the bond. Interest expense is not subtracted or added to the carrying amount directly.

451
Q

On July 1, year 6, Fox Company purchased 400 of the $1,000 face amount, 8% bonds of Dey Corporation for $369,200 to yield 10% per annum. The bonds, which mature on July 1, year 11, pay interest semi­annually on January 1 and July 1. Fox uses the effective interest method of amortization and the bonds are appropriately recorded as a long-term investment.

The bonds should be reported on Fox’s December 31, year 6, balance sheet at?

A
$397,540
B
$374,120
C
$371,660
D
$366,740
A

c

Explanation:
Fox purchased bonds with a full 6 months interest accrued. Discounts are typically not shown in a separate account for bond investments, but netted against the carrying amount. The most efficient method to determine the carrying amount for the bond investment is to plug the following journal entry:

Investment in Bonds $371,660 Plug
Interest Receivable 16,000 $400,000 * 8% * 6/12
Interest Income 18,460 $369,200 * 10% * 6/12
Cash 369,200 given in problem

452
Q
During the year, Lake Co. issued 3,000 of its 9%, $1,000 face value bonds at 101½. In connection with the sale of these bonds, Lake paid the following expenses:
Promotion costs	$ 20,000
Engraving and printing	25,000
Underwriters' commissions	200,000What amount should Lake record as bond issue costs to be amortized over the term of the bonds?
A
$0
B
$220,000
C
$225,000
D
$245,000
A

d

Explanation:
Engraving and printing costs, accounting and legal fees, commissions paid to underwriters, promotion costs, and other similar charges are incurred when bonds are issued. Bond issue costs should be recorded as a deferred charge and amortized over the life of the debt, in a manner similar to that used for discount on bonds. Therefore, $245,000 (i.e., $20,000 + $25,000 + $200,000) should be recorded as bond issue costs to be amortized over the term of the bonds.

453
Q
On December 30 of the current year, Fort Inc. issued 1,000 of its 8%, 10-year, $1,000 face value bonds with detachable stock warrants at par. Each bond carried a detachable warrant for one share of Fort's common stock at a specified option price of $25 per share. Immediately after issuance, the market value of the bonds without the warrants was $1,080,000 and the market value of the warrants was $120,000. In its December 31 current year balance sheet, what amount should Fort report as bonds payable?
A
$1,000,000
B
$ 975,000
C
$ 900,000
D
$ 880,000
A

c

Explanation:
To compute the amount at which the bonds payable should be reported, the proceeds from the bonds and the detachable stock warrants must be allocated between the two securities based on their aggregate relative fair values at the date of issue. Since the bonds are allocated $900,000 of the proceeds received, they were issued at a discount of $100,000 (i.e., $1,000,000 - $900,000). Therefore, the bonds should be reported in the balance sheet at face amount less unamortized discount.

Relative aggregate fair value FMV % Proceeds to be allocated Proceeds allocated to each
Bonds payable $1,080,000 90% $1,000,000 $ 900,000
Stock warrants 120,000 10% $1,000,000 100,000
$1,200,000 100% $1,000,000

454
Q
Main Co. issued bonds with detachable common stock warrants. Only the warrants had a known market value. The sum of the fair value of the warrants and the face amount of the bonds exceeds the cash proceeds. This excess is reported as
A
Discount on bonds payable.
B
Premium on bonds payable.
C
Common stock subscribed.
D
Contributed capital in excess of par-stock warrants.
A

a

Explanation:
The proceeds from the issuance of debt with detachable warrants is to be allocated to paid-in capital (the warrants) and to debt (the bonds) based on the relative fair values of the two securities at the time of issuance. When only the market value of the warrant is known, it is used to record the paid-in capital attributable to the issuance of the warrants. The remainder of the proceeds is recorded in debt accounts. A journal entry approach will be helpful in thinking through the rest of the question. Paid-in Capital is to be credited for the market value of the warrants. Bonds Payable is to be credited for the face amount of the bonds. Cash is to be debited for the cash proceeds. Because the question says that the fair value of the warrants and the face amount of the bonds exceeds the cash proceeds, the entry needs a debit to balance. That debit has to relate to the bonds so it must be to Discount on Bonds Payable. Thus, the amount of proceeds allocated to debt is less than the face amount of the bonds so the bonds are issued at a discount.

455
Q
On January 2, year 1, Burnt Co. issued 10-year convertible bonds at 105. During year 4, these bonds were converted into common stock having an aggregate par value equal to the total face amount of the bonds. At conversion, the market price of Burnt's common stock was 50 percent above its par value. Depending on whether the book value method or the market value method was used, Burnt would recognize gains or losses on conversion when using the
Book value method	Market value method
A	Either gain or loss	Gain
B	Either gain or loss	Loss
C	Neither gain or loss	Loss
D	Neither gain or loss	Gain
A

c

Explanation:
A major characteristic of the book value method is that neither a gain nor a loss is recognized on the conversion of bonds to stock; the carrying amount of debt is taken out of the debt accounts and recorded in stockholders’ equity accounts. The market value method may result in a gain or loss because the stock is to be recorded at the market value of the stock (or bonds) and the carrying amount of the debt is to be removed from liability accounts. A difference between the market value of the stock and the carrying amount of the debt is to be recorded as a gain or loss, whichever is appropriate. Because of the relationships of amounts involved, it is evident that the market value of the stock exceeds the carrying amount of the debt; therefore, a loss will be recorded on the conversion. Those relationships are as follows: (1) the aggregate par value of the stock was equal to the total face amount (par value) of the bonds, (2) the market value of the stock is 50% above its par, and (3) the carrying amount of the debt is less than 5% above its par (the bonds were issued in a prior year at a 5% premium and at least 30% of that premium had been amortized prior to year 4)..

456
Q
Verona Co. had $500,000 in short-term liabilities at the end of the current year. Verona issued $400,000 of common stock subsequent to the end of the year, but before the financial statements were issued. The proceeds from the stock issue were intended to be used to pay the short-term debt. What amount should Verona report as a short-term liability on its balance sheet at the end of the current year?
A
$0
B
$100,000
C
$400,000
D
$500,000
A

b

Explanation:
FASB ASC 470-10-45-14 requires that short-term obligations be reported as long-term liabilities if a company (1) intends to refinance the short-term obligation on a long-term basis and (2) demonstrates the ability to refinance it a long-term basis. The intent is stated in the problem. Verona’s issuance of common stock for $400,000 before the statements were issued demonstrates the ability to refinance $400,000 of the short-term obligations on a long-term basis. The balance of the obligation ($100,000) must be reported as a current liability.

457
Q
\_\_\_\_\_\_\_\_\_\_\_\_\_\_ funds result from the setting aside of specific assets, usually under the custody of a trustee for a particular purpose.
A
Special purpose
B
Preferred stock acquisition
C
Debt retirement
D
All of the above
A

d

Explanation:
Special purpose funds result from the setting aside of specific assets, usually under the custody of a trustee for a particular purpose. Some funds may be voluntarily created, such as preferred stock acquisition funds and plant expansion funds. Other funds result from contractual obligations, such as debt retirement funds. The funds may or may not be under the custody of a separate trustee. The accounting depends on the specific nature of the fund, legal requirements, etc. Since answers A., B., and C. are correct, answer D., all of the above, is the best choice

458
Q
On January 2 of the current year, West Co. issued 9% bonds in the amount of $500,000, which mature in ten years. The bonds were issued for $469,500 to yield 10%. Interest is payable annually on December 31. West uses the effective interest method of amortizing bond discount. In its June 30 current year balance sheet, what amount should West report as bonds payable?
A
$469,500
B
$470,475
C
$471,025
D
$500,000
A

b

Explanation:
$469,500 + $975 = $470,475

Bonds payable carrying amount, 1/2 $ 469,500
Effective interest rate (10% × 6/12) × 5%
Interest expense, 1/2 - 6/30 $ 23,475
Interest payment [$500,000 × (9% × 6/12)] (22,500)
Amortization of discount, 6/30 $ 975

459
Q
A company issued a short-term note payable with a stated 12% rate of interest to a bank. The bank charged a .5% loan origination fee and remitted the balance to the company. The effective interest rate paid by the company in this transaction would be
A
Equal to 12.5%.
B
More than 12.5%.
C
Less than 12.5%.
D
Independent of 12.5%.
A

b

Explanation:
The note’s 12% rate of interest and the bank’s 0.5% loan origination fee are based upon the face amount of the note. The cash received by the note holder is the face amount of the note less the loan origination fee. Since less cash is received than the amount on which the interest rate and loan origination fee is computed, the effective interest rate is more than 12.5%..

460
Q
On April 1, year 9, Ward Corp. issued $750,000 of 10% nonconvertible bonds at 102 that are due on March 31, year 19. Each $1,000 bond was issued with 40 detachable stock warrants, each of which entitled the bondholder to purchase one share of Ward $10 par common stock for $25. On April 1, year 9, the market value of Ward's common stock was $20 per share, and the market value of each warrant was $4. What amount of the proceeds from the bond issue should Ward record as an increase in stockholders' equity?
A
$ 15,000
B
$120,000
C
$300,000
D
$750,000
A

b

Explanation:
Allocation of the proceeds between the bond and the detachable stock purchase warrants, based on their relative fair values at the date of issue, is required. If the relative fair values are not known, then the fair value of either security is used. The question provides the fair value of the warrants, but not the fair value of the bonds without the warrants; therefore, the $4 fair value of the warrants is used. A stockholders’’ equity account, paid-in capital–stock purchase warrants, is increased by $120,000 [($750,000 / $1,000) bonds × 40 warrants × $4 fair value].

461
Q

Bondholders of Balm Co. converted their bonds into 90,000 shares of $5 par value common stock. In Balm’s accounting records, the bonds had a par value of $775,000 and unamortized discount of $23,000 at the time of conversion. What amount of additional paid-in capital from the conversion should Balm record?

A
$302,000
B
$325,000
C
$348,000
D
$798,000
A

a

Explanation:
Convertible bonds provide the bondholder the option of converting the bond to capital stock, typically common stock. Using the book value method, the conversion of the bonds into common stock is generally recorded by crediting the paid-in capital accounts for the carrying amount of the debt at the date of the conversion, less any cost associated with the conversion. The carrying amount of the bonds on the date of conversion is the $775,000 face value less the $23,000 unamortized discount. The journal entry would be:

Bonds Payable 775,000
Bond Discount 23,000
Common Stock (90,000 × $5 par) 450,000
APIC (to balance) 302,000
Option (B) is incorrect because $325,000 does not account for unamortized discount which should be credited.
Option (C) is incorrect because unamortized discount is debited, instead of crediting it.
Option (D) is incorrect because it is the sum of bonds payable and unamortized discount ($775,000 + $23,000). Unamortized discount should be credited and further common stock issued at par also needs to be credited.

462
Q

Which of the following is (are) true regarding convertible bonds?
A
The market value method recognizes no gain or loss upon conversion.
B
The market value method does not recognize costs associated with the conversion as an expense.
C
The market value method recognizes a gain or loss on retirement equal to the difference between the carrying amount of the debt at the conversion and the fair value of the shares issued upon conversion.
D
Both A. and B.

A

c

Explanation:
The market value method recognizes a gain or loss on retirement equal to the difference between the carrying amount of the debt at the date of the conversion and the fair value of the shares issued upon conversion.

463
Q

On June 1, Greendale Corp. issued $700,000, five-year bonds at 8%, with interest payable annually on May 31. The bonds sold for $728,700 when the market rate of interest was 7%. Greendale uses the effective interest method for amortizing premiums on bonds payable. What is the balance of the premiums on bonds payable account immediately following the first interest payment?

A
$22,960
B
$23,709
C
$33,691
D
$34,440
A

Explanation:
The correct answer is (B).

This question is asking to use the effective interest method for amortizing premiums on bonds payable. On June 1, Greendale Corp. issued $700,000, five-year bonds at 8%, with interest payable annually on May 31. The bonds sold for $728,700 when the market rate of interest was 7%. The balance on bonds payable subsequent to first interest payment:

Interest paid = $700,000 x 8% = $56,000
Interest expense = $728,700 x 7% = $51,009
Premium amortization = $28,700 ($728,700 - $700,000) - $4,991 ($56,000 - $51,009) = $23,709.

464
Q
On August 15 of the current year, Benet Co. sold goods for which it received a note bearing the market rate of interest on that date. The four-month note was dated this July 15. Note principal, together with all interest, is due November 15. When the note was recorded on August 15, which of the following accounts increased?
A
Unearned discount
B
Interest receivable
C
Prepaid interest
D
Interest revenue
A

b

Explanation:
When an interest-bearing instrument is sold between interest payment dates, the seller collects accrued interest from the buyer. The buyer will later collect interest for a full interest period at the next interest payment date. In the case at hand, inventory is being exchanged for the note. Because the market rate of interest is equal to the note’s stated rate, the note’s present value is equal to the face amount of the note plus the one month’s accrued interest. The journal entry to record the exchange involves a debit to Note Receivable for its face amount, a debit to Interest Receivable for the one month of accrued interest, and a credit to Sales Revenue for the sum of the note’s face amount and the one month of accrued interest.

465
Q
On December 30, year 7, Bart, Inc. purchased a machine from Fell Corp. in exchange for a noninterest bearing note requiring eight payments of $20,000. The first payment was made on December 30, year 7, and the others are due annually on December 30. At date of issuance, the prevailing rate of interest for this type of note was 11%. Present value factors are as follows:
Period	Present value of ordinary annuity of $1 at 11%	Present value of annuity in advance of $1 at 11%
7	4.712	5.231
8	5.146	5.712On Bart's December 31, year 7 balance sheet, the note payable to Fell was
A
$ 94,240
B
$102,920
C
$104,620
D
$114,240
A

a

Explanation:
The factor for the present value of an annuity in advance is used because the first payment of the note was made immediately (i.e., 12/30, year 7). In addition, the carrying amount of the note at 12/31, year 7 is reduced by the $20,000 payment on 12/30, year 7.

Periodic annual payments $ 20,000
Times: Present value of an annuity in advance of $1 at 11% for 8 periods × 5.712
Present value of all cash flows 114,240
Less: Payment, December 30, year 7 (20,000)
Carrying amount of note, December 31, year 7 $ 94,240

466
Q
The following information relates to noncurrent investments that Fall Corp. placed in trust as required by the underwriter of its bonds:
Bond sinking fund bal., 12/31, year1	$ 450,000
Year 2 additional investment	90,000
Dividends on investments	15,000
Interest revenue	30,000
Administration costs	5,000
Carrying amount of bonds payable	1,025,000What amount should Fall report in its December 31, Year 2, balance sheet related to its noncurrent investment for bond sinking fund requirements?
A
$585,000
B
$580,000
C
$575,000
D
$540,000
A

b

Explanation:
The bond sinking fund balance increases as a result of the additional investment and the income on the investments in the fund (i.e., the dividend and interest revenue). It decreases due to the expenses of the fund (i.e., the administrative costs incurred). The carrying amount of the bonds payable does not affect the bond sinking fund balance.

Bond sinking fund balance, 12/31, year 1 $450,000
Add: Additional investment, year 2 90,000
Dividends on investments 15,000
Interest revenue 30,000
Less: Administrative costs (5,000)
Bond sinking fund balance, 12/31, year 2 $580,000

467
Q
On October 1, year 2, Park Co. purchased 200 of the $1,000 face value, 10% bonds of Ott, Inc., for $220,000, including accrued interest of $5,000. The bonds, which mature on January 1, year 9, pay interest semiannually on January 1 and July 1. Park used the straight-line method of amortization and appropriately recorded the bonds as a long-term investment. On Park's December 31, year 3 balance sheet, the bonds should be reported at
A
$215,000
B
$214,400
C
$214,200
D
$212,000
A

d

Explanation:
The carrying amount of the bonds at 12/31, year 3 is determined by subtracting the amortization of the bond premium from the date of purchase from the cost of the bond investment.

Bond investment cost, 10/1, year 2($220,000 - $5,000) $215,000
Bond investment cost, 10/1, year 2 $ 215,000
Less: face amount of bonds (200 x $1,000) (200,000)
Bond premium $ 15,000
Divided by: months to maturity (10/1, yr 2 to 1/1, yr 9) / 75
Monthly premium amortization $ 200
Times: Months from issue × 15
Less amortization of premium to 12/31 (3,000)
Bond carrying amount, 12/31 of year 3 $212,000

468
Q
On July 1 of the current year, York Co. purchased as a long-term investment $1,000,000 of Park, Inc.'s 8% bonds for $946,000, including accrued interest of $40,000. The bonds were purchased to yield 10% interest. The bonds mature on January 1 seven years from now and pay interest annually on January 1. York uses the effective interest method of amortization. In its December 31 current year balance sheet, what amount should York report as investment in bonds?
A
$911,300
B
$916,600
C
$953,300
D
$960,600
A

a

Explanation:
The carrying amount of the bond investment is the amortization of the bond discount from the purchase date plus the cost of the bond investment ($906,000 + $5,300 = $911,300).

Bond investment cost ($946,000 - $40,000) $906,000
Times: effective interest rate (10% / 2) × 5%
Interest income, 7/1 - 12/31 45,300
Portion of annual interest payment applicable to 7/1 - 12/31 [($1,000,000 x 8%) / 2] (40,000)
Amortization of bond premium to 12/31 $ 5,300

469
Q

A company issues $1,500,000 of par bonds at 98 on January 1, year 1, with a maturity date of December 31, year 30. Bond issue costs are $90,000, and the stated interest rate of the bonds is 6%.

Interest is paid semiannually on January 1 and July 1. Ten years after the issue date, the entire issue was called at 102 and canceled.

The company uses the straight-line method of amortization for bond discounts and issue costs, and the result of this method is not materially different from the effective interest method.

The company should classify what amount as the loss on extinguishment of debt at the time the bonds are called?

A
$30,000
B
$50,000
C
$90,000
D
$110,000
A

Explanation:
The correct answer is (D).

The par bonds being sold at 98 indicate that they are being sold at a 2% discount. Total Discount is 2% x $1,500,000 = $30,000.

Discounts on bonds held long-term must be amortized from date of acquisition to maturity date, normally using the interest method to amortize these differences.

Other methods of amortization (straight-line) may be used if the effects are not material. Bond issue costs should be classified as a deferred charge (i.e., asset) and amortized over the life of the bonds as an increase to interest expense. When all or part of a bond issue is retired before maturity, it is necessary to write off both the principal and the pro rata portion of the unamortized premium or discount on the retired bonds.

If bond issue costs were incurred and recorded as an asset (i.e., as a deferred charge), it is also necessary to write off a pro rata portion of the bond issue costs (when retired before maturity). The amount of such write-off increases any loss or reduces any gain recognized on retirement.

The sale of bonds results in a gain or loss equal to the difference between the carrying amount of the bonds and the proceeds received on their disposal. In determining the carrying amount of the bonds, an adjustment must be made for the discount amortization to date of sale.

Bonds Payable (Face Value) $1,500,000
Unamortized Discount [($30,000 / 30) × 20 yrs.] ($20,000)
Unamortized Issue costs [($90,000 / 30) × 20 yrs.] ($60,000)
Carrying amount of bonds retired $1,420,000
Purchase price ($1,500,000 × 102%) $1,530,000
Loss on bond retirement, before income taxes ($110,000)

470
Q

A 15-year bond was issued in year 1 at a discount. During year 11, a 10-year bond was issued at face amount with the proceeds used to retire the 15-year bond at its face amount. The net effect of the year 11 bond transactions was to increase long-term liabilities by the excess of the 10-year bond’s face amount over the 15-year bond’s
A
Face amount.
B
Carrying amount.
C
Face amount less the deferred loss on bond retirement.
D
Carrying amount less the deferred loss on bond retirement.

A

b

Explanation:
The new 10-year bond was issued at its face amount which, from the facts, either equals or exceeds the face amount of the 15-year bond which exceeds the carrying amount of the 15-year bond because the 15-year bond was issued at a discount (a price less than the face amount). The excess of the retirement price (face amount) of the 15-year bond over its carrying amount will be recorded as a loss at retirement. Therefore, the issuance of the new bond and the retirement of the old bond will have the net effect of increasing the total of long-term liabilities by the excess of the new bond’s face amount over the old bond’s carrying amount.

471
Q

Blue Corp.’s December 31, year 1, balance sheet contained the following items in the long-term liabilities section:

9 3/4% registered debentures, callable in year 12, due in year 17 $700,000
9 1/2% collateral trust bonds, convertible into common stock beginning in year 10, due
in year 20 600,000
10% subordinated debentures ($30,000 maturing annually beginning in year 7) 300,000
What is the total amount of Blue’s term bonds?

A
$ 600,000
B
$ 700,000
C
$1,000,000
D
$1,300,000
A

d

Explanation:
Bond issues maturing on a single date are called term bonds. Bond issues maturing in installments are called serial bonds.

Since the debentures due in year 17 and the bonds due in year 20 each mature on a single date, the total amount of term bonds is $1,300,000 (i.e., $700,000 + $600,000).

The total amount of serial bonds is $300,000.

472
Q

A five-year term bond was issued by a company on January 1, year 6, at a discount. The carrying amount of the bond at December 31, year 7, would be
A
Higher than the carrying amount at December 31, year 6.
B
Lower than the carrying amount at December 31, year 6.
C
The same as the carrying amount at January 1, year 6.
D
Higher than the carrying amount at December 31, year 8.

A

a

Explanation:
When bonds are issued at a discount, the Discount on Bonds Payable account is deducted from the Bonds Payable account to determine the initial carrying amount of the bonds. As the bond account discount is amortized (written off), there is a smaller amount subtracted from the bonds and the carrying amount of the bond increases. Therefore, the carrying amount of the bonds in question at December 31, year 7, would be higher than their carrying amount at December 31, year 6.

473
Q

Cali, Inc., had a $4,000,000 note payable due on March 15 of the current year. On January 28 of the current year, before the issuance of its prior year financial statements, Cali issued long-term bonds in the amount of $4,500,000. Proceeds from the bonds were used to repay the note when it came due. How should Cali classify the note in its prior year December 31 financial statements?
A
As a current liability, with separate disclosure of the note refinancing.
B
As a current liability, with no separate disclosure required.
C
As a noncurrent liability, with separate disclosure of the note refinancing.
D
As a noncurrent liability, with no separate disclosure required.

A

c

Explanation:
The portion of long-term debt due within the next fiscal period is classified as a current liability if payment is expected to require the use of current assets or the creation of other current liabilities. Short-term debt expected to be refinanced on a long-term basis is to be excluded from current liabilities when the enterprise has the intent and ability to refinance the obligation on a long-term basis. One way intent and ability are demonstrated is by post-balance-sheet-date issuance of a long-term obligation or equity securities. Separate disclosure of such refinancing is required.

474
Q

A company issued a bond with a stated rate of interest that is less than the effective interest rate on the date of issuance. The bond was issued on one of the interest payment dates. What should the company report on the first interest payment date?
A
An interest expense that is less than the cash payment made to bondholders.
B
An interest expense that is greater than the cash payment made to bondholders.
C
A debit to the unamortized bond discount.
D
A debit to the unamortized bond premium.

A

b

Explanation:
A bond that is issued with a stated rate of interest that is less than the effective rate on the issuance date is a bond issued with a discount. A bond issued at a discount will have an increasing carrying value and an increasing amount of interest expense. On the first interest payment date the company should report an interest expense that is greater than the cash payment made to bondholders.

475
Q

Jent Corp. purchased bonds at a discount of $10,000. Subsequently, Jent sold these bonds at a premium of $14,000. During the period that Jent held this investment, amortization of the discount amounted to $2,000. What amount should Jent report as gain on the sale of bonds?

A
$12,000
B
$22,000
C
$24,000
D
$26,000
A

b

Explanation:
Since the bond was purchased at a discount, the initial carrying value of the bond investment is $10,000 less than the face amount. The amortization of the discount increases the bond investment and so, on the date of sale the bond investment is carried on Jent Corp.’s books at $8,000 less (i.e., $10,000 – 2,000) than the face amount. Therefore, the sale of the bond at a premium (i.e., at $14,000 more than the face amount of the bond) results in recognition of a $22,000 gain (i.e., $8,000 + $14,000).

Option (A)is incorrect because the amortization is added to the discount number ignoring the premium on sale.
Option (C) is incorrect because this is the difference between the discount and premium ignoring the amortization.
Option (D) is incorrect because here the amortization is added to the difference rather than deducted

476
Q

Straight-line amortization calls for the amortization of an equal amount of premium or discount each period over the life of the bonds. When is the use of straight-line amortization acceptable?
A
Only when the bond term is less than five years
B
Only when the premium or discount is immaterial
C
When the bond term is greater than five years
D
When the premium or discount is material

A

b

Explanation:
The straight-line method is acceptable only when the premium or discount is immaterial because it fails to determine the periodic interest expense in terms of the effective rate of interest.

477
Q

Andro Co. has a $10 million note payable that is due three months after year-end. The note payable was refinanced when long-term bonds were issued one month after year-end for $11 million. The December 31 financial statements were issued two months after year end.

How should Andro classify and disclose the note?

Classification of liability	Note disclosure required
A	Current	No
B	Current	Yes
C	Non-current	No
D	Non-current	Yes
A

Explanation:
The correct answer is (D)

Under GAAP, if a company demonstrates both the intent and ability to refinance short-term obligations on a long-term basis after the balance sheet date but before the issuance of the financial statements, then the obligations may be categorized as long-term on the financial statements and will be classified as a non-current liability.

As Andro Co. issued the bonds to refinance the notes payable, both these conditions are satisfied. The note payable will be classified as a non-current liability.

Per the disclosure requirements, if a short term obligation is excluded from current liabilities, the notes to financial statements should include the description of the financing agreement and the terms of the new obligation incurred or expected to be incurred issued as a result of refinancing.

478
Q
On January 1, year 2, Oak Co. issued 400 of its 8%, $1,000 bonds at 97 plus accrued interest. The bonds are dated October 1, year 1, and mature in fifteen years on October 1 . Interest is payable semiannually on April 1, and October 1. Accrued interest for the period October 1, year 1, to January 1, year 2, amounted to $8,000. On January 1, year 2, what amount should Oak report as bonds payable, net of discount?
A
$380,300
B
$388,000
C
$388,300
D
$392,000
A

b

Explanation:
The bonds payable account should reflect only the principal amount and does not include accrued interest. Therefore, the bonds payable account should be reported by Oak Co. as $388,000 (400 bonds x (1,000 x 97%)).

479
Q

How should bond issue costs be recorded?

A
Expensed in the period incurred.
B
Deducted from Carrying Value of Bonds and amortized using Effective Interest Method.
C
Classified as an asset.
D
All of the above
A

Explanation:
The correct answer is (B).

FASB issued Accounting Standards Update 2015-03 requiring Bond Issue Costs to deducted from Carrying Value of bonds and amortized using effective interest method - i.e., Bond Issue Costs is treated similar to discount/premium & reported as an adjustment to Bond Payable liability.

480
Q

Young Co. issues $800,000 of 10% bonds dated January 1, year 1. Interest is payable semiannually on June 30 and December 31. The bonds mature in five years. The current market for similar bonds is 8%. The entire issue is sold on the date of issue. The following values are given:

Present value of ordinary annuity Present value of $1

n=10; i=0.04 8.11090 0.67556
n=10; i=0.05 7.72173 0.61391
What amount of proceeds on the sale of bonds should Young report?

A
$799,997
B
$815,564
C
$849,317
D
$864,884
A

d

Explanation:
A premium or discount on bonds arises when the stated interest rate of the bonds is higher or lower, respectively, than the current market interest rate for similar securities. Premiums or discounts on bonds held as a long-term investment must be amortized from date of acquisition to maturity date and the interest method should be used to amortize these differences. To estimate the proceeds to be received from the issuance of bonds payable (ignoring bond issue costs), the present values of the bond principal and interest payments must be determined. A bond will sell at a premium (more than par) when the stated interest rate is greater than the market rate for similar debt.

Present value of principal payment [$800,000 × 0.67556 (PV of $1 for 10 periods at 4%)] $ 540,448
Present value of periodic interest payments [($800,000 × 10% / 2) × 8.11090] __324,436
Amount received from the issuance of the bonds $ 864,884
The stated rate of interest (10%) is above the market rate (8%); therefore, these bonds were sold at a premium. The following entry is made to record the bond
issuance.

Cash 864,884
Bonds Payable 800,000
Bond Premium (difference) 64,884
Option (A) is incorrect because it uses incorrect PV factors.

Amount($) PV Factor

Amount($)

Principal

800,000 0.61391 491,128
Interest @ 5%

40,000 7.72173 308,869

799,997

481
Q

East Corp., a calendar-year company, had sufficient retained earnings in year 1 as a basis for dividends, but was temporarily short of cash. East declared a dividend of $100,000 on April 1, year 1, and issued promissory notes to its stockholders in lieu of cash. The notes, which were dated April 1, year 1, had a maturity date of March 31, year 2, and a 10% interest rate. How should East account for the scrip dividend and related interest?
A
Debit retained earnings for $110,000 on April 1, year 1.
B
Debit retained earnings for $110,000 on March 31, year 2.
C
Debit retained earnings for $100,000 on April 1, year 1, and debit interest expense for $10,000 on March 31, year 2.
D
Debit retained earnings for $100,000 on April 1, year 1, and debit interest expense for $7,500 on December 31, year 1.

A

d

Explanation:
At the end of the year, interest is accrued.

At the date of declaration, April 1, year 1, the journal entry is as follows.
Retained Earnings 100,000
Notes Payable to Stockholders 100,000
And then on December 31, year 1
Interest Expense ($100,000 × 10% × 9/12) 7,500
Interest Payable 7,500The interest portion of the payment is not treated as part of the dividend. When the dividend is declared, it becomes a liability and is reported on the balance sheet at that time. If the entry to accrue the interest expense is made on December 31, year 1, and is reversed on January 1, year 2, then interest expense would be debited for $10,000 on March 31, year 2. However, since answer (c) does not state this, assume that the reversing entry was not made

482
Q
On May 18 of the current year, Sol Corp.'s board of directors declared a 10% stock dividend. The market price of Sol's 3,000 outstanding shares of $2 par value common stock was $9 per share on that date. The stock dividend was distributed on July 21, of this year, when the stock's market price was $10 per share. What amount should Sol credit to additional paid-in capital for this stock dividend?
A
$2,100
B
$2,400
C
$2,700
D
$3,000
A

a

Explanation:
The declaration of a’small’ stock dividend (i.e., less than 20-25% of the number of common shares outstanding) should be recorded by capitalizing a portion of retained earnings equal to the fair value of the shares to be issued. Fluctuations in the fair value of the shares between the declaration date and the issuance date are not recorded. The amount credited to Additional Paid-in-Capital is the excess of the fair market value of the stock over its par value, since Common Stock is credited for par value.

Retained earnings [(3,000 × 10%) × $9 FMV] 2,700
Common stock dividend distributable [(3,000 × 10%) × $2 PV] 600
Additional paid-in capital (to balance) 2,100To record the declaration of the 10% stock dividend.

483
Q
In the current year, Bal Corp. declared a $25,000 cash dividend on May 8 to stockholders of record on May 23 payable on June 3. As a result of this cash dividend, working capital
A
Was not affected.
B
Decreased on June 3.
C
Decreased on May 23.
D
Decreased on May 8.
A

d

Explanation:
On the declaration date, May 8, the liability for dividends payable is recorded by a debit to Retained Earnings and a credit to Cash Dividends Payable . Since the declaration of the cash dividend increases current liabilities without affecting current assets, working capital is decreased on this date. On May 23, the date of record, which stockholders will receive the cash dividend is determined. Assuming that the common shares outstanding did not change between the date of declaration and the date of record, no journal entry is made on the date of record and, thus, working capital is not affected on this date. On the payment date, June 3, the cash dividend is paid and is recorded by a debit to Cash Dividends Payable and a credit to Cash . Since current assets and current liabilities decrease by the same amount, working capital is not affected on this date.

484
Q

When Mill retired from the partnership of Mill, Yale, and Lear, the final settlement of Mill’s interest exceeded Mill’s capital balance. Under the bonus method, the excess
A
Was recorded as goodwill.
B
Was recorded as an expense.
C
Reduced the capital balances of Yale and Lear.
D
Had no effect on the capital balances of Yale and Lear.

A

c

Explanation:
The final settlement of Mill’s interest exceeded his capital balance. The excess payment represents Mill’s share of the unrecorded goodwill of the partnership. Under the bonus method, this excess payment would be recorded as a decrease in the remaining partners’ capital accounts.

485
Q
Jensen performed legal services to assist Balm Co. in accomplishing its initial organization. Jensen accepted 1,000 shares of $5 par common stock in Balm as payment for his services. The Balm shares were not yet publicly traded, but they had a book value of $4 per share. Jensen provided 48 hours of service, which is normally billed at $125 per hour. By what amount should the common stock account increase?
A
$1,000
B
$4,000
C
$5,000
D
$6,000
A

c

Explanation:
If stock is issued for noncash consideration, the property received and amount of contributed capital should be recorded at the fair value of the property received or the market value of the stock, whichever is more objectively determinable. If fair values are not determinable, then appraised values or values set by the board of directors may be used. The fair value of the stock is not determinable because it is not yet traded publicly. The fair value of the services is $6,000 (48 hours × $125 per hour). The common stock account would increase by the 1,000 shares × $5 par value = $5,000. The remaining $1,000 would be an increase to the additional paid-in capital account.

486
Q

On April 1, Ivy began operating a service proprietorship with an initial cash investment of $1,000. The pro­prietorship provided $3,200 of services in April and received full payment in May. The proprietorship incurred expenses of $1,500 in April which were paid in June. During May, Ivy drew $500 against her cap­ital account. What was the proprietorship’s income for the two months ended May 31, under the following methods of accounting?

Cash-basis	Accrual-basis
A	$1,200	$1,200
B	$1,700	$1,700
C	$2,700	$1,200
D	$3,200	$1,700
A

d

Explanation:
The sole proprietor’s drawing of $500 is recorded as a reduction of her capital account under both the cash-basis and accrual-basis methods of accounting.

Cash Accrual

Revenue for services provided in April, payment received May $3,200 $3,200
April expenses, paid June – (1,500)
Proprietorship’s income for two months ended May 31 $3,200 $1,700

487
Q
On May 1, Cobb and Mott formed a partnership and agreed to share profits and losses in the ratio of 3:7, respectively. Cobb contributed a parcel of land that cost him $10,000. Mott contributed $40,000 cash. The land was sold for $18,000 on that same date, immediately after formation of the partnership. What amount should be recorded in Cobb's capital account on formation of the partnership?
A
$18,000
B
$17,400
C
$15,000
D
$10,000
A

a

Explanation:
Using historical costs for assets such as inventory, land, or equipment would be inequitable to any partner investing appreciated property. Therefore, the contribution of noncash assets to a partnership should be recorded based on fair values. The land was sold on the same date for $18,000 so that would be used for its fair value amount.

488
Q

A company issued rights to its existing shareholders to purchase for $15 per share, 5,000 unissued shares of common stock with a par value of $10 per share. Common stock will be credited at:
A
$15 per share when the rights are exercised
B
$15 per share when the rights are issued
C
$10 per share when the rights are exercised
D
$10 per share when the rights are issued

A

c

Explanation:
A stock right issue generally is recorded by memorandum entry only (but must be disclosed in the financial statement notes). When the rights are exercised, common stock is credited at its par value, regardless of the option price or the stock’s FMV. The difference between the option price and the par value of the stock (i.e., $15 - $10 = $5) is credited to Additional Paid-In Capital when the stock is issued.

489
Q
The Flat and Iron partnership agreement provides for Flat to receive a 20% bonus on profits before the bonus. Remaining profits and losses are divided between Flat and Iron in the ratio of 2 to 3, respectively. Which partner has a greater advantage when the partnership has a profit or when it has a loss?
Profit	Loss
A	Flat	Iron
B	Flat	Flat
C	Iron	Flat
D	Iron	Iron
A

b

Explanation:
When there is a profit, Flat receives 20% of the profits before the bonus and 40% of the remaining 80% (i.e., 32%), for a total of 52% of the profits. In loss situations, Flat receives only 40% of the loss. Thus Flat has a greater advantage whether the partnership has a profit or loss.

490
Q
At December 31 of year 2 and year 3, Apex Co. had 3,000 shares of $100 par, 5% cumulative preferred stock outstanding. No dividends were in arrears as of December 31, year 1. Apex did not declare a dividend during year 2. During year 3, Apex paid a cash dividend of $10,000 on its preferred stock. Apex should report dividends in arrears in its year 3 financial statements as a (an)
A
Accrued liability of $15,000.
B
Disclosure of $15,000.
C
Accrued liability of $20,000.
D
Disclosure of $20,000.
A

d

Explanation:
If a cumulative preferred stock dividend is not paid in any given year, it becomes a dividend in arrears. No liability exists until the board of directors declares a dividend; a dividend in arrears is not accrued, but is disclosed in the financial statement notes. No dividend was declared in year 2, resulting in a dividend in arrears of $15,000 ($100 × 5% × 3,000 shares). Therefore, in its year 3 financial statements, Apex reports dividends in arrears of $20,000 ($15,000 from year 2 + $5,000 from year 3).

Dividend amount ($100 × 5% × 3,000 shares) $ 15,000
Less: Dividend paid, year 3 (10,000)
Dividend in arrears, year 3 $ 5,000

491
Q

A company had 400,000 shares of common stock issued and outstanding on January 1, year 1, and had the following equity transactions for year 1:

Transactions

Date

Issued 200,000 new shares for cash

April 1

Issued new shares as a result of a 2-for-1 stock split

July 1

Purchased 300,000 shares treasury stock for cash

October 1

What should the company use as the denominator for the calculation of basic earnings per share for the year ended December 31, year 1?

A
1,650,000
B
1,025,000
C
1,325,000
D
1,075,000
A

Explanation:
The correct answer is (B).

Calculation of Weighted average common stock outstanding:

Date

Particulars

No. of months outstanding

Calculation

Total outstanding

1/1

Outstanding 400,000 shares

12 months

400,000 x 12 /12

400,000

4/1

Issued 200,000 new shares for cash

9 months

200,000 x 9 /12

150,000

7/1

Issued new shares as a result of a 2 for 1 stock split (retroactively adjusted from the time of issue)

Retroactively

(400,000 x 2) + (150,000 x 2)

1,100,000

10/1

Purchased as treasury stock 300,000

3 months

300,000 x 3 /12

(75,000)

                           Weighted average common stock outstanding                                       1,025,000
492
Q

Which of the following is (are) true regarding dividends?
A
At the date of declaration, property dividends are recorded at the fair value of assets given up, and any difference between fair value and carrying amount of the asset is recorded as a gain or loss as a component of other comprehensive income.
B
Liquidating dividends represent a share in the profits of the company.
C
The shareholder has no income as a result of a stock dividend.
D
All of the above

A

c

Explanation:
The shareholder has no income as a result of the stock dividend, because the stock dividend (or stock split) is not a distribution division or severance of the corporate assets.

493
Q
\_\_\_\_\_\_\_\_\_\_\_\_\_ stock avoids the contingent liability involved with the issuance of other types of stocks at a discounted price.
A
Par value
B
No-par value
C
No-par with stated value
D
None of the above
A

b

Explanation:
No-par value stock avoids the contingent liability involved with the issuance of par value stock at a price below par (discount).

494
Q
During the previous year, Brad Co. issued 5,000 shares of $100 par convertible preferred stock for $110 per share. One share of preferred stock can be converted into three shares of Brad's $25 par common stock at the option of the preferred shareholder. On December 31 of the current year, when the market value of the common stock was $40 per share, all of the preferred stock was converted. What amount should Brad credit to Common Stock and to Additional Paid-in Capital--Common Stock as a result of the conversion?
Common stock	Additional paid-in capital
A	$375,000	$175,000
B	$375,000	$225,000
C	$500,000	$ 50,000
D	$600,000	$0
A

a

Explanation:
Common Stock should be credited for $375,000 [i.e., (5,000 × 3) × $25], the par value of the common shares issued to effect the conversion. Additional Paid-in Capital–Common Stock should be credited for $175,000, the excess of the carrying amount of the preferred stock converted over the par value of common shares issued to effect the conversion [i.e., (5,000 x $110) - $375,000].

495
Q
The following format was used by Gee, Inc. for its current year statement of owners' equity:
Common stock, $1 par	Additional
paid-in capital	Retained earnings
Balance at 1/1	$90,000	$800,000	$175,000
Additions and deductions:			
100% stock dividend			
5% stock dividend	\_\_\_\_\_\_\_\_\_	\_\_\_\_\_\_\_\_\_	\_\_\_\_\_\_\_\_\_
Balance at 12/31	\_\_\_\_\_\_\_\_\_	\_\_\_\_\_\_\_\_\_	\_\_\_\_\_\_\_\_\_When both the 100% and the 5% stock dividends were declared, Gee's common stock was selling for more than its $1 par value. How would the 100% stock dividend affect the additional paid-in capital and retained earnings amounts reported in Gee's year-end statement of owners' equity?
Additional
paid-in capital	Retained earnings
A	Increase	Increase
B	Increase	Decrease
C	No change	Increase
D	No change	Decrease
A

d

Explanation:
Since the 100% stock dividend exceeds 20 to 25% of the number of shares outstanding, it is considered to be a ‘large’ stock dividend. Thus, it should be recorded by capitalizing a portion of Retained Earnings equal to the par value of the shares issued. As a result of the stock dividend, Retained Earnings will decrease and Common Stock will increase by the par value of the shares issued. Additional Paid-in Capital will not change as a result of the ‘large’ stock dividend.

496
Q

On November 2 of year 2, Finsbury, Inc. issued warrants to its stockholders giving them the right to purchase additional $20 par value common shares at a price of $30. The stockholders exercised all warrants on March 1 of year 3. The shares had market prices of $33, $35, and $40 on November 2, year 2, December 31, year 2, and March 1, year 3, respectively. What were the effects of the warrants on Finsbury’s additional paid-in capital and net income?
Additional paid-in capital Net income
A Increased in year 3 No effect
B Increased in year 2 No effect
C Increased in year 3 Decreased in years 2 and 3
D Increased in year 2 Decreased in years 2 and 3

A

a

Explanation:
Stock rights have no impact on net income. No entry is required when stock rights are issued to existing stockholders (other than a memorandum entry). Therefore, APIC was not affected when the stock rights were issued. Stock issued upon the exercise of stock rights is recorded the same as any other issuance; therefore, when the stock rights were exercised APIC increased by the excess of the $30 exercise price over the $20 par value of the common stock.

497
Q

Long Co. had 100,000 shares of common stock issued and outstanding at January 1 of the current. During the year, Long took the following actions:

March 15 – Declared a 2-for-1 stock split, when the fair value of the stock was $80 per share.

December 15 – Declared a $.50 per share cash dividend.

In Long’s statement of stockholders’ equity for the current year, what amount should Long report as dividends?

A
$ 50,000
B
$100,000
C
$850,000
D
$950,000
A

b

Explanation:
Common shares outstanding, 1/1 100,000
Adjustment for 2-for-1 stock split, 3/15 × 2
Common shares outstanding, declaration date 200,000
Times: Cash dividend per common share × $0.50
Cash dividends declared during year $100,000

498
Q
East Co. issued 1,000 shares of its $5 par common stock to Howe as compensation for 1,000 hours of legal services performed. Howe usually bills $160 per hour for legal services. On the date of issuance, the stock was trading on a public exchange at $140 per share. By what amount should the additional paid-in capital account increase as a result of this transaction?
A
$135,000
B
$140,000
C
$155,000
D
$160,000
A

a

Explanation:
The acquisition of services by issuance of common stock is a nonmonetary exchange that should be recorded at the fair value of the stock issued or the services performed, whichever is more clearly evident. The fair value of the stock issued should be used to record this transaction because (1) on the date of issuance, the stock was trading on a public exchange and thus had the more clearly evident fair value and (2) the fair value of the stock issued is less than the amount usually billed for the services received. Thus, the increase in Additional Paid-in Capital as a result of the transaction is the excess of the fair value over the par value of the common stock multiplied by the number of common shares issued [i.e., $140 - $5 = $135 × 1,000 = $135,000].

499
Q
During the current year, Young and Zinc maintained average capital balances in their partnership of $160,000 and $100,000, respectively. The partners receive 10% interest on average capital balances, and residual profit or loss is divided equally. Partnership profit before interest was $4,000. By what amount should Zinc's capital account change for the year?
A
$ 1,000 decrease.
B
$ 2,000 increase.
C
$11,000 decrease.
D
$12,000 increase.
A

a

Explanation:
Young received $16,000 ($160,000 × 10%) and Zinc received $10,000 ($100,000 × 10%) for the interest on average capital balances. This makes the residual loss $22,000 ($4,000 - $16,000 - $10,000). As the residual loss is evenly divided between the two partners, Zinc gets $11,000 of loss. Zinc’s interest allowance less Zinc’s portion of the residual loss is the change in Zinc’s capital balance. $10,000 - $11,000 = $1,000 decrease.

500
Q

The per-share amount must be reported on the face of a public company’s income statement for which of the following items?

A
Income from continuing operations.
B
Preferred stock dividend.
C
U.S. Treasury Stock.
D
Compensation effect of fair value on stock options.
A

Explanation:
The correct answer is (A).

The earnings per share must be reported on the face of a public company’s income statement for the following items:

Income from continuing operations.
Income from discontinued operations.
Net income.
Earnings per share is the amount of current-period earnings that can be associated with a single share of a corporation’s common stock. All corporations must report per-share amounts for income from continuing operations and net income on the face of the income statement.

(B), (C) and (D) are incorrect as per the above explanation.

501
Q

On April 1 of the current year, Hyde Corp., a newly formed company, had the following stock issued and outstanding:
Common stock, no par, $1 stated value, 20,000 shares originally issued for $30 per share.
Preferred stock, $10 par value, 6,000 shares originally issued for $50 per share.
Hyde’s April 1 statement of stockholders’ equity should report
Common stock Preferred stock Additional paid-in capital
A $ 20,000 $ 60,000 $820,000
B $ 20,000 $300,000 $580,000
C $600,000 $300,000 $0
D $600,000 $ 60,000 $240,000

A

a

Explanation:
APIC = $580,000 + $240,000 = $820,000. If Hyde has only one APIC account, the issuance of the common and preferred stock would be recorded as follows.

Cash (20,000 × $30) 600,000
Common Stock (20,000 × $1) 20,000
Additional Paid-In Capital (to balance) 580,000
Cash (6,000 × $50) 300,000
Preferred Stock (6,000 × $10) 60,000
Additional Paid-In Capital (to balance) 240,000

502
Q

In the Adel-Brick partnership, Adel and Brick had a capital ratio of 3:1 and a profit and loss ratio of 2:1, respectively. The bonus method was used to record Colter’s admittance as a new partner. What ratio would be used to allocate, to Adel and Brick, the excess of Colter’s contribution over the amount credited to Colter’s capital account?
A
Adel and Brick’s new relative capital ratio.
B
Adel and Brick’s new relative profit and loss ratio.
C
Adel and Brick’s old capital ratio.
D
Adel and Brick’s old profit and loss ratio.

A

d

Explanation:
Under the bonus method, the excess of the partner’s contribution over the amount credited to the new partner’s capital account is viewed as a bonus to the original partners. The bonus is allocated to the original partners based upon their old profit and loss ratio.

503
Q

Aldrich Co. distributes cash dividends to its shareholders during the current year. The dividends are declared on March 9 and are payable to shareholders as of the date of record, which is April 15. The dividends are actually paid on May 19. At which of the following dates would the dividends become a liability to Aldrich?

A
March 9.
B
April 15.
C
May 19.
D
December 31.
A

Explanation:
The correct answer is (A)

Dividends become a liability at the date of declaration. On this date, the board of directors commit to the dividend. On March 9, the liability will be recorded.

Retained earnings XXX
Dividends payable XXX
(B) is incorrect because it is the date of record, on this date shareholder are identified as the ones entitled to the dividends. No J/E is passed.

(C) is incorrect because on the date of payment the dividend declared is distributed to the shareholders. Below J/E is passed:

Dividends Payable XXX
Cash XXX

504
Q
On June 30 of the current year the condensed balance sheet for the partnership of Eddy, Fox, and Grimm, together with their respective profit and loss sharing percentages, was as follows:
Assets, net of liabilities	$320,000
Eddy, capital (50%)	160,000
Fox, capital (30%)	96,000
Grimm, capital (20%)	    64,000
Total Equity	$320,000Eddy decided to retire from the partnership and by mutual agreement is to be paid $180,000 out of partnership funds for his interest. Total goodwill implicit in the agreement is to be recorded. After Eddy's retirement, what are the capital balances of the other partners?
Fox	Grimm
A	$ 84,000	$ 56,000
B	$ 102,000	$ 68,000
C	$ 108,000	$ 72,000
D	$ 120,000	$ 80,000
A

c

Explanation:
The goodwill method looks upon this transaction as an indication that the partnership possesses an actual value of $360,000 ($180,000 / 50%) prior to Eddy’s retirement. The partnership is reporting $320,000 in net assets, so a $40,000 valuation adjustment is required. This adjustment is recorded as goodwill and is allocated to Eddy, Fox, and Grimm by their respective profit and loss sharing percentages. Immediately after Eddy’s retirement, Fox’s capital account balance should be $108,000 ($96,000 + $12,000) and Grimm’s capital account balance should be $72,000 ($64,000 + $8,000).

Goodwill 40,000
Eddy, Capital ($40,000 × 50%) 20,000
Fox, Capital ($40,000 × 30%) 12,000
Grimm, Capital ($40,000 × 20%) 8,000
Eddy, Capital ($160,000 + $20,000) 180,000
Cash 180,000To record the payment of $180,000 to Eddy.

505
Q

If a corporation sells some of its treasury stock at a price that exceeds its cost, this excess should be
A
Reported as a gain in the income statement.
B
Treated as a reduction in the carrying amount of remaining treasury stock.
C
Credited to additional paid-in capital.
D
Credited to retained earnings

A

c

Explanation:
The answer to this question assumes that the cost method is used to account for the treasury stock. Under the cost method, if treasury stock is sold at a price that exceeds its cost, the excess should be credited to Additional Paid-in Capital . The excess is not reported as a gain in the income statement or as a credit to Retained Earnings under either the cost or par methods. Retained earnings may be decreased, but never increased, by treasury stock transactions.

506
Q

The partnership agreement of Axel, Berg & Cobb provides for the year-end allocation of net income in the following order:
First, Axel is to receive 10% of net income up to $100,000 and 20% over $100,000.
Second, Berg and Cobb each are to receive 5% of the remaining income over $150,000.
The balance of income is to be allocated equally among the three partners.
The partnership’s net income was $250,000 before any allocations to partners. What amount should be allocated to Axel?
A
$101,000
B
$103,000
C
$108,000
D
$110,000

A

c

Explanation:
The portion of partnership profit allocated to Axel is bonus and one-third of residual partnership profit ($204,000 × 1/3 = $68,000). ($40,000 + $68,000 = $108,000).

Profit prior to distribution $ 250,000
Deduct bonus to Axel [($100,000 × 10%) + ($250,000 - 100,000) × 20%] (40,000)
Less: Bonuses to Berg and Cobb [$250,000 - ($150,000 + $40,000)] × 5% × 2] (6,000)
Residual partnership profit $ 204,000

507
Q
Park Corp.'s stockholders' equity accounts at December 31 of the previous year were as follows:
Common stock, $20 par	$8,000,000
Additional paid-in capital	$2,550,000
Retained earnings	$1,275,000All shares of common stock outstanding at December 31 of the previous year were issued originally for $26 a share. On January 4 of the current year, Park reacquired 20,000 shares of its common stock at $24 a share and retired them. Immediately after the shares were retired, the balance in additional paid-in capital would be
A
$2,430,000
B
$2,470,000
C
$2,510,000
D
$2,590,000
A

b

Explanation:
In a treasury stock retirement, the Common Stock account decreases for the par value and the Additional Paid-In Capital account decreases up to the original increase to the APIC account when the retired shares were first issued. Immediately after the shares were retired, the APIC balance would be $2,470,000 ($2,550,000 - $120,000 + $40,000).

Common Stock (20,000 × $20 PV) 400,000
APIC - CS [20,000 × ($26- $20 PV)] 120,000
Cash (20,000 × $24) 480,000
APIC - Retirement of CS (to balance) 40,000To record retirement of common stock.

508
Q

On April 1 of the current year, Fay Corporation established an employee stock ownership plan (ESOP). Selected transactions relating to the ESOP during the year were as follows:
On April 1, Fay contributed $30,000 cash and 3,000 shares of its $10 par common stock to the ESOP. On this date, the market price of the stock was $18 a share.
On October 1, the ESOP borrowed $100,000 from Union National Bank and acquired 5,000 shares of Fay’s common stock in the open market at $17 a share. The note is for one year, bears interest at 10%, and is guaranteed by Fay.
On December 15, the ESOP distributed 6,000 shares of Fay common stock to employees of Fay in accordance with the plan formula.
In its year-end income statement, how much should Fay report as compensation expense relating to the ESOP?
A
$184,000
B
$120,000
C
$84,000
D
$60,000

A

Explanation:
Cash contributed $30,000
Common stock contributed (3,000 shares × $18 fair value) 54,000
ESOP compensation expense $84,000

509
Q
On February 1, Tory began a service proprietorship with an initial cash investment of $2,000. The proprietorship provided $5,000 of services in February and received full payment in March. The proprietorship incurred expenses of $3,000 in February, which were paid in April. During March, Tory drew $1,000 against the capital account. In the proprietorship's financial statements for the two months ended March 31, prepared under the cash basis method of accounting, what amount should be reported as capital?
A
$1,000
B
$3,000
C
$6,000
D
$7,000
A

c

Explanation:
The balance in the capital account at March 31, is the beginning balance plus the service revenues received less the owner’s draw. The expenses incurred in February and paid in April are not included in the calculation, as Tory has a cash basis of accounting. $2,000 + $5,000 - $1,000 = $6,000.

510
Q
When property other than cash is invested in a partnership, at what amount should the noncash property be credited to the contributing partner's capital account?
A
Fair value at the date of contribution.
B
Contributing partner's original cost.
C
Assessed valuation for property tax purposes.
D
Contributing partner's tax basis.
A

a

Explanation:
All identifiable assets (e.g., cash, inventory, land, patents, etc.) contributed to a partnership are recorded by the partnership at their fair values at the date of contribution.

511
Q

A retained earnings appropriation can be used to
A
Absorb a fire loss when a company is self-insured.
B
Provide for a contingent loss that is probable and reasonable.
C
Smooth periodic income.
D
Restrict earnings available for dividends.

A

d

Explanation:
The purpose of a retained earnings appropriation is to restrict a portion of retained earnings as to availability for dividends. A retained earnings appropriation is not used to absorb a fire loss when a company is self-insured, provide for a contingent loss that is probable and reasonably estimable, or to smooth periodic income.

512
Q

Jensen performed legal services to assist Balm Co. in accomplishing its initial organization. Jensen accepted 1,000 shares of $5 par common stock in Balm as payment for his services. The Balm shares were not yet publicly traded, but they had a book value of $4 per share. Jensen provided 48 hours of service, which is normally billed at $125 per hour. By what amount should the common stock account increase?

A
$1,000
B
$4,000
C
$5,000
D
$6,000
A

Explanation:
The correct answer is (C).

The compensatory stock option expense is reported by the company based on the estimated FV of the options when granted. On the execution date, the fair value of the services is $6,000 ($125 x 48 hours). Since Jensen received 1,000 shares of $5 par value Common Stock, $5,000 will be recorded as an increase to common stock with the excess of $1 per share or $1,000 to additional paid-in capital.

(A) is incorrect because it is the amount credited to APIC, not to C/S.

(B) is incorrect because it is the BV of the share, while the C/S is credited with the par value of the shares

(D) is incorrect because it is the total cost of the legal services provided by Jensen.

513
Q
Avers and Smith formed a partnership. Avers contributed cash of $50,000. Smith contributed property with a $36,000 carrying amount, a $40,000 original cost, and a fair value of $80,000. The partnership assumed the $35,000 mortgage attached to the property. What should Smith's capital account be on the partnership formation date?
A
$36,000
B
$40,000
C
$45,000
D
$80,000
A

c

Explanation:
All identifiable assets contributed to a partnership are recorded by the partnership at their fair values. All liabilities that the partnership assumes are recorded at their present values. The contributing partner’s capital account is credited for the fair value of the property less the assumed mortgage. Smith’s capital account would be the fair value of the property of $80,000 less the mortgage assumed by the partnership of $35,000, for a balance of $45,000.

514
Q
Rice Co. was incorporated on January 1 of the current year with $500,000 from the issuance of stock and borrowed funds of $75,000. During this first year of operations, net income was $25,000. On December 15, Rice paid a $2,000 cash dividend. No additional activities affected owners' equity in the year. At December 31, Rice's liabilities had increased to $94,000. In Rice's December 31 balance sheet, total assets should be reported at
A
$598,000
B
$600,000
C
$617,000
D
$692,000
A

c

Explanation:
Total assets equal the sum of liabilities and stockholders’ equity.

Total liabilities, 12/31 (given) $ 94,000
Proceeds from issuance of stock $500,000
Net income for the year 25,000
Cash dividend declared (2,000)
Total stockholders’ equity, 12/31 523,000
Total liabilities and stockholders’ equity (and thus total assets) at 12/31 $617,000

515
Q
In September of year 2, West Corp. made a dividend distribution of one right for each of its 120,000 shares of outstanding common stock. Each right was exercisable for the purchase of 1/100 of a share of West's $50 variable rate preferred stock at an exercise price of $80 per share. On March 20 of year 6, none of the rights had been exercised, and West redeemed them by paying each stockholder $0.10 per right. As a result of this redemption, West's stockholders' equity was reduced by
A
$ 120
B
$ 2,400
C
$12,000
D
$36,000
A

c

Explanation:
No entry (other than a memorandum entry) is made when stock rights are issued to existing stockholders. However, the redemption of the rights resulted in an outflow of cash. As this did not increase a noncash asset or reduce liabilities, it must affect an equity account. Stockholders' equity was reduced by 120,000 shares × $0.10 per share = $12,000.
516
Q
On January 15 of the current year, Rico Co. declared its annual cash dividend on common stock for the year ended January 31. The dividend was paid on February 9 to stockholders of record as of January 28 of this year. On what date should Rico decrease retained earnings by the amount of the dividend?
A
January 15
B
January 31
C
January 28
D
February 9
A

a

Explanation:
The date of declaration date is the date on which dividends are declared by the board of directors. Declared cash dividends are a liability. The journal entry required at the date of declaration includes a debit (decrease) to Retained Earnings . No journal entry is required at the date of record. The journal entry on the payment date reduces Cash and the liability, but has no effect on Retained Earnings.

517
Q
Universe Co. issued 500,000 shares of common stock in the current year. Universe declared a 30% stock dividend. The market value was $50 per share, the par value was $10, and the average issue price was $30 per share. By what amount will Universe decrease stockholders' equity for the dividend?
A
$0
B
$1,500,000
C
$4,500,000
D
$7,500,000
A

a

Explanation:
Stock dividends operate to transfer a part of the retained earnings to contributed capital (capitalization of retained earnings). In recording the stock dividend, a charge is made to retained earnings (thereby making a portion of retained earnings no longer available for distribution) and credits are made to paid-in capital accounts. Because the declaration and issuance of a stock dividend decreases retained earnings and increases paid-in capital by equal amounts, total stockholders’ equity is not affected.

518
Q
A company issued rights to its existing shareholders without consideration. The rights allowed the recipients to purchase unissued common stock for an amount in excess of par value. When the rights are issued, which of the following accounts will be increased?
Common stock	Additional paid-in capital
A	Yes	Yes
B	Yes	No
C	No	No
D	No	Yes
A

c

Explanation:
No entry (other than a memorandum entry) is made when stock rights are issued to existing stockholders without consideration.
519
Q
An accumulated balance of other comprehensive income (OCI) is a component of which of the following?
A
Corporation's equity
B
Partnership's equity
C
Sole proprietorship's equity
D
All of the above
A

a

Explanation:
A corporation’s equity consists of three main components: contributed capital, which includes capital stock and additional paid-in capital; retained earnings; and the accumulated balance of OCI

520
Q
Stock dividends on common stock should be recorded at their fair market value by the investor when the related investment is accounted for under which of the following methods?
Cost	Equity
A	Yes	Yes
B	Yes	No
C	No	Yes
D	No	No
A

d

Explanation:
Stock dividends received on common stock may be recorded only by memorandum entry, regardless of whether the investment is accounted for by the cost or equity method. Under the cost method, the cost of the common stock investment would be divided by the number of common shares owned after the stock dividend to compute a new cost basis per share. Under the equity method, the carrying amount of the investment would be divided by the increased number of common shares owned as a result of the stock dividend to compute a new carrying amount per share. These amounts would be used to determine any gain or loss on a subsequent disposal of the shares.

521
Q
Under the goodwill method, when a new partner's asset contribution is \_\_\_\_\_\_\_\_\_ than the ownership interest s/he is to receive, the excess assets are accounted for as goodwill attributable to the \_\_\_\_\_\_\_\_\_.
A
Greater; new partner
B
Greater; old partners
C
Less; old partners
D
Less; new and old partners equally
A

b

Explanation:
When a new partner’s asset contribution is greater than the ownership interest s/he is to receive, the excess assets are accounted for as goodwill attributable to the old partners.

522
Q

Murphy Co. had 200,000 shares outstanding of $10 par common stock on March 30 of the current year. Murphy reacquired 30,000 of those shares at a cost of $15 per share, and recorded the transaction using the cost method on April 15. Murphy reissued the 30,000 shares at $20 per share, and recognized a $50,000 gain on its income statement on May 20. Which of the following statements is correct?
A
Murphy’s comprehensive income for the current year is correctly stated.
B
Murphy’s net income for the current year is overstated.
C
Murphy’s net income for the current year is understated.
D
Murphy should have recognized a $50,000 loss on its income statement for the current year.

A

b

Explanation:
The net income for the current year is overstated, because no gains or losses are recognized on treasury stock transactions.

523
Q

A corporation declared a dividend, a portion of which was liquidating. How would this declaration affect each of the following:
Additional paid-in capital Retained earnings
A Decrease No effect
B Decrease Decrease
C No effect Decrease
D No effect No effect

A

b

Explanation:
Any dividend not based on earnings must be a reduction of additional paid-in capital and, to that extent, is a liquidating dividend.

Retained earnings XX
Additional paid-in capital XX
Cash dividends payable XXTo record a partially liquidating cash dividend..

524
Q
Ole Corp. declared and paid a liquidating dividend of $100,000. This distribution resulted in a decrease in Ole's
Paid-in capital	Retained earnings
A	No	No
B	Yes	Yes
C	No	Yes
D	Yes	No
A

d

Explanation:
Any dividend not based on earnings must be a reduction of corporate paid-in capital and, to that extent, is a liquidating dividend. Since no portion of the dividend is based on accumulated past earnings, Paid-in Capital decreases by the full dividend amount, while Retained Earnings is unaffected. The following journal entry illustrates the effects of the declaration of a $100,000 liquidating cash dividend.

Additional paid-in capital 100,000
Cash dividends payable 100,000

525
Q

Quoit, Inc. issued preferred stock with detachable common stock warrants. The issue price exceeded the sum of the warrants’ fair value and the preferred stocks’ par value. The preferred stocks’ fair value was not determinable. What amount should be assigned to the warrants outstanding?
A
Total proceeds.
B
Excess of proceeds over the par value of the preferred stock.
C
The proportion of the proceeds that the warrants’ fair value bears to the preferred stocks’ par value.
D
The fair value of the warrants.

A

d

Explanation:
The proceeds should be allocated between the preferred stock and the detachable stock purchase warrants based on their relative fair market values at date of issue. If the relative fair values are not known, then the fair value of either security is used. In this question, the fair value of the warrants is known, but the fair value of the preferred stock without the warrants is not. Therefore, the amount assigned to the warrants is the fair value of the warrants. The remaining amount of the proceeds are assigned to the preferred stock.

526
Q
On July 1 of the current year, Cove Corp., a closely held corporation, issued 6% bonds with a maturity value of $60,000, together with 1,000 shares of its $5 par value common stock, for a combined cash amount of $110,000. The market value of Cover's stock cannot be ascertained. If the bonds were issued separately, they would have sold for $40,000 on an 8% yield to maturity basis. What amount should Cove report for additional paid-in capital on the issuance of the stock?
A
$75,000
B
$65,000
C
$55,000
D
$45,000
A

b

Explanation:
Since the market value of both securities is not determinable, the incremental method should be used to allocate the cash proceeds between the two securities. Since the market value of the bonds is known, the cash proceeds in excess of this amount are allocated to the common stock. The excess of the proceeds allocated to the common stock over the par value of the common stock is recorded as Additional Paid-in Capital.

Lump sum cash proceeds $110,000
Less: Market value of bonds (40,000)
Cash proceeds allocated to common stock $ 70,000
Less: Par value of common stock (1,000 × $5) (5,000)
Additional paid-in capital on issuance of common stock $ 65,000

527
Q

Zinc Co.’s adjusted trial balance at December 31 includes the following account balances:

Common stock, $3 par $600,000
Additional paid-in capital 800,000
Treasury stock, at cost 50,000
Net unrealized loss on available-for-sale marketable debt securities 20,000
Retained earnings: Appropriated for uninsured earthquake losses 150,000
Retained earnings: Unappropriated 200,000
What amount should Zinc report as total stockholders’ equity in its December 31 balance sheet?

A
$1,680,000
B
$1,720,000
C
$1,780,000
D
$1,820,000
A

a

Explanation:
Common stock, $3 par $600,000
Additional paid-in capital 800,000
Total paid-in capital $1,400,000
Retained earnings, appropriated 150,000
Retained earnings, unappropriated 200,000
Total retained earnings 350,000
Less: Treasury stock, at cost (50,000)
Less: Net unrealized loss on AFS debt securities (20,000)
Total stockholders’ equity $1,680,000

528
Q
Preferred stock with a \_\_\_\_\_\_\_\_\_\_\_\_\_\_\_ feature allows preferred shareholders to exchange shares, at their option, for common stock.
A
Redeemable
B
Convertible
C
Callable
D
None of the above
A

b

Explanation:
A redeemable feature means that preferred shareholders may redeem shares, at their option, at a specified price per share. A convertible feature means that preferred shareholders may exchange shares, at their option, for common stock.

529
Q
Regarding stock rights, which of the following situations requires a journal entry instead of just a memorandum entry?
A
Stock rights are issued to existing stockholders.
B
Stock rights are exercised.
C
Stock rights are expired.
D
All of the above
A

b

Explanation:
An entry is required only when stock rights are exercised. Cash is debited for the number of common shares acquired times the exercise price of the shares. Common stock is credited for the par or stated value of the shares issued. Additional paid-in capital is credited for the excess of the cash received over the par or stated value of the shares.

530
Q

When preparing a draft of its year-end balance sheet, Mont, Inc., reported net assets totaling $875,000. Included in the asset section of the balance sheet were the following:

Treasury stock of Mont, Inc., at cost, which approximates market value on December 31 $24,000
Idle machinery 11,200
Cash surrender value of life insurance on corporate executives 13,700
Allowance for decline in market value of available-for-sale debt investments 8,400
At what amount should Mont’s net assets be reported in the December 31 year-end balance sheet?

A
$851,000
B
$850,100
C
$842,600
D
$834,500
A

a

Explanation:
The idle machinery, the cash surrender value of the life insurance on the corporate executives, and the contra account, Allowance for Decline in Market Value of Available-for-Sale Debt Securities, are all properly included in assets. The treasury stock is not an asset and should not be included in the computation.

Net assets, before adjustments $875,000
Less cost of treasury stock (24,000)
Net assets, 12/31 $851,000

531
Q
Eagle and Falk are partners with capital balances of $45,000 and $25,000, respectively. They agree to admit Robb as a partner. After the assets of the partnership are revalued, Robb will have a 25% interest in capital and profits, for an investment of $30,000. What amount should be recorded as goodwill to the original partners?
A
$0
B
$ 5,000
C
$ 7,500
D
$20,000
A

d

Explanation:
In the goodwill method of recording the admission of a new partner, the assets are revalued at their fair values and any excess valuation implied in the purchase price is recorded as goodwill. Robb’s 25% interest for an investment of $30,000 implies that total net assets are valued at $120,000 ($30,000/25%).

Net assets	$120,000
Less:	Robb investment	(30,000)
Eagle capital	(45,000)
Falk capital	  (25,000)
Goodwill	$20,000
532
Q
Two years ago Fogg, Inc. issued $10 par value common stock for $25 per share. No other common stock transactions occurred until March 31 of the current year when Fogg acquired some of the issued shares for $20 per share and retired them. Which of the following statements correctly states an effect of this acquisition and retirement?
A
Current year net income is decreased.
B
Current year net income is increased.
C
Additional paid-in capital is decreased.
D
Retained earnings is increased.
A

c

Explanation:
The effect of the acquisition and retirement of each common share is to decrease APIC by $10 (i.e., $15 - $5). A corporation cannot record a gain or loss on the acquisition and retirement of its own common stock. Retained earnings can be decreased, but never increased, as a result of the acquisition and retirement of its own common stock.

Common stock 10
APIC–common stock 15
Cash 20
APIC–retirement of stock 5To record the acquisition and retirement of one share of Fogg’s own common stock.

533
Q

Which of the following financial instruments issued by a public company should be reported on the issuer’s books as a liability on the date of issuance?
A
Cumulative preferred stock.
B
Preferred stock that is convertible to common stock five years from the issue date.
C
Common stock that contains an unconditional redemption feature.
D
Common stock that is issued at a 5% discount as part of an employee share purchase plan.

A

c

Explanation:
Financial instruments can be issued with an unconditional obligation on the part of the issuer to redeem the instruments by transferring assets to the purchaser at a specified date or upon an event that is certain to occur (e.g., the death or termination of the individual who holds it). These are referred to as mandatorily redeemable financial instruments, and are classified as liabilities because of the embodied obligation on the part of the issuer to transfer its assets.

534
Q
On December 1 of the current year, Line Corp. received a donation of 2,000 shares of its $5 par value common stock from a stockholder. On that date, the stock's market value was $35 per share. The stock was originally issued for $25 per share. By what amount would this donation cause total stockholders' equity to decrease?
A
$70,000
B
$50,000
C
$20,000
D
$0
A

d

Explanation:
The question does not indicate whether the donated shares are accounted for as treasury stock under the cost or par value methods, or whether the donated shares were canceled and retired. Therefore, this question cannot be answered by constructing a journal entry to record the donation of the shares. Instead, the question can be answered by knowing that the donation of the common stock would be accounted for entirely within stockholders’ equity accounts. Therefore, the amount reported for total stockholders’ equity would be unchanged as a result of the donation of the stock.

535
Q
The condensed balance sheet of Adams & Gray, a partnership, at December 31, year 1, follows:
Current assets	$250,000
Equipment (net)	    30,000
Total assets	$280,000
Liabilities	20,000
Adams, capital	160,000
Gray, capital	  100,000
Total liabilities and capital	$280,000On December 31, year 1, the fair values of the assets and liabilities were appraised at $240,000 and $20,000, respectively, by an independent appraiser. On January 2, year 2, the partnership was incorporated and 1,000 shares of $5 par value common stock were issued. Immediately after the incorporation, what amount should the new corporation report as additional paid-in capital?
A
$275,000
B
$260,000
C
$215,000
D
$0
A

Explanation:
Additional Paid-in Capital is credited for the difference between the fair value of the net assets contributed and the stock’s par value.

Assets (at fair value) 240,000
Liabilities (at fair value) 20,000
Common stock (1,000 × $5) 5,000
Additional paid-in capital (to balance) 215,000

536
Q

Knowing that a subscription is a contract to purchase one or more shares of stock in the future, and that a corporation does not issue shares until the full subscription price is paid, how may a corporation use the amounts already paid by a subscriber who defaults on the contract?
A
Return the amount in full to the defaulted subscriber
B
Retain the defaulted subscriber’s funds and issue an equivalent number of shares
C
Retain the defaulted subscriber’s funds to cover any losses on resale, and then return the balance, if any, to the defaulted subscriber
D
All of the above

A

d

Explanation:
If a subscriber defaults on a subscription contract, amounts paid to the corporation may be: returned in full; retained by the corporation and an equivalent number of shares issued; or retained to cover any losses on resale, and the balance, if any, returned to the defaulted subscriber. Since the answers for A., B., and C. are all correct, the best answer for this question is D., all of the above.

537
Q

Asp Co. was organized on January 2 with 30,000 authorized shares of $10 par common stock. During the year, the corporation had the following capital transactions:
January 5 Issued 20,000 shares at $15 per share
July 14 Purchase 5,000 shares at $17 per share
December 27 Reissued the 5,000 shares held in treasury at $20 per shareAsp used the par value method to record the purchase and reissuance of the treasury shares. In its December 31 Balance sheet, what amount should Asp report as additional paid-in capital in excess of par?
A
$100,000
B
$125,000
C
$140,000
D
$150,000

A

b

Explanation:
Under the par value method, the recording of the acquisition of treasury stock effectively removes the treasury stock from the accounts. The excess in acquisition price over the original issuance price ($17 - $15 = $2/share) is debited to Additional Paid-In-Capital From Treasury Stock , but only to the extent of any existing balance from prior treasury stock transactions. The difference, if any, is debited to Retained Earnings .

Additional Paid-In Capital:
20,000 shares @ $5/share over par, 1/ 5 $100,000
5,000 shares @ $5/share Treasury Stock, 7/14 (25,000)
5,000 shares @ ($20 - $10 par) = $10/per share, 12/27 50,000
APIC balance 12/31 $125,000

538
Q

How would a stock split affect each of the following?
Assets Total stockholders’ equity Additional paid-in capital
A Increase Increase No effect
B No effect No effect No effect
C No effect No effect Increase
D Decrease Decrease Decrease

A

b

Explanation:
A stock split consists of a reduction in the par value per share, together with a proportional increase in the number of shares outstanding. For instance, in a 2-for-1 split, the par value per share is halved, while the number of shares outstanding is doubled. Total par value, additional paid-in capital, stockholders’ equity, and total assets remain unchanged.

539
Q

Which of the following statements regarding legal capital is false?
A
The portion of contributed capital required by statute to be retained in the business for the protection of creditors is called legal capital.
B
Legal capital may not be used as a basis for dividends.
C
Acquisition of treasury stock is limited to the amount of legal capital.
D
The amount of legal capital cannot be reduced arbitrarily by the corporation.

A

c

Explanation:
Acquisition of treasury stock is limited to the amount of retained earnings, not legal capital.

540
Q

Each of the following transactions will cause a decrease in stockholders’ equity, except

A
The sale of treasury stock at less than cost
B
The declaration of a cash dividend
C
A loss on the sale of a discontinued segment
D
A loss from a foreign currency translation adjustment

A

Explanation:
The correct answer is (A).

Each of the following transactions will cause a decrease in stockholders’ equity:

The declaration of a cash dividend. Dividends will reduce retained earnings in equity.
A loss on the sale of a discontinued segment.
A loss from a foreign currency translation adjustment.
The sale of treasury stock at less than cost will not reduce equity rather increase it. This calculation involves the cost method. The sale of treasury stock at less than cost will result in debits to Cash and Additional Paid-In Capital-Treasury Stock (APIC-TS) and a credit to Treasury Stock (at cost). While APIC-TS is a stockholder equity account that will decrease, the increase in the Treasury Stock account will exceed such decrease that the net effect is still an increase to stockholder’s equity. Example: If treasury stock is sold for $100,000, which is $20,000 below cost, the net effect is an $80,000 increase in stockholders’ equity.

541
Q
A company whose stock is trading at $10 per share has 1,000 shares of $1 par common stock outstanding when the board of directors declares a 30% common stock dividend. Which of the following adjustments should be made when recording the stock dividend?
A
Treasury stock is debited for $300.
B
Additional paid-in capital is credited for $2,700.
C
Retained earnings is debited for $300.
D
Common stock is debited for $3,000.
A

c

Explanation:
In recording a stock dividend, a charge is made to retained earnings (thereby making a portion of retained earnings no longer available for distribution) and credits are made to paid-in capital accounts; total stockholders’ equity is not affected. The amount of retained earnings capitalized depends on the size of the stock dividend and the effect the dividend has on the market value of the shares. When the stock dividend is large (greater than 25% of the outstanding shares) only the par or stated value of the additional shares is capitalized. In this case, debit Retained Earnings for $300; credit Common Stock for $300.

542
Q
Beck Corp. issued 200,000 shares of common stock when it began operations two years ago and issued an additional 100,000 shares in the past year. Beck also issued preferred stock convertible to 100,000 shares of common stock. In the current year Beck purchased 75,000 shares of its common stock and held it in Treasury. At December 31 of the current year, how many shares of Beck's common stock were outstanding?
A
400,000
B
325,000
C
300,000
D
225,000
A

d

Explanation:
The 225,000 common shares outstanding are computed by subtracting the 75,000 Treasury shares from the 300,000 (i.e., 200,000 + 100,000) common shares issued. The number of common shares outstanding is not affected by the convertible preferred stock; there is no indication that any of the preferred stock converted.

543
Q
The following stock dividends were declared and distributed by Sol Corp.:
Percentage of common shares
outstanding at declaration date	Fair value 	Par value
10	$15,000	$10,000
28	40,000	30,800What aggregate amounts should be debited to retained earnings for these stock dividends?
A
$40,800
B
$45,800
C
$50,000
D
$55,000
A

b

Explanation:
The issuance of a “small” stock dividend (i.e., less than 20 to 25% of the number of shares outstanding) should be recorded by capitalizing a portion of retained earnings equal to the fair value of the shares issued. On the other hand, the issuance of a “large” stock dividend (i.e., more than 20 to 25% of the number of shares outstanding) should be recorded by capitalizing a portion of retained earnings equal to the par value of the shares issued. Thus, Retained Earnings should be debited for (1) the fair value of the 10% stock dividend ($15,000) and (2) the par value of the 28% stock dividend ($30,800), for a total of $45,800.

544
Q

Capital stock is best described as which of the following?
A
The par or stated value of the stock purchased by owners
B
Paid-in capital in excess of par or stated value
C
Retained earnings
D
The stock always used to build infrastructure

A

a

Explanation:
Capital stock is the par or stated value of the stock purchased by owners.

545
Q
Allen retired from the partnership of Allen, Beck, and Chale. Allen's cash settlement from the partnership was based on new goodwill determined at the date of retirement plus the carrying amount of the other net assets. As a consequence of the settlement, the capital accounts of Beck and Chale were decreased. In accounting for Allen's withdrawal, the partnership could have used the
Bonus method	Goodwill method
A	No	Yes
B	No	No
C	Yes	Yes
D	Yes	No
A

d

Explanation:
Allen, the withdrawing partner, is to receive cash or other assets equal to his current capital balance plus his share of unrecorded goodwill. If the bonus method were to be used to account for this transaction, the payment made to Allen for his share of the unrecorded goodwill would be recorded as a decrease in the remaining partners’ capital accounts. If the goodwill method were to be used to account for the withdrawal, a revaluation of partnership assets would be required. This adjustment would be recorded on the partnership’s books as goodwill and allocated among the existing partners (i.e., Allen, Beck, and Chale) by their respective profit and loss sharing percentages, thereby increasing their respective capital accounts. Since the capital accounts of the remaining partners (i.e., Beck and Chale) decreased as a result of the settlement, the withdrawal could only have been recorded using the bonus method.

546
Q
The following changes in Vel Corp.'s account balances occurred during the current year:  
Increase
Assets	$89,000
Liabilities	27,000
Capital stock	60,000
Additional paid-in capital	6,000Except for a $13,000 dividend payment and the year's earnings, there were no changes in retained earnings for the year. What was Vel's net income for the year?
A
$ 4,000
B
$ 9,000
C
$13,000
D
$17,000
A

b

Explanation:
During the year, stockholders’ equity increased by $62,000 (i.e., $89,000 - $27,000). Since paid-in capital increased by $66,000 (i.e., $60,000 + $6,000), retained earnings must have decreased by $4,000 (i.e., $66,000 - $62,000). Since the only charge to retained earnings was for a $13,000 dividend payment, net income for the year must have been $9,000.

547
Q
The Low and Rhu partnership agreement provides special compensation to Low for managing the business. Low receives a bonus of 15 percent of partnership net income before salary and bonus, and also receives a salary of $45,000. Any remaining profit or loss is to be allocated equally. During the current year, the partnership had net income of $50,000 before the bonus and salary allowance. As a result of these distributions, Rhu's equity in the partnership would
A
Increase.
B
Not change.
C
Decrease the same as Low's.
D
Decrease.
A

d

Explanation:
Low’s bonus results in a residual loss for the partnership for the year. Low’s equity would still increase due to the bonus Low received.

Partnership profit prior to distributions $ 50,000
Less: Bonus to Low (15% × $50,000) (7,500)
Salary to Low (45,000)
Residual partnership loss $ (2,500)
Times: Rhu’s loss percentage × 50%
Decrease in Rhu’s equity in partnership $ (1,250)

548
Q
Red and White formed a partnership in the previous year. The partnership agreement provides for annual salary allowances of $55,000 for Red and $45,000 for White. The partners share profits equally and losses in a 60/40 ratio. The partnership had earnings of $80,000 for the current year before any allowance to partners. What amount of these earnings should be credited to each partner's capital account?
Red	White
A	$40,000	$40,000
B	$43,000	$37,000
C	$44,000	$36,000
D	$45,000	$35,000
A

b

Explanation:
Each partner should be allocated their full salary allowance. Any gain or loss subsequent to this allocation should be allocated to each partner based on their profit and loss ratios. The division of partnership profits and losses must be based on the method specified in the partnership agreement. Therefore, the profit or loss allocated to each partner should be calculated after deducting the salary allowances due to each partner. $20,000 × 60% = $12,000

Partnership Profit Red White
Partnership profit prior to salary allowances $80,000
Deduct salary allowances (100,000) $55,000 $45,000
Residual partnership loss $(20,000)
60% of loss allocated to Red (12,000)
40% of loss allocated to White (8,000)
Credit to each partner’s capital account $43,000 $37,000

549
Q

The stockholders’ equity section of Brown Co.’s December 31, year 1, balance sheet consisted of the following:
Common stock, $30 par, 10,000 shares authorized and outstanding $ 300,000
Additional paid-in capital 150,000
Retained earnings (deficit) (210,000)On January 2, year 2, Brown put into effect a stockholder-approved quasi-reorganization by reducing the par value of the stock to $5 and eliminating the deficit against additional paid-in capital. Immediately after the quasi-reorganization, what amount should Brown report as additional paid-in capital?
A
$(60,000)
B
$150,000
C
$190,000
D

A

c

Explanation:
The balance in the additional paid-in capital account after the quasi-reorganization is the beginning balance, plus the difference in the old and new par values times the number of issued shares [($30 - $5) × 10,000 = $250,000], less the retained earnings deficit. $150,000 + $250,000 - $210,000 = $190,000.

550
Q
Kern and Pate are partners with capital balances of $60,000 and $20,000, respectively. Profits and losses are divided in the ratio of 60:40. Kern and Pate decided to form a new partnership with Grant, who invested land valued at $15,000 for a 20% capital interest in the new partnership. Grant's cost of the land was $12,000. The partnership elected to use the bonus method to record the admission of Grant into the partnership. Grant's capital account should be credited for
A
$12,000
B
$15,000
C
$16,000
D
$19,000
A

d

Explanation:
Under the bonus method, the total capital of the new partnership will equal the sum of the original partners’ capital balances plus the fair value of the identifiable asset (i.e., land) contributed by the new partner. Immediately after admission, the balance of the new partner’s capital account equals the total capital of the new partnership multiplied by the new partner’s capital interest.

Original partnership capital ($60,000 + $20,000) $80,000
Fair value of identifiable asset contributed by Grant 15,000
Total recorded capital of new partnership 95,000
Grant’s capital interest × 20%
Credit to Grant’s capital account $19,000
Journal entry:
Land (fair value) 15,000
Kern, capital ($4,000 x 60%) 2,400
Pate, capital ($4,000 x 40%) 1,600
Grant, capital (above) 19,000To record Grant’s admission into the partnership.

551
Q
On June 27 of the current year, Brite Co. distributed to its common stockholders 100,000 outstanding common shares of its investment in Quik, Inc., an unrelated party. The carrying amount on Brite's books of Quik's $1 par common stock was $2 per share. Immediately after the distribution, the market price of Quik's stock was $2.50 per share. In its income statement for the year ended June 30 what amount should Brite report as gain before income taxes on disposal of the stock?
A
$250,000
B
$200,000
C
$ 50,000
D
$0
A

c

Explanation:
Property dividends are recorded at the property’s fair value at the declaration date. The excess of the fair value over the carrying amount of the property distributed is recognized as a gain by the distributing corporation at the declaration date. Because there is no mention of a specific declaration date, we have to infer that the declaration date and distribution date are the same.

Fair value of distributed shares (100,000 × $2.50) $ 250,000
Less: Distributed shares’ carrying amount (100,000 × $2.00) (200,000)
Pretax gain on distributed shares $ 50,000

552
Q

Frey Inc. was organized on January 2 of the current year with the following capital structure:
10% cumulative preferred stock, par value $100 and
liquidation value $105; authorized, issued and
outstanding 1,000 shares $100,000
Common stock, par value $25; authorized 100,000
shares; issues and outstanding 10,000 shares 250,000Frey’s net income for the year ended December 31 was $450,000, but no dividends were declared. How much was Frey’s book value per preferred share at December 31?
A
$100
B
$105
C
$110
D
$115

A

d

Explanation:
The book value per preferred share is the portion of stockholders’ equity distributable to preferred stockholders in the event of liquidation (Liquidation value + Dividends in arrears), divided by the number of preferred shares outstanding. [($105 × 1,000 sh.) + (10% × $100,000)] / 1,000 shares = $115 share..

553
Q
At December 31 of year 1 and year 2, Carr Corp. had outstanding 4,000 shares of $100 par value 6% cumulative preferred stock and 20,000 shares of $10 par value common stock. At December 31, year 1, dividends in arrears on the preferred stock were $12,000. Cash dividends declared in year 2 totaled $44,000. Of the $44,000, what amounts were payable on each class of stock?
Preferred Stock	Common Stock
A	$44,000	$0
B	$36,000	$ 8,000
C	$32,000	$12,000
D	$24,000	$20,000
A

b

Explanation:
Cash dividends declared in year 2 $44,000
Dividends in arrears at 12/31, year 1 $12,000
Dividends for year 2 (4,000 × $100 × 6%) 24,000
Cash dividends payable to preferred stock: (36,000)
Cash dividends payable to common stock $ 8,000

554
Q

If 500 shares of $10 par value common stock are sold for $40 per share, the entry would be debit cash $20,000 and credit which of the following accounts for how much?
A
Common Stock $20,000
B
Common Stock $5,000; Additional Paid-in Capital—Common $15,000
C
Common Stock $15,000; Retain Earnings $5,000
D
Retained Earnings $5,000; Common Stock $15,000

A

b

Explanation:
Common Stock is credited $5,000 (500 shares × $10 per share), with the remaining $15,000 credited to Additional Paid-in Capital.

555
Q
Cor-Eng Partnership was formed on January 2 of the current year. Under the partnership agreement, each partner has an equal initial capital balance accounted for under the goodwill method. Partnership net income or loss is allocated 60% to Cor and 40% to Eng. To form the partnership, Cor originally contributed assets costing $30,000 with a fair value of $60,000 on January 2 while Eng contributed $20,000 in cash. Eng's initial capital balance in Cor-Eng is
A
$20,000
B
$25,000
C
$40,000
D
$60,000
A

d

Explanation:
Under the partnership agreement, each partner has an equal initial capital balance accounted for under the goodwill method. Cor is given an initial capital balance of $60,000, equal to the fair value of the identifiable assets Cor contributed to the partnership. Eng is also given an initial capital balance of $60,000. Since Eng only contributed identifiable assets with a fair value of $20,000, Eng must have also contributed an unidentifiable asset (i.e., goodwill) with a fair value of $40,000.

Cash	20,000	
Identifiable noncash assets	60,000	
Goodwill	40,000	
Cor, capital		60,000
Eng, capital		60,000
556
Q
Blau and Rubi are partners who share profits and losses in the ratio of 6:4, respectively. On May 1 of the current year their respective capital accounts were as follows:
Blau	$60,000
Rubi	$50,000On that date, Lind was admitted as a partner with a one-third interest in capital and profits for an investment of $40,000. The new partnership began with total capital of $150,000. Immediately after Lind's admission, Blau's capital should be
A
$50,000
B
$54,000
C
$56,667
D
$60,000
A

b

Explanation:
Because the total capital of the partners following the admission of Lind will be equal to the sum of the original partner’s capital balances plus the cash contributed by Lind (i.e., $60,000 + $50,000 + $40,000 = $150,000), no goodwill is recognized.

Blau’s capital balance, pre-admission $60,000
Credit to Lind for 1/3 of new partnership capital ($150,000 × 1/3) $50,000
Less: Cash contributed by Lind 40,000
Bonus to Lind from Blau and Rubi $10,000
Times Blau’s share of bonus [60% / (60% + 40%)] × 60%
Decrease in Blau’s capital account (6,000)
Blau’s capital balance, post admission $54,000

557
Q

On December 30 of the current year, Hale Corp. paid $400,000 cash and issued 80,000 shares of its $1 par value common stock to its unsecured creditors on a pro rata basis pursuant to a reorganization plan under Chapter 11 of the bankruptcy statutes. Hale owed these unsecured creditors a total of $1,200,000. Hale’s common stock was trading at $1.25 per share on December 30. As a result of this transaction, Hale’s total stockholder’s equity had a net increase of

A
$1,200,000
B
$ 800,000
C
$ 100,000
D
$ 80,000
A

b

Explanation:
A troubled debt restructuring has occurred because the unsecured creditors, for reasons related to Hale’s financial difficulties, granted concessions to Hale that they would not otherwise consider [i.e., the unsecured creditors accepted $400,000 cash and 80,000 shares of Hale’s common stock worth $100,000 (80,000 x $1.25 FMV) in satisfaction of Hale’s $1,200,000 debt]. Hale recognizes a $700,000 [$1,200,000 - ($400,000 + $100,000) gain on the restructuring equal to the excess of the carrying amount of the debt over the sum of the cash and the fair value of the equity interest transferred to the unsecured creditors. If income taxes are ignored, Hale’s retained earnings account increases by the full amount of the gain. In addition, Hale’s paid-in capital accounts increase by the $100,000 fair market value of the common shares issued to effect the restructuring. Thus, Hale’s total stockholders’ equity had a net increase of $800,000 (i.e., $700,000 + $100,000).

Although not required, the following journal entry would be used to record the troubled debt restructuring on Hale’s books:

Liabilities 1,200,000
Cash 400,000
Common stock (80,000 x $1) 80,000
APIC-CS [(80,000 x ($1.25 - $1.00)] 20,000
Gain on restructuring [$1,200,000 - ($400,000 +$100,000)] 700,000
Alternatively, this question can be solved using the accounting equation. Since assets decreased by $400,000, and liabilities decreased by $1,200,000, Hale’s stockholders’ equity must have increased by $800,000.

558
Q

On June 30 of the current year the condensed balance sheet for the partnership of Eddy, Fox, and Grimm, together with their respective profit and loss sharing percentages, was as follows:

Assets, net of liabilities	$320,000
Eddy, capital (50%)	160,000
Fox, capital (30%)	96,000
Grimm, capital (20%)	     64,000
Total Equity	$320,000
Hamm is admitted as a new partner with a 25% interest in the capital of the new partnership for a cash payment of $140,000. Total goodwill implicit in the transaction is to be recorded. Immediately after the admission of Hamm, Eddy's capital account balance should be
A
$280,000
B
$210,000
C
$160,000
D
$140,000
A

b

Explanation:
The correct answer is (B). The goodwill method looks upon this transaction as an indication that the partnership possesses an actual value of $560,000 ($140,000/25%) after Hamm’s admission. Since, even with Hamm’s investment, the partnership is reporting only $460,000 ($320,000 + $140,000) in net assets, a valuation adjustment of $100,000 is required. This adjustment is recorded as goodwill and is allocated to the original partners—Eddy, Fox, and Grimm—by their respective profit and loss sharing percentages. Immediately after admission of Hamm, Eddy’s capital account balance is $210,000 ($160,000 + $50,000).

Goodwill 100,000
Eddy, Capital ($100,000 x 50%) 50,000
Fox, Capital ($100,000 x 30%) 30,000
Grimm, Capital ($100,000 x 20%) 20,000

Cash 140,000
Hamm, Capital 140,000

559
Q
Aldrich Co. distributes cash dividends to its shareholders during the current year. The dividends are declared on March 9 and are payable to shareholders as of the date of record, which is April 15. The dividends are actually paid on May 19. At which of the following dates would the dividends become a liability to Aldrich?
A
March 9
B
April 15
C
May 19
D
December 31
A

a

Explanation:
The date of declaration, March 9, is the date on which dividends are formally declared by the board of directors and the declared dividends become a liability. On that date Aldrich would debit Retained Earnings and credit Dividends Payable. The date of record, April 15, is the date used to establish those stockholders who will receive the declared dividends. No journal entry is required unless the number of shares outstanding have changed from the date of declaration. The distribution of assets is made on the date of payment, May 19. On that date Aldrich would credit Cash and debit Dividends Payable. December 31 is merely the end of the year; once the dividends were paid on May 19 they would no longer be a liability.

560
Q
On March 1 of the current year, Rya Corp. issued 1,000 shares of its $20 par value common stock and 2,000 shares of its $20 par value convertible preferred stock for a total of $80,000. At this date, Rya's common stock was selling for $36 per share, and the convertible preferred stock was selling for $27 per share. What amount of the proceeds should be allocated to Rya's convertible preferred stock?
A
$60,000
B
$54,000
C
$48,000
D
$44,000
A

c

Explanation:
The amount of the proceeds received that should be allocated to the preferred stock is $48,000 [i.e., ($54,000 / $90,000) × $80,000]. Since the fair value is available for each class of security, the lump sum received should be allocated to the two classes of securities by their relative fair value, as follows:

Fair value of common stock (1,000 × $36) $36,000
Fair value of preferred stock (2,000 × $27) 54,000
$90,000

561
Q
Porter Co. began its business last year and issued 10,000 shares of common stock at $3 per share. The par value of the stock is $1 per share. During January of the current year, Porter bought back 500 shares at $6 per share, which were reported by Porter as treasury stock. The treasury stock shares were reissued later in the current year at $10 per share. Porter used the cost method to account for its equity transactions. What amount should Porter report as paid-in capital related to its treasury stock transactions on its balance sheet for the current year?
A
$ 1,500
B
$ 2,000
C
$ 4,500
D
$20,000
A

b

Explanation:
Treasury stock is the corporation’s common or preferred stock that has been reacquired by purchase, by settlement of an obligation to the corporation, or through donation. The cost method views the purchase and subsequent disposition of stock as one transaction. The treasury stock is recorded (debited), carried, and then reissued at acquisition cost. If the stock is reissued at a price in excess of the acquisition cost, the excess is credited to an appropriately titled paid-in capital (PIC) account. If the stock is reissued at less than the acquisition cost, the deficit is first charged against any existing balance in the treasury stock paid-in capital account and then excess, if any, is then charged against retained earnings. Porter reissued the 500 shares of treasury stock at a price ($10 per share) in excess of the acquisition cost ($6 per share). The excess ($10 - $6 = $4 per share × 500 shares = $2,000) is credited to the PIC account related to treasury stock on Porter’s balance sheet for the current year.

562
Q
In the current year, on December 1, Nilo Corp. declared a property dividend of marketable securities to be distributed on December 31 to stockholders of record on December 15. On December 1 the marketable securities had a carrying amount of $60,000 and a fair value of $78,000. What is the effect of this property dividend on Nilo's current year retained earnings, after all nominal accounts are closed?
A
$0
B
$18,000 increase
C
$60,000 decrease
D
$78,000 decrease
A

c

Explanation:
A transfer of a nonmonetary asset to a stockholder in a non-reciprocal transfer should be recorded at the fair value of the asset transferred, and a gain or loss should be recognized on the disposition of the asset equal to the difference between the fair value and carrying amount of the asset. After all nominal accounts (e.g., Gain on Disposition of Investment) are closed, the effect of this property dividend is to decrease Retained Earnings by $60,000 (i.e., $78,000 - $18,000). Nilo records the following entries at the declaration date.

Marketable securities ($78,000 - $60,000) 18,000
Gain on disposition of investment 18,000
Retained earnings 78,000
Property dividend payable 78,000

563
Q

Instead of the usual cash dividend, Evie Corp. declared and distributed a property dividend from its overstocked merchandise. The excess of the merchandise’s carrying amount over its market value should be

A
Ignored
B
Reported as a separately disclosed reduction of retained earnings
C
Reported as discontinued operations loss, net of income taxes
D
Reported as a reduction in income from continuing operations

A

d

Explanation:
The correct answer is (D).

Dividend paid in the form of property is recorded at Fair Market Value as on the declaration date and any difference between the fair value and book value is recognized as gain or loss as if the asset is sold at the fair value.

In this question, the merchandise’s market value is less than its carrying amount and therefore a loss has to be recognized for the difference as a reduction in income from continuing operations.

The retained earnings would be reduced for the amount of dividend payable and amount of loss on property transferred.

Options (A), (B) and (C) are incorrect as per above explanation.

564
Q

Which of the following statements regarding dividends on preferred stock is false?
A
If all or part of the stated dividend on cumulative preferred stock is not paid in a given year, the unpaid portion accumulates.
B
No dividends can be paid on common stock until the accumulated dividends are paid on cumulative preferred stock.
C
Dividends in arrears should be reported as a liability.
D
If a dividend on noncumulative preferred stock is not paid in a given year, the dividend is lost forever.

A

c

Explanation:
Dividends in arrears are not a liability; however, they should be disclosed parenthetically or in the footnotes to the financial statements.

565
Q

If the agreement specifies how profits are to be shared, but is silent as to losses, how should losses be shared?
A
In the same manner as profits
B
According to the ratio of partners’ capital balances as of a particular date
C
According to the partners’ salaries or bonuses
D
Equally, or in a specified ratio

A

a

Explanation:
If the agreement specifies how profits are to be shared, but is silent as to losses, losses are to be shared in the same manner as profits. Conversely, if the agreement specifies the sharing of losses, but is silent as to profits, profits are shared in the same manner as losses.

566
Q
Which of the following would be reported in the income statement of a proprietorship?
Proprietor's draw	Depreciation
A	Yes	Yes
B	Yes	No
C	No	Yes
D	No	No
A

c

Explanation:
A sole proprietorship’s equity consists of a single proprietor’s equity account, Owner’s Equity or Net Worth. This is the sum of the beginning capital balance, plus additional investments during the period, plus net income (or minus net loss) minus withdrawals. The proprietor’s draw is not reported separately on the income statement, but rather is included in Owner’s Equity or Net Worth. Depreciation is not included in the Owner’s Equity or Net Worth account. It is reported on the income statement of a proprietorship.

567
Q

On December 1 of the current year, shares of authorized common stock were issued on a subscription basis at a price in excess of par value. A total of 20% of the subscription price of each share was collected as a down payment on December 1 with the remaining 80% of the subscription price of each share due in the next year. Collectibility was reasonably assured. At December 31 the stockholders’ equity section of the balance sheet would report additional paid-in capital for the excess of the subscription price over the par value of the shares of common stock subscribed and
A
Common stock issued for 20% of the par value of the shares of common stock subscribed.
B
Common stock issued for the par value of the shares of common stock subscribed.
C
Common stock subscribed for 80% of the par value of the shares of common stock subscribed.
D
Common stock subscribed for the par value of the shares of common stock subscribed.

A

d

Explanation:
The common stock’s par value is recorded as Common Stock Subscribed, and the excess of the subscription price over the common stock’s par value is recorded as Additional Paid-In Capital.

Cash (20% down payment) XX
Subscriptions receivable (balance due) XX
Common stock subscribed (shares × par value) XX
Additional paid-in capital (to balance) XX

568
Q
Bain Corp. owned 20,000 common shares of Tell Corp. purchased several years ago for $180,000. On December 15 of the previous year, Bain declared a property dividend of all of its Tell Corp. shares on the basis of one share of Tell for every 10 shares of Bain common stock held by its stockholders. The property dividend was distributed on January 15 of the current year. On the declaration date, the aggregate market price of the Tell shares held by Bain was $300,000. The entry to record the declaration of the dividend would include a debit to retained earnings (or property dividends declared) of
A
$0
B
$120,000
C
$180,000
D
$300,000
A

d

Explanation:
A transfer of a nonmonetary asset to a stockholder in a nonreciprocal transfer should be recorded at the fair value of the asset transferred, and a gain or loss should be recognized on the disposition of the asset. Bain records the following entries at the declaration date.

Investment in Stock of Tell Corp. ($300,000 - $180,000) 120,000
Gain on Disposal of Investment 120,000
Retained Earnings 300,000
Property Dividend Payable 300,000

569
Q
The following format was used by Gee, Inc. for its current year statement of owners' equity:
Common
stock,
$1 par	Additional
paid-in
capital	Retained
earnings
Balance at 1/1	$90,000	$800,000	$175,000
Additions and deductions:			
100% stock dividend			
5% stock dividend	\_\_\_\_\_\_\_\_\_	\_\_\_\_\_\_\_\_\_	\_\_\_\_\_\_\_\_\_
Balance at 12/31	\_\_\_\_\_\_\_\_\_	\_\_\_\_\_\_\_\_\_	\_\_\_\_\_\_\_\_\_When both the 100% and the 5% stock dividends were declared, Gee's common stock was selling for more than its $1 par value. How would the 5% stock dividend affect the additional paid-in capital and retained earnings amounts reported in Gee's year-end statement of owners' equity?
Additional paid-in capital	Retained earnings
A	Increase	Decrease
B	Increase	Increase
C	No Change	Decrease
D	No change	Increase
A

a

Explanation:
Since the 5% stock dividend is less than 20 to 25% of the number of shares outstanding, it is considered to be a’small’ stock dividend. Therefore, it should be recorded by capitalizing a portion of Retained Earnings equal to the fair value of the shares issued. As a result of the’small’ stock dividend, Retained Earnings will decrease by the fair value of the shares issued, Common Stock will increase by the par value of the shares issued, and Additional Paid-in Capital will increase by the excess of the fair value over the par value of the shares issued.

570
Q

On January 1 of the current year the partners of Cobb, Davis, and Eddy, who share profits and losses in the ratio of 5:3:2, respectively, decided to liquidate their partnership. On this date the partnership condensed balance sheet was as follows:
Cash $ 50,000 Liabilities $ 60,000
Other assets 250,000 Cobb, capital 80,000
$300,000 Davis, capital 90,000
Eddy, capital 70,000
$300,000On January 15 of the current year the first cash sale of other assets with a carrying amount of $150,000 realized $120,000. Safe installment payments to the partners were made the same date. How much cash should be distributed to each partner?
Cobb Davis Eddy
A $15,000 $51,000 $44,000
B $40,000 $45,000 $35,000
C $55,000 $33,000 $22,000
D $60,000 $36,000 $24,000

A

a

Explanation:
The maximum possible loss assumes that nothing will be received from the disposition of the remaining $100,000 of other assets. The January 5 sale generates an additional $120,000 of cash and includes a $30,000 loss. This $30,000 loss is allocated to Cobb ($15,000), Davis ($9,000), and Eddy ($6,000). From the $170,000 balance of cash, the $60,000 debts are paid, leaving $110,000 in cash. As the maximum possible loss is $100,000, this amount is tenatively distributed to the capital accounts [Cobb ($50,000), Davis ($30,000), and Eddy ($20,000)] to determine the remaining amount in each capital account. This remaining amount is the safe cash distribution [Cobb ($15,000), Davis ($51,000), and Eddy ($44,000)] for each partner.

571
Q
Plack Co. purchased 10,000 shares (2% ownership) of Ty Corp. on February 14 of the current year. Plack received a stock dividend of 2,000 shares on April 30 when the market value per share was $35. Ty paid a cash dividend of $2 per share on December 15. In its current year income statement, what amount should Plack report as dividend income?
A
$20,000
B
$24,000
C
$90,000
D
$94,000
A

b

Explanation:
A stock dividend is not income under the cost or equity methods of accounting for equity investments, but it does impact the number of shares owned at the dividend declaration date. (10,000 + 2,000) × $2 = $24,000.

572
Q
A company declared a cash dividend on its common stock on December 15 of the previous year, payable on January 12 of the current year. How would this dividend affect stockholders' equity on the following dates?
December 15,
previous year	December 31,
previous year	January 12,
current year
A	Decrease	No effect	Decrease
B	Decrease	No effect	No effect
C	No effect	Decrease	No effect
D	No effect	No effect	Decrease
A

b

Explanation:
A cash dividend reduces retained earnings (and, therefore, net stockholders’ equity) on the date of declaration. A cash dividend will have no effect on net stockholders’ equity at the date of distribution. A cash dividend is recorded when declared by a debit to Retained Earnings and a credit to Cash Dividends Payable . There is no adjusting entry at the end of the period. The entry at the payment date includes a debit to Cash Dividends Payable and a credit to Cash. Generally, the dividend reduces stockholders’ equity only at the declaration date. The fact that the accounting period ends between the declaration date and the payment date has no effect on stockholders’ equity.

573
Q

The following condensed balance sheet is presented for the partnership of Alfa and Beda, who share profits and losses in the ratio of 60:40, respectively:
Cash $ 45,000 Accounts payable $120,000
Other assets 625,000 Alfa, capital 348,000
Beda, loan 30,000 Beda, capital 232,000
$700,000 $700,000The assets and liabilities are fairly valued on the balance sheet. Alfa and Beda decide to admit Capp as a new partner with a 20% interest. No goodwill or bonus is to be recorded. What amount should Capp contribute in cash or other assets?
A
$110,000
B
$116,000
C
$140,000
D
$145,000

A

d

Explanation:
Total current capital ($348,000 + $232,000) $580,000
Divided by: Percentage current capital is of new total capital (100% - Capp @ 20%) / 80%
Total new capital, including Capp $725,000
Less: Alfa & Beda Capital (current capital) (580,000)
Capp’s contribution for 20% interest $145,000

574
Q

Retained Earnings include which of the following?

A
Dividends declared
B
Gains from treasury stock transactions
C
Both A and B
D
None of the above
A

Explanation:
The correct answer is (D).

The Retained Earnings account is used to close out all profit and loss accounts.

The balance represents the accumulated income of the corporation less dividends paid (not declared) and amounts transferred to paid-in capital accounts.

Retained earnings should not include the following:

Gains from treasury stock transactions
Gifts of property
Additions to owners’ equity attributable to reappraisals of property
Accumulated balance of other comprehensive income

575
Q

The following condensed balance sheet is presented for the partnership of Smith and Jones, who share profits and losses in the ratio of 60:40, respectively:
Other assets $450,000 Accounts payable $120,000
Smith, loan 20,000 Smith, capital 195,000
$470,000 Jones, capital 155,000
$470,000The partners have decided to liquidate the partnership. If the other assets are sold for $385,000, what amount of the available cash should be distributed to Smith?
A
$136,000
B
$156,000
C
$159,000
D
$195,000

A

a

Explanation:
Smith’s capital balance of $195,000 is reduced by his $20,000 loan from the partnership ($195,000 - $20,000). The cash available to be distributed to each partner is determined after the loss on the sale of other assets is figured and the payment of liabilities. $175,000 - $39,000 = $136,000.

576
Q
Jones Co. had 50,000 shares of $5 par value common stock outstanding at January 1. On August 1, Jones declared a 5% stock dividend followed by a two-forone stock split on September 1. What amount should Jones report as common shares outstanding at December 31?
A
105,000
B
100,000
C
52,500
D
50,000
A

a

Explanation:
Jones should report 105,000 shares outstanding (50,000 shares x 1.05 x 2)

577
Q
On January 2 of the current year, Smith purchased the net assets of Jones' Cleaning, a sole proprietorship, for $350,000, and commenced operations of Spiffy Cleaning, a sole proprietorship. The assets had a carrying amount of $375,000 and a market value of $360,000. In Spiffy's cash-basis financial statements for the year ended December 31, Spiffy reported revenues in excess of expenses of $60,000. Smith's drawings during the year were $20,000. In Spiffy's financial statements, what amount should be reported as Capital--Smith?
A
$390,000
B
$400,000
C
$410,000
D
$415,000
A

a

Explanation:
Capital–Smith, 1/2 (i.e., cost of net assets contributed by owner at 1/2) $ 350,000
Add: Income of sole proprietorship 60,000
Less: Smith’s drawings (20,000)
Capital–Smith, 12/31 $ 390,000

578
Q
Rudd Corp. had 700,000 shares of common stock authorized and 300,000 shares outstanding at December 31 of the previous year. The following events occurred during the current year:
January 31	Declared 10% stock dividend
June 30	Purchased 100,000 shares
August 1	Reissued 50,000 shares
November 30	Declared 2-for-1 stock splitAt December 31 of the current year, how many shares of common stock did Rudd have outstanding?
A
560,000
B
600,000
C
630,000
D
660,000
A

a

Explanation:
Common shares outstanding, 12/31, previous year 300,000
10% stock dividend, 1/31 (300,000 × 10%) 30,000
Treasury shares purchased, 6/30 (100,000)
Treasury shares reissued, 8/1 50,000
2-for-1 stock split, 11/30 (300,000 + 30,000 - 100,000 + 50,000) 280,000
Common shares outstanding, 12/31, current year 560,000

579
Q
Abel and Carr formed a partnership and agreed to divide initial capital equally, even though Abel contributed $100,000 and Carr contributed $84,000 in identifiable assets. Under the bonus approach to adjust the capital accounts, Carr's unidentifiable asset should be debited for
A
$46,000
B
$16,000
C
$ 8,000
D
$0
A

d

Explanation:
Carr must have made an intangible contribution to the partnership because Abel and Carr have agreed to divide initial capital equally, even though Carr contributed less in identifiable assets ($84,000 < $100,000). Because the bonus method is used to record the formation of the partnership, and the bonus method assumes that an intangible contribution does not constitute a partnership asset with a measurable cost, no unidentifiable asset (i.e., goodwill) is recognized by the partnership. The partnership formation adjusts the capital accounts of the two partners without recognizing goodwill.

Identifiable Assets ($100,000 + $84,000) 184,000
Abel, capital ($184,000 × 50%) 92,000
Carr, capital ($184,000 × 50%) 92,000

580
Q

The following condensed balance sheet is presented for the partnership of Alfa and Beda, who share profits and losses in the ratio of 60:40, respectively:
Cash $ 45,000 Accounts payable $120,000
Other assets 625,000 Alfa, capital 348,000
Beda, loan 30,000 Beda, capital 232,000
$700,000 $700,000Alfa and Beda decide to liquidate the partnership. If the other assets are sold for $500,000, what amount should be distributed to Alfa?
A
$255,000
B
$273,000
C
$327,000
D
$348,000

A

b

Explanation:
Total available cash is distributed after allowing for loan and loss on sale of other assets.

Cash $ 45,000
Plus: Sale of other assets 500,000
Less: Accounts payable (120,000)
Total cash to distribute $ 425,000
Total (100%) Alfa (60%) Beda (40%)
Beginning capital $580,000 $348,000 $232,000
Reduce Beda capital for loan (30,000) (30,000)
Allocate loss on sale of assets ($500,000 - $625,000) (125,000) (75,000) (50,000)
Distribution of cash $425,000 $273,000 $152,000

581
Q
On January 2 of the current year, Lake Mining Co.'s board of directors declared a cash dividend of $400,000 to stockholders of record on January 18 and payable on February 10 of the current year. The dividend is permissible under law in Lake's state of incorporation. Selected data from Lake's previous year December 31 balance sheet are as follows:
Accumulated depletion	$100,000
Capital stock	500,000
Additional paid-in capital	150,000
Retained earnings	300,000The $400,000 dividend includes a liquidating dividend of
A
$0.
B
$100,000.
C
$150,000.
D
$300,000.
A

b

Explanation:
Any dividend that does not come out of retained earnings is a reduction of corporate paid-in capital and, to that extent, is a liquidating dividend.

Dividend $400,000
Retained earnings (300,000)
Liquidating dividend $100,000

582
Q
The following information pertains to a sale of real estate by Ryan Co. to Sud Co. on December 31, year 1:
Carrying amount		$2,000,000
Sales price:		
Cash	$ 300,000	
Purchase money mortgage	2,700,000	3,000,000The mortgage is payable in nine annual installments of $300,000 beginning December 31, year 2, plus interest of 10%. The December 31, year 2, installment was paid as scheduled, together with interest of $270,000. Ryan uses the cost recovery method to account for the sale. What amount of income should Ryan recognize in year 2 from the real estate sale and its financing?
A
$570,000
B
$370,000
C
$270,000
D
$0
A

d

Explanation:
Under the cost recovery method, no profit is recognized until cash payments by the buyer exceed the seller’s cost of goods sold. Therefore, Ryan should not recognize any income in year 2 from the real estate sale and its financing because the accumulated cash payments from the buyer are less than the carrying amount of the real estate. [($300,000 down payment + $300,000 year 2 payment + $270,000 year 2 interest payment) < $2,000,000]

583
Q

Which of the following is a method of recognizing revenue over time?

A
Output method
B
Throughput method
C
Conservation method
D
Completion method
A

Explanation:
Answer (A): The output method is one of the two methods for recognizing revenue over time.

If the performance obligation is met by the “over time “ entity, the revenue is also recognized as “over time “ according to the following two methods:

  • Output Method: Revenue shall be recognized by the value of the goods and services transferred to the customer to date.
  • Input Method: The revenue is recognized on the basis of the efforts of the entity or its inputs.
584
Q
On day 1, Clothes Co., sells clothing to Link Corp. for $40,000. Clothes ships the clothing on day 1 and Link is obligated to pay Clothes within six months. Link is given 12 months to return any of the clothing for a refund if they experience low demand. Link is also given 18 months to exchange any clothing due to low demand. At the time of sale, Clothes cannot reasonably estimate returns, but estimates $5,000 in exchanged goods. Clothes should recognize revenue for the aforementioned transaction
A
On the day of the sale.
B
Six months after the date of sale.
C
12 months after the date of sale.
D
18 months after the date of sale.
A

Explanation:
When the buyer has the right to return the product, revenue should be recognized at the time of sale only if certain conditions are met. One of those conditions is that the amount of future returns can be reasonably estimated. Clothes can estimate the $5,000 of exchanged goods, but not the amount of returns. If the conditions are not met, revenue must be recognized when the return privilege has substantially expired or when the conditions are subsequently met, whichever comes first. There was no indication the conditions were ever met, so the revenue would be recognized 12 months after the sale date when the return privilege expired.

585
Q

A construction company recognizes revenue at a point in time. Which of the following is one of the criteria for recognizing revenue at a point in time?

A
Simultaneous receipt and consumption of customer benefits as and when the company performs.
B
Significant risks and rewards of ownership of the asset are with the customer
C
The performance of the entity creates or enhances an asset controlled by the customer.
D
The performance of the entity does not create an asset with an alternative use.

A

Explanation:
The correct answer is (B).

Significant risks and rewards of ownership of the asset are with the customer.

All other options are the criteria for revenue to be recognized over time.

To recognize revenue at a point in time, the following criteria must be met:

  • Entity currently has a right to collect payment for the asset
  • The legal title for the asset is with the customer
  • Physical possession of the asset is transferred by the entity to the customer
  • Significant risks and rewards of ownership of the asset are with the customer
  • Asset has been accepted by the customer
586
Q
Lin Co., a distributor of machinery, bought a machine from the manufacturer in November for $10,000. On December 30, Lin sold this machine to Zee Hardware for $15,000 under the following terms: 2% discount if paid within 30 days, 1% discount if paid after 30 days but within 60 days, or payable in full within 90 days if not paid within the discount periods. However, Zee had the right to return this machine to Lin if Zee was unable to resell the machine before expiration of the 90-day payment period, in which case Zee's obligation to Lin would be canceled. In Lin's net sales for the year ended December 31, how much should be included for the sale of this machine to Zee?
A
$0
B
$14,700
C
$14,850
D
$15,000
A

a

Explanation:
Zee has the right to return the machine to Lin if Zee is not able to resell the machine before expiration of the 90-day payment period. If an enterprise sells its product but gives the buyer the right to return the product, revenue from the sales transaction is not recognized at the time of sale if the buyer is obligated to pay the seller and the obligation is contingent upon resale of the product.

587
Q

On September 21, 2018 Big Ltd. sold 10 laptops to Small Ltd. for $10,000. For this price, promise was given by Big Ltd. that these laptops complied with agreed upon specifications and would operate as promised for one year from the date of purchase. Big Ltd. also agrees to send its personnel every four months for a period of two years for an on-site maintenance of the laptops as Small Ltd. was one of its major customers. Big Ltd. estimated that it would incur a cost of $1,000 per annum for such a service and the company usually earns a profit of 20% for such a service. Big Ltd. records for this transaction by crediting revenue by $10,000. What should be the correcting entry?

A
Credit revenue with a further $2,400.
B
Credit revenue with a further $1,200.
C
Debit revenue with $2,400.
D
No correction required.
A

c

Explanation:
In the given example, two kinds of warranty are given:-

Promise was given by Big Ltd. that these laptops complied with agreed upon specifications and would operate as promised for one year from the date of purchase and
Big Ltd. also agrees to send its personnel every four months for a period of two years for an on-site maintenance of the laptops as Small Ltd. was one of its major customers.
The first promise is an assurance type of warranty as the customer does not have the option of purchasing it separately and it just gives assurance that the product complies with the agreed upon procedures. This type of warranty is not a separate performance obligation, and thus no transaction price is allocated to it. To account for an assurance-type warranty, the vendor should estimate and accrue a warranty liability when the promised good or service is delivered to the customer.

Service type of warranty arises when (1) a customer has the option to purchase it separately or (2) it provides service beyond assuring that the product complies with promised specifications. A service-type warranty represents a distinct performance obligation, thus a portion of the transaction price is allocated to it. Revenue is recognized as the warranty obligation is fulfilled, which is likely over the term of the warranty.

Thus, for $10,000 Big Ltd. agrees to give 10 laptops and provide maintenance service for 2 years. The value of maintenance service would be $1,000 + 20% per annum = $1,200. The service is promised for two years and would have revenue worth $2,400. This revenue has to be recognized over time as and when Big Ltd. provides the maintenance service. As revenue has to be recognized over time, the revenue will be recognized either using the output method or the input method. The first maintenance service is due only in January the following year, there would be no output or input as of December this year towards maintenance service and no revenue has to be recognized for this year for the maintenance service. This implies the revenue to be recognized this year for the sale of the laptop would be $10,000 - $2,400 = $7,600.

As revenue is already credited with $10,000, the correcting entry would be to debit revenue with $2,400.

Options (A), (B) and (D) are incorrect based on the above explanation

588
Q

The following information pertains to the transfer of real estate pursuant to a troubled debt restructuring by Knob Co. to Mene Corp. in full liquidation of Knob’s liability to Mene.
Carrying amount of liability liquidated $150,000
Carrying amount of real estate transferred 100,000
Fair value of real estate transferred 90,000What amount should Knob report as the total amount of pretax gain (loss) as a result of this troubled debt restructuring?
A
$(10,000)
B
$0
C
$50,000
D
$60,000

A

c

Explanation:
Knob reports a $10,000 (i.e., $100,000 - $90,000) ordinary loss equal to the excess of the carrying amount over the fair value of the real estate transferred. Knob also reports a $60,000 gain on the debt restructuring. The net gain is reported in income from continuing operations [($10,000) + $60,000 = $50,000].

589
Q

Which of the following statements is correct regarding deferred revenues recorded by a company that provides services to customers?

A
Deferred revenue is a liability until the service has been performed.
B
Deferred revenues represent revenues earned but not yet received in cash.
C
Deferred revenues result from services that have been performed but have not been billed.
D
A deferred revenue on the books of one company is an accrued expense on the books of another company

A

a

Explanation:
Deferred revenue or unearned revenue is accounted when the cash is received, but revenue is not earned as the entity has not yet satisfied its performance obligation. It is accounted as a liability till the entity satisfies the performance obligation.

590
Q
North Co. entered into a franchise agreement with South Co. for an initial fee of $50,000. North received $10,000 at the agreement's signing. The remaining balance was to be paid at a rate of $10,000 per year, beginning the following year. North's services per the agreement were not complete in the current year. Operating activities will commence next year. What amount should North report as franchise revenue in the current year?
A
$0
B
$10,000
C
$20,000
D
$50,000
A

a

Explanation:
Initial franchise fees from franchise sales ordinarily must be recognized (with provision for estimated uncollectible amounts) when all material services or conditions relating to the sale have been substantially performed or satisfied by the franchisor. North’s services per the agreement were not complete in the current year so no franchise revenue would be reported by North in the current year.

591
Q
Toddler Care Co. offers three payment plans on its 12-month contracts. Information on the three plans and the number of children enrolled in each plan for the September 1, Year 1, through August 31, Year 2, contract year follows:
Plan	Initial payment per child	Monthly fees per child	Number of children
#1	$500	$ --	15
#2	200	30	12
#3	----	50	 9 
36Toddler received $9,900 of initial payments on September 1, Year 1, and $3,240 of monthly fees during the period September 1 through December 31, Year 1. In its December 31, Year 1 balance sheet, what amount should Toddler report as deferred revenues?
A
$3,300
B
$4,380
C
$6,600
D
$9,900
A

c

Explanation:
Todd Care received $9,900 [i.e., ($500 × 15) + ($200 x 12)] of initial payments on its 12-month contracts for the 9/1 year 1 through 8/31 year 2 contract year. Since four months of the initial payments on the contracts have been earned as of 12/31 of year 1, eight months of the initial payments on the contracts should be reported as deferred revenue at that date (i.e., $9,900 × 8/12 = $6,600).

592
Q
On December 31, year 1, Rice, Inc., authorized Graf to operate as a franchisee for an initial franchise fee of $150,000. Of this amount, $60,000 was received upon signing the agreement and the balance, represented by a note, is due in three annual payments of $30,000 each beginning December 31, year 2. The present value on December 31, year 1, of the three annual payments appropriately discounted is $72,000. According to the agreement, the nonrefundable down payment represents a fair measure of the services already performed by Rice; however, substantial future services are required of Rice. Collectibility of the note is reasonably certain. In Rice's December 31, year 1, balance sheet, unearned franchise fees from Graf's franchise should be reported as
A
$132,000
B
$100,000
C
$ 90,000
D
$ 72,000
A

d

Explanation:
The $60,000 received in year 1 was for services already performed and is, therefore, recognized as income in year 1. The three payments of $30,000 have not yet been earned as of the signing of the agreement because Rice is required to perform substantial future services. Such payments may not be recognized as revenue until the services are performed. The annual payments should be discounted and reported as unearned franchise fees at their present value of $72,000 at 12/31, year 1.

593
Q

A retail store sold gift certificates that are redeemable in merchandise. The gift certificates lapse one year after they are issued. How would the deferred revenue account be affected by each of the following?
Redemption of certificates Lapse of certificates
A Decrease Decrease
B Decrease No effect
C No effect Decrease
D No effect No effect

A

a

Explanation:
When the redeemable gift certificates are sold, the sales price collected represents unearned revenue and an unearned revenue account is credited (i.e. increased). As the certificates are redeemed, the earned revenue is recognized, decreasing the unearned portion. Lapsed certificates would also decrease unearned revenue, but would increase Gain on Lapsed Certificates rather than Revenue account. In either case, the Deferred Revenue account is decreased.

594
Q

State Co. recognizes construction revenue and expenses using an adjusted version of the percentage-of-completion method in accordance with the input method prescribed in ASC 606 and revenue is recognized over time. During year 1, a single long-term project was begun, which continued through year 2. Information on the project follows:

Year 1 Year 2

Accounts receivable from construction contract $100,000 $300,000
Construction expenses 105,000 192,000
Construction in progress 122,000 364,000
Partial billings on contract 100,000 420,000
It should be assumed that all costs incurred to date, as well as all estimated future costs, will advance the progress of the project and no spoilage or waste has occurred. Additionally, the performance by State Co. does not create an asset with an alternative use and State Co. has an enforceable right to payment for the completed performance to date.

Profit recognized from the long-term construction contract in year 2 should be

A
$50,000
B
$108,000
C
$128,000
D
$228,000
A

Explanation:
The correct answer is (A)

Given the facts, the profit recognized from the contract in year 2 can be computed by subtracting the contract costs incurred in year 2 from the increase in the Construction In Progress account during year 2.

Construction in progress, 12/31, year 2 $ 364,000
Construction in progress, 12/31, year 1 (122,000)
Increase during year 2 (contract costs incurred and profit recognized in year 2) $ 242,000
Contract costs incurred in year 2 (192,000)
Profit recognized in year 2 $ 50,000
Note:

Revenue recognized over time is similar to the Percentage-of-Completion Method allowed under the previous standard.
Revenue recognized at a point in time is similar to the Completed Contract Method allowed under the previous standard.

595
Q

Regal Department Store sell gift certificates, redeemable for store merchandise that expires one year after their issuance. Regal has the following information pertaining to its gift certificates sales and redemptions:

Unredeemed at 12/31/Yr1 $75,000
Yr2 sales $250,000
Yr2 redemptions of prior year sales $25,000
Yr2 redemptions of current year sales $175,000
Regal’s experience indicates that 10% of gift certificates sold will not be redeemed. In its December 31, Yr2, balance sheet, what amount should Regal report as unearned revenue?

A
$125,000
B
$112,500
C
$50,000
D
$100,000
A

Explanation:
The correct answer is (C).

Unearned revenue is when the cash is received, but revenue is not earned. Unearned revenue is accounted for as liability (as it is an obligation to perform a service in the future) and revenue is recognized as ‘earned’. As on December 31, Yr2, the balance sheet of Regal should report unearned revenue at $50,000 calculated as:

Ref Summary Amount
a Gift certificates sold in Yr2 $250,000
b Expected redemption of gift certificates (i.e. $250,000 x 0.9) $225,000
c Gift certificates redeemed in Yr2 $175,000
d Unearned revenue for Yr2 (b-c) $50,000
Note: Any gift certificate unredeemed will be recognized as revenue at the expiry of the gift certificate validity period as the contract will be deemed terminated and consideration received is non-refundable.

596
Q

Bear Co., which began operations on January 2, Year 1 recognizes revenue over time in accordance with ASC 606. The performance by Bear Co. does not create an asset with an alternative use and Bear Co., has an enforceable right to payment for the completed performance to date. The following information is available for the year:

Year 1 Year 2

Total Contract Price $300,000 $300,000
Costs incurred $70,000 $110,000
Estimated costs to complete at Year-end $200,000 $92,000
What is the income (or loss) recognized in Year 2?

A
$10,751
B
$7,778
C
$28,000
D
$18,529
A

a

Explanation:
The correct answer is (A).

Gross Profit recognized in each period of the contract on accrual basis and is calculated as follows:

Gross Profit = [(Cost incurred to date / Total Estimated Cost) x Estimated Profit] - Profit Already Recognized

Total Contract Price $300,000 $300,000
Actual Cost to Date $70,000 $180,000
Estimated Costs to Complete $200,000 $92,000
Estimated Total Costs at Completion $270,000 $272,000
Estimated Gross Profit $30,000 $28,000
Profit Recognized $7,778* $10,751**
*Year 1: ($70,000/$270,000) x $30,000 = $7,778

**Year 2: ($180,000/$272,000) x $28,000 - $7,778 = $10,751

Note:

Revenue recognized over time is similar to the Percentage-of-Completion Method allowed under the previous standard.
Revenue recognized at a point in time is similar to the Completed Contract Method allowed under the previous standard.

597
Q

Howard Co. had the following first-year amounts for a $7,000,000 construction contract:

Actual costs $2,000,000
Estimated costs to complete $6,000,000
Progress billings $1,800,000
Cash collected $1,500,000
Howard Co recognizes revenue over time in accordance with ASC 606. What amount should Howard recognize as gross profit (loss)?

A
($1,000,000)
B
($200,000)
C
$800,000
D
$1,750,000
A

Explanation:
The correct answer is (A).

Howard Co., recognizes revenue over time and would report a gross loss of ($1,000,000). Construction cost (actual + estimate) exceeds the value of the contract resulting in a loss.

Summary

Amount

Actual costs incurred

$2,000,000

Estimated costs to complete

$6,000,000

Total cost to complete

$8,000,000

Total contract value

$7,000,000

Gross loss on the construction contract

$(1,000,000)

598
Q

Which of the following statements is correct regarding deferred revenues recorded by a company that pro­vides services to customers?

A
Deferred revenue is a liability until the service had been performed.
B
Deferred revenues represent revenues earned but not yet received in cash.
C
Deferred revenues result from services that have been performed but have not been billed.
D
A deferred revenue on the books of one company is an accrued expense on the books of another company.

A

a

Explanation:
Deferred revenue is revenue collected or collectible in cash but not yet earned. When a company provides services to customers, deferred revenue is a liability until the service has been performed because the company has not yet earned that revenue.

599
Q

A company provides the following information:

 	Year 1	Year 2	Year 3
Cash receipts from customers:	 	 	 
From year 1 sales	$95,000	$120,000	 
From year 2 sales	 	$200,000	$ 75,000
From year 3 sales	 	$ 50,000	$225,000
What is the accrual-based revenue for year 2?
A
$200,000
B
$275,000
C
$320,000
D
$370,000
A

b

Explanation:
Accrual accounting recognizes and reports the effects of transactions and other events on the assets and liabilities of a business enterprise in the time periods to which they relate rather than only when cash is received or paid. The accrual-based revenue for year 2 would be all the $275,000 total cash receipts from cus­tomers for year 2 sales ($200,000 in year 2 + $75,000 in year 3 = $275,000).

600
Q
On January 2 of the current year, Boulder Co. assigned its patent to Castle Co. for royalties of 10% of patent-related sales. The assignment is for the remaining four years of the patent's life. Castle guaranteed Boulder a minimum royalty of $100,000 over the life of the patent and paid Boulder $50,000 against future royalties during the year. Patent-related sales for the year were $300,000. In its current year income statement, what amount should Boulder report as royalty revenue?
A
$ 25,000
B
$ 30,000
C
$ 50,000
D
$100,000
A

b

Explanation:
Revenue generally is recognized when both the earnings process is complete and an exchange has taken place. Deferred revenue is revenue collected, but not yet earned. Royalty revenue to Boulder would be $30,000 ($300,000 × .10). The remaining $20,000 balance of Castle’s payment against future royalties is deferred revenue.

601
Q

Ward, a consultant, keeps her accounting records on a cash basis. During the current year, Ward collected $200,000 in fees from clients. At December 31 of the previous year, Ward had accounts receivable of $40,000. At December 31 of the current year, Ward had accounts receivable of $60,000, and unearned fees of $5,000. On an accrual basis, what was Ward’s service revenue for the current year?

A
$175,000
B
$180,000
C
$215,000
D
$225,000
A

c

Explanation:
Collection of fees in the year	$200,000
Less: Accounts receivable, 1/1	(40,000)
Plus: Accounts receivable, 12/31	60,000
Less: Unearned revenue, 12/31	( 5,000)
Service revenue for the year	$215,000
602
Q

Which of the following is not one of the five revenue recognition steps under ASC 606?

A
Allocate the transaction price to the performance obligations in the contract.
B
Determine the transaction price.
C
Identify the contract(s) with a customer.
D
The customer can benefit from the goods or services either on a standalone basis or with other resources that the customer can easily access
Explanation:

A

Explanation:
The correct answer is (D).

The customer can benefit from the goods or services either on a standalone basis or with other resources that the customer can easily access is one of the criteria for a distinct good or service.

The 5-Step approach to Revenue Recognition:

Step 1: Identify the contract(s) with a customer.

Step 2: Identify the performance obligations in the contract.

Step 3: Determine the transaction price.

Step 4: Allocate the transaction price to the performance obligations in the contract.

Step 5: Recognize revenue when (or as) the entity satisfies a performance obligation.

603
Q

A clothing retailer allows customers to return shoes within 90 days of purchase. The company estimates that 10% of sales will be returned within the 90-day period. During the month, the company has sales of $500,000 and returns of sales made in prior months of $30,000. What amount should the company record as net sales revenue for new sales made during the month?

A
$420,000
B
$450,000
C
$500,000
D
$75,000
A

Explanation:
The correct answer is (B).

Sale with a Right of Return: In some contracts, an entity transfers control of a product to a customer and also grants the customer the right to return the product for various reasons (such as dissatisfaction with the product) and receive any combination of the following:

Full or partial refund of any consideration paid
Credit that can be applied against amounts owed or that will be owed, to the entity
Another product in exchange
To account for the transfer of goods with a right of return (and for some services that are provided subject to a refund), an entity should recognize all of the following:

Revenue for the transferred products in the amount of consideration to which the entity expects to be entitled; therefore, revenue would not be recognized for the products expected to be returned
Refund liability based on expectations about the number of refunds
Asset (and the corresponding adjustment to cost of sales) for its right to recover products from customers on settling the refund liability. The company will have to make a provision for returns from the sales revenue amount. The actual returns in the prior months will not affect the provision to be made for future months. Thus the net sales revenue for the month will be $450,000 [500,000 - (500,000 x 10%)].
(A) is incorrect because it also deducts the returns of sales in prior months
(C) is incorrect because it adds the returns of sales in prior months.
(D) is incorrect because it does not make provision for returns from the sales.

604
Q

Bear Co., which began operations on January 2, Year 1 recognizes revenue over time in accordance with ASC 606. The performance by Bear Co. does not create an asset with an alternative use and Bear Co., has an enforceable right to payment for the completed performance to date. The following information is available for the year:

Year 1 Year 2 Year 3

Total Contract Price $300,000 $300,000 $300,000
Costs Incurred $70,000 $110,000 $93,000
Estimated costs to complete at year end $200,000 $92,000 N/A
Calculate the income (or less) recognized in Year 3:

A
$8,471
B
$10,751
C
$27,000
D
$300,000
A

Explanation:
The correct answer is (A).

Gross Profit recognized in each period of the contract on accrual basis and is calculated as follows:

Gross Profit = [(Cost incurred to date / Total Estimated Cost) x Estimated Profit] - Profit Already Recognized

Total Contract Price $300,000 $300,000 $300,000
Actual Cost to Date $70,000 $180,000 $273,000
Estimated Costs to Complete $200,000 $92,000 N/A
Estimated Total Costs at Completion $270,000 $272,000 $273,000
Estimated Gross Profit $30,000 $28,000 $27,000
Profit Recognized $7,778* $10,751** $8,471***
*Year 1: ($70,000/$270,000) x $30,000 = $7,778

**Year 2: ($180,000/$272,000) x $28,000 - $7,778 = $10,751

***Year 3: ($273,000/$273,000) x $27,000 - $18,529 = $8,471

Note:

Revenue recognized over time is similar to the Percentage-of-Completion Method allowed under the previous standard.
Revenue recognized at a point in time is similar to the Completed Contract Method allowed under the previous standard.

605
Q

Which of the following is not one of the criteria for recognizing revenue over time?

A
Simultaneous receipt and consumption of customer benefits as and when the company performs.
B
The legal title for the asset is with the customer.
C
The performance of the entity creates or enhances an asset controlled by the customer.
D
The performance of the entity does not create an asset with an alternative use.

A

Explanation:
Answer (B): The legal title for the asset is with the customer is one of the criteria for recognizing revenue at a point in time, not over time.

An entity transfers control of goods or services over time and fulfills an obligation to perform and recognizes revenue over time if one of the following criteria is met:

  • Simultaneous receipt and consumption of customer benefits as and when the company performs.
  • The performance of the entity creates or enhances an asset controlled by the customer.
  • The performance of the entity does not create an asset with an alternative use of the entity and the entity has an enforceable right to payment for the completed performance to date.
606
Q

Which of the following is not an indicator that an entity is an agent

A
The entity does not have inventory risk before or after the goods have been ordered by a customer, during shipping, or on return.
B
The entity does not have discretion in establishing prices for the other party’s goods or services and, thus, the benefit that the entity can receive from those goods or services is limited.
C
The entity is exposed to credit risk for the amount receivable from a customer in exchange for the other party’s goods or services.
D
Another party is primarily responsible for fulfilling the contract.

A

Explanation:
The correct answer is (C) An agent is not exposed to credit risk for the amount receivable from a customer in exchange for the other party’s goods or services. The following are indicators that an entity is an agent – a. Another party is primarily responsible for fulfilling the contract. b. The entity does not have inventory risk before or after the goods have been ordered by a customer, during shipping, or on return. c. The entity does not have discretion in establishing prices for the other party’s goods or services and, thus, the benefit that the entity can receive from those goods or services is limited. d. The entity’s consideration is in the form of a commission. e. The entity is not exposed to credit risk for the amount receivable from a customer in exchange for the other party’s goods or services. (A), (B) and (D) are incorrect as they are all indicators that an entity is an agent

607
Q
On December 30, Devlin Co. sold goods to Jensen Co. for $10,000, under an arrangement in which (1) Jensen has an unlimited right of return and (2) Jensen's obligation to pay Devlin is contingent upon Jensen's reselling the goods. Past experience has shown that Jensen ordinarily resells 60% of goods and returns the other 40%. What amount should Devlin include in sales revenue for this transaction on its December 31 income statement?
A
$10,000
B
$6,000
C
$4,000
D
$0
A

d

Explanation:
Revenue from sales when the buyer has the right to return the product should be recognized at the time of sale only if all of several conditions are met. One condition is that the buyer has paid the seller, or is obligated to pay the seller, and the obligation is not contingent on resale of the product. While the amount of returns can be estimated based on past experience, the obligation for Jensen to pay Devlin is contingent upon Jensen reselling the goods. Thus, Devlin has no sales revenue for this transaction.

608
Q

Under IFRS, which of the following is not an allowed method of accounting for long-term construction contracts?

A
Fair value through profit and loss method
B
Percentage of completion method
C
Revenue recognition to the extent of cost incurred
D
Completed contract method
A

d

Explanation:
The completed contract method is not allowed under IFRS. IFRS requires the percentage of com­pletion method for long-term construction projects. If the percentage cannot be reasonably estimated, then reve­nue is recognized only to the extent of cost.

609
Q

H&S Ltd. is a manufacturer of huge cardboard boxes to help in transport of huge items. Many shops acquire these boxes for their customers. H&S ships the boxes with full payment due immediately. Legal title does not transfer and H&S retains the right to require shipment of any unsold boxes to other shops. When this right is exercised a full refund is provided. Similarly, shops may return any unsold boxes to H&S for a full refund. Is this a consignment arrangement and should revenue be recognized now?

Consignment Arrangement	Revenue Recognition
A	Yes	Yes
B	Yes	No
C	No	Yes
D	No	No
A

b

Explanation:
H&S has received payment and has transferred physical possession of the boxes, which are indicators that control has transferred. However, because shops can return the boxes and H&S can require shops to ship the boxes to other shops, the risks and rewards of ownership have not transferred. Additionally, legal title remains with H&S. In aggregate, it appears the customer is not able to obtain substantially all of the remaining benefits or the boxes of direct the use of the boxes. Furthermore, all three indicators of a consignment arrangement (control by the vendor, vendor’s ability to require return or transfer and no unconditional obligation of consignee to pay) are present. Consequently, this arrangement is a consignment. Because of this, revenue recognition must be deferred until the subsequent sale to the end-consumer.

610
Q

It is proper to recognize revenue prior to the sale of merchandise when

The revenue will be reported as a sale on a Consignment Basis.
The revenue will be reported under the Percentage of Completion Method.
A
I only
B
II only
C
Both I and II
D
Neither I nor II
A

d

Explanation:
The correct answer is (D). Both the sale on Consignment Basis and Percentage of Completion methods report revenue based upon satisfaction of performance obligation based on ASC 606’s five-step approach to recognizing revenue.

611
Q

Which of the following statements is false concerning long-term construction contract accounting?

A
Revenue recognized at a point in time reflects the periodic recognition of income as it is earned
B
Revenue recognized over time reflects the status of uncompleted contracts that is provided through the current estimates of costs to complete or of progress toward completion
C
Revenue recognized at a point in time does not permit the recording of income prior to completion, or substantial completion, of the contract
D
Revenue recognized at a point in time does not reflect current performance when the contract extends into more than one accounting period

A

Explanation:
The correct answer is (A)

Revenue recognized over time reflects the periodic recognition of income as it is earned.

Note:

Revenue recognized over time is similar to the Percentage-of-Completion Method allowed under the previous standard.
Revenue recognized at a point in time is similar to the Completed Contract Method allowed under the previous standard.

612
Q

In June, Northan Retailers sold refundable merchandise coupons. Northan received $10 for each coupon redeemable from July 1 to December 31 for merchandise with a retail price of $11. At June 30, how should Northan report these coupon transactions?
A
Unearned revenues at the merchandise’s retail price
B
Unearned revenues at the cash received amount
C
Revenues at the merchandise’s retail price
D
Revenues at the cash received amount

A

b

Explanation:
When the refundable merchandise coupons are sold, the sales price collected represents unearned revenue and an unearned revenue account is credited. Earned revenue will be recognized later when the coupons are redeemed.

613
Q
\_\_\_\_\_\_\_\_\_\_\_\_\_\_\_\_\_\_\_ is(are) used to alter the terms and conditions of recorded sales transactions to entice customers to accept the delivery of goods and services.
A
Side agreements
B
Channel stuffing
C
Related-party transactions
D
Trade loading
A

a

Explanation:
Side agreements are used to alter the terms and conditions of recorded sales transactions to entice customers to accept the delivery of goods and services. They may create obligations or contingencies relating to financing arrangements or to product installation or customization that may relieve the customer of some of the risks and rewards of ownership.

614
Q

Under ASC 606, cash collection is a critical event for income recognized

Over Time	at a Point in Time
A	No	No
B	Yes	Yes
C	No	Yes
D	Yes	No
A

Explanation:
The correct answer is (A).

Cash collection is neither a critical event for income recognized at a “Point in Time” or “Over Time” under ASC 606.

Performance Obligation Satisfied at a “point in time”:

If an obligation to perform is not fulfilled over time, the obligation to perform is fulfilled at a point in time:

  • Entity currently has a right to collect payment for the asset
  • The legal title for the asset is with the customer
  • Physical possession of the asset is transferred by the entity to the customer
  • Significant risks and rewards of ownership of the asset are with the customer
  • The asset has been accepted by the customer

Performance Obligation Satisfied “over time”:

An entity transfers control of goods or services over time and fulfills an obligation to perform and recognizes revenue over time if one of the following criteria is met:

  • Simultaneous receipt and consumption of customer benefits as and when the company performs.
  • The performance of the entity creates or enhances an asset controlled by the customer.
  • The performance of the entity does not create an asset with alternative use of the entity and the entity has an enforceable right to payment for the completed performance to date.

If the performance obligation is met by the “over time “ entity, the revenue is also recognized as “over time “ according to the following two methods:

  • Output Method: Revenue shall be recognized by the value of the goods and services transferred to the customer to date.
  • Input Method: The revenue is recognized on the basis of the efforts of the entity or its inputs.
615
Q

In year 2, May Corp. acquired land by paying $75,000 down and signing a note with a maturity value of $1,000,000. On the note’s due date, December 31, year 7, May owed $40,000 of accrued interest and $1,000,000 principal on the note. May was in financial difficulty and was unable to make any payments. May and the bank agreed to amend the note as follows:
The $40,000 of interest due on December 31, year 7, was forgiven.
The principal of the note was reduced from $1,000,000 to $950,000 and the maturity date extended 1 year to December 31, year 8.
May would be required to make one interest payment totaling $30,000 on December 31, year 8.
As a result of the troubled debt restructuring, May should report a gain, before taxes, in its year 7 income statement of
A
$40,000
B
$50,000
C
$60,000
D
$90,000

A

c

Explanation:
When the aggregate payments under new terms of an impaired loan are less than the amount of the obligation, the debtor records a gain equal to the difference between the carrying amount of the payable, including accrued interest, and the aggregate future cash payments required under the new terms. May Corp. is the debtor and owes $1,000,000 principal plus $40,000 of accrued interest. Under the new terms, May is required to pay future cash payments of $950,000 plus $30,000. The gain, before taxes, is $1,040,000 - $980,000 = $60,000. Note that the creditor (in this case, the bank) would use the present value, not the aggregate, of the future cash payments in its accounting for this loan.

616
Q
On January 1, Dell Inc. contracted with the city of Little to provide custom built desks for the city schools. The contract made Dell the city's sole supplier and required Dell to supply no fewer than 4,000 desks and no more than 5,500 desks per year for two years. In turn, Little agreed to pay a fixed price of $110 per desk. During the year, Dell produced 5,000 desks for Little. At December 31, 500 of these desks were segregated from the regular inventory and were accepted and awaiting pickup by Little. Little paid Dell $450,000 during the year. What amount should Dell recognize as contract revenue in this year?
A
$450,000
B
$495,000
C
$550,000
D
$605,000
A

c

Explanation:
The amount of cash paid by Little to Dell does not affect the amount of revenue to be recognized. The contract revenue should be accounted for under the completion of production method. Therefore, Dell should recognize the full amount of revenue pertaining to the desks produced in the year. Revenue should be recognized only for those units produced by year-end, despite the 5,500 maximum. Dell should recognize contract revenue of $550,000 (5,000 × $110) in the year. Dell properly recognizes the full amount of the contract revenue pertaining to the 5,000 desks produced in the year because (1) the number of desks produced is within the parameters of the contract for the year and (2) the earnings process is virtually complete.

617
Q

How would the proceeds from the advance sale of nonrefundable tickets for a dance performance be reported in the seller’s financial statements before the performance?

A
Revenue for the entire proceeds
B
Unearned revenue for the entire proceeds
C
Revenue to the extent of related costs expended
D
Unearned revenue to the extent of related costs expended
A

Explanation:
The correct answer is (B).

Proceeds from the advance sale of tickets for a theatrical performance should be reported as unearned revenue as the services/performances are not yet rendered. Revenues will be recognized when the product is delivered or services are rendered to customers. The fact that the tickets are non-refundable doesn’t impact the performance obligation. The company still has to perform.

Deferred revenue or unearned revenue is accounted when the cash is received, but revenue is not earned as the entity has not yet satisfied its performance obligation. It is accounted as a liability until the entity satisfies the performance obligation. Revenue should be recognized when it is realized and earned.

The 5-step approach to revenue recognition:

Step 1: Identify the Contract(s) with a customer.
Step 2: Identify the Performance obligations in the contract.
Step 3: Determine the transaction price (i.e., Amount).
Step 4: Allocate transaction Price to Performance obligations in the contract.
Step 5: Revenue Recognition when/as the entity satisfies a performance obligation.

618
Q

Lew Co. sold 200,000 corrugated boxes for $2 each. Lew’s cost was $1 per unit. The sales agreement gave the customer the right to return up to 60% of the boxes within the first six months, provided an appropriate reason was given. It was immediately determined, with appropriate reason, that 5% of the boxes would be returned. Lew absorbed an additional $10,000 to process the returns and expects to resell the boxes. What amount should Lew report as operating profit from this transaction?

A
$170,000
B
$179,500
C
$180,000
D
$200,000
A

Explanation:
The correct answer is (C).

When an entity sells goods on right to return basis, we recognize revenue and the corresponding cost of goods sold for the portion of goods not expected to be returned.

Estimated returns:
200,000 x 5% = 10,000 units
Net Sales = 200,000 – 10,000 = 190,000 units
Sales (190,000 x $2)	$380,000
Cost of Sales ($190,000 x $1)	  -190,000
Gross Profit	190,000
Less processing costs	-10,000
Operating Profit	180,000
619
Q
On October 20 of the current year, Grimm Co. consigned 40 freezers to Holden Co. for sale at $1,000 each and paid $800 in transportation costs. On December 30, Holden reported the sale of 10 freezers and remitted $8,500. The remittance was net of the agreed 15% commission. What amount should Grimm recognize as consignment sales revenue for the current year?
A
$ 7,700
B
$ 8,500
C
$ 9,800
D
$10,000
A

d

Explanation:
Consignment sales revenue is recognized at the time of sale. Thus, consignment sales revenue of $10,000 (10 freezers × $1,000 sales price) is reported in the current year. The transportation costs and the 15% commission are reported as selling expense; they are not netted against sales revenue.

Cash 8,500
Consignment sales commissions 1,500
Consignment sales revenues 10,000To record consignment sale

620
Q

If an entity pays or receives royalties, it should do which as the following?
A
Recognize royalty revenue and royalty expense under the rules of accrual accounting.
B
Not recognize royalties received in advance as revenue at the date of the royalty agreement.
C
Generally compute the amount of royalty revenue or expense for a period by multiplying the period’s sales applicable to the royalty agreement, by the royalty percentage.
D
All of the above.

A

d

Explanation:
Royalty revenue and royalty expense are recognized under the rules of accrual accounting, royalties received in advance are not recognized as revenue at the date of the royalty agreement, and an entity generally should compute the amount of royalty revenue or expense for a period by multiplying the period’s sales applicable to the royalty agreement by the royalty percentage. Since answers A., B., and C. are all correct, answer D., all of the above, is the best answer choice.

621
Q

When should an anticipated loss on be recognized when recognizing revenue for a project at a point in time, and recognizing revenue over time, respectively?

Over Time	Point in Time
A	Over life of project	Contract complete
B	Immediately	Contract complete
C	Over life of project	Immediately
D	Immediately	Immediately
A

d

Explanation:
Under the conservatism principle, the full amount of an anticipated loss must be recognized immediately under both revenue recognition methods. The recognition of an anticipated loss cannot be deferred to future periods under either method.

622
Q
Class Corp. maintains its accounting records on the cash basis but restates its financial statements to the accrual method of accounting. Class had $60,000 in cash-basis pretax income for year 2. The following information pertains to Class's operations for the years ended December 31, year 2 and year 1:
Year 2	Year 1
Accounts receivable	$40,000	$20,000
Accounts payable	15,000	30,000Under the accrual method, what amount of income before taxes should Class report in its December 31, year 2, income statement?
A
$25,000
B
$55,000
C
$65,000
D
$95,000
A

d

Explanation:
Accrual accounting (1) recognizes revenue in the period it is earned rather than only when the related cash is received and (2) recognizes expenses in the period incurred rather than only when the related cash is paid.

Cash basis pretax income, year 2 $60,000
Add: Increase in accounts receivable–revenues earned but not yet collected in cash ($40,000 - $20,000) 20,000
Add: Decrease in accounts payable–payments made not not representing current year expenses ($30,000 - $15,000) 15,000
Accrual basis pretax income, year 2 $95,000

623
Q
On October 1, year 1, Acme Fuel Co. sold 100,000 gallons of heating oil to Karn Co. at $3 per gallon. Fifty-thousand gallons were delivered on December 15, year 1, and the remaining 50,000 gallons were delivered on January 15, year 2. Payment terms were: 50% due on October 1, year 1, 25% due on first delivery, and the remaining 25% due on second delivery. What amount of revenue should Acme recognize from this sale during year 1?
A
$75,000
B
$150,000
C
$225,000
D
$300,000
A

b

Explanation:
Revenues are only considered earned when the entity has accomplished substantially what it must do to be entitled to the revenues. Acme has only delivered 50,000 gallons to Karn during year 1. Therefore, Acme should only recognize revenue of $150,000 (i.e., 50,000 gallons × $3) during year 1 from this sale.

624
Q

For $50 a month, Rawl Co. visits its customers’ premises and performs insect control services. If customers experience problems between regularly scheduled visits, Rawl makes service calls at no additional charge. Instead of paying monthly, customers may pay an annual fee of $540 in advance. For a customer who pays the annual fee in advance, Rawl should recognize the related revenue
A
When the cash is collected.
B
At the end of the fiscal year.
C
At the end of the contract year after all of the services have been performed.
D
Evenly over the contract year as the services are performed.

A

d

Explanation:
Accrual accounting recognizes revenue in the period(s) it is earned. Therefore, the revenue should be recognized evenly over the contract year as the services are performed.

625
Q

Tara Co. owns an office building and leases the offices under a variety of rental agreements involving rent paid in advance monthly or annually. Not all tenants make timely payments of their rent. Tara’s balance sheets contained the following data:

Year 1 Year 2

Rentals receivable $ 9,600 $12,400
Unearned rentals 32,000 24,000
During year 2, Tara received $80,000 cash from tenants. What amount of rental revenue should Tara record for year 2?

A
$90,800
B
$85,200
C
$74,800
D
$69,200
A

a

Explanation:
The $2,800 increase in rentals receivable represents rental revenue earned but not yet received in cash. The $8,000 decrease in unearned rentals represents rental revenue earned that had been received in cash in a prior period.

Cash (given)	$80,000
Rentals Receivable ($12,400 - $9,600)	2,800
Unearned Rentals ($32,000 - $24,000)	  8,000
Rental Revenue (to balance)	$90,800
626
Q

Which of the following is one of the criteria for recognizing revenue at a point in time?

A
Simultaneous receipt and consumption of customer benefits as and when the company performs.
B
The performance of the entity does not create an asset with an alternative use of the entity.
C
The performance of the entity creates or enhances an asset controlled by the customer.
D
Entity currently has a right to collect payment for the asset

A

Explanation:
Answer (D): Entity currently has a right to collect payment for the asset.

All other options are criteria to recognize revenue over time.

To recognize revenue at a point in time, the following criteria must be met:

  • Entity currently has a right to collect payment for the asset
  • The legal title for the asset is with the customer
  • Physical possession of the asset is transferred by the entity to the customer
  • Significant risks and rewards of ownership of the asset are with the customer
  • Asset has been accepted by the customer
627
Q

For a customer to have obtained control of a product in a bill-and-hold arrangement, which of the following criteria must be met:

i. Product must be identified separately as belonging to the customer
ii. Product must be ready for physical transfer to the customer
iii. Entity cannot use the product or direct it to another customer
iv. Substantive reason for the bill-and-hold arrangement

A
i, ii and iv
B
i and ii
C
i, ii, iii and iv
D
I,III, and iv
A

c

Explanation:
The correct answer is (C).

A bill-and-hold arrangement is a contract under which an entity bills a customer for a product but the entity retains (i.e., holds) physical possession of the product until it is transferred to the customer at a point in time in the future. Revenue should be recognized only when the customer obtains control of the product.

For a customer to have obtained control of a product in a bill-and-hold arrangement, all of the following criteria must be met:

i. Substantive reason for the bill-and-hold arrangement (e.g., the customer has requested the arrangement).
ii. Product must be identified separately as belonging to the customer.
iii. Product currently must be ready for physical transfer to the customer.
iv. Entity cannot use the product or direct it to another customer.

628
Q

Ninja Company reports under IFRS and has a calendar year end. Samurai Company purchases a service-type warranty from Ninja for $20,000. The warranty is effective for 5 years beginning January 1, 20X5. What is the balance in the unearned warranty revenue account on December 31, 20X8.

A
$16,000
B
$12,000
C
$8,000
D
$4,000
A

d

Explanation:
$4,000. The $20,000 warranty is for a 5 year period and must be recognized as unearned revenue at the inception of the contract. The warranty revenue can only be realized over the life of the warranty. So each year $4,000 of revenue can be recognized. Therefore on December 20X5 the balance in the unearned warranty revenue account will be $16,000. On December 31, 20X6 it will be $12,000. On December 31, 20X7 it will be $8,000. On December 31, 20X8 it will be $4,000 and finally on December 31, 20X9 it will be fully recognized.

629
Q

Under IFRS 15 – Revenue Recognition, which of the following is not a step in the revenue recognition process?

A
Identify the contract with a customer
B
Allocate the transaction price to the performance obligations in the contract
C
Identify the performance obligations in the contract
D
Confirm the contract complies with all applicable laws and regulations

A

d

Explanation:
The first steps of revenue recognition under IFRS 15 are: (1) Identify the contract with a customer; (2) Identify the performance obligations in the contract; (3) Determine the transaction price; (4) Allocate the transaction price to the performance obligations in the contract; (5) Recognize revenue when (or as) the entity satisfied a performance obligation. Therefore “D” is not a step in the revenue recognition process.

630
Q
The following information pertains to Eagle Co.'s current year sales:
Cash sales
Gross	$80,000
Returns and allowance	4,000
Credit sales
Gross	$120,000
Discounts	6,000On January 1 of the current year customers owed Eagle $40,000. On December 31, customers owed Eagle $30,000. Eagle uses the direct write-off method for bad debts. No bad debts were recorded in the year. Under the cash basis of accounting, what amount of net revenue should Eagle report for the current year?
A
$ 76,000
B
$170,000
C
$190,000
D
$200,000
A

d

Explanation:
The decrease in accounts receivable represents collections in the current period of credit sales of a prior period.

Gross cash sales	$ 80,000	
Less: Returns and allowances	  (4,000)	
Receipts from cash sales		$ 76,000
Gross credit sales	120,000	
Less: Discounts	  (6,000)	
Net credit sales	114,000	
Add: Decrease in accounts receivable ($40,000 - $30,000)	  10,000	
Receipts from credit sales		  124,000
Receipts from cash and credit sales		$200,000
631
Q

ABC Ltd. sells a product to a customer on December 1, Y1 for $121 that is payable 24 months after delivery. The customer obtains control of the product on December 1, Y1. The contract permits the customer to return the product within ninety days. The product is new and the entity has no relevant historical evidence of product returns or other available market evidence. The cash selling price of the product is $100, which represents the amount that a customer would pay upon delivery for the same product sold under otherwise identical terms and conditions as at contract inception. The entity’s cost of the product is $80. When will ABC recognize revenue for this transaction and for what amount?

A
December 1, Y1 for $100.
B
December 1, Y1 for $121.
C
March 1, Y2 for $100.
D
March 1, Y2 for $121.
A

c

Explanation:
The entity does not recognize revenue when control of the product transfers to the customer on December 1, Y1. This is because of the existence of the right of return and the lack of relevant historical evidence about the product returns. This means that the entity cannot for sure conclude that it is probable that a significant reversal in the amount of cumulative revenue recognized will not occur. Consequently, revenue is recognized only after ninety days on March 1, Y2 when the right of return lapses.

The contract includes a significant financing component. This is evident from the difference between the amount of promised consideration of $121 and the cash selling price of $100 at the date that the goods are transferred to the customer. The contract includes an implicit interest rate of 10 percent (that is, the interest rate that over 24 months discounts the promised consideration of $121 to the cash selling price of $100).

Thus, revenue is recognized on March 1, Y2 for $100. Until the entity receives the cash payment from the customer, interest income would be recognized consistently. The entity would accrete the receivable up to $121 from the time the right of return lapses until customer payment.

632
Q

Deer Co., which began operations on January 2, Year 1 recognizes revenue at a point in time in accordance with ASC 606. The performance by Deer Co. does not create an asset with an alternative use and Deer Co., has an enforceable right to payment for the completed performance to date. The following information is available for the year:

 	Year 1	Year 2
Total Contract Price	$300,000	$300,000
Costs Incurred	$70,000	$110,000
Estimated costs to complete at year end	$200,000	$72,000
What is Deer's revenue in Year 2?
A
$0
B
$7,778
C
$28,000
D
$300,000
A

Explanation:
The correct answer is (A).

Under the input method prescribed in ASC 606 that recognizes revenue at a point in time, gross profit is recognized only on completion of the contract. Since the project started in Year 1 and at the end of Year 2 $720,000 of the cost is yet to be incurred. The project is not complete and accordingly, no revenue will be recognized.

Note:

Revenue recognized over time is similar to the Percentage-of-Completion Method allowed under the previous standard.
Revenue recognized at a point in time is similar to the Completed Contract Method allowed under the previous standard.

633
Q

Which of the following statements regarding the milestone method of revenue recognition is true?
A
It is an application of the proportional performance method.
B
It is the only revenue recognition method that is available for research and development arrangements.
C
With the issuance of ASU 2010-17, reporting entities will not be able to utilize other revenue recognition methods.
D
All of the above are true.

A

Explanation:

The milestone method of revenue recognition is, at its core, an application of the proportional performance method.

634
Q

Barr Corp. started a long-term construction project in the current year. The following data relate to this project:

Contract price $4,200,000
Costs incurred in the year $1,750,000
Estimated costs to complete $1,750,000
Progress billings $900,000
Collections on progress billings $800,000
The project is accounted for in accordance with the input method prescribed in ASC 606 and revenue is recognized over time. It should be assumed that all costs incurred to date, as well as all estimated future costs, will advance the progress of the project and no spoilage or waste has occurred.

Additionally, the performance by Barr does not create an asset with an alternative use and Barr has an enforceable right to payment for the completed performance to date.

In Barr’s current year income statement, what amount of gross profit should be reported for this project?

A
$350,000
B
$150,000
C
$133,333
D
$100,000
A

Explanation:
The correct answer is (A)

Since revenue is recognized over time, the performance by Barr does not create an asset with an alternative use, and Barr has an enforceable right to payment for the completed performance to date, the gross profit recognized in the first year of the contract is determined by multiplying the estimated total gross profit of the , by the estimated percentage of completion.

Contract price	 	$4,200,000
Costs incurred to date	$1,750,000	 
Estimated costs to complete	$1,750,000	 
Less estimated total costs	 	$(3,500,000)
Estimated total gross profit	 	$700,000
Costs incurred to date	$1,750,000	 
Estimated total costs	/$3,500,000	 
Times estimated % of completion	 	×         50%
Gross profit recognized	 	$350,000
635
Q

ABC Ltd., after considering many companies, signed on January 1, Y1, a two year contract with S&S entertainers to provide entertainment during any of the events organized by the company during the period of the contract. On March 1, Y1, ABC was holding a 5 day international conference. The following were the costs incurred by S&S entertainers in relation to the contract:

Cost to print pamphlets to obtain contract - $750.
Travel cost of the marketing team to ABC Ltd. to obtain the contract - $1,000.
Legal fees for drafting the contract - $250.
Sales commissions to marketing department - $1,000.
Building a removable stage, that could be used for all future programs organized by S&S - $1,200.
Making props for the show which could be used for any future programs of S&S - $800.
Printing brochures about S&S to be distributed to members of the conference - $750.
Salary of employee employed primarily to organize events at ABC. This employee could also be used for other shows of S&S - $2,250.
ABC is confident that the costs incurred could be recovered from the price charged for performance. What is the cost that should be capitalized?

A
$3,250
B
$2,450
C
$5,250
D
$3,000
A

Explanation:
The correct answer is Option (A).

Some costs incurred to obtain or fulfill a contract with a customer are capitalized along with cost incurred in fulfilling a contract.

A. Incremental costs of obtaining a contract are the costs that the entity would not have incurred if the contract had not been obtained. The incremental costs incurred by S&S were:

Particulars

Amount

Cost to print pamphlets to obtain contract $750
Travel cost of the marketing team to ABC Ltd. to obtain the contract $1,000
Legal fees for drafting the contract $250
Sales commissions to marketing department $1,000
However, only costs incurred because the contract had been obtained has to be capitalized. The cost of pamphlets and travel cost of marketing department would have been incurred even if the contract would not have been obtained. Thus, only the legal fee to draft the contract and the sales commissions given because of obtaining the contract has to be capitalized (i.e. $250 + $1,000) = $1,250.

B. Costs incurred in fulfilling a contract are capitalized if:

Relate directly to a contract (or a specific anticipated contract).
Generate or enhance resources of the entity that will be used in satisfying performance obligations in the future.
Are expected to be recovered.
S&S has to consider the following costs to determine if it has to be capitalized.

Particulars

Amount

Rationale

Building a removable stage, that could be used for all future programs organized by S&S $1,200 This has to be capitalized as this relates directly to the contact to provide entertainment for the conference and also enhances the resources of S&S as this can be used for all the entertainment programs organized by S&S during the two year contract with ABC.
Making props for the show which could be used for any future programs of S&S $800 This also has to be capitalized as this relates directly to the contact to provide entertainment for the conference and also enhances the resources of S&S as this can be used for all the entertainment programs organized by S&S for ABC or outside
Printing brochures about S&S to be distributed to members of the conference $750 This should not be capitalized as this does not relate to the contract . This is a mere advertisement for S&S to obtain future business
Salary of employee employed primarily to organize events at ABC $2,250 This employee could also be used for other shows of S&S. Though the employee salary is incurred to organize the entertainment and relates directly to the contract, these costs do not generate or enhance resources of the S&S, thus, should not be capitalized and must be recognized as payroll expense when incurred
Total cost incurred while fulfilling the contract that has to be capitalized = $1,200 + $800 = $2,000.

Total cost to be capitalized = Incremental cost of $1,250 + Cost to fulfill the contract $2,000 = $3,250.

Options (B), (C) and (D) are incorrect based on the above explanation as these use a different combination of the costs incurred by S&S.

636
Q
Anchor Co. is experiencing financial difficulties. Anchor negotiated a settlement of $100,000 in debt owed to Bowden, Inc. in exchange for Anchor’s gross receivables of $100,000. The receivables have an allowance for uncollectible accounts of $25,000. The impact of this transaction on Anchor’s net income is a $25,000
A
Increase in bad debt expense
B
Gain on restructuring of payables
C
Loss on restructuring of payables
D
Decrease in bad debt expense
A

Explanation:
To account for transfer of goods with a right of return (and for some services that are provided subject to a refund), an entity should recognize all of the following: Revenue for the transferred products in the amount of consideration to which the entity expects to be entitled; therefore, revenue would not be recognized for products expected to be returned. Revenue cannot be recognized at the time of sale because neither collection of payment from the buyer is reasonably assured nor can the company reasonably estimate the returns. The buyer’s right to return expires only after 12 months after which revenue can be recognized. The right to exchange goods is not a consideration for revenue recognition as an exchange does not affect the amount of collection. Therefore Aqua Co. will recognize revenue after twelve months after date of sale.

637
Q
Seco Corp. was forced into bankruptcy and is in the process of liquidating assets and paying claims. Unsecured claims will be paid at the rate of forty cents on the dollar. Hale holds a $30,000 noninterest-bearing note receivable from Seco collateralized by an asset with a book value of $35,000 and a liquidation value of $5,000. The amount to be realized by Hale on this note is
A
$ 5,000
B
$12,000
C
$15,000
D
$17,000
A

8 907 2J Revenue Recognition Incorrect 00:00:02
9 903 2J Revenue Recognition Incorrect 00:00:04
10 512 2J Revenue Recognition Incorrect 00:00:09
11 901 2J Revenue Recognition Correct 00:00:03
12 567 2J Revenue Recognition Incorrect 00:00:03
13 914 2J Revenue Recognition Incorrect 00:00:11
14 939 2J Revenue Recognition Incorrect 00:00:02
15 930 2J Revenue Recognition Incorrect 00:00:01
16 3642 2J Revenue Recognition Incorrect 00:00:02
17 876 2J Revenue Recognition Incorrect 00:00:02
18 874 2J Revenue Recognition Correct 00:00:02
19 908 2J Revenue Recognition Incorrect 00:00:02
20 883 2J Revenue Recognition Correct 00:00:04
21 919 2J Revenue Recognition Incorrect 00:00:03
22 932 2J Revenue Recognition Incorrect 00:00:02
23 933 2J Revenue Recognition Incorrect 00:00:02
24 923 2J Revenue Recognition Correct 00:00:02
25 940 2J Revenue Recognition Incorrect 00:00:02
26 955 2J Revenue Recognition Incorrect 00:00:08
27 1504 2J Revenue Recognition Incorrect 00:00:02
28 3518 2J Revenue Recognition Incorrect 00:00:01
29 948 2J Revenue Recognition Incorrect 00:00:04
30 928 2J Revenue Recognition Incorrect 00:00:01
31 557 2J Revenue Recognition Correct 00:00:03
32 960 2J Revenue Recognition Incorrect 00:00:02
33 946 2J Revenue Recognition Incorrect 00:00:02
34 598 2J Revenue Recognition Incorrect 00:00:02
35 952 2J Revenue Recognition Correct 00:00:01
36 942 2J Revenue Recognition Incorrect 00:00:03
37 954 2J Revenue Recognition Incorrect 00:00:02
38 897 2J Revenue Recognition Correct 00:00:02
39 910 2J Revenue Recognition Incorrect 00:00:02
40 925 2J Revenue Recognition Incorrect 00:00:02
41 878 2J Revenue Recognition Incorrect 00:00:02
42 1005 2J Revenue Recognition Correct 00:00:25
43 867 2J Revenue Recognition Incorrect 00:00:02
44 864 2J Revenue Recognition Incorrect 00:00:03
45 922 2J Revenue Recognition Correct 00:00:25
46 937 2J Revenue Recognition Incorrect 00:00:02
47 3552 2J Revenue Recognition Incorrect 00:00:02
48 3551 2J Revenue Recognition Correct 00:00:02
49 882 2J Revenue Recognition Incorrect 00:00:05
50 902 2J Revenue Recognition Correct 00:00:01
51 934 2J Revenue Recognition Incorrect 00:00:03
52 950 2J Revenue Recognition Incorrect 00:00:02
53 911 2J Revenue Recognition Correct 00:00:19
54 3517 2J Revenue Recognition Incorrect 00:00:03
55 600 2J Revenue Recognition Incorrect 00:00:03
56 894 2J Revenue Recognition Incorrect 00:00:02
57 604 2J Revenue Recognition Correct 00:00:01
58 926 2J Revenue Recognition Incorrect 00:00:03
59 941 2J Revenue Recognition Correct 00:00:03
60 917 2J Revenue Recognition Incorrect 00:00:02
61 945 2J Revenue Recognition Incorrect 00:00:04
62 913 2J Revenue Recognition Incorrect 00:00:03
63 957 2J Revenue Recognition Incorrect 00:00:03
64 3520 2J Revenue Recognition Correct 00:00:02
65 912 2J Revenue Recognition Correct 00:00:04
66 920 2J Revenue Recognition Incorrect 00:00:19
67 921 2J Revenue Recognition Incorrect 00:00:02
68 595 2J Revenue Recognition Incorrect 00:00:01
69 916 2J Revenue Recognition Incorrect 00:00:02
70 509 2J Revenue Recognition Incorrect 00:00:02
71 909 2J Revenue Recognition Correct 00:00:03
72 918 2J Revenue Recognition Incorrect 00:00:02
73 936 2J Revenue Recognition Incorrect 00:00:03
74 927 2J Revenue Recognition Incorrect 00:00:05
75 924 2J Revenue Recognition Incorrect 00:00:05
76 872 2J Revenue Recognition Incorrect 00:00:02
77 3516 2J Revenue Recognition Correct 00:00:02
78 947 2J Revenue Recognition Incorrect 00:00:04
79 935 2J Revenue Recognition Incorrect 00:00:02
80 601 2J Revenue Recognition Incorrect 00:00:02
81 949 2J Revenue Recognition Incorrect 00:00:02
82 953 2J Revenue Recognition Incorrect 00:00:03
83 592 2J Revenue Recognition Correct 00:00:02
84 3508 2J Revenue Recognition Incorrect 00:00:02
85 938 2J Revenue Recognition Correct 00:00:05
86 3569 2J Revenue Recognition Correct 00:00:03
87 944 2J Revenue Recognition Incorrect 00:00:03
88 873 2J Revenue Recognition Correct 00:00:02
89 931 2J Revenue Recognition Incorrect 00:00:02
90 951 2J Revenue Recognition Incorrect 00:00:02
51
52
53
54
55
56
57
58
59
60
Question #604, Blueprint Area: Revenue Recognition
Seco Corp. was forced into bankruptcy and is in the process of liquidating assets and paying claims. Unsecured claims will be paid at the rate of forty cents on the dollar. Hale holds a $30,000 noninterest-bearing note receivable from Seco collateralized by an asset with a book value of $35,000 and a liquidation value of $5,000. The amount to be realized by Hale on this note is
A
$ 5,000
B
$12,000
C
$15,000
D
$17,000

638
Q

A construction company recognizes revenue at a point in time. Which of the following is not one of the criteria for recognizing revenue at a point in time?

A
The legal title for the asset is with the customer
B
The performance of the entity creates or enhances an asset controlled by the customer
C
Physical possession of the asset is transferred by the entity to the customer
D
Significant risks and rewards of ownership of the asset are with the customer

A

Explanation:
Answer (B): The performance of the entity creates or enhances an asset controlled by the customer is one of the criteria for recognizing revenue over time, not a point in time.

To recognize revenue at a point in time, the following criteria must be met:

  • Entity currently has a right to collect payment for the asset
  • The legal title for the asset is with the customer
  • Physical possession of the asset is transferred by the entity to the customer
  • Significant risks and rewards of ownership of the asset are with the customer
  • Asset has been accepted by the customer
639
Q

When would a company recognize revenue over time?

A
When collectability of payments is not predictable
B
When repossessions of merchandise sold on the Percentage of Completion may result in a future gain or loss.
C
When costs to complete and estimates of progress towards completion cannot be estimated.
D
When costs to complete and estimates of progress towards completion are reasonably dependable

A

Explanation:
The correct answer is (D).

The Input Method prescribed in ASC 606 recognizes revenue over time for long term contracts and is used when costs to complete and estimates of progress towards completion are reasonably dependable.

Revenue is recognized on each period in the contract on an accrual basis in proportion to cost incurred.

Note:

Revenue recognized over time is similar to the Percentage-of-Completion Method allowed under the previous standard.
Revenue recognized at a point in time is similar to the Completed Contract Method allowed under the previous standard.

640
Q

Haft Construction Co. uses the input method prescribed in ASC 606 and revenue is recognized over time. On January 10, year 1, Haft began work on a $3,000,000 construction contract. At the inception date, the estimated cost of construction was $2,250,000. The following data relate to the progress of the contract:

Income recognized at 12/31, year 1 $ 300,000
Costs incurred 1/10, year 1 through 12/31, year 2 1,800,000
Estimated cost to complete at 12/31, year 2 600,000
It should be assumed that all costs incurred to date, as well as all estimated future costs, will advance the progress of the project and no spoilage or waste has occurred.

Additionally, the performance by Haft does not create an asset with an alternative use and Haft has an enforceable right to payment for the completed performance to date.

In its income statement for the year ended December 31, year 2, what amount of gross profit should Haft report?

A
$450,000
B
$300,000
C
$262,500
D
$150,000
A

Explanation:
The correct answer is (D)

Given the facts, the gross profit recognized in the second year of the contract is determined by multiplying the estimated total gross profit of the contract by the estimated percentage of completion and then subtracting the gross profit recognized in the first year.

Contract price $3,000,000
Costs incurred to date $1,800,000
Estimated cost to complete 600,000
Less estimated total costs (2,400,000)
Estimated total gross profit $600,000
Costs incurred to date $1,800,000
Estimated total costs / 2,400,000
Times estimated % of completion × 75%
Gross profit recognizable to date $ 450,000
Less: Gross profit previously recognized 300,000
Gross profit recognized in year 2 $ 150,000
Note:

Revenue recognized over time is similar to the Percentage-of-Completion Method allowed under the previous standard.
Revenue recognized at a point in time is similar to the Completed Contract Method allowed under the previous standard.

641
Q
Several of Fox, Inc.'s customers are having cash flow problems. Information pertaining to these customers for the years ended March 31, year 1 and year 2, follows:
3/31, year 1	3/31, year 2
Sales	$10,000	$15,000
Cost of sales	8,000	9,000
Cash collections		
on year 1 sales	7,000	3,000
on year 2 sales	---	12,000If the cost recovery method is used, what amount would Fox report as gross profit from sales to these customers for the year ended March 31, year 2?
A
$ 2,000
B
$ 3,000
C
$ 5,000
D
$15,000
A

Explanation:
The cost recovery method defers the recognition of all gross profit until cash collections of revenue are equal to the cost of the item sold; all remaining cash collections are recorded as gross profit. No profit was recognized on the year 1 sales for the year ended 3/31, year 1 because the $7,000 collected in the period did not recover the $8,000 cost of sales. For the year ended 3/31, year 2: (1) collections to date on the year 1 sales were $10,000 (i.e., $7,000 + $3,000), permitting the recognition of $2,000 profit above the year 1 cost of sales of $8,000 and (2) collections on the year 2 sales were $12,000, permitting the recognition of $3,000 profit above the year 2 cost of sales of $9,000. Therefore, the amount of gross profit that Fox should recognize for the year ended 3/31, year 2 is $5,000 (i.e., $2,000 + $3,000).

642
Q

Which of the following is not one of the benefits of the revenue recognition standard under ASC 606:

A
Enhances the interchangeability of revenue recognition practices.
B
Eliminates inconsistencies and weaknesses of existing revenue requirements
C
Provides a more robust revenue management framework
D
Simplifies the preparation of financial statements by reducing the number of requirements referred to by an organization.

A

Explanation:
Answer (A): Enhances the interchangeability of revenue recognition practices.

ASC 606 enhances the comparability of revenue recognition practices.

5 Benefits of the revenue recognition standard under ASC 606:

  • Eliminates inconsistencies and weaknesses of existing revenue requirements
  • Provides a more robust revenue management framework
  • Enhances the comparability of revenue recognition practices.
  • Provides users of financial statements with more useful information through improved disclosure requirements
  • Simplifies the preparation of financial statements by reducing the number of requirements referred to by an organization.
643
Q
A shoe retailer allows customers to return shoes within 90 days of purchase. The company estimates that 5% of sales will be returned within the 90-day period. During the month, the company has sales of $200,000 and returns of sales made in prior months of $5,000. What amount should the company record as net sales revenue for new sales made during the month?
A
$185,000
B
$190,000
C
$195,000
D
$200,000
A

Explanation:
Revenue from sales when the buyer has the right to return the product should be recognized at the time of sale only if all of several conditions are met. Since the amount of returns can be estimated based on past experience, the amount reported for net sales should be reduced for the 5% of sales expected to be returned within 90 days. Net sales would not be reduced for returns from prior months, as that sales revenue would have already been reduced in compliance with the matching concept.

Sales, current month $200,000
Less: Estimated sales that will be returned [$200,000 × 5%] 10,000
Net sales, current month $190,000

644
Q

A retail store sold gift certificates that are redeemable in merchandise. The gift certificates lapse one year after they are issued. How would the deferred revenue account be affected by each of the following?

Redemption of certificates:	Lapse of certificates:
A	Decrease	Decrease
B	Decrease	No effect
C	No effect	Decrease
D	No effect	No effect
A

Explanation:
Upon receipt of a prepayment from a customer, an entity should recognize a contract liability (deferred revenue in this case). Thereafter, entity should derecognize that contract liability (and recognize revenue) when it transfers those goods or services and, therefore, satisfies its performance obligation. The deferred revenue account is credited when gift certificates are sold. When the certificates are redeemed, sales revenues are recognized by debiting deferred revenues. When certificates lapse, deferred revenues are reduced as they are not a liability anymore.

645
Q

Lake Construction Company has consistently used the input method prescribed in ASC 606 and recognizes revenue over time. During year 1, Lake entered into a fixed-price contract to construct an office building for $10,000,000. Information relating to the contract is as follows:

At December 31
Year 1 Year 2

Percentage of completion 20% 60%
Estimated total cost at completion $7,500,000 $8,000,000
Income recognized (cumulative) 500,000 1,200,000

The performance by Lake does not create an asset with an alternative use and Lake has an enforceable right to payment for the completed performance to date.

Contract costs incurred during year 2 were

A
$3,200,000
B
$3,300,000
C
$3,500,000
D
$4,800,000
A

Explanation:
The correct answer is (B)

Year 1 Year 2

Contract price $10,000 $10,000
Estimated total cost at completion (7,500) (8,000)
Estimated gross profit $ 2,500 $ 2,000

Estimated total cost at completion $ 7,500 $ 8,000
GP recognized to date $ 500 $ 1,200
Estimated total GP / 2,500 / 2,000
Percentage of completion × 20% × 60%
Contract costs to date 1,500 4,800
Contract costs, prior years (0) (1,500)
Contract costs, current year $ 1,500 $ 3,300
Note:

Revenue recognized over time is similar to the Percentage-of-Completion Method allowed under the previous standard.
Revenue recognized at a point in time is similar to the Completed Contract Method allowed under the previous standard.

646
Q

When an entity incurs costs in fulfilling a contract with a customer, which of the following is one of the criteria for recognizing those costs as an asset?

A
The costs should be recovered.
B
The costs generate or enhance the entity's resources to fulfill performance obligations in the future.
C
The costs pertain directly to a contract or to an expected contract.
D
All of the above.
A

Explanation:
The correct answer is (D).

Costs incurred in fulfilling the contract should be recognized as an asset only if they meet all the following criteria:

  • The costs pertain directly to a contract or to an expected contract.
  • The costs generate or enhance the entity’s resources to fulfill performance obligations in the future.
  • The costs should be recovered.
647
Q

Frame construction company’s contract requires the construction of a bridge in three years. The expected total cost of the bridge is $2,000,000, and Frame will receive $2,500,000 for the project. The actual costs incurred to complete the project were $500,000, $900,000, and $600,000, respectively, during each of the three years. Progress payments received by Frame were $600,000, $1,200,000, and $700,000, respectively.

Frame uses the input method prescribed in ASC 606 that recognizes revenue over time. It should be assumed that all costs incurred to date, as well as all estimated future costs, will advance the progress of the project and no spoilage or waste has occurred.

Additionally, the performance by Frame does not create an asset with an alternative use and Frame has an enforceable right to payment for the completed performance to date.

What amount of gross profit would Frame report during the last year of the project?

A
$120,000
B
$125,000
C
$140,000
D
$150,000
A

Explanation:
The correct answer is (D)

Given the facts, in the final year of the contract, the final revenue recognition would take place. The calculation would be the total revenue earned over the entire contract less the actual total costs incurred less the revenue previously recognized.

Contract price $2,500,000
Costs incurred to date (2,000,000)
Estimated total gross profit $ 500,000
Costs incurred to date $2,000,000
Estimated total costs / 2,000,000
Times estimated % of completion × 100%
Gross profit recognizable to date $ 500,000
Less: Gross profit recognized in year 1 & 2 350,000
Gross profit recognized in year 3 $ 150,000
Gross Profit recognized in year 1 & 2:

(Cost Incurred to Date/Estimated Total Cost) x Estimated Total Gross Profit = Gross Profit Recognized.
($1,400,000/$2,000,000) x $500,000 = 70% x $500,000 = $350,000
Note:

Revenue recognized over time is similar to the Percentage-of-Completion Method allowed under the previous standard.
Revenue recognized at a point in time is similar to the Completed Contract Method allowed under the previous standard.

648
Q

Hull Company is indebted to Apex under a $500,000, 12%, three-year note dated December 31, Year 2. Because of Hull’s financial difficulties developing in year 3, Hull owed accrued interest of $60,000 on the note at December 31, Year 4. Hull restructured debt in year 3 on a loan to another creditor. Under a troubled debt restructuring, on December 31, Year 4, Apex agreed to settle the note and accrued interest for a tract of land having a fair value of $450,000.Hull’s acquisition cost of the land is $360,000. Ignoring income taxes, on its year 4 income statement Hull should report as a result of the troubled debt restructuring

Income from Continuing Operations	Income from Discontinued Operations
A	$200,000	$0
B	$140,000	$0
C	$ 90,000	$ 50,000
D	$ 90,000	$110,000
A

Explanation:
The correct answer is (A).

The debtor recognizes a gain or loss equal to the difference between the fair value and the carrying amount of the asset transferred.

The amount is reported in income from continuing operations.

The debtor recognizes a gain on the troubled debt restructuring equal to the difference between the carrying amount of the obligation settled and the fair value of the asset transferred. Both the ordinary gain and gain on troubled debt restructuring is reported as income from continuing operations at $200,000 ($110,000 + $90,000).

Debt principal $500,000
Add: Accrued interest at restructure date 60,000
Carrying amount of obligation at restructure date $560,000
Less fair value of land transferred (450,000)
Gain on restructure $110,000

Fair value of land transferred $450,000
Less: Carrying amount of the land (360,000)
Gain on disposition $ 90,000

649
Q

This question is based on the following data pertaining to Pell Co.’s construction jobs, which commenced during the current year:

Project 1 Project 2

Contract price $420,000 $300,000
Cost incurred during the year 240,000 280,000
Estimated costs to complete 120,000 40,000
Billed to customers during the year 150,000 270,000
Received from customers during year 90,000 250,000
Pell recognizes revenue over time in accordance with ASC 606. It should be assumed that all costs incurred to date, as well as all estimated future costs, will advance the progress of the project and no spoilage or waste has occurred.

Additionally, the performance by Pell does not create an asset with an alternative use and Pell has an enforceable right to payment for the completed performance to date.

What amount of gross profit (loss) would Pell report in its current year income statement?

A
($20,000)
B
$40,000
C
$22,500
D
$20,000
A

Explanation:
The correct answer is (D)

Given the facts, Pell recognizes a gross profit of $20,000 on the two projects because (1) $40,000 of the estimated gross profit on Project 1 is recognized in the year, and (2) the full amount of the $20,000 anticipated loss on Project 2 is recognized in the year because no portion of the anticipated loss can be deferred to future periods.

Project 1 Project 2

Contract price $420,000 $300,000
Costs incurred to date $240,000 $280,000
Estimated costs to complete 120,000 40,000
Estimated total costs (360,000) 320,000
Estimated total gross profit (loss) $60,000 ($20,000)
Costs incurred to date $240,000
Estimated total costs / 360,000
Percentage-of-completion × 2/3
Gross profit recognized in the year $ 40,000
Note:

Revenue recognized over time is similar to the Percentage-of-Completion Method allowed under the previous standard.
Revenue recognized at a point in time is similar to the Completed Contract Method allowed under the previous standard.

650
Q

Winn Co. sells subscriptions to a specialized directory that is published semiannually and shipped to subscribers on April 15 and October 15. Subscriptions received after the March 31 and September 30 cutoff dates are held for the next publication. Cash from subscribers is received evenly during the year and is credited to deferred subscription revenue. Data relating to the current year are as follows:
Deferred subscription revenue 1/1 $ 75,000
Cash receipts from subscribers 3,600,000In its current year December 31 balance sheet, Winn should report deferred subscription revenue of
A
$2,700,000
B
$1,800,000
C
$1,650,000
D
$ 900,000

A

Explanation:
Because the deferred subscription revenues at 12/31 of the previous year were earned in the current year, they do not affect the computation of deferred subscription revenue at 12/31 of the current year.

Cash receipts from customers during the year $3,600,000
Portion received after 9/30 cutoff date to be earned in the future × 3/12
Deferred subscription revenue at 12/31 $ 900,000

651
Q
During 2002, Fleet Co.'s trademark was licensed to Hitch Corp. for royalties of 10% of net sales of the trademarked items. Returns were estimated to be 1% of gross sales. On signing the licensing agreement, Hitch paid Fleet $75,000 as an advance against future royalty earnings. Gross sales of the trademarked items during the year were $600,000. What amount should Fleet report as royalty income for 2002?
A
$54,000
B
$59,400
C
$60,000
D
$75,000
A

Explanation:
Royalty revenue is recognized in the period(s) the royalties are earned. The $75,000 is an advance and has not yet been earned. There were $600,000 in gross sales with $6,000 (1% of gross sales) estimated for returns leaving a total of net sales at $594,000. With royalties calculated at 10% of net sales there would be $59,400 ($594,000 × 0.10) earned and reported as royalty income for the year.

652
Q

Which of the following is one of the criteria for recognizing revenue at a point in time?

A
Simultaneous receipt and consumption of customer benefits as and when the company performs.
B
The performance of the entity creates or enhances an asset controlled by the customer.
C
The performance of the entity does not create an asset with an alternative use of the entity
D
None of the above.

A

Explanation:
The correct answer is (D).

All of the answer choices are criteria for recognizing revenue over time, not a point in time.

To recognize revenue at a point in time, the following criteria must be met:

  • Entity currently has a right to collect payment for the asset
  • The legal title for the asset is with the customer
  • Physical possession of the asset is transferred by the entity to the customer
  • Significant risks and rewards of ownership of the asset are with the customer
  • Asset has been accepted by the customer
653
Q

According to the input method prescribed in ASC 606 and recognizes revenue over time, gross profit on contracts are to be recognized in income

A
On the date of sale
B
On the date the final cash collection is received
C
In proportion to the satisfaction of the performance obligation
D
After cash collections equal to the cost of sales have been received

A

Explanation:
The correct answer is (C).

An entity that recognizes revenue over time allows revenue to be deferred and recognized each year in proportion to the work done during that year.

Note:

Revenue recognized over time is similar to the Percentage-of-Completion Method allowed under the previous standard.
Revenue recognized at a point in time is similar to the Completed Contract Method allowed under the previous standard.

654
Q

The following data pertains to Pell Co.’s construction jobs, which commenced during the current year:

Project 1 Project 2

Contact price $420,000 $300,000
Cost incurred during the year 240,000 280,000
Estimated costs to complete 120,000 40,000
Billed to customers during the year 150,000 270,000
Received from customers during year 90,000 250,000
If Pell recognizes revenue at a point in time, and the performance obligations were not fulfilled in the current year, what amount of gross profit (loss) would Pell report in its current year income statement?

A
$(20,000)
B
$0
C
$340,000
D
$420,000
A

Explanation:
If revenue is recognized at a point in time, Pell recognizes a gross loss of $20,000 on the two pro­jects because:

(1) no portion of the $60,000 estimated gross profit on Project 1 can be recognized in the current year since the performance obligation has not been satisfied:

  • Entity currently has a right to collect payment for the asset
  • The legal title for the asset is with the customer
  • Physical possession of the asset is transferred by the entity to the customer
  • Significant risks and rewards of ownership of the asset is with the customer
  • Asset has been accepted by the customer

(2) the full amount of the $20,000 anticipated loss on Pro­ject 2 must be recognized in the current year because the anticipated loss cannot be deferred to future periods. Amounts below have been rounded (000s).

Project 1 Project 2

Contract price $420,000 $300,000
Cost incurred to date $240,000 $280,000
Less: Estimated costs to complete 120,000 40,000
Less: Estimated total costs (360,000) (320,000)
Estimated total gross profit $ 60,000 $(20,000)

655
Q

During year 1, Mitchell Corp. started a construction job with a total contract price of $600,000. The job was completed on December 15, year 2. Additional data are as follows:

 	Year 1	Year 2
Actual costs incurred	$225,000	$255,000
Estimated remaining costs	225,000	---
Billed to customer	240,000	360,000
Received from customer	200,000	400,000
Mitchell recognizes revenue for the project at a point in time and did not satisfy any of the performance obligations for revenue recognition in year 1. What amount should Mitchell recognize as gross profit for year 2?
A
$45,000
B
$72,000
C
$80,000
D
$120,000
A

d

Explanation:
At 12/31, year 1, the estimated total gross profit of the contract is $150,000, determined as follows:

Contract price $600,000
Estimated total costs:
Actual costs incurred $225,000
Estimated cost to complete 225,000 (450,000)
Estimated total gross profit $150,000
Because none of the performance obligations were satisfied in year 1, no portion of the estimated gross profit can be recognized in year 1. The full amount of the gross profit of the contract should be recognized in year 2, the final year of the contract.

Contract price $600,000
Actual cost incurred ($225,000 + $255,000) (480,000)
Total gross profit of contract $120,000
The amounts billed to and received from the customer do not enter into the computation of the amount of gross profit recognized from the contract.

656
Q

UVW Broadcast Co. entered into a contract to exchange unsold advertising time for travel and lodging services with Hotel Co. As of June 30, advertising commercials of $10,000 were used. However, travel and lodging services were not provided. How should UVW account for advertising in its June 30 financial state­ments?

A
Revenue and expense is recognized when the agreement is complete
B
An asset and revenue for $10,000 is recognized
C
Both the revenue and expense of $10,000 are recognized
D
Not reported

A

b

Explanation:
Accrual accounting recognizes and reports the effects of transactions and other events on the assets and liabilities of a business enterprise in the time periods to which they relate rather than only when cash is received or paid. Revenues are recognized when earned. A deferred (prepaid) expense is an expense paid or payable in cash but not yet incurred. A prepaid expense is reported in the financial statements as an asset.

657
Q

XYZ Ltd. enters into a three years extended warranty service contract. The payment terms are $100,000 payable annually in advance This amount is standalone selling price of services at contract inception. At the beginning of third year (after the customer had paid the $100,000 for that year), the entity agrees to reduce the price for third year of service to $80,000. In addition, the customer agrees to pay an additional $220,000 for an extension of the contract for three years. The standalone selling price of the services at the beginning of the third year is $80,000 per year. How should the contract modification be reflected in third year?

A
Modification of the existing contract with revenue recognized of $73,333 per year.
B
Termination of the existing contract and creation of a new contract with revenue of $75,000 per year.
C
Separate contract with revenue of $80,000 per year.
D
Contract modification should be reflected only in the fourth year from the new warranty period.

A

c

Explanation:
Contract modifications is a change in the scope or price (or both) of a contract that is approved by the parties to the contract. This is to be accounted for as a separate contract if BOTH of the following conditions are present:

Scope of contract increases due to promise of additional goods / services that are distinct.
Price of contract increases by an amount of consideration that reflects the entity’s standalone selling prices (less any adjustments / discounts) of the additional goods / services.
A good or service is distinct if both of the following criteria are met:

Capable of being distinct - Customer can benefit from the good / service either on its own or together with other resources that are readily available to the customer.
Distinct within the context of the contract - The promise to transfer the good / service is separately identifiable from other promises in the contract.
The extension of the contract is deemed to be distinct because the customer will benefit from the extended warranty service separately and this service is separately identifiable.

The standalone selling price of the contract per year is $80,000 at the beginning of third year when the warranty period is extended. Thus, the standalone selling price of the contract for three years would be $240,000 (i.e. $80,000 x 3). The contract has been modified and the price payable for the third year of the existing contract is $80,000. The customer however has already paid $100,000, that is $20,000 more. This $20,000 could be adjusted in the future payments due from the customer. Under the extension of the contract for another three years the customer agrees to pay $220,000. This would be the standalone price of the contract (i.e. $240,000 less the $20,000 paid extra in the third year)

As both the above conditions are satisfied the contract is a separate contract with revenue of $80,000 recognized each year.

(A) is incorrect because the contact is treated as a modification if the increase in the scope of the contract is not treated as distinct

(B) is incorrect because termination of existing contract and creating a new contract will be done only if the extended contract does not reflect the standalone price of the service. If such is the case the revenue would be recognized for the remaining agreed price of the contract divided by the remaining period = ($80,000 + $220,000) / 4 = $75,000.

(D) is incorrect because the changes to the contract are to be recognized in the year of change

658
Q

Deer Co., which began operations on January 2, Year 1 recognizes revenue at a point in time in accordance with ASC 606. The following information is available for the year:

Year 1 Year 2 Year 3

Total Contract Price $300,000 $300,000 $300,000
Costs incurred $70,000 $110,000 $273,000
Estimated costs to complete at Year-end $200,000 $72,000 $0
The performance by Deer Co. does not create an asset with an alternative use and Deer Co. has an enforceable right to payment for the completed performance to date. Calculate the income (or loss) recognized in Year 3?

A
$0
B
$8,471
C
$27,000
D
$30,000
A

Explanation:
The correct answer is (C).

When revenue is recognized at a point in time, gross profit is recognized only on completion of the contract. Since the project started in Year 1 and is completed in Year 3 as evidenced by the fact that the estimated remaining costs the end of Year 3 is $0. The project is completed and accordingly, all of the profit will be recognized i.e. $27,000 ($300,000 – 273,000).

Note:

Revenue recognized over time is similar to the Percentage-of-Completion Method allowed under the previous standard.
Revenue recognized at a point in time is similar to the Completed Contract Method allowed under the previous standard.

659
Q

Wren Corp.’s trademark was licensed to Mont Co. for royalties of 15% of sales of the trademarked items. Royalties are payable semiannually on March 15 for sales in July through December of the prior year, and on September 15 for sales in January through June of the same year. Wren received the following royalties from Mont:

March 15 September 15

Year 2 $10,000 $15,000
Year 3 $12,000 $17,000
Mont estimated that sales of the trademark items would total $60,000 for July through December Year 3. In Wren’s Year 3 income statement, the royalty revenue should be:

A
$26,000
B
$29,000
C
$38,000
D
$41,000
A

Explanation:
The correct answer is (A).

The amount to be recognized is the amount of consideration to which the entity expects to be entitled. Since the actual sales are unknown, the royalty revenue for the period between July to December Year 3 would be based on estimated sales of $60,000.

Royalty revenue Year 3 = Revenue for Jan to June Year 3 + Revenue for Jul to Dec Year 3

= $17,000 + 9,000 (i.e. 15% of $60,000) = $26,000.

(B) is incorrect because it is taking the royalty revenue received on March 15, Year 3 and September 15, Year 3. The royalties received on March 15, Year 3 should not be included as they are royalties received for sales in July to December Year 2. Also, royalties received for the period July to December Year 3 needs to be included.

(C) is incorrect because it is taking the royalty revenue received on March 15, Year 3, September 15, Year 3 and for the period July through December, Year 3. The royalties received on March 15, Year 3 should not be included as they are royalties received for sales in July to December Year 2.

(D) is incorrect because it is taking the royalty revenue on September 15, Year 2, September 15, Year 3 and for the period July through December Year 3. The royalties received on September 15, Year 2 should not be included as they are royalties received for sales in January to June Year 2.

660
Q

For a troubled debt restructuring involving only a modification of terms, which of the following items specified by the new terms would be compared to the carrying amount of the debt to determine if the debtor should report a gain on restructuring?
A
The total future cash payments.
B
The present value of the debt at the original interest rate.
C
The present value of the debt at the modified interest rate.
D
The amount of future cash payments designated as principal repayments.

A

a

Explanation:
Accounting for debtors when the restructuring is a modification of terms is determined by whether the sum of the cash payments under the new terms (not discounted to present value) equal or exceed the amount of the obligation.

661
Q

The calculation of the income recognized in the third year of a five-year construction contract accounted for using the uses input method prescribed in ASC 606 that recognizes revenue over time includes the ratio of

A
Total costs incurred to date to total estimated costs.
B
Cost incurred in year three to total estimated costs.
C
Costs incurred in year three to total billings to date.
D
Total costs incurred to date to total billings.

A

a

Explanation:
The correct answer is (A).

When income is recognized over time per ASC 606, Gross Profit is calculated using the following:

[ (Cost incurred to date / Total Estimated Cost) x Estimated Profit) ] - Profit already recognized

The ratio included in the formula is the total costs to date to the total estimated costs.

662
Q
Fenn Stores, Inc. had sales of $1,000,000 during December. Experience has shown that merchandise equaling 7% of sales will be returned within 30 days and an additional 3% will be returned within 90 days. Returned merchandise is readily resalable. In addition, merchandise equaling 15% of sales will be exchanged for merchandise of equal or greater value. What amount should Fenn report for net sales in its income statement for the month of December?
A
$900,000
B
$850,000
C
$780,000
D
$750,000
A

a

Explanation:
The amount reported for net sales should not be reduced for the 15% of sales expected to be exchanged for merchandise of equal or greater value. However, net sales should be reduced for the 7% of sales expected to be returned within 30 days and the additional 3% of sales expected to be returned within 90 days

Sales, December $1,000,000
Less: Estimated sales that will be returned [$1,000,000 × (7% + 3%)] 100,000
Net sales, December $ 900,000

663
Q

On October 15, Year 1, Kam Corp. informed Finn Co. that Kam would be unable to repay its $100,000 note due on October 31 to Finn. Finn agreed to accept title to Kam’s computer equipment in full settlement of the note. The equipment’s carrying value was $80,000 and its fair value was $75,000. This was an unusual and infrequent event. What amounts should Kam report as ordinary gain (loss) or the year ended September 30, Year 2?

Income from Continuing Operations	Other Comprehensive Income
A	$ (5,000)	$17,500
B	$0	$20,000
C	$0	$14,000
D	$20,000	$0
A

d

Explanation:
The correct answer is (D).

The debtor recognizes a loss equal to the excess of the carrying amount over the fair value of the computer equipment transferred [$75,000 - $80,000 = ($5,000)] and a gain equal to the difference between the carrying amount of the obligation and the fair value of the equipment transferred at the restructure date [$100,000 - $75,000 = $25,000].

The net ordinary gain equals $20,000 [($5,000) + $25,000].

Unusual and/or Infrequent is always classified as Income from Continuing Operations.

664
Q

A shoe retailer allows customers to return shoes within 90 days of purchase. The company estimates that 5% of sales will be returned within the 90-day period. During the month, the company has sales of $200,000 and returns of sales made in prior months of $5,000. What amount should the company record as net sales revenue for new sales made during the month?

A
$185,000
B
$190,000
C
$195,000
D
$200,000
A

b

Explanation:
The company will have to make a provision for returns from the sales revenue amount. The actual returns in the prior months will not affect the provision to be made for future months. Thus the net sales revenue for the month will be $190,000 [200,000 - (200,000 x 5%)].

Option (A) is incorrect because it also deducts the returns of sales in prior months.

Option (C) is incorrect because it adds the returns of sales in prior months.

Option (D) is incorrect because it does not make provision for returns from the sales

665
Q

Bear Co., which began operations on January 2, Year 1 appropriately uses the input method prescribed in ASC 606 and recognizes revenue over time. The performance by Bear Co. does not create an asset with an alternative use and Bear Co., has an enforceable right to payment for the completed performance to date. The following information is available for the year:

Total Contract Price $300,000
Costs incurred $70,000
Estimated costs to complete at year end $200,000
Calculate the income (or loss) recognized in Year 1?

A
$0
B
$7,778
C
$30,000
D
$300,000
A

Explanation:
The correct answer is (B).

Gross Profit recognized in each period of the contract on an accrual basis and is calculated as follows:

Gross Profit = (Cost incurred to date / Total Estimated Cost) x Estimated Profit

Total Contract price $300,000
Actual cost to date $70,000
Estimated costs to complete $200,000
Estimated total costs at completion $270,000
Estimated Gross Profit $30,000
Profit Recognized in Year 1 ($70,000/$270,000) x $30,000 $7,778
Note:

Revenue recognized over time is similar to the Percentage-of-Completion Method allowed under the previous standard.
Revenue recognized at a point in time is similar to the Completed Contract Method allowed under the previous standard.

666
Q

On day 1 Clothes Co. sells clothing to Link Corp. for $40,000. Clothes ships the clothing on day 1 and Link is obligated to pay Clothes within six months. Link is given 12 months to return any of the clothing for a refund if they experience low demand. Link is also given 18 months to exchange any clothing due to low demand. At the time of sale, Clothes cannot reasonably estimate returns but estimates $5,000 in exchanged goods. Clothes should recognize revenue for the aforementioned transaction.

A
On the day of the sale.
B
Six months after the date of sale.
C
12 months after the date of sale.
D
18 months after the date of sale.
A

Explanation:
The correct answer is (C)

A 5-step approach for revenue recognition:

Step 1: Identify the Contract(s) with a customer.
Step 2: Identify the Performance obligations in the contract.
Step 3: Determine the transaction price (i.e., Amount).
Step 4: Allocate transaction Price to Performance obligations in the contract.
Step 5: Revenue Recognition when/as an entity satisfies a performance obligation.
Recognize revenue for the transferred products in the amount of consideration to which the entity expects to be entitled; therefore, revenue would not be recognized for the products expected to be returned. Revenue cannot be recognized at the time of sale because neither collection of payment from the buyer is reasonably assured nor can the company reasonably estimate the returns. The buyer’s right to return expires only 12 months, during which revenue can be recognized. The right to exchange goods is not a consideration for revenue recognition as an exchange does not affect the amount of collection. Therefore, Clothes Co. will recognize revenue after twelve months after the day of sale.

667
Q

Go Ltd. enters into a contract to provide internet services to a customer for one year. Go charges a $192 activation fee for which Go provides a router and sets up the account and connection for the customer. The customer is required to make monthly payments of $72 for the continuing service. A renewal option now exists that enables the customer to renew the contract after the first year without paying an additional activation fee. Based on historical customer data, the company expects each customer to renew for one additional year before changing service providers. What is the revenue to be recognized each month?

A
$192 in January and $ 72 every month after that
B
$264 in January and $72 every month after that
C
$88 each month
D
$80 each month
A

Explanation:
The correct answer is (D).

In many cases, even though a nonrefundable upfront fee relates to an activity that the entity is required to undertake at or near contract inception to fulfill the contract, that activity does not result in the transfer of a promised good/service to the customer. Instead, the upfront fee is an advance payment for future goods/services and, would be recognized as revenue when those future goods/services are provided. Revenue recognition period may extend beyond the initial contractual period if the entity grants the customer the option to renew the contract and that option provides the customer with a material right. If the nonrefundable upfront fee relates to a good/service, the entity should evaluate whether to account for the good/service as a separate performance obligation. In this case, the renewal option creates a material right for additional services for the customer.

Based on historical data, the company expects each customer to renew for one additional year before changing service providers.

With the upfront fee of $192 received, Go has a performance obligation for the next two years if the additional $72 is paid each month.

The estimate of the total consideration to be received is $192 + (24 months x $72) = $1,920. The company would recognize $80 dollars per month over the two years ($1,920/24).

(A) and (B) are incorrect because the upfront fee paid is to be recognized over the period of the contract based on the above explanation.

(C) is incorrect because the upfront fee paid is divided over a period of only 12 months. However, it is estimated that the contract will most likely be renewed for about 12 months, the upfront fee is to be divided over a period of 24 months.

668
Q
At the beginning of the current year, Hayworth Co. sold equipment with a two-year service contract for a single payment of $20,000. The fair value of the equipment was $18,000. Hayworth recorded this transaction with a debit of $20,000 to cash and a credit of $20,000 to sales revenue. Which of the following statements is correct regarding Hayworth's current-year financial statements?
A
The financial statements are correct.
B
Net income will be overstated.
C
Total assets will be overstated.
D
Total liabilities will be overstated.
A

b

Explanation:
Of the $20,000 Hayworth received, $18,000 should have been for sales revenue from the equipment and $2,000 should have been allocated to the two-year service contract. Because only one year has passed, Hayworth has only earned $1,000 from that service contract and still has a liability for the next year. Recording the transaction the way Hayworth did means net income will be overstated, the financial statements are not correct, total assets are not affected, and total liabilities will be understated.

669
Q

Deer Co., which began operations on January 2, Year 1 recognizes revenue at a point in time. The performance by Deer Co. does not create an asset with an alternative use and Deer Co., has an enforceable right to payment for the completed performance to date. The following information is available for the year:

Total Contract Price $300,000
Costs Incurred $70,000
Estimated costs to complete at year end $200,000
Calculate the income (or loss) recognized in Year 1?

A
$0
B
$7,778
C
$30,000
D
$300,000
A

Explanation:
The correct answer is (A).

Under ASC 606, when an entity recognizes revenue at a point in time, gross profit is recognized only on completion of the contract.

Since the project started in Year 1 and $200,000 of the cost is yet to be incurred. The project is not complete and accordingly, no revenue will be recognized.

Note:

Revenue recognized over time is similar to the Percentage-of-Completion Method allowed under the previous standard.
Revenue recognized at a point in time is similar to the Completed Contract Method allowed under the previous standard.

670
Q
Amar Farms produced 300,000 pounds of cotton during the 20X0 season. Amar sells all of its cotton to Brye Co., which has agreed to purchase Amar's entire production at the prevailing market price. Recent legislation assures that the market price will not fall below $.70 per pound during the next two years. Amar's costs of selling and distributing the cotton are immaterial and can be reasonably estimated. Amar reports its inventory at expected exit value. During year 1, Amar sold and delivered to Brye 200,000 pounds at the market price of $.70. Amar sold the remaining 100,000 pounds during year 2 at the market price of $.72. What amount of revenue should Amar recognize in year 1?
A
$140,000
B
$144,000
C
$210,000
D
$216,000
A

c

Explanation:
Amar recognizes revenue of $210,000 (i.e., 300,000 lbs. × $0.70/lb.) in year 1 for the cotton produced in year 1. It is appropriate for Amar to recognize revenue when the cotton is produced because (1) there is a relatively stable market for the cotton, (2) Amar’s costs of selling and distributing the cotton are immaterial and can be reasonably estimated, and (3) the units of cotton are homogeneous.

671
Q

In a compensatory stock option plan for which the grant, measurement, and exercise date are all different, the stock options outstanding account should be
reduced at the

A
Date of grant.
B
Measurement date.
C
Beginning of the service period.
D
Exercise date.
A

Explanation:
The correct answer is (D)

The cost of the services received from employees in exchange for awards of share-based compensation generally shall be measured based on the grant-date fair value of the options. The Stock Options Outstanding account is increased on the grant date.

The subsequent exercising, forfeiture, or lapsing of the stock options reduce this account.

Therefore, the Stock Options Outstanding account is increased on the grant date and reduced on the exercise date.

672
Q

Webb Co. implemented a defined benefit pension plan for its employees on January 1, year 5. During years 5 and 6, Webb’s contributions fully funded the plan. The following data are provided:

Year 8 Estimated Year 7 Actual

Projected benefit obligation, December 31 $750,000 $700,000
Accumulated benefit obligation, December 31 520,000 500,000
Plan assets at fair value, December 31 675,000 600,000
Projected benefit obligation in excess of plan assets 75,000 100,000
Pension expense 90,000 75,000
Employer’s contribution ? 50,000
What amount should Webb contribute in order to report an underfunded pension liability of $15,000 in its December 31, year 8, balance sheet?

A
$50,000
B
$60,000
C
$75,000
D
$100,000
A

b

Explanation:
The pension liability in year 8 would be the amount the projected benefit obligation exceeded the fair value of plan assets.(i.e., $750,000 – $675,000 = $75,000).In order to report a pension liability of $15,000 in its 12/31, year 8 balance sheet, Webb must make a funding payment of $60,000 (i.e., $75,000 – $60,000) in year 8.

673
Q

A company has an underfunded defined benefit pension plan. During the current year, the company uses the years-of-service method to amortize its prior service cost. What effect will the amortization of prior service cost have on the company’s current-year financial statements?
A
Total liabilities will be decreased.
B
Net income will be increased.
C
Current period expenses will be decreased.
D
Other comprehensive income will be increased.

A

d

Explanation:
Prior service cost amortization is the spreading of the cost of retroactive benefits generated by a plan amendment that granted increased benefits based on service rendered in prior periods. Retroactively increasing benefits increases the projected benefit obligation at the date of the amendment and the cost shall be recognized as a charge (increase) to other comprehensive income.

674
Q

A company sponsors two defined benefit pension plans. The following information relates to the plans at year end:
Plan A Plan B
Fair value of plan assets $ 800,000 $1,000,000
Projected benefit obligation 1,000,000 700,000What amount(s) should the company report in its balance sheet related to the plans?
A
Liability of $200,000; asset of $300,000
B
Asset of $100,000
C
Asset of $1,800,000; liability of $1,700,000
D
Liability of $100,000

A

a

Explanation:
Employers with more than one postretirement benefit plan are required to aggregate all overfunded plans and report one net asset amount ($300,000) and to aggregate all underfunded plans and report one net liability amount ($200,000). The current and noncurrent portions of the liability are reported separately in a classified balance sheet. A current liability is reported for the amount by which the fair value of plan assets is exceeded by the expected benefits to be paid over the next 12 months or over the operating cycle if it is longer. Net postretirement benefit assets will always be classified as noncurrent.

675
Q
At December 31, year 9, the following information was provided by the Kerr Corp. pension plan administrator:
Fair value of plan assets	$3,450,000
Accumulated benefit obligation	4,300,000
Projected benefit obligation	5,700,000What is the amount of the pension liability that should be shown on Kerr's December 31, year 9 balance sheet?
A
$5,700,000
B
$2,250,000
C
$1,400,000
D
$ 850,000
A

b

Explanation:
The amount of pension liability to be shown on the balance sheet is the amount the projected benefit obligation exceeds the fair value of plan assets.

Projected benefit obligation $ 5,700,000
Fair value of plan assets (3,450,000)
Unfunded projected benefit obligation $ 2,250,000

676
Q

On January 1, year 1, the board of directors of a corporation granted 10,000 stock options to the CEO. Each option permits the purchase of one share of stock at $25 per share, the current market price of the stock. The options are exercisable on December 31, year 4, as long as the CEO is still employed. The options expire on December 31, year 5. The grant date fair value of each option is $5. The corporation must recognize:

A
$50,000 of compensation expense when the options are exercised
B
$50,000 of compensation expense in year 1
C
$12,500 of compensation expense per year for four years
D
$10,000 of compensation expense per year for five years

A

c

Explanation:
The correct answer is (C).

Stock options may not only be granted as inducements to employees to purchase company stock but also as compensation in many cases. Fair value on the grant date is used to recognize the expense on a straight-line basis over the period of service required by the employee before the option can be exercised. The service period required is 4 years (January 1, year 1 to December 31, year 4). The corporation must recognize the expense at $12,500 for 4 years [i.e. ($5 x 10,000 stock) / 4].

(A) is incorrect because $50,000 is the total expense which should be recognized on a straight line basis over the period of service required (4 years) instead of recognizing complete expense at once when options are exercised.

(B) is incorrect because $50,000 is the total expense which should be recognized on a straight line basis over the period of service required (4 years) instead of recognizing complete expense at once when options are granted.

(D) is incorrect because option expense should be spread over 4 years instead of 5 years [i.e. $10,000 = ($5 x 10,000) / 5].

677
Q
The \_\_\_\_\_\_\_\_\_\_\_\_ is the basis for determining payments to which participants may be entitled under a pension plan.
A
Pension benefit formula
B
Prior service cost
C
Projected benefit obligation
D
Unfunded accumulated benefit obligation
A

a

Explanation:
The pension benefit formula is the basis for determining payments to which participants may be entitled under a pension plan. Pension benefit formulas usually refer to the employee’s service or compensation or both.

678
Q

A company has multiple defined benefit pension plans. A pension asset reported in the statement of financial position represents the amount by which the

A
Total fair value of plan assets exceeds the total projected benefit obligation for all overfunded and underfunded plans
B
Total fair value of all plans exceeds the total accumulated benefit obligation for all overfunded and underfunded plans
C
Fair value of plan assets exceeds the projected benefit obligation for the company’s overfunded plans
D
Fair value of plan assets exceeds the accumulated benefit obligation for the overfunded plans

A

c

Explanation:
The correct answer is (C).

A company has multiple defined benefit pension plans. A pension asset reported in the statement of financial position represents the amount by which the fair value of plan assets exceeds the projected benefit obligation for the company’s overfunded plans.

Fair value of plan assets > Projected benefit obligation = Overfunded plan.
Fair value of plan assets < Projected benefit obligation = Underfunded plan.

679
Q

What are the two required financial statements of a defined contribution retirement plan?

A
A statement of financial position and a statement of activities.
B
A statement of fiduciary net assets and a statement of changes in fiduciary net assets.
C
A statement of net assets available for benefits of the plan and a statement of changes in fiduciary net assets.
D
A statement of net assets available for benefits of the plan and a statement of changes in net assets available for benefits.

A

d

Explanation:
The correct answer is (D).

Under a defined contribution plan, an employer contributes specific periodic amounts of contributions to the plan during the time the employee is in service such that after retirement, the employee would receive these contributions and along with any returns generated.

The two required financial statements of a defined contribution retirement plan are a Statement of Net Assets Available for Benefits of the Plan and a Statement of Changes in Net Assets Available for Benefits.

680
Q
Bounty Co. provides postretirement health care benefits to employees who have completed at least 10 years service and are aged 55 years or older when retiring. Employees retiring from Bounty have a median age of 62, and no one has worked beyond age 65. Fletcher is hired at 48 years old. The attribution period for accruing Bounty's expected postretirement health care benefit obligation to Fletcher is during the period when Fletcher is aged
A
48 to 65
B
48 to 58
C
55 to 65
D
55 to 62
A

b

Explanation:
The attribution period for accruing the expected postretirement health care benefit obligation begins with the date of hire and ends at the full eligibility date. Bounty provides postretirement health care benefits to employees who have completed at least 10 years of service and are aged 55 years or older when retiring. Fletcher is hired at 48 years old. Therefore, the attribution period for accruing Bounty’s expected postretirement health care benefit obligation to Fletcher is during the period when Fletcher is aged 48 to 58, because at age 58, Fletcher will have completed 10 years of service and be 55 years or older.

CLOSE

681
Q

A company provides a defined benefit pension plan for all of its employees. The fair value of the plan assets at year-end is $45,000,000. The values of the accumulated benefit obligation and projected benefit obligation at year-end are $46,000,000 and $60,000,000, respectively. The company expects to make benefit payments totaling $2,000,000 next year. What amount should the company report in the year-end financial statements as a liability in connection with the defined benefit pension plan?

A
$17,000,000
B
$15,000,000
C
$3,000,000
D
$1,000,000
A

Explanation:
The correct answer is (B)

Overfunding = FV of plan assets > PBO = Pension asset.
Underfunding = FV of plan assets < PBO = Pension liability.
Planis underfunded by $15,000,000 (i.e. $45,000,000 - $60,000,000) resulting in liability.

Expected Benefit payments for next year will not affect the Pension plan asset or liability in the current year.

682
Q

Harmony Co. has a single-employer defined benefit pension plan. Harmony should report a liability related to the plan equal to which of the following amounts?

A
The unfunded projected benefit obligation.
B
The accumulated benefit obligation.
C
The projected benefit obligation.
D
The unfunded vested benefit obligation.
A

a

Explanation:
The correct answer is (A).

The funded status of the pension plan is equal to the fair value of plan assets less the projected benefit obligation. This funded status must be reported on the balance sheet as either an asset (when plan assets exceed the projected benefit obligation) or a liability (when the projected benefit obligation exceeds plan assets).

A liability on the balance sheet represents the incremental, or unfunded, portion of the projected benefit obligation that exceeds plan assets.

Underfunding = FV of plan assets < PBO = Pension liability.

683
Q
An entity should accrue a liability at year-end for the estimated cost of material compensated absences. This would include which of the following?
A
Vacation days
B
Holidays
C
Accumulated, nonvesting sick leave, if an entity customarily pays or allows its employees to take compensated absences for accumulated, nonvesting, sick leave days, even though employees are not actually absent as a result of illness
D
All of the above
A

d

Explanation:
A liability is accrued at year-end for the estimated cost of material compensated absences, which include vacation days and holidays. An entity should accrue a liability if the entity customarily pays or allows compensated absences for employees with accumulated, nonvesting sick leave days, even though employees are not actually absent as a result of illness. Since the answers for A., B., and C. are all correct, the best answer for this question is D., all of the above.

684
Q
The following information pertains to the currrent year activity of Ral Corp.'s defined benefit pension plan:
Service cost	$300,000
Return on plan assets	80,000
Interest cost on pension benefit obligation	164,000
Amortization of actuarial loss	30,000
Amortization of unrecognized net obligation	70,000Ral's current year pension cost was
A
$316,000
B
$484,000
C
$574,000
D
$644,000
A

b

Explanation:
Service cost $300,000
Return on plan assets (80,000)
Interest cost on projected benefit obligation 164,000
Amortization of actuarial loss 30,000
Amortization of unrecognized net obligation 70,000
Pension cost for the current year $484,000

685
Q

Payne, Inc., implemented a defined-benefit pension plan for its employees on January 2, year 3. The following data are provided for the year, as of December
31, year 3:

Projected benefit obligation	$103,000
Plan assets at fair value	78,000
Net periodic pension cost	90,000
Employer's contribution	70,000
What amount should Payne record as additional minimum pension liability at December 31, year 3?
A
$0
B
$ 5,000
C
$20,000
D
$45,000
A

Explanation:
The amount of the total liability to be recognized equals the unfunded projected benefit obligation.

Projected benefit obligation $103,000
Less: Plan assets at fair value (78,000)
Amount of total pension liability $ 25,000
Payne would have already recorded a pension liability in the difference between the net periodic pension cost and the employer’s contribution.

Net periodic pension cost $90,000
Less: Employer’s contribution (70,000)
Pension liability recognized $ 20,000
So, the additional minimum pension liability is $25,000 - $20,000 = $5,000.

686
Q

When a single employer has a defined benefit pension plan and records actuarial gains and losses to the extent recognized, it is not in compliance with authoritative guidance if it does which of the following?
A
Excludes unrealized gains and losses from its computations and analyses
B
Computes part of the gain or loss on plan assets as the difference between the actual return on assets and the expected return on assets, for a given accounting period
C
Determines the expected return on assets by considering the expected long-term rate of return and the market-related value of plan assets
D
Determines the expected long-term rate of return as the average rate of earnings expected on the funds invested, including the return expected to be available for reinvestment

A

a

Explanation:
Actuarial gains and losses to the extent recognized refers to changes in the amount of either the projected benefit obligation or plan assets resulting from experience different from that assumed, and also changes in assumptions. It includes both realized and unrealized gains and losses.

687
Q
A
The unfunded projected benefit obligation
B
The accumulated benefit obligation
C
The projected benefit obligation
D
The unfunded vested benefit obligation
A

a

Explanation:
If Harmony is reporting a liability for its single-employer defined benefit pension plan then the plan must be underfunded, meaning the benefit obligation is larger than the fair value of plan assets. In that case a net liability equal to the unfunded projected benefit obligation should be reported. The accumulated benefit obligation is the actuarial present value of benefits attributed by the pension benefit formula to employee services rendered before a specified date and based on employee services and compensation (if applicable) before that date. The projected benefit obligation is the actuarial present value as of a date of all benefits attributed by the pension benefit formula to employee service rendered prior to that date. The vested benefit obligation the actuarial present value of vested benefits (benefits for which the employee’s right to receive is no longer contingent on remaining in the service of the employer).

688
Q

The following information pertains to Gali Co.’s defined benefit pension plan for the current year:
Fair value of plan assets at 1/1 $350,000
Fair value of plan assets at 12/31 525,000
Employer contributions 110,000
Benefits paid 85,000In computing pension expense, what amount should Gali use as actual return on plan assets?
A
$65,000
B
$150,000
C
$175,000
D
$260,000

A

Explanation:
Increase in fair value of plan assets ($525,000 - $350,000) $ 175,000
Less: Increase due to employer contributions (110,000)
Add: Decrease due to benefit payments 85,000
Actual return on plan assets for the year $ 150,000

689
Q
For a defined benefit postretirement plan, the cost of benefit improvements is the increase in which of the following at the date of a plan amendment?
Accumulated postretirement
benefit obligation	Expected postretirement
benefit obligation
A	No	No
B	No	Yes
C	Yes	No
D	Yes	Yes
A

c

Explanation:
The cost of benefit improvements is the increase in the accumulated benefit obligation as a result of a plan amendment, measured at the date of the amendment.

690
Q
\_\_\_\_\_\_\_\_\_\_\_\_ is the actuarial present value of benefits (whether vested or non-vested) attributed by the pension benefit formula to employee services rendered before a specified date and based on employee services and compensation (if applicable) prior to that date.
A
Attribution
B
Expected return on plan assets
C
Accumulated benefit obligation
D
Market-related value of plan assets
A

c

Explanation:
The accumulated benefit obligation is the actuarial present value of benefits (whether vested or nonvested) attributed by the pension benefit formula to employee services rendered before a specified date and based on employee services and compensation (if applicable) prior to that date.

691
Q

Which of the following disclosures is not required of companies with a defined benefit pension plan?

A
A description of the plan.
B
The amount of pension expense by component.
C
Benefits expected to be paid in each of the next 5 years and in the aggregate for 5 years thereafter.
D
Benefits expected to be paid in each of the next 10 years and in the aggregate for 5 years thereafter.

A

Explanation:
The correct answer is (D).

Required pension disclosures include:

Components of Net Periodic Pension Cost
Reconciliation of Pension Benefit Obligation and of Fair Value of Plan Assets showing the components separately
Accumulated Benefits Obligations for defined benefit plans
Funded status of the plans
Impact on Other Comprehensive Income
Detailed description of the plan including employee groups covered
Expected benefits & contributions
Benefits expected to be paid in each of the next 5 years and in the aggregate for 5 years thereafter
Contributions expected to be paid to the plan during the rest of the fiscal year
Rates & Assumptions on a weighted-average basis
Amortization method used to amortize prior service costs or unrecognized gains or losses on transition
Explanation of any significant changes in plan assets or benefit obligations
The required disclosure is for benefits expected to be paid in each of the next 5 years (not 10 years) and in the aggregate for 5 years thereafter.

692
Q

An entity sponsors a defined benefit pension plan that is underfunded by $800,000. A $500,000 increase in the fair value of plan assets would have which of the following effects on the financial statements of the entity?
A
An increase in the assets of the entity.
B
An increase in accumulated other comprehensive income of the entity for the full amount of the increase in the value of the assets.
C
A decrease in accumulated other comprehensive income of the entity for the full amount of the increase in the value of the assets.
D
A decrease in the liabilities of the entity.

A

d

Explanation:
The recognition of a liability, or asset, is required if the projected benefit obligation differs from the fair value of plan assets for a defined benefit pension plan. A defined pension plan that is underfunded indicates the projected benefit obligation exceeds the fair value of plan assets and a liability is required. A $500,000 increase in the fair value of plan assets would lower the amount underfunded to only $300,000 and would in effect decrease the liabilities of the entity.

693
Q
On January 2 of the current year, Kine Co. granted Morgan, its president, compensatory stock options to buy 1,000 shares of Kine's $10 par common stock. The options call for a price of $20 per share and are exercisable for 3 years following the grant date. The options are valued at $35,000 on the date of the grant. Morgan exercised the options on December 31 of the current year. The market price of the stock was $50 on January 2 and $70 on December 31. By what net amount should stockholders' equity increase as a result of the grant and exercise of the options?
A
$20,000
B
$30,000
C
$50,000
D
$70,000
A

a

Explanation:
Deferred compensation expense is a contra stockholders’ equity account. Therefore, the net effect on stockholders’ equity when the options are granted is zero. Stockholders’ equity is increased $45,000 + $10,000 - $35,000 = $20,000. The journal entries follow:

Deferred Compensation Cost (value of options at date of grant) 35,000
Stock Options Outstanding 35,000
To record the grant.
Cash (1,000 shares × $20) 20,000
Stock Options Outstanding 35,000
Common Stock (1,000 shares × $10) 10,000
Add’l Paid-in Cap., Common (to balance) 45,000
To record the exercise of options.

694
Q

On September 1, year 1, Howe Corp., offered special termination benefits to employees who had reached the early retirement age specified in the company’s pension plan. The termination benefits consisted of lump-sum and periodic future payments. Additionally, the employees accepting the company offer receive the usual early retirement pension benefits. The offer expired on November 30, year 1. Actual or reasonably estimated amounts at December 31, year 1, relating to the employees accepting the offer are as follows:
Lump-sum payments totaling $475,000 were made on January 1, year 2.
Periodic payments of $60,000 annually for three years will begin January 1, year 3. The present value at December 31, year 1, of these payments was $155,000.
Reduction of accrued pension costs at December 31, year 1, for the terminating employees was $45,000.
In its December 31, year 1, balance sheet, Howe should report a total liability of special termination benefits of
A
$475,000
B
$585,000
C
$630,000
D
$655,000

A

c

Explanation:
An employer may provide benefits to employees in connection with their termination of employment. Special termination benefits are termination benefits that are offered for only a short period of time. An employer that offers special termination benefits to employees recognizes a liability and a loss when the employees accept the offer and the amount can be reasonably estimated. The cost of termination benefits recognized as a liability and loss is the amount of any lump-sum payments and the present value of any expected future payments. The amount to be reported as the total liability for special termination benefits at December 31, year 1, for the company in question is determined as follows:

Lump-sum payments made 1/1, year 2 $475,000
Present value of periodic payments beginning 1/1, year 3 155,000
Liability for special termination benefits, 12/31, year 1 $630,000The reduction of accrued pension cost for the terminating employees does not affect the liability reported for special termination benefits.

695
Q

A defined benefit pension plan had the following activity during the fiscal year:

Dividends and interest received $ 92,000
Contributions received from employers and employees 340,000
Administrative expenses 45,400
Investments purchased 155,000
Increase in fair value of investments at year-end 36,750
What should be reported as the total additions in the pension plan’s statement of changes in net assets available for benefits?

A
$313,750
B
$423,350
C
$432,000
D
$468,750
A

Explanation:
The correct answer is (D).

Statement of changes in net assets available for benefits are calculated as follows:

Beginning plan assets

Add: Contributions during the period

Add: Returns (actual) on plan assets during the period

Less: Benefits paid out during the year

Ending plan assets

So additions in the pension plan’s statement of changes in net assets available for benefits will include Contributions during the period ($340,000) and Returns (actual) on plan assets during the period which will include Dividends and interest received ($92,000) and Increase in fair value of investments at year end ($36,750) and will total to $468,750.

696
Q

How does the accounting for postretirement benefits other than pensions differ from the accounting for pensions?
A
Accounting for postretirement benefits differs by assigning benefit costs on a years-of-service approach.
B
Accounting for postretirement benefits differs by allocating benefit costs over the approximate service years of employees.
C
Accounting for postretirement benefits differs by allowing the option of recognizing immediately in net income, as a change in accounting, the transition amounts for postretirement benefits other than pensions.
D
None of the above.

A

c

Explanation:
The transition amounts for postretirement benefits other than pensions may be recognized immediately in net income as a change in accounting principle, as an alternative to amortization. This option is not available in accounting for pension costs.

697
Q

At the beginning of year 1, a company amends its defined benefit pension plan for an additional $500,000 in prior service cost. The amendment covers employees with a 10-year average remaining service life. At the end of year 1, what is the net entry to accumulated other comprehensive income, ignoring income tax effects?

A
A $450,000 debit.
B
A $500,000 debit.
C
A $550,000 credit.
D
A $450,000 credit.
A

Explanation:
The correct answer is (A)

Defined benefit pension plan:

Prior service costs ↑; Excess of projected benefit obligation over the fair value of plan assets ↑.

Other comprehensive income (OCI) $500,000
Project benefit obligation $500,000
Since it covers employees with an average 10-year remaining service life, it will be amortized at the rate of $50,000 per year with an increase, or debit, to pension expense and a credit to OCI.

Pension expense $50,000
Other comprehensive income $50,000
The net effect on OCI is a debit of $450,000.

(B) is incorrect because it reflects beginning balance debit to OCI of $500,000.

(C) is incorrect because it is adding amortization amount to the existing prior year cost.

(D) is incorrect because OCI has a debit balance and not a credit balance.

698
Q

The following information pertains to Lee Corp.’s defined benefit pension plan for for the current year:

Service cost $160,000
Actual and expected gain on plan assets 35,000
Unexpected loss on plan assets related to a disposal of a subsidary 40,000
Amortization of prior service cost 5,000
Annual interest on pension obligation 50,000
What amount should Lee report as pension expense in its current year-end income statement?

A
$250,000
B
$220,000
C
$210,000
D
$180,000
A

Explanation:
A loss on plan assets related to a subsidiary disposal is not included in the determination of net pension cost. A gain or loss directly related to a disposal is
recognized as a part of the gain or loss associated with that event.

Service cost $160,000
Annual interest on pension obligation 50,000
Actual and expected gain on plan assets (35,000)
Amortization of prior service cost 5,000
Net pension cost $180,000

699
Q

Note section disclosures in the financial statements for pensions do not require inclusion of which of the following?
A
The components of period pension costs
B
The amount of unrecognized prior service cost
C
The differences in executive and non-executive plans
D
A detailed description of the plan including employee groups covered

A

c

Explanation:
The difference in executive and non-executive plans is not a required disclosure. The components of period pension costs is a required disclosure. The amount of unrecognized prior service cost and a detailed description of the plan including employee groups covered are required disclosures.

700
Q

A company granted its employees 100,000 stock options on January 1, year 1. The stock options had a grant date fair value of $15 per option and a three-year vesting period. On January 1, year 2, the company estimated the fair value of the stock options to be $18 per option. Assuming that the company did not grant any additional options or modify the terms of any existing option grants during year 2, what amount of share-based compensation expense should the company report for the year ended December 31, year 2?

A
$500,000
B
$600,000
C
$700,000
D
$800,000
A

Explanation:
The correct answer is (A).

When compensatory stock options are granted to employees, the compensation expense is reported by the company based on the fair value of the option when granted. The total compensation expense for 3 years is equal to $1,500,000 (i.e. 100,000 stock options x $15 per option). The vesting period is the period over which the employee is required to provide services. Compensation cost for share-based employee compensation classified as equity is amortized straight-line over the requisite period. The above stock options of value $1,500,000 have a vesting period of 3 years. The amount of share-based compensation expense that the company should report for the year ended December 31, year 2 = $1,500,000 / 3 years = $500,000.

701
Q
Some costs cannot be directly related to particular revenues but are incurred to obtain benefits that are exhausted in the period in which the costs are incurred. An example of such a cost is
A
Salespersons' monthly salaries.
B
Salespersons' commissions.
C
Transportation to customers.
D
Prepaid insurance.
A

a

Explanation:
GAAP guidance specifically mentions salesmen’s monthly salaries as an example that fits the description of a cost that cannot be directly related to particular revenues, but rather it is incurred in order to obtain benefits that are exhausted during the period. Salespersons’ commissions and transportation to customers are mentioned as examples of items that are directly related to sales revenues. Prepaid insurance is mentioned as an asset yielding benefits over several periods.

702
Q
In the financial statements of employee benefit pension plans and trusts, the plan investments are reported at
A
Fair value
B
Historical cost
C
Net realizable value
D
Lower of historical cost or market
A

a

Explanation:
For purposes of reporting overfunded and underfunded plans, along with required disclosures, plan investments shall be measured at their fair value as of the measurement date.

703
Q

Which of the following components should be included in net pension cost by an employer sponsoring a defined benefit pension plan?
Amortization of unrecognized prior service cost Fair value of plan assets
A Yes No
B Yes Yes
C No Yes
D No No

A

a

Explanation:
The following components are included in net pension cost: (1) service cost, (2) interest cost, (3) actual return on plan assets, if any, (4) amortization of unrecognized prior service cost, if any, (5) gain or loss (including the effects of changes in assumptions) to the extent recognized, and (6) amortization of the unrecognized net obligation (and loss or cost) or unrecognized net asset (and gain) existing at the date of initial application of GAAP. The fair value of plan assets should not be included in net pension cost.

704
Q
How should plan investments be reported in a defined benefit plan's financial statements?
A
At actuarial present value.
B
At cost.
C
At net realizable value.
D
At fair value.
A

d

Explanation:
For purposes of reporting overfunded and underfunded plans, along with required disclosures, plan investments shall be measured at their fair value as of the measurement date.

705
Q
On January 2, year 2, Loch Co. established a noncontributory defined benefit plan covering all employees and contributed $1,000,000 to the plan. At December 31, Loch determined that the service and interest costs on the plan were $620,000 for the year. The expected and the actual rate of return on plan assets for the year was 10%. There are no other components of Loch's pension expense. Prior to comparing the fair value of plan assets to the projected benefit obligation, what amount should Loch report in its December 31, year 2 balance sheet as a pension asset?
A
$280,000
B
$380,000
C
$480,000
D
$620,000
A

c

Explanation:
Loch contributed $1,000,000 to the plan. Therefore, in its 12/31, year 2 balance sheet, Loch should report a pension asset of $480,000 (i.e., $1,000,000 - $520,000) prior to comparing the fair value of plan assets to the projected benefit obligation.

Service and interest cost $620,000
Return on plan assets ($1,000,000 × 10%) (100,000)
Pension cost, year 2 $520,000Note: The asset recognized on the balance sheet is the amount the fair value of plan assets exceeds the projected benefit obligation.

706
Q

On January 2, East Corp. established a defined benefit pension plan. The plan’s service cost of $150,000 was fully funded at the end of the year. Prior service cost was funded by a contribution of $60,000 and amortization of prior service cost was $24,000. At December 31, what amount should East report as a pension asset prior to comparing the fair value of plan assets to the projected benefit obligation?

A
$90,000
B
$84,000
C
$60,000
D
$36,000
A

d

Explanation:
In the current year, East accrued pension cost of $174,000 (i.e., $150,000 service cost + $24,000 prior service cost amortization) and made funding payments of $210,000 (i.e.,$150,000 for service cost+ $60,000 for prior service cost). The amount of East’s pension asset at 12/31 is $36,000 (i.e., $210,000 funding payments – $174,000 pension cost), prior to comparing the fair value of plan assets to the projected benefit obligation.

707
Q
An overfunded single-employer defined benefit postretirement plan should be recognized in a classified statement of financial position as a
A
Noncurrent liability
B
Current liability
C
Noncurrent asset
D
Current asset
A

c

Explanation:
The current and noncurrent portions of any liability are reported separately in a classified balance sheet. A current liability is reported for the amount by which the fair value of plan assets is exceeded by the expected benefits to be paid over the next 12 months or over the operating cycle if it is longer. Net postretirement benefit assets will always be classified as noncurrent.

708
Q
Nome Co. sponsors a defined benefit plan covering all employees. Benefits are based on years of service and compensation levels at the time of retirement. Nome determined that, as of September 30, year 2, its projected benefit obligation was $380,000, and its plan assets had a $290,000 fair value. Nome's September 30, year 2, trial balance showed a pension asset of $20,000. To report the proper pension liability in its September 30, year 2 balance sheet, what amount should Nome report as an adjustment?
A
$110,000
B
$360,000
C
$380,000
D
$400,000
A

a

Explanation:
This is a poorly worded question. It is included so candidates learn to handle such questions. The excess of the projected benefit obligation over the fair value of plan assets (i.e., $90,000) is the amount of the pension liability to be reported in the 9/30 balance sheet. The pension asset of $20,000 and the amount of liability of $90,000 to be reported are netted into one amount of the $110,000 required adjustment.

The amount of the liability to be reported equals the unfunded projected benefit obligation.

Projected benefit obligation $380,000
Less: Fair value of plan assets (290,000)
Unfunded projected benefit obligation 90,000
Add: Pension asset on books 20,000
Additional liability to be recorded $110,000

709
Q

Able Co. provides an incentive compensation plan under which its president receives a bonus equal to 10% of the corporation’s income before income tax but after deduction of the bonus. If the tax rate is 40% and net income after bonus and income tax was $360,000, what was the amount of the bonus?

A
$36,000
B
$60,000
C
$66,000
D
$90,000
A

b

Explanation:
Net income after bonus and income taxes is $360,000. The income tax rate is 40%. Therefore, income after bonus but before income taxes is $600,000 [i.e., $360,000 / (100% – 40%)]. The bonus is equal to 10% of the corporation’s income after bonus but before income taxes. Therefore, the amount of the bonus is $60,000 (i.e., $600,000 × 10%).

710
Q
On January 2, year 3, Farm Co. granted an employee an option valued at $32,000 to purchase 1,000 shares of Farm's $10 common stock at $40 per share. The option became exercisable on December 31, year 3, after the employee had completed one year of service, and was exercised on that date. The market prices of Farm's stock were as follows:
January 2, year 3	$50
December 31, year 3	$65What amount should Farm recognize as compensation expense for year 3?
A
$0
B
$30,000
C
$32,000
D
$60,000
A

c

Explanation:
The cost of services received from employees in exchange for awards of share-based compensation generally shall be measured based on the grant-date fair value of the options. The account Stock Options Outstanding is increased on the grant date. The subsequent exercising, forfeiture, or lapsing of the stock options reduces this account.

January 2, Year 3
Deferred Compensation Cost (value of options at date of grant)	32,000	
Stock Options Outstanding		32,000
December 31, Year 3
Wages and Compensation Expense	32,000	
Deferred Compensation Cost		32,000
Cash (1,000 shares × $40)	40,000	
Stock Options Outstanding	32,000	
Common Stock (1,000 × $10)		10,000
Add'l Paid-in Cap., Common Stock (to balance)		62,000
711
Q
Visor Co. maintains a defined benefit pension plan for its employees. The service cost component of Visor's net periodic pension cost is measured using the
A
Unfunded accumulated benefit obligation.
B
Unfunded vested benefit obligation.
C
Projected benefit obligation.
D
Expected return on plan assets.
A

c

Explanation:
The service cost component of net periodic pension cost is a portion of the projected benefit obligation. Therefore, it is measured using the projected benefit obligation.

712
Q

At year-end, a company has a defined benefit pension plan with a projected benefit obligation of $350,000, a net gain of $140,000 that was not previously recognized in net periodic pension cost and prior service cost of $210,000 that was not previously recognized in net periodic pension cost. What amount should be reported in accumulated other comprehensive income related to the company’s defined benefit pension plan at year end?

A
A credit balance of $420,000
B
A debit balance of $420,000
C
A credit balance of $70,000
D
A debit balance of $70,000
A

Explanation:
The correct answer is (D).

To start, prior service cost is the cost of an amendment to a pension plan that increases pension benefits granted to employees for services already rendered. It is recognized as a debit to AOCI on the date of the plan amendment. Prior service cost of $210,000 is reported as a debit balance in accumulated other comprehensive income (AOCI). The net gain of $140,000 is reported as a credit in AOCI because it was not previously recognized in net periodic pension cost. This net gain (loss) consists of asset gains or losses (the difference between the actual return on plan assets and the expected return on plan assets) and the liability gains or losses due to unexpected changes in Projected Benefit Obligation (PBO). Thus, a debit balance of $70,000 ($210,000 debit - $140,000 credit) should be reported in AOCI.

713
Q

Effective January 1 of the previous year, Flood Co. established a defined benefit pension plan with no retroactive benefits. The first of the required equal annual contributions was paid on December 31 of that previous year. A 10% discount rate was used to calculate service cost and a 10% rate of return was assumed for plan assets. All information on covered employees for the previous and current year is the same. How should the service cost for the current year compare with the previous year, and should the previous year balance sheet report an underfunded or an over funded funding status?

Service cost for current year
compared to the previous year	Funding status reported on the
previous year balance sheet
A	Equal to	Underfunded
B	Equal to	Overfunded
C	Greater than	Underfunded
D	Greater than	Overfunded
A

d

Explanation:
Service cost for a period is the increase in the projected benefit obligation due to services rendered by employees during that period.All information on covered employees for the previous and current year is the same, and the current year is one year closer to the payment of retirement benefits.Service cost will be greater in the current year as compared to the previous year because the present value computation will be greater due to one less year of discounting.The plan requires equal annual contributions, but service costs are less in the beginning and more later.The contribution in the previous year will exceed the previous year service costs, resulting in an over funded pension status to be reported on the previous year balance sheet.

714
Q
The following information pertains to Seda Co.'s pension plan for the current year:
Actuarial estimate of projected benefit obligation at 1/1	$72,000
Assumed discount rate	10%
Service costs for the year	$18,000
Pension benefits paid during the year	$15,000If no change in actuarial estimates occurred during the year, Seda's projected benefit obligation at December 31 of the current year was
A
$64,200
B
$75,000
C
$79,200
D
$82,200
A

d

Explanation:
Interest cost represents the increase in the projected benefit obligation due to the passage of time, and is computed by multiplying the projected benefit obligation at 1/1 by the assumed discount (settlement) rate. The projected benefit obligation is increased by the interest costs and service costs incurred in the year and decreased by any pension benefits paid during the year.

Projected benefit obligation at 1/1 $72,000
Interest cost for the year ($72,000 × 10%) 7,200
Service costs for the year 18,000
Pension benefits paid during the year (15,000)
Projected benefit obligation, 12/31 $82,200

715
Q

Interest cost included in the net pension cost recognized by an employer sponsoring a defined benefit pension plan represents the
A
Amortization of the discount on unrecognized prior service costs.
B
Increase in the fair value of plan assets due to the passage of time.
C
Increase in the projected benefit obligation due to the passage of time.
D
Shortage between the expected and actual returns on plan assets.

A

c

Explanation:
Interest cost included in net pension cost is the increase in the projected benefit obligation due to the passage of time.

716
Q
The funded status of a defined benefit pension plan for a company should be reported in
A
The income statement.
B
The statement of cash flows.
C
The statement of financial position.
D
The notes to the financial statements only.
A

c

Explanation:
The funded-status amount is measured as the difference between the fair value of plan assets and the benefit obligation, with the benefit obligation including all actuarial gains and losses, prior service cost, and any remaining transition amounts. If the benefit obligation is larger than the fair value of plan assets, the plan is underfunded, and a net liability is reported. Conversely, if the fair value of the plan assets is larger, the plan is overfunded, and a net asset is reported on the statement of financial position (i.e. balance sheet).

717
Q

Zach Corp. pays commissions to its sales staff at the rate of 3% of net sales. Sales staff are not paid sala­ries but are given monthly advances of $15,000. Advances are charged to commission expense, and recon­ciliations against commissions are prepared quarterly. Net sales for the current year ended March 31 were $15,000,000. The unadjusted balance in the commissions expense account on March 31 was $400,000. March advances were paid on April 3. In its income statement for the current year ended March 31, what amount should Zach report as commission expense?

A
$465,000
B
$450,000
C
$415,000
D
$400,000
A

b

Explanation:
Accrual accounting recognizes expenses in the period they are incurred rather than when paid. Zach pays commissions to its sales staff at the rate of 3% of net sales. For the year ended 3/31, Zach should report commission expense of $450,000 ($15,000,000 × 3%).

718
Q
On which of the following dates is a public entity required to measure the cost of employee services in exchange for an award of equity interests, based on the fair market value of the award?
A
Date of grant.
B
Date of restriction lapse.
C
Date of vesting.
D
Date of exercise.
A

a

Explanation:
An award of equity interests in exchange for employee services is considered a compensatory share-based payment plan. With a compensatory share-based payment plan the compensation cost is recognized using the fair value method of option pricing and the measurement date is the date of the grant.

719
Q
On July 31 of the current year, Tern Co. amended its single employee defined benefit pension plan by granting increased benefits for services provided prior to the current year. This prior service cost will be reflected in the financial statement(s) for
A
Years before the current year only.
B
The current year only.
C
The current year, and years before and following the current year.
D
The current year and following years.
A

Explanation:
The plan amendment granted increased benefits for services provided prior to the current year. Because the plan amendment was granted with the expectation that Tern will realize economic benefits in future periods, the cost of providing the retroactive benefits should not be included in pension cost entirely in the year of amendment. Instead, the prior service cost should be recognized during the future service periods of those employees active at the date of the amendment who are expected to receive benefits under the plan (i.e., in the current year and following years).

720
Q
As of December 31, year 7, the projected benefit obligation and plan assets of a noncontributory defined benefit plan sponsored by Reed, Inc., were:
Projected benefit obligation	$780,000
Plan assets at fair value	  600,000
Transition obligation	$180,000Reed elected to apply the GAAP provisions for pension plans in its financial statements for the year ended December 31, year 8. At December 31 of year 7, all amounts accrued as net periodic pension cost had been contributed to the plan. The average remaining service period of active plan participants expected to receive benefits was estimated to be 10 years at the date of transition. Some participants' estimated service periods are 20 and 25 years. To minimize an accrual for pension cost, what amount of transition obligation should Reed amortize?
A
$ 7,200
B
$ 9,000
C
$12,000
D
$18,000
A

Explanation:
The $180,000 of transition obligation at the beginning of year 8, is computed as the excess at that date of the projected benefit obligation over the fair value of plan assets ($780,000 - $600,000 = $180,000), since at that date all amounts accrued as net periodic pension cost had been contributed to the plan. Typically, the transition obligation should be amortized on a straight-line basis over the average remaining service period of employees expected to receive benefits under the plan. However, because the average remaining service period is less than 15 years (10 < 15), Reed may elect to use a 15-year period. Therefore, the amount of transition obligation that Reed should amortize to minimize its accrual for pension cost is $12,000 ($180,000 / 15).

721
Q

For a compensatory stock option plan for which the date of grant and the measurement date are different, compensation cost should be recognized in the income statement
A
At the later of grant or measurement date.
B
At the exercise date.
C
At the adoption date of the plan.
D
Of each period in which the services are rendered.

A

d

Explanation:
Compensation costs of a stock option plan should be recognized (i.e., expensed) in the period(s) in which the employee performs the services. Deferred compensation costs are recognized on the date of the grant.

722
Q

Which of the following is a (are) fundamental aspect(s) that shape financial reporting in the application of accrual accounting to pensions?

A
Immediate recognition of all events
B
Reporting gross cost
C
Offsetting liabilities and assets
D
All of the above
A

c

Explanation:
There are three fundamental aspects shape financial reporting in the application of accrual account­ing to pensions.They are: delaying recognition of certain events;reporting net cost;and off setting liabilities and assets.

723
Q
What is the present value of all future retirement payments attributed by the pension benefit formula to employee services rendered prior to that date only?
A
Service cost
B
Interest cost
C
Projected benefit obligation
D
Accumulated benefit obligation
A

d

Explanation:
An accumulated benefit obligation is the actuarial present value of benefits attributed by the pension benefit formula to employee services rendered before a specified date and based on employee services and compensation prior to that date. Service and interest costs are components of the net periodic pension cost. The projected benefit obligation is the actuarial present value as of a date of all benefits attributed by the pension benefit formula to employee service rendered prior to that date.

724
Q
On January 2 of the current year, Loch Co. established a noncontributory defined-benefit pension plan covering all employees and contributed $400,000 to the plan. At December 31 Loch determined that the service and interest costs on the plan were $720,000 for the year. The expected and the actual rate of return on plan assets for the year was 10%. There are no other components of Loch's pension expense. What amount would Loch have as a pension liability on its December 31 year-end balance sheet?
A
$280,000
B
$320,000
C
$360,000
D
$720,000
A

Explanation:
Loch contributed only $400,000 to the plan and collected 10% return on plan assets. Therefore at year-end, Loch would have a pension liability of $280,000.

Service and interest costs $ 720,000
Less: Amount contributed (400,000)
Less: Actual return on plan assets (10% of $400,000) (40,000)
Unfunded pension liability $280,000

725
Q

Jan Corp. amended its defined benefit pension plan, granting a total credit of $100,000 to four employees for services rendered prior to the plan’s adoption. The employees, A, B, C, and D, are expected to retire from the company as follows: “A” will retire after three years. “B” and “C” will retire after five years. “D” will retire after seven years.

What is the amount of prior service cost amortization in the first year?

A
$0
B
$ 5,000
C
$ 20,000
D
$ 25,000
A

c

Explanation:
The cost of retroactive benefits generated by a plan amendment is amortized by assigning an equal amount to each year of future service for each employee active at the date of the amendment expected to receive benefits under the plan. The use of simplified methods is permitted, including the use of the straightline method that amortizes the cost over the average remaining service life of the active participants. The average remaining service life is 5 years, calculated by adding the expected remaining years of service of the participants (3 + 5 + 5 + 7 = 20 years) and dividing by the number of participants (4). The total credit of $100,000 would thus be amortized over 5 years, at $20,000 per year. If instead the amortization is calculated by prorating for each year over the next 7 years, during the first year 4 participants remain in active service; 4/20 x ($100,000) = $20,000. In this problem, during each of the first three years the amortization is the same under either method.

726
Q
Parker Co. amended its pension plan on January 2 of the current year. It also granted $600,000 of unrecognized prior service costs to its employees. The employees are all active and expect to provide 2,000 service years in the future, with 350 service years this year. What is Parker's unrecognized prior service cost amortization for the year?
A
$0
B
$2,000
C
$105,000
D
$600,000
A

c

Explanation:
Prior service cost amortization is the spreading of the cost of retroactive benefits generated by a plan amendment that granted increased benefits based on service rendered in prior periods. These costs are to be amortized as a component of net periodic pension cost by assigning an equal amount to each year of future service of each employee active at the date of the amendment who is expected to receive benefits under the plan. The $600,000 unrecognized prior service costs divided by the 2,000 future service years equals $300 to be amortized per service year. In the current year there were 350 service years provided. Multiplying the 350 service years by the $300 to be amortized per service year results in $105,000 unrecognized prior service cost amortization for the year.

727
Q

Which of the following information should be included in disclosures by a company providing health care benefits to its retirees?

The assumed health care cost trend rate used to measure the expected cost of benefits covered by the plan.
The accumulated post-retirement benefit obligation.
A
I and II.
B
I only.
C
II only.
D
Neither I nor II.
A

a

Explanation:
An employer sponsoring a benefit plan should disclose, among other things, the assumed health care cost trend rate(s) for the next year and a general description of the direction and pattern of change as well as the ultimate trend rate(s) and when the rate is expected to occur. The employer must also disclose the accumulated post-retirement benefit obligation.

728
Q

A company that maintains a defined benefit pension plan for its employees reports a pension liability. This cost represents the amount that the
A
Projected benefit obligation exceeds the fair value of plan assets.
B
Projected benefit obligation exceeds the vested benefit obligation.
C
Vested benefit obligation exceeds the fair value of plan assets.
D
Vested benefit obligation exceeds contributions to the plan.

A

a

Explanation:
A pension liability is recognized for the amount the projected benefit obligation exceeds the fair value of plan assets.

729
Q
Which of the following costs are unique to postretirement healthcare benefits?
A
Per capita claims.
B
Service.
C
Prior service.
D
Interest.
A

a

Explanation:
The editors suggest that you answer this question through process of elimination. If you are well prepared, you should recognize the terms used in choices b-d are commonly used in both pension and postretirement accounting, but per capita claims may not be as familiar to you. Per capita claims are the current cost of providing postretirement health care benefits for one year at each age from the youngest age to the oldest age at which plan participants are expected to receive benefits under the plan. Per capita costs are unique to postretirement benefits only.

730
Q

On January 1, year 1, a company grants 5,000 non-qualified stock options to an employee with a strike price of $3 per option and fair value of $8 per option. All of the options vest at the end of five years from the grant date. At the end of year 1, the company’s stock price was $10 per share. What amount of annual stock compensation cost should the company report for year 1?

A
$0
B
$3,000
C
$5,000
D
$8,000
A

Explanation:
The correct answer is (D).

When stock options are offered to a company’s employees, a stock compensation cost is required to be recognized in the books every year until the options are vested.

The cost is calculated as:

Stock Compensation Cost = (No. of options x fair value per option) / Vesting period

In the given case, the stock compensation cost can be calculated: (5,000 x 8) / 5 = $8,000

731
Q

Gavin Co. grants all employees two weeks of paid vacation for each full year of employment. Unused vacation time can be accumulated and carried forward to succeeding years and will be paid at the salaries in effect when vacations are taken or when employment is terminated. There was no employee turnover in the current year. Additional information relating to the current year ended December 31 is as follows:
Liability for accumulated vacations at 12/31 of the prior year $35,000
Pre-current year accrued vacations taken from 1/1 to 9/30 of the current year (the authorized period for vacations) 20,000
Vacations earned for work in the current year (adjusted to current rates) 30,000Gavin granted a 10% salary increase to all employees on October 1 of the current year, its annual salary increase date. For the current year ended December 31, Gavin should report vacation pay increase of
A
$45,000
B
$33,500
C
$31,500
D
$30,000

A

c

Explanation:
The amount of expense recognized is the increase in the cost of the pre-current year accrued vacations and the cost of vacations earned for work performed in the current year.

Liability for accrued vacations, 12/31 prior year $35,000
Less: Accrued vacations taken 1/1 to 9/30 current year (20,000)
Liability for pre-current year vacations at salary increase, 10/1 15,000
Times: 10% salary increase × 10%
Additional cost of pre-current year vacations $ 1,500
Cost of vacations earned for work in the year 30,000
Vacation pay expense $31,500

732
Q

Each of the following is a component of the changes in the net assets available for benefits of a defined benefit pension plan trust, except
A
The net change in fair value of each significant class of investments.
B
The net change in the actuarial present value of accumulated plan benefits.
C
Contributions from the employer and participants.
D
Benefits paid to participants.

A

b

Explanation:
Net plan assets include amounts contributed by the employer (and by employees for a contributory plan) and amounts earned from investing the contributions, less benefits paid. The net change in the actuarial present value of accumulated plan benefits is not part of net plan assets.

733
Q
At the beginning of year 1, a company amends its defined benefit pension plan for an additional $500,000 in prior service cost. The amendment covers employees with a 10-year average remaining service life. At the end of year 1, what is the net entry to accumulated other comprehensive income, ignoring income tax effects?
A
A $450,000 debit.
B
A $500,000 debit.
C
A $550,000 credit.
D
A $450,000 credit.
A

a

Explanation:
Retroactively increasing benefits increases the projected benefit obligation at the date of the amendment and the cost shall be recognized as a charge to other comprehensive income. These costs are to be amortized as a component of net periodic pension cost by assigning an equal amount to each year of future service of each employee active at the date of the amendment whom is expected to receive benefits under the plan. Plan amendments can reduce projected benefit obligations. These reductions reduce any other prior service costs and any excess is amortized. Other comprehensive income is adjusted each period as prior service cost is amortized. The $500,000 amortized over 10 years would be $50,000 per year. Accumulated other comprehensive income would first be debited the $500,000 at the beginning of year 1 and then credited the $50,000 amortization resulting in a net $450,000 debit entry.

734
Q

Which of the following is a pension plan settlement?
A
An employer purchases nonparticipating annuity contracts for vested benefits and continues to provide defined benefits for future service in the same plan.
B
Benefits to be accumulated in future periods are reduced, but the plan remains in existence and continues to pay benefits, to invest assets, and to receive contributions.
C
A plan is terminated and not replaced by a successor defined benefit plan.
D
Both A. and C.

A

Explanation:
If benefits to be accumulated in future periods are reduced, but the plan remains in existence and continues to pay benefits, to invest assets, and to receive contributions, a curtailment has occurred, but not a settlement. If a plan is terminated and not replaced by a successor, both a settlement and a curtailment have occurred. A settlement has also occurred if an employer purchases nonparticipating annuity contracts for vested benefits and continues to provide defined benefits for future service in the same plan. Since answers A. and C. are correct, answer D., both A. and C., is the best choice.

735
Q

The following information relates to a company’s defined benefit pension plan at December 31:

Accumulated benefit obligation

$1,035,000

Projected benefit obligation

$1,250,000

Prior service cost

$113,000

Net gain on plan assets

$167,000

Plan assets (fair value)

$737,000

What amount should the company report as its pension liability at December 31?

A
$567,000
B
$513,000
C
$400,000
D
$352,000
A

Explanation:
The correct answer is (B).

Reported Pension Liability Calculation = Projected Benefit Obligation (PBO) of $1,250,000 -Pension plan assets of $737,000 = $513,000.

The $513,000 reflects the amount by which the pension plan is underfunded (PBO > Plan assets). This is a clear shortage of plan assets accumulated to pay benefits compared with the present value of those benefits or PBO. In other words, only $737,000 of PBO is covered as of the balance sheet date. Accumulated benefit obligation, prior service cost, and net gain on plan assets are not used in the calculation of pension liability on December 31st.

(A), (C) and (D) are incorrect because of improper calculations. Accumulated benefit obligations, prior service cost or net gain on plan asset would not be considered for calculating pension liability.

736
Q
North Corp. has an employee benefit plan for compensated absences that gives employees 10 paid vacation days and 10 paid sick days. Both vacation and sick days can be carried over indefinitely. Employees can elect to receive payment in lieu of vacation days; however, no payment is given for sick days not taken. At December 31 of the current year, North's unadjusted balance of liability for compensated absences was $21,000. North estimated that there were 150 vacation days and 75 sick days available at December 31. North's employees earn an average of $100 per day. In its current year December 31 balance sheet, what amount of liability for compensated absences is North required to report?
A
$36,000
B
$22,500
C
$21,000
D
$15,000
A

d

Explanation:
Employers are required to accrue a liability for employee compensated absences that meet all of the following conditions: (1) the obligation is attributable to employees’ services already rendered, (2) the obligation relates to rights that vest or accumulate, (3) payment of the compensation is probable, and (4) the amount can be reasonably estimated. Thus, it would appear that North should accrue a liability for both the vacation benefits and the future sick day benefits. However, an employer is not required to accrue a liability for non-vesting accumulating rights to receive sick pay benefits. Therefore, since North’s employees’ rights to receive sick pay benefits do not vest, North should report only the $15,000 (i.e., 150 x $100) cost of the vacation days available at 12/31 of the current year as the liability for compensated absences at that date.

737
Q
The following information pertains to Kane Co.'s defined benefit pension plan:
Prepaid pension cost, January 1	$ 2,000
Service cost	19,000
Interest cost	38,000
Actual return on plan assets	22,000
Amortization of unrecognized prior service cost	52,000
Employer contributions	40,000The fair value of plan assets exceeds the accumulated benefit obligation. In its December 31 year-end balance sheet, what amount should Kane report as unfunded accrued pension cost?
A
$45,000
B
$49,000
C
$67,000
D
$87,000
A

a

Explanation:
Unfunded accrued pension costs consists of the net periodic pension cost adjusted for employer contributions and previous prepaid costs or accrued costs.

Net Periodic Pension Cost:

Service cost $19,000
Interest cost 38,000
Amortization of unrecognized prior service cost 52,000
Actual return on plan assets (22,000)
Net periodic pension cost for the year $87,000
Less: Prepaid pension cost, 1/1 $ 2,000
Employer contributions 40,000 (42,000)
Unfunded accrued pension cost 12/31 $45,000

738
Q

P Corporation has reported following items as a component of its net pension expense for the year end December 31, 2017.

Service Cost:

Factory Workers: $4,000
Sales Staff: $6,000
How much of the above expenses should be included in Cost of Goods Sold for the year ended?

A
$6,000
B
$10,000
C
$0
D
$4,000
A

d

Explanation:
$4,000- FASB issued Accounting Standards Update 2017-07 effective Dec 15, 2017, for public companies, and Dec 15, 2018, for non-public companies which changed the presentation guidelines for reporting of Service Cost. Service cost component is now reported in the same line item(s) as other compensation costs arising from services rendered by the employees during the period E.g., COGS for factory workers, Selling for Salesmen, G&A for officers. Other components of net benefit cost are required to be presented in I/S separately from the Service cost component as Non-operating item. Prior to this update components of Net Pension Expense used to be aggregated and presented as Operating Expenses on I/S.

739
Q
A pension liability, or asset, is required if the projected benefit obligation differs from:
A
The vested benefit obligation
B
The accumulated benefit obligation
C
Contributions to the plan
D
The fair value of plan assets
A

d

Explanation:
The recognition of a liability, or asset, is required if the projected benefit obligation differs from the fair value of the plan assets. A liability is required if the projected benefit obligation exceeds the fair value of the plan assets. This liability amount equals the unfunded projected benefit obligation. An asset is recognized if the fair value of the plan assets exceeds the projected benefit obligation. The asset amount equals the overfunded projected benefit obligation.

740
Q
Dell Co. adopted a defined benefit pension plan on January 1, year 1. Dell amortizes the prior service cost over 16 years and funds prior service cost by making equal payments to the fund trustee at the end of each of the first ten years. The service (normal) cost is fully funded at the end of each year. The following data are available for the current year:
Service (normal) cost for the year	$220,000
Prior service cost:	
Amortized	83,400
Funded	114,400Dell's pension asset at December 31, prior to comparing the fair value of plan assets to the projected benfit obligation, is
A
$114,400
B
$83,400
C
$31,000
D
$0
A

c

Explanation:
During the year, Dell accrued pension cost of $303,400 ($220,000 + $83,400) and made funding payments of $334,400 ($220,000 + $114,400). At 12/31, Dell’s pension asset, prior to comparing the fair value of plan assets to the projected benefit obligation, is $31,000 ($334,400 - $303,400).

Note: The asset recognized on the balance sheet is the amount the fair value of plan assets exceeds the projected benefit obligation.

741
Q

For a defined benefit pension plan, the discount rate used to calculate the projected benefit obligation is determined by the
Expected return on plan assets Actual return on plan assets
A Yes Yes
B No No
C Yes No
D No Yes

A

b

Explanation:
The projected benefit obligation is determined using the settlement rate and not the expected return plan assets nor the actual return on plan assets. These two rates, however, do have an impact on the determination of net periodic pension cost.

742
Q
A company has a defined benefit pension plan for its employees. On December 31, year 1, the accumulated benefit obligation is $45,900, the projected benefit obligation is $68,100, and the fair value of the plan assets is $62,000. What amount, if any, related to the defined benefit plan should be recognized in the balance sheet at December 31, year 1?
A
An asset of $16,100.
B
A liability of $6,100.
C
Nothing, as the fair value of the plan assets exceeds the accumulated benefit obligation.
D
An unrealized loss of $6,100.
A

Explanation:
The amount of the total liability to be recognized equals the unfunded projected benefit obligation.

Projected benefit obligation $68,100
Less: Plan assets at fair value (62,000)
Amount of total liability to be recognized $ 6,100

743
Q

On June 1 of the previous year, Oak Corp. granted stock options valued at $8,000 to certain key employees as additional compensation for the year. The
options were for 1,000 shares of Oak’s $2 par value common stock at an option price of $15 per share. Market price of this stock on June 1 of the previous
year was $20 per share. The options were exercisable beginning January 2 of the current year and expire on December 31 of next year. On April 1 of the
current year, when Oak’s stock was trading at $21 per share, all the options were exercised. What amount of pretax compensation should Oak report in the
previous year in connection with the options?

A
$8,000
B
$5,000
C
$2,500
D
$2,000
A

Explanation:
The correct answer is (A).

Stock options are granted to specific employees to purchase the entity’s stock at a price that offers special benefits. As these are a compensatory stock option, Oak Corp. would record compensation expense which is equal to grant date fair value of the options.

Fair Value of the options is already given to $8,000.

Also, the account Stock Options Outstanding is increased on the grant date. The subsequent exercising, forfeiture, or lapsing of the stock options reduces this account.
The following Journal Entry would be recorded:

Compensation Cost $8,000
Stock Options Outstanding $8,000

Note: If the fair value of the option was not available, an entity would have estimated the fair value of options using an option pricing-model as follows:

Fair Value of Options = Market Price of Share – Exercise Price of Share

= $20 - $15 = $5 per option or $5,000 for 1,000 options.

744
Q
In the terminology of single-employer defined benefit pension plans, \_\_\_\_\_\_\_\_\_\_\_ means the actuarial present value, as of a date, of all benefits attributed by the pension benefit formula to employee service rendered prior to that date, and may be measured using assumptions as to future compensation levels.
A
Accumulated benefit obligation
B
Projected benefit obligation
C
Unfunded accumulated benefit obligation
D
Vested benefit obligation
A

b

Explanation:
A projected benefit obligation is the actuarial present value, as of a date, of all benefits attributed by the pension formula to employee service rendered prior to that date. The projected benefit obligation is measured using assumptions as to future compensation levels if the pension benefit formula is based on those future compensation levels (pay-related, final-pay, final-average-pay, or career-average-pay plans).

745
Q
The stockholders of Meadow Corp. approved a stock-option plan that grants the company's top three executives options to purchase a maximum of 1,000 shares each of Meadow's $2 par common stock for $19 per share. The options were granted on January 1 when the fair value of the stock was $20 per share. Meadow determined that the fair value of the compensation is $300,000 and the vesting period is three years. What amount of compensation expense from the options should Meadow record in the year the options were granted?
A
$ 20,000
B
$ 60,000
C
$100,000
D
$300,000
A

c

Explanation:
The cost of services received from employees in exchange for awards of share-based compensation generally shall be measured based on the grant-date fair value of the options. The account Stock Options Outstanding is increased on the grant date. The subsequent exercising, forfeiture, or lapsing of the stock options reduces this account.

January 1, Year 1
Deferred Compensation Cost (option/compensation value at grant date)	300,000	
Stock Options Outstanding		300,000
December 31, Year 1, 2, 3
Wages and Compensation Expense	100,000	
Deferred Compensation Cost		100,000
746
Q
On January 1, year 1, a company issued its employees 10,000 shares of restricted stock. On January 1, year 2, the company issued to its employees an additional 20,000 shares of restricted stock. Additional information about the company’s stock is as follows:
Date	Fair value of stock (per share)
January 1, year 1	$20
December 31, year 1	22
January 1, year 2	25
December 31, year 2	30The shares vest at the end of a four-year period. There are no forfeitures. What amount should be recorded as compensation expense for the 12-month period ended December 31, year 2?
A
$175,000
B
$205,000
C
$225,000
D
$500,000
A

a

Explanation:
Restricted stock gives employees the right to acquire or receive shares, by gift or purchase, once certain restrictions, such as working a certain number of years. Usually, the vesting restriction lapses if the employee continues to work for the company for a certain number of years, say three to five. Restricted stock accounting parallels option accounting in most respects. If the only restriction is time-based vesting, companies account for restricted stock by recording total compensation cost using fair value on the award date. The cost is then amortized over the vesting period. In this case, one-fourth of the year 1 and onefourth of year 2 compensation cost would be recognized as compensation expense for year 2 [(10,000$201/4) + (20,000$251/4) = $175,000].

747
Q
\_\_\_\_\_\_\_\_\_\_\_\_ is the contribution called for by the plan for the period in which the individual renders services.
A
Defined contribution
B
Defined benefit pension
C
Net pension cost
D
Attribution
A

c

Explanation:
The net pension cost is the contribution called for by the defined contribution plan for the period in which the individual renders services.

748
Q

As of December 31, of the current year, the accumulated pension benefit obligation and plan assets of a defined benefit pension plan sponsored by Crouse, Inc., were:

Accumulated pension benefit obligation $500,000
Plan assets at fair value 425,000
Transition obligation $ 75,000
Crouse elected to apply GAAP provisions for employers’ accounting for pension benefits other than pensions, in its financial statements for the current year ended December 31 and recognize the transition amount on a delayed basis as a component of net periodic pension benefit cost. The average remaining service period of active plan participants expected to receive benefits was estimated to be 10 years at the date of transition. Some participants’ estimated service periods are 25 years. To minimize an accrual for pension benefit-cost, what amount of transition obligation should Crouse amortize?

A
$3,000
B
$5,000
C
$3,750
D
$7,500
A

Explanation:
The correct answer is (B).

The $75,000 of transition obligation at the beginning of the current year, the first fiscal year in which Crouse applied GAAP, is computed as the excess at that date of the accumulated pension benefit obligation over the fair value of plan assets (i.e., $500,000 - $425,000).

Typically, if the transition amount is recognized on a delayed basis as a component of net periodic pension benefit cost, it should be amortized on a straight-line basis over the average remaining service period of employees expected to receive benefits under the plan.

However, if the average remaining service period is less than 15 years, the employer may elect to use a 15-year period. Since in this question, the average remaining service life is 10 years, (which is less than 15, obviously), we would amortize over 15 years. If the remaining service life was 20, however, we would amortize it over the remaining life (20) in that instance.

Therefore, the amount of the transition obligation that Crouse should amortize to minimize its accrual for pension benefit-cost is $5,000 (i.e., $75,000 / 15).

749
Q
In situations where a loss results in a tax refund or tax savings, the provision is referred to as which of the following?
A
Pretax financial income
B
Income tax expense
C
Income tax benefit
D
Provision for income taxes
A

c

Explanation:
Where a loss situation results in a tax refund or tax savings, the provision is referred to as income tax benefit. Expenses incurred for the period are deducted from revenues earned for the period to arrive at pretax financial income. Income tax expense (often called provision for income taxes) is deducted from pretax financial income to arrive at net income.

750
Q
Quinn Co. reported a net deferred tax asset of $9,000 in its December 31, year 1, balance sheet. For year 2, Quinn reported pretax financial statement income of $300,000. Temporary differences of $100,000 resulted in taxable income of $200,000 for year 2. At December 31, year 2, Quinn had cumulative taxable differences of $70,000. Quinn's effective income tax rate is 30%. In its December 31, year 2, income statement, what should Quinn report as deferred income tax expense?
A
$12,000
B
$21,000
C
$30,000
D
$60,000
A

c

Explanation:
Deferred income tax expense reported for a period is determined by the net change during the year in the deferred tax accounts on the balance sheet. For Quinn, the elimination of the beginning $9,000 net deferred tax asset and the creation of the ending $21,000 ($70,000 cumulative taxable differences × 30% effective income tax rate) net deferred tax liability results in $30,000 of deferred income expense for year 2.

751
Q

Dunn Co.’s current year income statement reported $90,000 income before provision for income taxes. To compute the provision for federal income taxes, the following data are provided:

Rent received in advanced $16,000
Income from exempt municipal bonds 20,000
Depreciation deducted for income tax purposes in excess of
depreciation reported for financial statement purposes 10,000
Estimated tax payments 0
Enacted corporate income tax rate 21%
What amount of current federal income tax liability should be reported in Dunn’s December 31 balance sheet?

A
$12,600
B
$15,960
C
$18,060
D
$20,160
A

Explanation:
To determine Dunn’s current federal income tax liability, Dunn’s pretax financial income is adjusted to its taxable income; then, taxable income is multiplied by the current year’s enacted corporate income tax rate.

Pretax financial income $90,000
Rent received in advance 16,000
Income from municipal bonds, tax-exempt (20,000)
Depreciation deducted for income tax purposes in excess of reported for book purposes (10,000)
Taxable income 76,000
Enacted corporate income tax rate × 21%
Current federal income tax liability $15,960

752
Q

For the year ended December 31, year 1, Mont Co.’s books showed income of $600,000 before provision for income tax expense. To compute taxable income for federal income tax purposes, the following items should be noted.

Income from exempt municipal bonds $ 60,000
Depreciation deducted for tax purposes in excess of depreciation
recorded on the books 120,000
Proceeds received from life insurance from the death of an officer 100,000
Estimated tax payments 0
Enacted corporate rate 21%
What amount should Mont report on December 31, year 1, as its current federal income tax liability?

A
$67,200
B
$79,800
C
$105,000
D
$113,400
A

Explanation:
The correct answer is (A).

Tax liability is based on taxable income, so first reconcile book income of $600,000 to taxable income, then compute the tax liability. All three items listed are IN book income but are NOT in taxable income. $600,000 - 60,000 - 120,000 - 100,000 = $320,000 × 21% = $67,200.

(Note that depreciation is a temporary difference and is a future taxable amount; the remaining two items are permanent differences).

(B) is incorrect because income from municipal bonds is to be deducted from book income to calculate taxable income.

(C) is incorrect because income from municipal bonds and excess depreciation recorded for tax purposes are to be deducted from book income to calculate taxable income.

(D) is incorrect because proceeds received from life insurance due to the death of an officer and excess depreciation recorded for tax purposes are to be deducted from book income to calculate taxable income.

753
Q

Fry Corp., a construction company, appropriately uses an adjusted version of the percentage-of-completion method in accordance with the input method prescribed in ASC 606 and recognizes revenue “over time” in its financial statements and at a “point in time” for income tax reporting. Pertinent data at December 31, year 2, the close of Fry’s first year of operations, are as follows.

Date
contract began	Estimated
completion date	Income recognized in
year 2 on each contract
3/1/2	9/1/3	$600,000
6/1/2	12/1/3	300,000
9/1/2	3/1/4	200,000
12/1/2	6/1/4	100,000
Fry's enacted income tax rates are 30% for year 2, 25% for year 3, and 20% for year 4. Assuming Fry expects taxable income in all future periods, what amount should be included in the deferred income tax liability at December 31, year 2, for these transactions?
A
$360,000
B
$330,000
C
$285,000
D
$240,000
A

c

Explanation:
Deferred tax liability is always based on future taxable amounts times future tax rates. Subtotal the temporary differences in year 3 and year 4, then apply the respective future rate as follows: (900,000 × 25%) + (300,000 × 20%) = 225,000 + 60,000 = $285,000 deferred income tax liability.

754
Q

Shear, Inc., began operations in year 1. Included in Shear’s year 1 financial statements were bad debt expenses of $1,400. For tax purposes, the bad debts will be deducted in year 3. The enacted tax rates are 30% in year 1 and 25% in year 3. Shear expects to be profitable in all future years. In its year 1 income statement, what amount should Shear report as deferred income tax assets?

A
$300
B
$350
C
$650
D
$780
A

Explanation:
The correct answer is (B).

Deferred tax Assets is always computed on temporary timing differences giving rise to future taxable amounts or future deductible amounts, at the future tax rate. Since Bad Debts will not be allowed as a deduction in Year 1.

Tax Income is higher than Book Income giving rise to deferred tax assets.


The temporary timing differences = $1,400 × 25% = $350 Deferred Tax Assets.

755
Q

In its first four years of operations ending December 31, year 2, Alder, Inc.’s depreciation for income tax purposes exceeded its depreciation for financial statement purposes. This temporary difference was expected to reverse in years 3, 4 and 5. Alder had no other temporary difference. Alder’s year 2 balance sheet should include
A
A noncurrent contra asset for the effects of the difference between asset bases for financial statement and income tax purposes.
B
Both current and noncurrent deferred tax assets.
C
A current deferred tax liability only.
D
A noncurrent deferred tax liability only.

A

d

Explanation:
A temporary difference caused by the excess of depreciation deducted for tax purposes over the depreciation reported for financial statement purposes will result in net taxable amounts in the periods the temporary difference reverses. A deferred tax liability should be reported for the amount of deferred tax consequences attributable to temporary differences that will result in net taxable amounts in future years. Deferred tax accounts are classified as current or noncurrent based on the classification of the related asset or liability. Depreciation relates to plant assets which are a noncurrent asset classification. Hence, the entire related deferred tax liability is a noncurrent liability.

756
Q

Tell Corp.’s year 5 income statement had pretax financial income of $38,000 in its first year of operations. Tell uses an accelerated cost recovery method on its tax return and straight-line depreciation for financial reporting.

The differences between the book and tax deductions for depreciation over the five-year life of the assets acquired in year 5, and the enacted tax rates for years 5 to 9, are as follows:

Year	Book over (under) tax	Tax rates
5	$ (8,000)	35%
6	(13,000)	30%
7	(3,000)	30%
8	10,000	25%
9	14,000	25%There are no other temporary differences. Taxable income is expected in all future years. In Tell's December 31, year 5 balance sheet, the deferred income tax liability and the income taxes currently payable should be
Deferred income tax liability	Income taxes currently payable
A	$1,200	$13,300
B	$1,200	$10,500
C	$2,800	$13,300
D	$2,800	$10,500
A

Explanation:
To determine the amount of Tell’s income taxes currently payable at December 31, year 5, Tell’s year 5 taxable income must be multiplied by the year’s income tax rate [($38,000 - $8,000) × 35% = $10,500].

The amount of deferred income tax liability that should be reported at December 31, year 5 is computed by considering the future tax effects of the temporary difference for depreciation. While straight-line depreciation is used in computing financial income, an accelerated cost recovery method is used for tax purposes. This causes the amount of an asset to exceed its tax basis which will result in deductible amounts of $13,000 in year 6 and $3,000 in year 7 and taxable amounts of $10,000 in year 8 and $14,000 in year 9 when the asset is recovered through use. The future taxable (deductible) amounts are tax-effected at the tax rates enacted for the individual particular future years affected by this temporary difference. The $1,200 net liability amount is determined by the following schedule.

Taxable in Current Year ($30,000)
Taxable in Future Years Year 6 Year 7 Year 8 Year 9 Totals
Future taxable amounts (deductible) $(13,000) $(3,000) $10,000 $14,000 $8,000
Enacted tax rates 30% 30% 25% 25% _____
Deferred tax liability (asset) $ (3,900) + $ (900) + $ 2,500 + $ 3,500 = $1,200

757
Q

At December 31, Dorr, Inc., has a net operating loss carryforward of $90,000 available to offset future taxable income. At this date, Dorr has temporary differences that will result in taxable amounts of $60,000 during the operating loss carryforward period. The company has sufficient positive evidence to support an assumption that the benefits of the carryforward will be realized in the near future. Assuming a present and future enacted income tax rate of 30%, what amount of the tax benefit of the operating loss carryforward should be recognized in the income statement for the year ended December 31?

A
$0
B
$ 9,000
C
$18,000
D
$27,000
A

Explanation:
The correct answer is (D).

Per the Tax Cuts and Jobs Act (TCJA), C Corporations are not allowed to carry back losses. They can only be carried forward.

The Net operating loss can be fully claimed. The income tax benefit realized is $27,000 (i.e. $90,000 x 30%).

A deferred tax asset is to be recognized for the future tax benefits of a loss carryforward ($90,000 × 30% future tax rate = $27,000).

A valuation allowance is to be established only if it is more likely than not that a portion or all of the asset will not be realized.

Dorr expects to realize the asset and does not need a valuation allowance.

758
Q
Kent, Inc.'s reconciliation between financial statement and taxable income for year 7 follows:
Pretax financial income	$150,000
Permanent difference	  (12,000)
138,000
Temporary difference: depreciation	   (9,000)
Taxable income	$129,000
Additional information:
At 12/31, year 6	At 12/31, year 7
Cumulative temporary differences (future taxable amounts)	$11,000	$20,000The enacted tax rate was 34% for year 6, and 40% for year 7 and years thereafter. In its year 7 income statement, what amount should Kent report as current portion of income tax expense?
A
$51,600
B
$55,200
C
$55,860
D
$60,000
A

Explanation:
Current portion of income tax expense is always based on the current taxable income at the current tax rate, so $129,000 × 40% = $51,600..

759
Q

In accounting for income taxes, justification for the method of determining periodic deferred tax expense is based on the concept of
A
Matching of periodic expense to periodic revenue.
B
Objectivity in the calculation of periodic expense.
C
Recognition of assets and liabilities.
D
Consistency of tax expense measurements with actual tax planning strategies.

A

Explanation:
In accounting for income taxes, an enterprise shall recognize a deferred tax liability or asset for all temporary differences. After computing the balances of deferred tax accounts at a balance sheet date, the amount of deferred income tax expense is computed by determining the change required during the period in the deferred tax asset and liability accounts. That change is a by-product of the amount of deferred taxes (asset and/or liability) to be reported on the balance sheet at the balance sheet date. The method prescribed is often called the ‘liability’ method or the ‘asset-liability’ method.

760
Q
Black Co., organized on January 2, year 1, had pretax accounting income of $500,000 and taxable income of $800,000 for the year ended December 31, year 1. The only temporary difference is accrued product warranty costs which are expected to be paid as follows:
Year 2	$100,000
Year 3	50,000
Year 4	50,000
Year 5	100,000The enacted income tax rates are 35% for year 1, 30% for years 2 through 4, and 25% for year 5. Taxable income is expected in all future years. In Black's December 31, year 1 balance sheet, the deferred income tax asset should be
A
$75,000
B
$85,000
C
$90,000
D
$105,000
A

Explanation:
The amount of the deferred tax asset is computed by considering the tax effects of the temporary difference for accrued product warranty costs. While product warranty costs are accrued in computing financial income, they are deductible for tax purposes only when they are paid. This causes the amount of a liability (i.e., product warranty liability) to exceed its tax basis (zero) which will result in deductible amounts in future years when the liability is settled. A deferred tax asset should be recognized for the expected tax benefits of the deductible amounts. Because taxable income is expected in all future years, the enacted tax rates for particular years are used to tax effect the individual deductible amounts scheduled to occur in each of those years. Black reports a deferred tax asset of $85,000 [30% ($100,000 + $50,000 + $50,000) + 25% ($100,000)].

761
Q

In year 1, Lobo Corp. reported for financial statement purposes the following revenue and expenses which were not included in taxable income:

Premiums on officers’ life insurance under which the
corporation is the beneficiary $5,000
Interest revenue on qualified state or municipal bonds 10,000
Depreciation deducted for income tax purposes in excess of depreciation reported
for financial statement purposes 10,000
Estimated future warranty costs to be paid in
year 2 and year 3 60,000
Lobo’s enacted tax rate for the current and future years is 30%. Lobo expects to operate profitably in the future. There were no temporary differences in prior years. The deferred tax asset is

A
$18,000
B
$19,500
C
$21,000
D
$22,500
A

Explanation:
The correct answer is (A).

The amount of deferred tax benefit to be applied against current income tax expense is computed by considering the tax effects of the difference for accrued product warranty costs.

Product warranty costs are accrued in computing financial income and deductible for tax purposes only when paid.

This causes the product warranty liability to exceed its tax basis (zero), which will result in deductible amounts in years when the warranty liability is settled.

A deferred tax asset (DTA) is recognized for the tax benefit of the future deductible amounts ($60,000 × 30% = $18,000).

There is no apparent need for a valuation allowance against that DTA; Lobo expects to operate profitably in the future.

Lobo had no temporary differences in prior years.

Recognition of the DTA requires a debit to Deferred Tax Asset and a credit to Income Tax Expense for $18,000.

The insurance premiums and interest revenue do not affect the recognition of the DTA because they are events recognized in financial statements that do not have tax consequences as they are permanent differences, so they do not affect deferred taxes.

Depreciation deducted for income tax purposes in excess of depreciation reported for financial statement purposes will result in a deferred tax liability.

762
Q

Purl Corporation’s income statement for the year ended December 31, shows the following:

Income before income tax $900,000
Gain on life insurance coverage– included in the above $900,000 income amount 100,000
Purl’s tax rate for the year is 30%. How much should be reported as the provision for income tax in Purl’s income statement?

A
$150,000
B
$180,000
C
$240,000
D
$270,000
A
Explanation:
Income before income taxes 	$900,000
Less: Nontaxable gain on life insurance coverage	(100,000)
Income before income taxes 	800,000
Applicable tax rate	×      30%
Provision for income taxes	$240,000
763
Q

On its current year, December 31 balance sheet, Shin Co. had income taxes payable of $13,000 and a current deferred tax asset of $20,000 before determining the need for a valuation account. Shin had reported a current deferred tax asset of $15,000 at December 31 of the previous year. No estimated tax payments were made during the current year. At December 31, current year, Shin determined that it was more likely than not, that 10% of the deferred tax asset would not be realized. In its current year income statement, what amount should Shin report as total income tax expense?

A
$ 8,000
B
$ 8,500
C
$10,000
D
$13,000
A

Explanation:
Income taxes payable $13,000
Plus: Deferred tax expense due to valuation allowance (10% × $20,000) $2,000
Less: Deferred tax benefit due to increase in deferred tax asset ($20,000 - $15,000) $(5,000)
Total Income tax expense $10,000
Option (a) is incorrect because it does not provide for the creating of valuation account of 10% thereby reducing closing balance of deferred asset by $2,000.Option (b) wrongly creates a valuation account on the opening deferred asset balance. A need for valuation account was made only in the current year and would therefore affect only the closing balance. Option (d) is incorrect because this incorrectly includes only the current tax payable and ignores the deferred tax asset for the year.

764
Q

Pine Corp.’s books showed pretax income of $800,000 for the current year ended December 31. In the computation of federal income taxes, the following data were considered:

Gain on an involuntary conversion (Pine has elected to replace the
property within the statutory period using total proceeds.) $350,000
Depreciation deducted for tax purposes in excess of depreciation
deducted for book purposes 50,000
Federal estimated tax payments 70,000
Enacted federal tax rates 30%
What amount should Pine report as its current federal income tax liability on its December 31 balance sheet?

A
$50,000
B
$65,000
C
$120,000
D
$135,000
A

Explanation:
Pine Corp.’s as on December 31, 20X6 balance sheet should report current federal income tax liability at $50,000. Gain on involuntary conversion was included inaccounting income will be deferred for tax purposes. Depreciation deducted for tax purposes in excess of book depreciation causes taxable income to be less than accounting income.

Ref

Summary

Amount

a

Income before taxes

$800,000

b

Gain on an involuntary conversion

(350,000)

c

Depreciation deducted for tax purposes in excess of depreciation

deducted for book purposes

(50,000)

d

Taxable Income (a - b - c)

$400,000

e

Corporate income tax rate

30%

f

Tax Payable (d x e)

$120,000

g

Federal estimated tax payments, 20X6

$70,000

h

Current federal income tax liability (f - g)

$50,000

Option (b) is incorrect because depreciation adjustment is not made to the taxable income {i.e. $65,000 = [($800,000 - $350,000) x 30%] - $70,000}. Option (c) is incorrect because federal estimated tax payments is not adjusted to arrive at current tax liability [i.e. $120,000 = ($800,000 - $350,000 - $50,000) x 30%]. Option (d) is incorrect because depreciation adjustment is not made to the taxable income and federal estimated tax payments is not considered in arriving at current liability [i.e. $135,000 = ($800,000 - $350,000) x 30%].

765
Q
Paxton Inc.'s reconciliation between financial statement and taxable income for year 6 follows.
Pretax financial income	$750,000
Permanent difference	  (25,000)
725,000
Temp difference: depreciation	(60,000)
Temp difference: disallowed warranty expense	   50,000
Taxable income	$715,000The enacted tax rate was 30% for year 6 and 35% for years thereafter. Paxton estimates that 10% of any deferred tax asset will never be realized. In its year 6 income statement, what would Paxton report as current income tax expense?
A
$214,500
B
$250,250
C
$219,750
D
$218,000
A

Explanation:
Income tax expense is always computed on current taxable income at current tax rates, as follows: $715,000 x 30% = $214,500.

766
Q
Under GAAP, enterprises should recognize the largest amount of tax benefit that is greater than \_\_\_\_\_ percent likely of being realized upon ultimate settlement with the taxing authority.
A
10
B
25
C
50
D
90
A

Explanation:
Under GAAP, enterprises should recognize the largest amount of tax benefit that is greater than 50 percent likely of being realized upon ultimate settlement with the taxing authority.

767
Q

Which of the following is true regarding the enacted tax rate?
A
The enacted tax rate is the expected tax rate applied to taxable income in the years that the liability is expected to be settled or the asset recovered, and is used to measure deferred tax liabilities and deferred tax assets.
B
The presently enacted changes in tax rates and laws must be considered when determining the tax rate to apply to temporary differences reversing in that year(s).
C
The enacted tax rate is the tax rate for the current year if no changes have been enacted for future years.
D
All of the above are true.

A

Explanation:
The tax rate that is used to measure deferred tax liabilities and deferred tax assets is the enacted tax rate(s) expected to apply to taxable income in the years that the liability is expected to be settled or the asset recovered. Presently enacted changes in tax laws and rates that become effective for a particular future year or years must be considered when determining the tax rate to apply to temporary differences reversing in that year(s). Tax laws and rates for the current year are used if no changes have been enacted for future years. Since answers A., B., and C. are correct, answer D., all of the above, is the best answer choice.

768
Q

Hut Co. has temporary taxable differences that will reverse during the next year and add to taxable income. These differences relate to noncurrent assets.
Deferred income taxes based on these temporary differences should be classified in Hut’s balance sheet as a

A
Current asset.
B
Noncurrent asset.
C
Current liability.
D
Noncurrent liability
A

Explanation:
The correct answer is (D).

Per FASB ASU 2015-17, deferred tax assets and liabilities are always netted and treated as non-current on the Balance Sheet.

769
Q

For interim financial reporting, a company’s income tax provision for the second quarter of the year should be determined using the
A
Effective tax rate expected to be applicable for the full year as estimated at the end of the first quarter.
B
Effective tax rate expected to be applicable for the full year as estimated at the end of the second quarter.
C
Effective tax rate expected to be applicable for the second quarter of the year.
D
Statutory tax rate for the year.

A

Explanation:
At the end of each interim period, a company should make its best estimate of the effective tax rate expected to be applicable for the full fiscal year. The rate so determined should be used in providing for income taxes on a current year-to-date basis.

770
Q

Which of the following statements is correct regarding valuation allowances in accounting for income taxes?

A
The effect of a change in the opening balance of a valuation allowance that results from a change of circumstances ordinarily is included in income from operations.
B
Both deferred tax assets and deferred tax liabilities can be reduced by a valuation allowance.
C
Only negative evidence, not positive evidence, should be considered when determining whether a valuation allowance is needed.
D
A valuation allowance is necessary when the realistic probability standard of evidence is satisfied.

A

Explanation:
The correct answer is (A)

The effect of a change in the opening balance of a valuation allowance that results from a change of circumstances ordinarily is included in income from operations.

Valuation Account Allowance: If it is more likely (>50%) that part or all the deferred tax assets will not be realized, a valuation allowance is recognized such that the net deferred tax assets equal that portion which is more likely than not to be realized. The effect of a change in the opening balance of a valuation allowance that results from a change of circumstances ordinarily is included in income from operations.

771
Q

Orleans Co., a cash basis taxpayer, prepares accrual basis financial statements. In its current year balance sheet, Orleans’ deferred income tax liabilities increased compared to the previous year. Which of the following changes would cause this increase in deferred income tax liabilities?

An increase in prepaid insurance.
An increase in rent receivable.
An increase in warranty obligations.
A
I only.
B
I and II.
C
II and III.
D
III only.
A

Explanation:
An increase in prepaid insurance can cause an increase in deferred tax liabilities because an expense deducted this period for tax purposes but deferred for financial accounting purposes will cause future taxable amounts. An increase in rent receivable can cause an increase in deferred tax liabilities because a reve­nue accrued for book purposes but not recognized for tax purposes until it is collected will give rise to future taxable amounts. A deferred tax liability represents the deferred tax consequences attributable to taxable tem­porary differences. An increase in cumulative temporary differences giving rise to future taxable amounts results in an increase in deferred tax liabilities. An increas in warranty obligation can cause an increase in deferred tax assets rather than deferred tax liabilities. An expense accrued for book purposes, but deducted for tax purposes when paid, causes a temporary difference which gives rise to future deductible amounts. A deferred tax asset is the deferred tax consequences attributable to deductible temporary differences and carry forwards.Options (a), (c) and (d) are incorrect based on the above explanation.

772
Q

Which of the following should be disclosed in a company’s financial statements related to deferred taxes?

The types and amounts of existing temporary differences
The types and amounts of existing permanent differences
The nature and amount of each type of operating loss and tax credit carry forward
A
I and II only
B
I and III only
C
II and III only
D
I, II, and III

A

Explanation:
The recognition and measurement of a deferred tax liability or asset is based on the future effects on income taxes, as measured by the provisions of enacted tax laws, resulting from temporary differences and operating loss and tax credit carry forwards at the end of the current year. An enterprise is to identify (disclose) the types and amounts of existing temporary differences and the nature and amount of each type of operating loss and tax credit carry forward (and remaining length of carry forward). Options (a), (c) and (d) are incorrect as per the above explanation.

773
Q
The amount to be reported for deferred taxes on a balance sheet is calculated by using \_\_\_\_\_\_\_\_\_\_\_\_\_\_\_ and applying them to cumulative \_\_\_\_\_\_\_\_\_ differences based on when and how those differences are expected to affect the tax return in the future.
A
Enacted tax rates; permanent
B
Future tax rates; temporary
C
Last year's tax rates; temporary
D
Current tax rates; permanent
A

Explanation:
The amount to be reported for deferred taxes on a balance sheet is calculated by using future tax rates and applying them to cumulative temporary differences based on when and how the temporary differences are expected to affect the tax return in the future.

774
Q

Taft Corp. uses the equity method to account for its 25% investment in Flame, Inc. During the current year, Taft received dividends of $30,000 from Flame and recorded $180,000 as its equity in the earnings of Flame. Additional information follows:

All the undistributed earnings of Flame will be distributed as dividends in future periods.
The dividends received from Flame are eligible for the 80% dividends received deduction.
There are no other temporary differences.
Enacted income tax rates are 30% for this year and thereafter.
In its December 31 balance sheet, what amount should Taft report for deferred income tax liability?

A
$9,000
B
$10,800
C
$45,000
D
$54,000
A

Explanation:
The investor recognizes investment income for financial purposes based on its equity in the investee’s earnings; for income tax purposes, investment income is recognized when cash dividends are received. Deferred income taxes are recorded on this temporary difference which will result in a future taxable amount.

Equity in investee’s earnings, book

$ 180,000

Less: Dividends received

  (30,000)

Temporary difference before DRD consideration

 150,000

Less: Dividends-received deduction applicable to temporary difference ($150,000×80%)

(120,000)

Taxable portion of temporary difference

   30,000

Times: Enacted tax rate for future periods

× 30%

Deferred tax liability

$ 9,000

775
Q

Concerning income tax guidance provided by GAAP, which of the following statements is (are) true?
A
GAAP does not provide guidance on accounting for uncertainty in income taxes recognized in an enterprise’s financial statements.
B
GAAP does not prescribe a recognition threshold and measurement attribute for the financial statement recognition and measurement of a tax position taken or expected to be taken in a tax return.
C
GAAP provides guidance on derecognition, classification, interest and penalties, accounting in interim periods, disclosure, and transition related to uncertain tax positions.
D
All of the above

A

Explanation:
GAAP provides guidance for uncertainty in income taxes recognized in an enterprise’s financial statements and prescribes a recognition threshold and measurement attribute for the financial statement recognition and measurement of a tax position taken or expected to be taken in a tax return.

776
Q

Income tax expense or benefit for the year should be allocated among
Discontinued operations Prior period adjustments
A Yes Yes
B Yes No
C No Yes
D No No

A

Explanation:
Income tax expense or benefit for the year shall be allocated among continuing operations, discontinued operations, extraordinary items, and items charged or credited directly to shareholders’ equity’ such as prior period adjustments, certain changes in accounting principles, and changes in market values of investments in marketable equity securities classified as noncurrent assets.

777
Q

Mill, which began operations on January 1, year 1, recognizes income from long-term construction contracts by applying an adjusted version of the percentage-of-completion method in accordance with the input method prescribed in ASC 606 and recognizes revenue “over time” in its financial statements and at a “point in time” for income tax reporting. Income under each method follows:

Year Recognition: Point in Time Recognition: Over Time
1 $ — $300,000
2 400,000 600,000
3 700,000 850,000
The income tax rate was 30% for years 1 through 3. For years after year 3, the enacted tax rate is 25%. There are no other temporary differences. Assuming Mill does not expect any tax losses in the near future, Mill should report in its December 31, year 3 balance sheet, a deferred income tax liability of

A
$ 87,500
B
$105,000
C
$162,500
D
$195,000
A

c

Explanation:
The $650,000 [($300,000 + $600,000 + $850,000) - ($400,000 + $700,000)] of contract income recognized in the financial statements in excess of that included in taxable income results in taxable amounts in future years. The enacted tax rate for later years is used to determine the deferred income tax liability (i.e., $650,000 × 25% = $162,500).

778
Q

Under current generally accepted accounting principles, which approach is used to determine income tax expense?

A
Asset and liability approach
B
"With and without" approach
C
Net of tax approach
D
Periodic expense approach
A

a

Explanation:
Income tax expense (deferred) is reported using the ‘liability method’ where the income statement figure flows from the balance sheet calculations of deferred asset and liability {think of it as the balance sheet is prepared first and then the income statement}. Thus, income tax expense is as per the asset and liability approach. Options (b), (c) and (d) are incorrect based on the above explanation.

779
Q

Generally, the manner of reporting the tax benefit of an operating loss carryforward or carryback is determined by the source of the

Income or loss
in the current year	Operating loss carry forward
or taxes paid in a prior year
A	Yes	Yes
B	Yes	No
C	No	Yes
D	No	No
A

Explanation:
The correct answer is (B).

Except for certain areas such as business combinations and quasi-reorganizations, the manner of reporting the tax benefits of an operating loss carryforward or carryback is determined by the source of the income (or loss) in the current year and not by the source of the operating loss carryforward or taxes paid in a prior year.

For example, the tax benefit of an operating loss carryforward reduces income tax expense from continuing operations if the source of that tax benefit is from continuing operations.

780
Q

On December 31, year 1, a publicly-traded entity identified a tax position that will result in a $100,000 tax benefit that qualifies for measurement and should be recognized. The entity has considered the amounts and possible outcomes of the position being sustained upon examination as follows:

Possible estimated outcome Individual probability of occurring Cumulative probability of occurring
$100,000 20% 20%
30,000 35% 55%
10,000 45% 100%
100%
What amount should be recognized as the tax benefit as of December 31, year 1?

A
$0
B
$10,000
C
$30,000
D
$100,000
A

Explanation:
The correct answer is (C).

Uncertain tax positions are accounted for in the financial statements only when the management has a more-likely-than-not level of confidence that the said position will be sustained by the tax authorities on technical grounds. If so, the largest amount of tax benefits which has more than 50% likeliness to be realized is ultimately recognized in the books.

In the given case, $30,000 is the largest tax benefit with a probability exceeding 50%. It is recognized as of December 31.

781
Q
For its first year of operations, Cable Corp. recorded a $100,000 expense in its tax return that will not be recorded in its accounting records until next year. There were no other differences between its taxable and financial statement income. Cable's effective tax rate for the current year is 45%, but a 40% rate has already been passed into law for next year. In its year-end balance sheet, what amount should Cable report as a deferred tax asset (liability)?
A
$40,000 asset
B
$40,000 liability
C
$45,000 asset
D
$45,000 liability
A

Explanation:
The balance in a tax liability account is the amount of taxes expected to be paid in the future as a result of the turn-around (reversal) of the temporary difference(s). Because Cable deducted an expense from its income tax before it was reportable in the financial statements, the transaction results in a future tax liability. The amount of temporary difference is multiplied times the future tax rate(s) already enacted into law to arrive at the future tax asset or liability. ($100,000 × .40 = $40,000).

782
Q

Busy Corp. prepared the following reconciliation between pretax accounting income and taxable income for the year ended December 31:
Pretax accounting income $250,000
Taxable income (150,000)
Difference $100,000
Analysis of difference:
Interest on municipal bonds $ 25,000
Excess of tax over book depreciation 75,000
$100,000
Busy’s effective income tax rate for the year is 30%. The depreciation difference will reverse in equal amounts over the next three years at an enacted tax rate of 40%.

In Busy’s income statement, what amount should be reported as the current portion of its provision for income taxes?

A
$45,000
B
$67,500
C
$75,000
D
$82,500
A
Explanation:
Pretax financial income	$250,000
Permanent difference - interest	(25,000)
Temporary difference - excess tax depreciation	  (75,000)
Taxable income	$150,000
Tax rate	×     30%
Current tax expense	$ 45,000
783
Q

Mobe Co. reported the following operating income (loss) for its first three years of operations:

2017

           $300,000

2018

            (700,000)

2019

           1,200,000

For each year, there were no deferred income taxes, and Mobe’s effective income tax rate was 30%. In its 2017 income tax return, Mobe elected to carry back the maximum amount of loss possible. In its 2019 income statement, what amount should Mobe report as total income tax payable?

A
$120,000
B
$360,000
C
$150,000
D
$240,000
A

Explanation:
The correct answer is (C).

The Tax Cuts and Jobs Act repeals pre-enactment NOL carryback provisions.

Under the new law, NOLs arising in years ending after December 31, 2017, may not be carried back (with limited exceptions for certain farming losses).

For NOLs arising in tax years ending after December 31, 2017, NOLs may be carried forward indefinitely. Additionally, NOLs originating in tax years beginning after December 31, 2017, are limited to 80% of taxable income.

Corporations will need to track NOLs for pre-12/31/2017 NOLs and post 12/31/2017 NOLs separately as only post 12/31/2017 losses will be subject to the 80% of taxable income limitation.

For the loss incurred in 2018, a carryback would not be allowed and will only be carried forward to offset the loss in next year. In this scenario, losses would be deductible only to the extent of 80% of taxable income i.e. 80% of $1,200,000 = $960,000.

The entire loss of $700,000 would be carried forward and tax payable is calculated on the remaining profit of $500,000 which will give a tax liability of $500,000 x 30% = $150,000.

784
Q

Dodd Corp.is preparing its December 31 current year financial statements and must determine the proper accounting treatment for the following situations:

For the current year ended December 31, Dodd has a loss carry forward of $180,000 available to off set future taxable income.However ,there are no temporary differences.Based on an analysis of both positive and negative evidence, Dodd has reason to believe it is more likely than not that the benefits of the entire loss carry forward will be realized with in the carry forward period.
On 12/31 of this year, Dodd received a $200,000 offer for its patent. Dodd’s management is considering whether to sell the patent. The offer expires on 2/28 of next year. The patent has a carrying amount of $100,000 at 12/31.
Assume a current and future income tax rate of 30%. In its current year income statement, Dodd should recognize an increase in net income of

A
$0
B
$54,000
C
$70,000
D
$124,000
A

Explanation:
A deferred tax asset is recognized for the future benefits of a loss carry forward. Therefore, Dodd will increase income by $54,000 ($180,000 × 30% future tax rate) when the benefits of the NOL carry forward are recorded in the current year by a debit to Deferred Tax Asset and a credit to Benefits of Loss Carry forward (a component of income tax expense on the income statement). No valuation allowance is required (which would reduce the deferred tax asset and the described impact on income) because the company expects the benefits of the loss carry forward to be realized in the future. In addition, the potential gain of $100,000 (i.e., $200,000 offer - $100,000 carrying amount) from the possible sale of the patent should not be recognized in the current year. Dodd Corp. has not sold the patent as of December 31; it is only considering whether to sell the patent. The excess of fair value over book value of the patent does serve, however, as some positive evidence in evaluating the realizability of the deferred tax asset related to the tax loss carry forward.

785
Q
Which of the following should be used to measure the deferred tax consequences of temporary differences that will result in taxable amounts in future years?
Enacted changes in tax laws and
rates scheduled for future years	Anticipated change in tax laws
and rates for future years
A	Yes	Yes
B	Yes	No
C	No	Yes
D	No	No
A

Explanation:
A deferred tax liability is computed at the date of the financial statements by applying tax law provisions to measure the deferred tax consequences of differences that will result in taxable amounts in each future year. Enacted changes in tax laws and rates that are scheduled for a particular future year (or years) are used to measure a liability for the deferred tax consequences of taxable amounts that will arise in that year (or years). Tax laws and rates for the current year are used if no changes have been enacted for future years.

786
Q

Ajax Corp. has an effective tax rate of 30%. On January 1, year 1, Ajax purchased equipment for $100,000. The equipment has a useful life of 10 years.

What amount of current tax benefit will Ajax realize during year 1 by using the 150% declining balance method of depreciation for tax purposes instead of the straight-line method?

A
$1,500
B
$3,000
C
$4,500
D
$5,000
A

Explanation:
The correct answer is (A).

The question is asking about the additional tax benefit Azax would receive for using the Double Declining Method instead of the Straight-Line Method.

Using the straight-line method of depreciation, Ajax would have a depreciation expense of $10,000 ($100,000 purchase price / 10 years).

The current tax benefit would be $3,000 ($10,000 depreciation expense × 30% effective tax rate).

Using the 150% declining balance method of depreciation, Ajax would have a depreciation expense of $15,000 ($100,000 purchase price / 10 years × 150%).

The current tax benefit would be $4,500 ($15,000 depreciation expense × 30% effective tax rate).

The amount of tax benefit realized by using the 150% declining method instead of the straight-line method would be the difference ($4,500 - $3,000).

787
Q

Rein Inc. reported deferred tax assets and deferred tax liabilities at the end of the previous year and at the end of the current year. For the current year ended, Rein should report deferred income tax expense or benefit equal to the
A
Decrease in the deferred tax assets.
B
Increase in the deferred tax liabilities.
C
Amount of the current tax liability plus the sum of the net changes in deferred tax assets and deferred tax liabilities.
D
Sum of the net changes in deferred tax assets and deferred tax liabilities.

A

d

Explanation:
Deferred income tax expense or benefit is calculated using the asset and liability method. Under this approach, deferred income tax expense or benefit is equal to the sum of the net changes in deferred taxes assets and deferred tax liabilities on the balance sheet.

788
Q

At the end of year 3, the tax effects of temporary differences for Thorn Co. were as follows:

Accelerated tax depreciation ($75,000) Noncurrent liability
Additional costs in inventory for tax purposes $25,000 Noncurrent asset
($50,000)
A valuation allowance was not considered necessary. Thorn anticipates that $10,000 of the deferred tax liability will reverse in year 4. In Thorn’s December
31, year 3 balance sheet, what amount should Thorn report as a deferred tax liability?

A
$40,000
B
$75,000
C
$65,000
D
$50,000
A

Explanation:
Deferred tax assets and liabilities are classified as non-current only.

Further, US GAAP allows separate netting of deferred tax assets and liabilities resulting in a single non-current amount to be reported on the balance sheet.

The deferred tax liability is $50,000 ($75,000 - $25,000).

789
Q

Bard Co., a calendar-year corporation, reported income before income tax expense of $10,000 and income tax expense of $1,500 in its interim income statement for the first quarter of the year. Bard had income before income tax expense of $20,000 for the second quarter and an estimated effective annual rate of 25%. What amount should Bard report as income tax expense in its interim income statement for the second quarter?

A
$3,500
B
$5,000
C
$6,000
D
$7,500
A

Explanation:
The taxes are accounted for the total taxable income for the year. The effective tax rate is updated at each interim date and the provision for income taxes in later quarters will be based on current estimated rate applied to cumulative income reduced by provisions reported in early periods. The calculation for income tax expense in second quarter is as follows:

Ref	Summary	Amount ($)
a	Q1 income	10,000
b	Q2 income	20,000
c	Total (a+b)	30,000
d	Tax expense @25% on total Income	7,500
e	Less: Tax expense Q1	(1,500)
f	Tax expense Q2 (d+e)	6,000
Options (A), (B) and (D) are incorrect based on the above calculation.
790
Q

Because Jab Co. uses different methods to depreciate equipment for financial statement and income tax purposes, Jab has temporary differences that will reverse during the next year and add to taxable income. Deferred income taxes that are based on these temporary differences should be classified in Jab’s balance sheet as a

A
Contra account to current assets.
B
Contra account to noncurrent assets.
C
Current liability.
D
Noncurrent liability.
A

Explanation:
Deferred taxes are reported as assets or liabilities on the balance sheet, not as contra accounts. Always classify deferred tax assets and liabilities as non-current. FASB issued Accounting Standards Update 2015-17 requiring companies to classify all deferred tax assets and liabilities as non-current on the balance sheet [IFRS always had this requirement]. The Board released the new guidance as part of its simplification initiative, which,as explained in the ASU, is intended to “identify, evaluate, and improve areas of US GAAP for which cost and complexity can be reduced while maintaining or improving the usefulness of the information provided to the users of the financial statements”. This is effective from December 15, 2016 for public companies and December 15, 2017 for non-public companies. Earlier: Deferred tax assets and liabilities were segregated into current and non-current amounts. Thus, it is a taxable temporary difference resulting in a non-current deferred tax liability.Options (a), (b) and (c) are incorrect based on the above explanation

791
Q

Concerning the measurement of deferred tax accounts, which of the following statements is false?

A
A current tax liability or asset is recognized for the estimated taxes payable or refundable on tax returns for the current year.
B
A deferred tax liability or asset is recognized for the estimated future tax effects attributable to temporary differences and carryforwards.
C
The measurement of current and deferred tax liabilities and assets is based on the provisions of future changes in tax laws and rates.
D
The measurement of deferred tax assets is reduced, if necessary, by the amount of any tax benefits that, based on available evidence, are not expected to be realized.
Explanation:

A

Explanation:
The measurement of current and deferred tax liabilities and assets is based on provisions of the enacted tax law; the effects of future changes in tax laws or rates are not anticipated.Options (a), (b) and (d) are incorrect as per above explanation.

CLOSE

792
Q

Which of the following items is not subject to the application of intra-period income tax allocation?

I. Discontinued operations
II. Income from continuing operations
III. Operating income

A
I, II, and III
B
I only
C
II only
D
III only
A

Explanation:
The correct answer is (D).

Intra-period tax allocation is allocating income taxes to key components of the income statement. Operating income is not subject to intra-period tax allocation.

Tax is deducted from Income from Continuing Operations and Income (Loss) from Discontinued Operations is presented net of tax.

793
Q

Paxton Inc.’s reconciliation between financial statement and taxable income for year 6 follows.

Pretax financial income $750,000
Permanent difference (25,000)
725,000
Temp difference: depreciation (60,000)
Temp difference: disallowed warranty expense 50,000
Taxable income $715,000
The enacted tax rate was 30% for year 6 and 35% for years thereafter. Paxton estimates that 10% of any deferred tax asset will never be realized. In its December 31, year 6 balance sheet, what amount would Paxton report as deferred income tax liability?

A
$18,000
B
$21,000
C
$18,900
D
$17,500
A

Explanation:
A deferred tax liability is generated from a temporary difference that is classified as a future taxable amount, computed using future tax rates. The excess depreciation expense is deductible currently for tax purposes, but not for book purposes. The deferred tax liability is computed as follows: 60,000 × 35% = $21,000.

794
Q
A deferred tax liability or asset should be adjusted for the effect of a change in
Tax laws	Tax rates
A	No	No
B	No	Yes
C	Yes	No
D	Yes	Yes
A

Explanation:
Deferred tax liabilities and assets shall be adjusted for the effect of a change in tax laws or rates. The effect shall be included in income from continuing operations for the period that includes the enactment date.

795
Q

Lion Co.’s income statement for its first year of operations shows pretax income of $6,000,000. In addition, the following differences existed between Lion’s tax return and records:

Tax return Accounting records

Uncollectible accounts expense $220,000 $250,000
Depreciation expense 860,000 570,000
Tax-exempt interest revenue — 50,000
Lion’s current year tax rate is 30% and the enacted rate for future years is 40%. What amount should Lion report as deferred tax expense in its income statement for the year?

A
$148,000
B
$124,000
C
$104,000
D
$78,000
A

Explanation:
Deferred tax liabilities or assets are recognized for the future tax consequences of, among other things, revenues, expenses, gains, or losses that are included in taxable income of an earlier or later year than the year in which they are recognized. The difference in uncollectible accounts expense between the tax return and the accounting records would result is a $30,000 deferred tax asset. The difference in depreciation expense between the tax return and the accounting records would result in a $290,000 deferred tax liability. The differ­ence in tax-exempt interest revenue between the tax return and the accounting records results in no tax asset or liability because the revenue was tax exempt. This results in a net future tax liability of $260,000. The$260,000 times the future tax rate of 40% equals $104,000.Option (a) is incorrect because it is adding the deferred tax asset on uncollectible accounts instead of deducting it from deferred tax liability and included permanent difference arising on tax-exempt revenue in deferred taxes [$148,000 = ($30,000 +$290,000 + $50,000) x 40%]. The permanent difference will not result in deferred taxes. Option (b) is incorrect because it includes tax-exempt revenue in deferred tax expense [$124,000 = ($290,000 + $50,000 - $30,000) x 40%]. Option (d) is incorrect because it uses the current year’s tax rate, instead of future tax rate at 40%. [$78,000 = ($290,000 - $30,000) x 30%].

796
Q

In year 2, Ajax, Inc. reported taxable income of $400,000 and pretax financial statement income of $300,000. The difference resulted from $60,000 of nondeductible premiums on Ajax’s officers’ life insurance and $40,000 of rental income received in advance. Rental income is taxable when received. Ajax’s effective tax rate is 30%. In its year 2 income statement, what amount should Ajax report as income tax expense?

A
$ 90,000
B
$102,000
C
$108,000
D
$120,000
A

Explanation:
The correct answer is (D).

Pretax financial statement income is determined on an accrual basis. Most revenues and expenses are reported on the tax return in the same period that they are reported on the income statement. However, tax laws often differ from the recognition and measurement requirements of financial accounting standards, and it is common to find differences between the amount of pretax financial income and the amount of taxable income for a period. The $60,000 difference for officer’s life insurance premiums is a permanent difference that does not give rise to any deferred tax assets or liabilities. The $40,000 difference for rental income received in advance is a temporary difference that will result in a deductible amount when the liability is settled.The income tax expense for the year is the $400,000 taxable income × the 30% tax rate = $120,000.

Option (a) is incorrect because it calculates income tax expense based on pre tax financial statement income. ($90,000 = $300,000 x 30%).

Option (b) is incorrect because it is the income tax expense if the non-deductible premiums are excluded from the taxable income. [$102,000 = ($400,000-$60,000)x 30%].

Option (c) is incorrect because it is the income tax expense if the rental income is excluded from the taxable income. [$108,000 = ($400,000-$40,000) x 30%].

797
Q
For the current year ended December 31, Tyre Co. reported pretax financial statement income of $750,000. Its taxable income was $650,000. The difference is due to accelerated depreciation for income tax purposes. Tyre's effective income tax rate is 30%, and Tyre made estimated tax payments during the year of $90,000. What amount should Tyre report as current income tax expense for the year?
A
$105,000
B
$135,000
C
$195,000
D
$225,000
A

Explanation:
The amount to be reported as current tax expense is computed by multiplying taxable income by the effective income tax rate ($650,000 × 30% = $195,000).

798
Q

Which of the following should be recognized for the amount of deferred tax consequences attributable to temporary differences that will result in taxable amounts in future years?
Deferred tax asset Deferred tax liability
A Yes Yes
B Yes No
C No Yes
D No No

A

Explanation:
A deferred tax liability is recognized for the amount of deferred tax consequences attributable to temporary differences that will result in taxable amounts in future years. The liability is the amount of taxes that will be payable on those taxable amounts in future years based on the provisions of the tax law. On the other hand, a deferred tax asset is recognized for the amount of deferred tax consequences attributable to temporary differences that will result in tax deductions in future years which will reduce taxes payable in those future years.

799
Q

Which of the following statements is (are) true regarding temporary differences?

A
All temporary differences must be identified with a particular asset or liability for financial reporting.
B
When there is a temporary difference that results from an event that has been recognized in the financial statements, that difference will result in taxable or deductible amounts in future years.
C
Interest earned on state and municipal obligations is an example of a temporary difference.
D
All of the above

A

Explanation:
When there is no related, identifiable asset or liability for financial reporting, but there is a temporary difference that results from an event that has been recognized in the financial statements, that difference will result in taxable or deductible amounts in future years.

800
Q

Bart, Inc., a newly organized corporation, uses the equity method of accounting for its 30% investment in Rex Co’s common stock. During the current year, Rex paid dividends of $300,000 and reported earnings of $900,000. In addition:

The dividends received from Rex are eligible for the 65% dividends-received deduction.
All the undistributed earnings of Rex will be distributed in future years.
There are no other temporary differences.
Bart’s current year income tax rate is 30%.
The enacted income tax rate after the current year is 25%.
Bart expects taxable income in all future years.
In Bart’s December 31 current year balance sheet, the deferred income tax liability should be

A
$10,800
B
$15,750
C
$5,400
D
$4,500
A

Explanation:
The correct answer is (B).

This question involves a temporary difference because the investor recognizes investment income for financial purposes based on its equity in the investor’s earnings, whereas for tax purposes, investment income is recognized on the cash basis when dividends are received.

Deferred taxes must be recorded on this temporary difference. The increase in the investor’s deferred income tax liability for this temporary difference is computed as follows:

Income per Books ($900,000 x 30%) (a) $270,000
Permanent Difference (65% DRD) [b = 65% x a] ($175,500)
Taxable Income per Books (c) = (a) – (b) $94,500
Temporary Difference (c) – (d) ($63,000)
Income Per Taxes (Dividend - 65% of Dividend) (Dividend x 35%) ($300,000 x 30% x 35%) (d) $31,500
Deferred Tax Liability = Temporary Difference x Tax Rate

= $63,000 x 25%

= $15,750

801
Q
In its year 1 income statement, Noll Corp. reported depreciation of $400,000 and interest revenue on municipal obligations of $60,000. Noll reported depreciation of $550,000 on its year 1 income tax return. The difference in depreciation is the only temporary difference, and it will reverse equally over the next three years. Noll's enacted income tax rates are 35% for year 1, 30% for year 2 and 25% for year 3 and year 4. Assuming Noll expects to report taxable income in all future years, what amount should be included in the deferred income tax liability in Noll's December 31, year 1 balance sheet?
A
$40,000
B
$52,500
C
$63,000
D
$73,500
A

Explanation:
Interest income on municipal obligations is a tax-exempt revenue; therefore, no current or future tax consequences will result and no deferred taxes are recorded for this difference. The $150,000 ($550,000 - $400,000) difference due to depreciation is the only temporary difference existing at the balance sheet date. It will reverse equally over the next three years, resulting in future taxable amounts of $50,000 ($150,000 / 3). Noll expects to report taxable income in all future periods; therefore, the future taxable amounts are tax effected at the rates scheduled for individual future years. Therefore, the enacted rate of 30% should be used for year 2 and the enacted rate of 25% should be used for years 3 and 4.

Amount	Rate	Deferred Tax Liability
Future taxable amounts:			
Depreciation			
Year 2	$ 50,000	30%	$15,000
Years 3 and 4	$100,000	25%	25,000
Deferred tax liability			$40,000
802
Q

Which of the following statements is a primary objective of accounting for income taxes?

A
To compare an enterprise’s federal tax liability to its state tax liability
B
To identify all of the permanent and temporary differences of an enterprise
C
To estimate the effect of the tax consequences of future events
D
To recognize the amount of deferred tax liabilities and deferred tax assets reported for future tax consequences

A

Explanation:
The correct answer is (D).

According to FASB, the primary objective of accounting for income taxes is to recognize the amount of:

Taxes payable or refundable for the year.
Deferred tax liabilities and assets for the future tax consequences of events that have been recognized in an enterprise’s financial statements or tax returns.
Among the given options, the primary objective of accounting for income taxes is to recognize the amount of deferred tax liabilities and deferred tax assets reported for future tax consequences.

803
Q
Temporary differences arise when expenses are deductible for tax purposes
After they are recognized
in financial income	Before they are recognized
in financial income
A	No	No
B	No	Yes
C	Yes	Yes
D	Yes	No
A

Explanation:
Expenses that are deductible before or after they are recognized in financial income create a difference between the tax basis of an asset or liability and its reported amount in the financial statements. These differences result in taxable or deductible amounts in a future period(s) when the reported amount of the related asset or liability is recovered or settled, respectively. Temporary differences include all existing differences that will result in taxable or deductible amounts in future years.

804
Q

Busy Co. applied the provisions of the tax law to its taxable loss figure for the period. What period of time is Busy allowed to carryback and carryforward a net operating loss?

Carryback	Carryforward
A	2 years	15 years
B	0 years	Indefinitely
C	3 years	15 years
D	3 years	20 years
A

Explanation:
The correct answer is (B).

As per the TCJA, NOLs for tax years beginning in 2018 and onwards may be carried forward indefinitely but may be deducted only up to 80% of taxable income (wherein the taxable income is computed without regard to any NOL carryover).

805
Q

Rom Corp. began business in year 1 and reported taxable income of $50,000 on its year 1 tax return. Rom’s enacted tax rate is 21% for year 1 and future
years. The following is a schedule of Rom’s December 31, year 1, temporary differences in thousands of dollars:

 	Book basis over
(under) tax basis	Future taxable
(deductible) amounts
 	12/31, yr 1	Yr 2	Yr 3	Yr 4	Yr 5
Warranty liability	(20)	(10)	(10)	 	 
Deferred compensation liability	(15)	 	(5)	 	(10)
Installment receivables	30	10	 	20	 
What amount should Rom report as deferred tax assets in its December 31, year 1, balance sheet?
A
$1,050
B
$0
C
$3,150
D
$4,200
A

Explanation:
Deferred tax liabilities and assets are classified on the balance sheet as noncurrent.

Net Differences [(20) + (15) + 30] $(5,000)
Future Enacted Tax Rate 21%
Net Deferred Tax Assets $1,050

806
Q

Senlo Co., which uses a one-year operating cycle, recognized profits for both financial statement and tax purposes during its two years of operation.
Depreciation for tax purposes exceeded depreciation for financial statement purposes in each year. These temporary differences are expected to reverse in years 3, 4, and 5. At the end of year 2, the deferred tax liability shown as a noncurrent liability is based on the

A
Enacted tax rates for years 3, 4, and 5.
B
Enacted tax rates for years 4 and 5.
C
Enacted tax rate for year 3.
D
Tax rates for years 1 and 2.
A

a

Explanation:
A difference caused by the excess of depreciation taken for tax purposes over the depreciation reported for financial statement purposes will result in taxable amounts in the periods that the difference reverses. The deferred tax liability related to this difference at the end of year 2 is calculated by scheduling the taxable amounts that are to occur in each future year because of the temporary difference and by applying enacted tax rates for those years to the amount of taxable amounts scheduled for those years. Deferred tax accounts are classified based on the classification of a related asset or liability.

807
Q

Leer Corp.’s pretax income in the current year was $100,000. The temporary differences between amounts reported in the financial statements and the tax return are as follows:
Depreciation in the financial statements was $8,000 more than tax depreciation.
The equity method of accounting resulted in financial statement income of $35,000. A $25,000 dividend was received during the year, which is eligible for the 80% dividends received deduction.
Leer’s effective income tax rate was 30%. In its current year income statement, Leer should report a current provision for income taxes of
A
$26,400
B
$23,400
C
$21,900
D
$18,600

A

Explanation:
To adjust the reported pretax financial income to taxable income for the calculation of current income taxes, the depreciation recorded in the financial statements in excess of depreciation deducted for tax purposes must be added to pretax financial income.

Pretax financial income $100,000
Less investment income (book) in excess of dividend income (tax) [$35,000 - ($25,000 × 20%)] (30,000)
Add: Depreciation recorded in financial statements in excess of tax deduction 8,000
Taxable income 78,000
Effective tax rate × 30%
Current provision for income taxes $ 23,400

808
Q

During the first quarter of the current year, Tech Co. had income before taxes of $200,000, and its effec­tive income tax rate was 15%. Tech’s previous year effective annual income tax rate was 30%, but Tech expects its current year effective annual income tax rate to be 25%. In its first quarter interim income state­ment, what amount of income tax expense should Tech report?

A
$0
B
$30,000
C
$50,000
D
$60,000
A

Explanation:
When preparing annual Financial statements, the company’s taxable income for the year is known & income tax provision can be computed with full knowledge of the applicable tax rates & available tax credits. However, when computing income taxes at an interim date the company needs to estimate the effective annual tax rate that it believes will be applicable for the entire year. In the given scenario, Tech Co. expects its 20X3 effective annual income tax rate to be 25%. We need to apply this rate on income before taxes of $200,000 to arrive at a tax expense of $50,000 that Tech should report.

Option (A) is incorrect because provision for tax is required to be calculated for each quarter. Tech Co. had income of $200,000 on which tax expense will be $50,000 using effective annual income tax rate of 25%.
Option (B) is incorrect because we need to use effective annual income tax rate of 25% & not the effective rate of 15% that was for the first quarter of 20X3.
Option (D) is incorrect because it uses the effective rate of last year rather than the effective annual income tax rate for the current year.

809
Q

A temporary tax difference, due to a difference between the reported amount and the tax basis of an asset or liability, may arise from which of the following?
A
An investment tax credit accounted for by the deferred method
B
An increase in the tax basis of an asset because of indexing when the local currency is the functional currency
C
A business combination that is accounted for by the acquisition method
D
All of the above

A

Explanation:
Although an investment tax credit accounted for by the deferred method may cause temporary differences, it is also true that: (1) an increase in the tax basis of an asset because of indexing when the local currency is the functional currency may cause temporary differences; and (2) a business combination that is accounted for by the acquisition method may also cause temporary differences. The best choice for this question is answer D., all of the above.

810
Q

Which of the following statements is correct regarding valuation allowances in accounting for income taxes?
A
The effect of a change in the opening balance of a valuation allowance that results from a change of circumstances ordinarily is included in income from operations.
B
Both deferred tax assets and deferred tax liabilities can be reduced by a valuation allowance.
C
Only negative evidence, not positive evidence, should be considered when determining whether a valuation allowance is needed.
D
A valuation allowance is necessary when the realistic probability standard of evidence is satisfied.

A

Explanation:
The effect of a change in the beginning-of-the-year balance of a valuation allowance that results from a change in circumstances that causes a change in judgment about the realizability of the related deferred tax asset in future years ordinarily shall be included in income from continuing operations. Only deferred tax assets, and not deferred tax liabilities, are to be reduced by a valuation allowance if it is more likely than not, a likelihood of more than 50%, that some or all of the deferred tax assets will not be realized. The valuation allowance should be sufficient to reduce the deferred tax asset to the amount that is more likely than not to be realized. All available evidence, both positive and negative, should be considered to determine whether, based on the weight of that evidence, a valuation allowance is needed.

811
Q

Black Co., organized on January 2, year 1, had pretax financial statement income of $500,000 and taxable income of $800,000 for the year ended December 31, year 1. The only temporary differences are accrued product warranty costs, which Black expects to pay as follows:

Year 2	$100,000
Year 3	50,000
Year 4	50,000
Year 5	100,000
The enacted income tax rates are 25% for year 1,30% for years 2 through 4, and 35% for year 5. Black believes that future years’ operations will produce profits. In its December 31,year1, balance sheet, what amount should Black report as deferred tax asset?
A
$50,000
B
$75,000
C
$90,000
D
$95,000
A

Explanation:
A deferred tax asset or liability is calculated by multiplying temporary differences by the enacted tax rate expected to apply to taxable income in the periods in which the deferred tax liability or asset is expected to be settled or realized.

Amount Rate Def. Tax Asset

Years 2 through 4 $200,000 × 30% = $60,000
Year 5 100,000 × 35% = 35,000
Deferred tax asset $95,000
Options (a), (b) and (c) are incorrect because of inaccurate calculations

812
Q

Zeff Co. prepared the following reconciliation of its pretax financial statement income to taxable income for the current year ended December 31, its first year of operations:

Pretax financial income $160,000
Nontaxable interest received on municipal securities (5,000)
Long-term loss accrual in excess of deductible amount 10,000
Depreciation in excess of financial statement amount (25,000)
Taxable income $140,000
Zeff’s tax rate for the year is 40%.

In its December 31 balance sheet, what should Zeff report as deferred income tax liability?

A
$2,000
B
$4,000
C
$6,000
D
$8,000
A

Explanation:
The permanent differences only affect current taxes. The non-taxable interest received on municipal securities is a permanent difference and will not affect deferred taxes.On the other side, temporary difference effect deferred taxes. The accruals as per financial statements effect taxes only when there is a related cash flow. Excess accrual in income statement on account of long term losses is a temporary7difference and would result in more tax income today The taxes on this difference calculated as per the future enacted tax rate, would result in non-current deferred tax asset of $4,000(i.e. $10,000 x 40%). The excess depreciation in tax return is also a temporary difference and would result in more book income today. The taxes on this difference, calculated as per the future enacted tax rate would result in non-current deferred tax liability of $10,000 (i.e. $25,000 x 40%). Since both are non-current, they would be netted and would result in net deferred tax liability of $6,000 ($10,000 -$4000).

Option (a) is incorrect because non-taxable interest received on municipal securities is a permanent difference and do not affect deferred taxes.Option (b) is incorrect because long-term loss accrual per income statement in excess of deductible amount per tax return would result in more tax income today and therefore would create deferred tax asset. Further, depreciation in excess of financial statement amount is a temporary difference and should also be used for the calculation.Option (d) is incorrect because non-taxable interest received on municipal securities is a permanent difference and do not affect deferred taxes.

813
Q

Brass Co. reported income before income tax expense of $60,000 for year 3. Brass had no permanent or temporary timing differences for tax purposes. Brass has an effective tax rate of 30% and a $40,000 net operating loss carryforward from year 2. What is the maximum income tax benefit that Brass can realize from the loss carryforward for year 3?

A
$12,000
B
$18,000
C
$20,000
D
$40,000
A

Explanation:
The correct answer is (A).

For NOLs arising in tax years ending after December 31, 2017, NOLs may not be carried back but can be carried forward indefinitely. Additionally, NOLs originating in tax years beginning after December 31, 2017, are limited to 80% of taxable income.

The deduction for NOL is limited to the lesser of:
The aggregate of net operating loss carryovers to such year, or
80% of taxable income computed without regard to deduction allowable.

Here, Brass Corp has an NOL of $40,000 in Year 2 which it must carry forward.

The amount of deduction of loss in Year 3, is limited to the lesser of NOL carried forward to that year ($40,000) or 80% of the taxable income of Year 3 of $48,000 ($60,000 x 80%).

The NOL deduction allowed is lower $40,000 in year 3 and the resultant tax benefit that Brass can realize from the loss carryforward for year 3 is $40,000 x 30% = $12,000.

814
Q

At December 31, Bren Co. had the following deferred income tax items:

  • A deferred income tax liability of $15,000 related to a noncurrent asset
  • A deferred income tax asset of $3,000 related to a noncurrent liability
  • A deferred income tax asset of $8,000 related to a current liability

Which of the following should Bren report in the noncurrent section of its December 31 balance sheet?

A
A noncurrent tax asset of $3,000 and a noncurrent tax liability of $15,000.
B
A noncurrent tax liability of $12,000
C
A noncurrent tax asset of $11,000 and a noncurrent tax liability of $15,000.
D
A noncurrent tax liability of $4,000.
A

Explanation:
Always classify deferred tax assets and liabilities as non-current. GAAP allows separate netting of deferred tax assets and liabilities resulting in a single non-current amount to be reported on the balance sheet. Thus, the deferred non-current liability will be $4,000 (i.e. $15,000 - $3,000 - $8,000). Options (a), (b) and (c) are incorrect based on the above explanation.

815
Q

PQR Ltd. reported a net deferred tax liability of $75,000 and pretax financial statement income of $1,850,000 in its December 31, Y1, financial statements. Taxable income was$1,380,000 for Y2. At December 31, Y2, PQR had cumulative deductible differences of$420,000. PQR’s effective income tax rate is 40%. What should PQR report as the deferred portion of income tax expense on its December 31, Y2, statement of income?

A
$243,000
B
$168,000
C
$470,000
D
$93,000
A

Explanation:
A positive temporary difference implies a deferred tax asset and a negative temporary difference implies a deferred tax liability. Y1 would have had a negative temporary difference leading to a deferred tax liability. Y2 has a cumulative positive temporary difference of $420,000, creating a total deferred asset of $168,000 (i.e. 40% of 420,000). From a deferred liability of $75,000 in Y1, Y2 has a cumulative deferred asset of $168,000 implies that to calculate the deferred portion of income tax expense for Y2, the deferred tax account must go from a credit balance of$75,000 to a debit balance of $168,000, which is equal to $243,000 (i.e. $168,000 – (-$75,000)).

Option (b) is incorrect because the cumulative difference is given as $420,000, 40%of that $168,000 would be the cumulative deferred tax asset. The temporary difference of the current year has to be calculated to get the deferred portion of income tax expense of Y2.Option (c) is incorrect because this amount is arrived at by subtracting the Y2taxable income with the Y1 book income, which is not correct.Option (d) is incorrect because this amount is arrived at by subtracting $75,000 from$168,000. As $75,000 is a negative balance (or a liability), and $168,000 is a positive balance, the difference between the two would be $243,000 based on the above explanation.

816
Q

Which of the following circumstances would result in a deferred tax asset for the current year?
A
Expenses that are recognized in financial income this year and deductible next year
B
Expenses that are deductible this year and recognized in financial income next year
C
Revenues that are recognized in financial income this year and taxable next year
D
Revenues that are recognized in financial income this year but are not subject to taxation

A

Explanation:
A deferred tax liability or asset is recognized for the estimated future tax effects attributable to temporary differences and carryforwards. Temporary differences that will result in deductible amounts in future years when the related liabilities are settled are often called deductible temporary differences. Expenses that are recognized in financial income this year and deductible next year would result in a deferred tax asset for the current year. Expenses that would be recognized in financial income next year could never be deductible in the current year. Revenues that are recognized in financial income this year and taxable next year would be a deferred tax liability. Revenues that are recognized in financial income this year but are not subject to taxation are considered permanent differences. Events that do not have tax consequences do not give rise to temporary differences and, therefore, do not give rise to deferred tax assets or liabilities.

817
Q

__________ is a permanent difference resulting from a deduction that is allowed in computing taxable income, but is not allowed in computing financial income.
A
An expense incurred in generating tax-exempt income
B
A premium paid for life insurance on officers when the entity is the beneficiary
C
The excess of percentage depletion, as per statutory allowance, over the cost of natural resources
D
None of the above

A

Explanation:
The excess of percentage depletion, as per statutory allowance, over the cost of natural resources, is an example of a permanent difference resulting from an expense not allowed in computing financial income, but permitted in computing taxable income.

818
Q
Fern Co. has net income, before taxes, of $200,000, including $20,000 interest revenue from municipal bonds and $10,000 paid for officers' life insurance premiums where the company is the beneficiary. The tax rate for the current year is 30%. What is Fern's effective tax rate?
A
27.0%
B
28.5%
C
30.0%
D
31.5%
A

Explanation:
The effective tax rate (28.5%) is the amount due to the government ($57,000) divided by net income before taxes ($200,000). The amount due to the government $57,000 ($190,000 x 30%) is net income before taxes adjusted for tax-exempt revenues and nondeductible expenses $190,000 ($200,000 - 20,000 + 10,000) times the current year tax rate (30%).

819
Q

Kent, Inc.’s reconciliation between the financial statement and taxable income for year 7 follows:

Pretax financial income $150,000
Permanent difference (12,000)
138,000
Temporary difference: depreciation (9,000)
Taxable income $129,000
Additional information:
At 12/31, year 6 At 12/31, year 7
Cumulative temporary differences (future taxable amounts) $11,000 $20,000
The enacted tax rate was 34% for year 6, and 40% for year 7 and years thereafter. In its December 31, year 7 balance sheet, what amount should Kent report
as deferred income tax liability?

A
$3,600
B
$6,800
C
$7,340
D
$8,000
A

Explanation:
The correct answer is (D).

Permanent differences never reverse, so they are never part of the deferred tax computation. Deferred tax liability is always based on the future tax amount at the future tax rate, so $20,000 × 40% = $8,000.

The balance sheet shows the total deferred income tax liability for all the cumulative temporary differences up to the balance sheet date.

The closing balance at 12/31/X3 shows a cumulative temporary difference of $20,000. Deferred tax liability = temporary difference x future enacted tax rate = $20,000 x 40% = $8,000.

Note: Deferred tax is calculated on the future tax rate for the year in which Deferred Tax Assets or Liabilities are expected to reverse which is 40% for all future years and hence will be calculated at total DTL X 40% = $8,000.

820
Q

Zeff Co. prepared the following reconciliation of its pretax financial statement income to taxable income for the current year ended December 31, its first year of operations:
Pretax financial income $160,000
Nontaxable interest received on municipal securities (5,000)
Long-term loss accrual in excess of deductible amount 10,000
Depreciation in excess of financial statement amount (25,000)
Taxable income $140,000
Zeff’s tax rate for the year is 40%.

In its income statement, what amount should Zeff report as income tax expense - current portion?

A
$52,000
B
$56,000
C
$62,000
D
$64,000
A

Explanation:
To determine the current portion of income tax expense, taxable income is multiplied by the current year’s enacted corporate income tax rate. $140,000 × 40% = $56,000.

821
Q
Cory, Inc., uses the accrual method of accounting for financial reporting purposes and appropriately uses the installment method of accounting for income tax purposes. Installment income of $250,000 will be collected in the following years when the enacted tax rates are:
Year	Collection of income	Enacted tax rates
2	$ 25,000	35%
3	50,000	30%
4	75,000	30%
5	100,000	25%The installment income is Cory's only temporary difference. Taxable income is expected in all future years. What amount should be included in the deferred income tax liability in Cory's December 31, year 2 balance sheet?
A
$62,500
B
$71,250
C
$78,750
D
$87,500
A

Explanation:
The deferred tax liability balance represents the amount of taxes expected to be paid in the future when installment receivables are collected. (The revenue from installment sales has already been included in the financial statements but will not be reported for tax purposes until the related receivables are collected.)

Period Amount Rate Deferred Tax Liability
Years 3 and 4 $125,000 30% $37,500
Year 5 100,000 25% 25,000
Total $62,500

822
Q

At the end of year 1, Cody Co. reported a profit on a partially-completed construction contract by applying an adjusted version of the percentage-of-completion method in accordance with the input method prescribed in ASC 606 and recognized revenue “over time”.

By the end of year 2, the total estimated profit on the contract at completion in year 3 had been drastically reduced from the amount estimated at the end of year 1. Consequently, in year 2, a loss equal to one-half of the previous year profit was recognized.

Conversely, Cody recognized revenue at a “point in time” for income tax purposes and had no other contracts. The year 2 balance sheet should include a deferred tax

Asset	Liability
A	Yes	Yes
B	No	Yes
C	Yes	No
D	No	No
A

Explanation:
At the end of year 1, a cumulative difference exists which is equal to the contract profit recognized on the income statement in the previous year. The cumulative difference will result in future taxable amounts, so a deferred tax liability is established for an amount equal to the cumulative temporary difference multiplied by the tax rate enacted for the year(s) in which the temporary difference is expected to reverse.

In year 2, half of the cumulative temporary difference reverses because of the recognition of a loss to offset half of the profit reported in the previous year. With no change in enacted future tax rates, this reversal results in a decrease in the related deferred tax liability account. Therefore, at the end of year 2, Cody has a deferred tax liability bal­ance equal to half of the balance that was in that account at the end of year 1. Options (a), (c) and (d) are incorrect based on the above explanation.

823
Q

Concerning permanent differences, which of the following statements is false?
A
All events recognized in financial statements will have tax consequences under the regular U.S. tax system.
B
Certain revenues are exempt from taxation and certain expenses are not deductible.
C
Events that do not have tax consequences do not give rise to temporary differences and, therefore, do not give rise to deferred tax assets or liabilities.
D
Differences that will not have future tax consequences are often referred to as permanent differences.

A

Explanation:

Some events recognized in financial statements do not have tax consequences under the regular U.S. tax system.

824
Q
For interim financial reporting, the computation of a company's second quarter provision for income taxes uses an effective tax rate expected to be applicable for the full fiscal year. The effective tax rate should reflect anticipated
Foreign tax rates	Available tax
planning alternatives
A	No	Yes
B	No	No
C	Yes	No
D	Yes	Yes
A

Explanation:
The best estimate of the effective tax rate to be applicable for the full fiscal year be made at the end of each interim period. This rate should reflect foreign tax rates, percentage depletion, capital gains rates, and other available tax planning alternatives.

825
Q

Marco Corp., a construction company, appropriately uses an adjusted version of the percentage-of-completion method in accordance with the input method prescribed in ASC 606 and recognizes revenue “over time” in its financial statements and at a “point in time” for income tax reporting. Pertinent data at December 31, year 2, the close of Marco’s first year of operations, are as follows.

Date contract began Estimated completion date Income recognized in year 2
on each contract
3/1, year 2 9/1, year 3 $300,000
6/1, year 2 12/1, year 3 $150,000
9/1, year 2 3/1, year 4 $100,000
12/1, year 2 6/1, year 4 $50,000
Marco’s enacted tax rates are 30% for year 2, 25% for year 3, and 20% for year 4. Assuming Marco expects taxable income in all future periods, what amount should be included in the deferred tax liability at December 31, year 2, for these transactions?

A
$180,000
B
$165,000
C
$120,000
D
$142,500
A

Explanation:
Deferred tax liability is based on future taxable amounts times future tax rates. The first two contracts will end in year 3 and recognize $450,000, so a liability of $112,500 is created [450,000 * 25%]. The last two contracts close in year 4 with $150,000 income, so a liability of $30,000 is created [150,000 * 20%], for a total deferred tax liability of $142,500 at the end of year 2 for temporary timing differences.

826
Q

Ram Corp. prepared the following reconciliation of income per books with income per tax return for the current year ended December 31:
Book income before income taxes $750,000
Add temporary difference
Construction contract revenue which will reverse in 4 years 100,000
Deduct temporary difference
Depreciation expense which will reverse in equal amounts in each of the next 4 years (400,000)
Taxable Income $450,000Ram’s effective income tax rate for the year is 34%. What amount should Ram report in its current year income statement as the current provision for income taxes?
A
$34,000
B
$153,000
C
$255,000
D
$289,000

A

Explanation:
The current portion of the provision for income taxes (which is the same as the amount of taxes due to the government for the current period) is computed by multiplying the taxable income figure by the statutory tax rate for the current period. The statutory tax rate is not given. However, because there are no permanent differences and because there are no temporary differences which are expected to reverse at tax rates different from the current statutory rate, the effective rate for the current year must be the same as the statutory tax rate. Thus, $450,000 × 34% = $153,000.

827
Q
During the first quarter of the calendar year, Worth Co. had income before taxes of $100,000, and its effective income tax rate was 15%. Worth's effective annual income tax rate for the previous year was 30%. Worth expects that its effective annual income tax rate for the current year will be 25%. The statutory tax rate for the current year is 35%. In its first quarter interim income statement, what amount of income tax expense should Worth report?
A
$15,000
B
$25,000
C
$30,000
D
$35,000
A

Explanation:
The objectives of accounting for income taxes are to recognize the amount of taxes payable or refundable for the current year and deferred tax liabilities and assets for the future tax consequences of events that have been recognized in an enterprise’s financial statements or tax returns. One of the following basic principles applied in accounting for income taxes at the date of the financial statements is that a current tax liability or asset is recognized for the estimated taxes payable or refundable on tax returns for the current year. The effective annual income tax rate for the entire year will be 25%, thus $25,000 (25% of the first quarter $100,000 income) should be reported as income tax expense in the first quarter interim income statement. The first quarter effective rate, previous year effective rate, and statutory tax rate are not considered.

828
Q

Which of the following tax positions is unlikely to be affected by the accounting for uncertain tax positions?
A
A deduction taken on the tax return for a current expenditure that the taxing authority may assert should be capitalized and amortized over future periods.
B
The calculation of the amount of a research and experimentation credit.
C
The determination as to whether an entity is subject to tax in a particular jurisdiction.
D
Deductions for ordinary salaries paid to employees.

A

Explanation:
Tax benefits associated with positions that are highly certain and not likely to be questioned by the taxing authorities, such as deductions for ordinary salaries paid to employees, are unlikely to be affected by the guidance related to uncertain tax positions.

829
Q

A company reported the following financial information:

Taxable income for current year

$120,000

Deferred income tax liability, beginning of year

50,000

Deferred income tax liability, end of year

55,000

Deferred income tax asset, beginning of year

10,000

Deferred income tax asset, end of year

16,000

Current and future years’ tax rate

35%

What amount is the current year’s income tax expense?

A
$41,000
B
$42,000
C
$43,000
D
$53,000
A

Explanation:
The correct answer is (A).

Income tax expense = Income tax expense (current) + Income tax expense (deferred) = Income tax expense (current) + Deferred tax liability (for the year) - Deferred tax asset (for the year).
Income tax expense (current) = Taxable income x current tax rate = $120,000 x 35% = $42,000.
Deferred tax liability (for the year) = Deferred tax liability, end of the year - Deferred tax liability, beginning of the year = $55,000 - $50,000 = $5,000.
Deferred tax asset (for the year) = Deferred tax asset, end of the year - Deferred tax asset, beginning of the year = $16,000 - $10,000 = $6,000.
Income tax expense = $42,000 + $5,000 - $6,000 = $41,000.

830
Q

A deferred tax liability may result from which of the following items?

A
Penalties paid for legal violations
B
Life insurance proceeds received on the death of key employees
C
Depreciation of tangible assets
D
Interest on municipal bonds
A

Explanation:
There would be a deferred tax Liability if there is More Book income today that is, there is a temporary difference that will reverse in future years. Permanent difference would not lead to deferred taxes since permanent differences do not reverse in future years. Depreciation of tangible assets is a temporary difference due to the difference of depreciation methods between the financial books and the tax books. That would create a deferred tax liability if there is greater depreciation in the tax books compared to the financial books, as IRS allows accelerated depreciation. Options (a), (b), and (d) are incorrect because penalties, life insurance proceeds on key employees and municipal bond interest are all permanent differences and would not lead to deferred taxes.

831
Q

When accounting for income taxes, a temporary difference occurs in which of the following scenarios?

A
An item is included in the calculation of net income, but is neither taxable nor deductible.
B
An item is included in the calculation of net income in one year and in taxable income in a different year.
C
An item is no longer taxable due to a change in the tax law.
D
The accrual method of accounting is used.

A

Explanation:
Differences between book and taxable income resulting from temporary differences are those which occur due to items which will reverse themselves in a matter of time (timing difference). Thus, an item included in the calculation of net income in one year and in taxable income in a different year would lead to a temporary difference. Options (a) and (c) are incorrect because both would lead to a permanent difference. Option (d) is incorrect because if accrual basis of accounting is used for both book and taxable income, this would not lead to a difference.

832
Q

Paxton Inc.’s reconciliation between financial statement and taxable income for year 6 follows.

Pretax financial income $750,000
Permanent difference (25,000)
725,000
Temp difference: depreciation (60,000)
Temp difference: disallowed warranty expense 50,000
Taxable income $715,000
The enacted tax rate was 30% for year 6 and 35% for years thereafter. Paxton estimates that 10% of any deferred tax asset will never be realized. In its December 31, year 6 balance sheet, what amount would Paxton report as deferred income tax asset?

A
$17,500
B
$21,000
C
$15,750
D
$18,900
A

Explanation:
A deferred tax asset is generated from a temporary difference that is classified as a future deductible amount, computed using future tax rates. The warranty expense is not deductible currently for tax purposes but will be in the future once the expense is paid, although it is estimated that 10% will never be realized.
The deferred tax asset is computed as follows: (50,000 – 5,000) × 35% = $15,750

833
Q

Which of the following circumstances would result in a deferred tax asset for the current year?

A
Expenses that are recognized in financial income this year and deductible next year.
B
Expenses that are deductible this year and recognized in financial income next year.
C
Revenues that are recognized in financial income this year and taxable next year.
D
Revenues that are recognized in financial income this year but are not subject to taxation.

A

Explanation:
The correct answer is (A).

Expenses that are recognized in financial income this year and deductible next year in tax return would create a scenario of a deferred tax asset for the current year. A deferred tax asset results from a temporary timing difference between what is reported on the financial statements and what is reported on the tax forms. A deferred tax asset is recognized when taxable income reported on the tax return exceeds pre-tax income on the income statement. The expense would be recorded in the current year on the income statement but may not be deductible on the tax forms until the next year. This will cause pretax income to be lower on the income statement than taxable income on the tax forms.

(B) and (C) are incorrect because it results in deferred tax liability, not asset.

(D) is incorrect because they will be a permanent difference, not a deferred tax asset.

834
Q
As a result of differences between depreciation for financial reporting purposes and tax purposes, the financial reporting basis of Noor Co.'s sole depreciable asset, acquired in the current year, exceeded its tax basis by $250,000 at December 31 of the current year. This difference will reverse in future years. The enacted tax rate is 30% for the current year, and 40% for future years. Noor has no other temporary differences. In its December 31 current year balance sheet, how should Noor report the deferred tax effect of this difference?
A
As an asset of $75,000.
B
As an asset of $100,000.
C
As a liability of $75,000.
D
As a liability of $100,000.
A

Explanation:
The calculation to determine a deferred tax effect is based on a future tax rate, which in this problem is 40%. Thus, 40% of the $250,000 difference is $100,000. Because the financial reporting basis exceeded the tax basis, the $100,000 represents the amount of tax expected to be paid in future years as a result of the turn-around of this temporary difference. This is shown on the balance sheet as a deferred tax liability.

835
Q

According to the federal tax law, a net operating loss (NOL) may be carried back ___ years and carried forward ___ years.

A
2; 5
B
0; Indefinitely
C
3; 10
D
3; 20
A

Explanation:
The correct answer is (B).

As per the TCJA, NOLs for tax years beginning in 2018 and onwards may be carried forward indefinitely but may be deducted only up to 80% of taxable income (wherein the taxable income is computed without regard to any NOL carryover).

836
Q
For calendar year 2, Clark Corp. reported depreciation of $300,000 in its income statement. On its year 2 income tax return, Clark reported depreciation of $500,000. Clark's income statement also included $50,000 accrued warranty expense that will be deducted for tax purposes when paid. Clark's enacted tax rates are 30% for years 2 and 3, and 25% for years 4 and 5. Taxable income is expected in all future years. The depreciation difference and warranty expense will reverse over the next three years as follows:
Year	Depreciation difference	Warranty expense
3	$80,000	$10,000
4	70,000	15,000
5	50,000	25,000
$200,000	$50,000These were Clark's only temporary differences. In Clark's year 2 income statement, the deferred portion of its provision for income taxes should be
A
$67,000
B
$45,000
C
$41,000
D
$37,500
A

Explanation:
The deferred portion of Clark’s provision for income taxes (which means the deferred portion of income tax expense) is determined by the net change during the year in deferred tax accounts on the balance sheet. The temporary difference due to depreciation will result in taxable amounts in future years; the temporary difference due to accrued warranty expense will result in deductible amounts in future years.

Future taxable amounts:	Amount	Rate	Deferred Tax Accts.
Depreciation Expense			
Year 3	$ 80,000	30%	$24,000
Years 4 - 5	120,000	25%	30,000
Deferred tax liability			$54,000
Future deductible amounts:			
Warranty Expense			
Year 3	$ 10,000	30%	$ 3,000
Years 4 - 5	40,000	25%	10,000
Deferred tax asset			$13,000
Deferred tax liability			$ 54,000
Deferred tax asset			(13,000)
Tax expense - deferred			$ 41,000
The net deferred tax liability is $41,000 [i.e., $54,000 + $(13,000)].

There is no mention of a beginning balance of deferred taxes; therefore, the entry to record the $54,000 deferred tax liability and the $13,000 deferred tax asset will involve a credit to Deferred Tax Liability for $54,000, a debit to Deferred Tax Asset for $13,000, and a debit to Income Tax Expense for $41,000. This debit to expense represents deferred income tax expense, and it is referred to as the deferred portion of the provision for income taxes.

837
Q
On January 2, year 1, Ross Co. purchased a machine for $70,000. This machine has a 5-year useful life, a residual value of $10,000, and is depreciated using the straight-line method for financial statement purposes. For tax purposes, depreciation expense was $25,000 for year 1 and $20,000 for year 2. Ross' year 2 income, before income taxes and depreciation expense, was $100,000 and its tax rate was 30%. If Ross had made no estimated tax payments during year 2, what amount of current income tax liability would Ross report in its December 31, year 2, balance sheet?
A
$26,400
B
$25,800
C
$24,000
D
$22,500
A
Explanation:
Pretax financial income before depreciation	$ 100,000
Depreciation for tax purposes	  (20,000)
Taxable income	$ 80,000
Tax rate expected	×     30%
Current income tax expense	$ 24,000
Estimated tax payments	         (0)
Current income tax liability	$ 24,000
838
Q

A deferred tax asset of $100,000 was recognized in the year 1 financial statements by the Chaise Company when a loss from discontinued segments was carried forward for tax purposes. A valuation allowance of $100,000 was also recognized in the year 1 statements because it was considered more likely than not that the deferred tax asset would not be realized. Chaise had no temporary differences. The tax benefit of the loss carried forward reduced current taxes payable on year 3 continuing operations. The year 3 income statement would include the tax benefit from the loss brought forward in

A
Income from continuing operations
B
Gain or loss from discontinued segments
C
Capital gains
D
Cumulative effect of accounting changes
A

Explanation:
The correct answer is (A).

Except for certain areas such as business combinations and quasi-reorganizations, the manner of reporting the tax benefits of an operating loss carryforward is determined by the source of income or loss in the current year and not by the source of the operating loss carryforward or taxes paid in a prior year.

Therefore, the tax benefit of the operating loss in question reduces income tax expense from income from continuing operations because the realization of the tax benefit results from income from continuing operations.

839
Q

In its current year income statement, Cere Co. reported income before income taxes of $300,000. Cere estimated that, because of permanent differences, taxable income would be $280,000. During the year Cere made estimated tax payments of $50,000, which were debited to income tax expense. Cere is subject to a 30% tax rate. What amount should Cere report as income tax expense?

A
$34,000
B
$50,000
C
$84,000
D
$90,000
A

Explanation:
The current income tax = current taxable income x current income tax rate.Cere’s taxable income is $280,000 and the difference between book income and taxable income is on account of permanent differences which will not reverse in future and will not affect current taxes. Therefore, Cere would report $84,000 (i.e. $280,000 x 30%) as its current income tax expenses in its income statement.Options (a) is incorrect because this represents the current tax liability. We do not need to deduct the estimated tax payments.Option (b) is incorrect because this represents the estimated tax payments made.Option (d) is incorrect because income tax expense is based on the taxable income and not the financial statement income.

840
Q

On January 1 of the current year, Pesto Co. purchased 35% of the voting common stock of Surry Co. and appropriately accounts for the investment by the equity method. During the current year, Surry reported earnings of $200,000 and paid dividends of $40,000. Pesto’s current enacted income tax rate is 28%. In addition:

The dividends received from Surry are eligible for the 80% dividends received deduction.
The income tax rate enacted for future periods is 30%.
In Pesto’s December 31 balance sheet the deferred income tax liability should be

A
$16,800
B
$15,680
C
$ 3,136
D
$ 3,360
A

Explanation:
The correct answer is (D).

This question involves a temporary difference because the investor recognizes investment income for financial purposes based on its equity in the investor’s earnings, whereas for tax purposes, investment income is recognized on the cash basis when dividends are received. Deferred taxes must be recorded on this temporary difference. The increase in the investor’s deferred income tax liability for this temporary difference is computed as follows:

Income per Books ($200,000 x 35%) (a) $70,000
Permanent Difference (80% DRD) [b = 80% x a] ($56,000)
Taxable Income per Books (c) = (a) – (b) $14,000
Temporary Difference (c) – (d) ($11,200)
Income Per Taxes (Dividend - 80% of Dividend) (Dividend x 20%) ($40,000 x 35% x 20%) (d) $2,800
Deferred Tax Liability = Temporary Difference x Tax Rate

= $11,200 x 30%

= $3,360

841
Q

In its year 1 income statement, Tow, Inc. reported proceeds from an officer’s life insurance policy of $90,000 and depreciation of $250,000. Tow was the owner and beneficiary of the life insurance on its officer. Tow deducted depreciation of $370,000 in its year 1 income tax return when the tax rate was 30%. Data related to the reversal of the excess tax deduction for depreciation follow:

Year Reversal of excess tax deduction Enacted tax rates
2 $50,000 35%
3 40,000 35%
4 20,000 25%
5 10,000 25%
There are no other temporary differences. Tow expects to report profits (rather than losses) for tax purposes for all future years. In its December 31, year 1, balance sheet, what amount should Tow report as a deferred income tax liability?

A
$36,000
B
$39,000
C
$63,000
D
$66,000
A

Explanation:
The proceeds from the officer’s life insurance policy is a tax-exempt revenue; therefore, no current or future tax consequences will result and no deferred taxes are to be recorded for this difference. The $120,000 ($370,000–$250,000) difference due to depreciation is the only temporary difference existing at the balance sheet date. The tax rate that is used to measure the deferred tax consequences is the enacted tax rate(s) expected to apply to taxable income in the years that this difference reverses.

 	Amount	Rate	Deferred Tax Liability
Depreciation	 	 	 
Years 2 - 3	$90,000	35%	$31,500
Years 4 - 5	$30,000	25%	7,500
Deferred tax liability	 	 	$39,000
842
Q

On January 1 of the current year, Lundy Corp. purchased 40% of the voting common stock of Glen, Inc., and appropriately accounts for its investment by the equity method. During the year, Glen reported earnings of $225,000 and paid dividends of $75,000. Lundy assumes that all of Glen’s undistributed earnings will be distributed as dividends in future periods when the enacted tax rate will be 30%. Ignore the dividends-received deduction. Lundy’s current enacted income tax rate is 25%. Lundy uses the liability method to account for temporary differences and expects to have taxable income in all future periods. The increase in Lundy’s deferred income tax liability for this temporary difference is

A
$45,000
B
$37,500
C
$27,000
D
$18,000
A

Explanation:
The investor recognizes investment income for financial purposes based on its equity in the inves­tee’s earnings; for tax purposes,investment income is recognized on the cash basis when dividends are received. Deferred taxes are recorded on this temporary difference. In this question, we are told to ignore the 80% dividends-received deduction (DRD).

Equity in investor’s earnings, book ($225,000 × 40%) $90,000
Less: Dividends received ($75,000 × 40%) 30,000
Temporary difference (without consideration of the dividends-received deduction) 60,000
Times enacted tax rate for future periods × 30 %
Increase in deferred tax liability $18,000

843
Q
Barnel Corp. owns and manages 19 apartment complexes. On signing a lease, each tenant must pay the first and last months' rent and a $500 refundable security deposit. The security deposits are rarely refunded in total, because cleaning costs of $150 per apartment are almost always deducted. About 30% of the time, the tenants are also charged for damages to the apartment, which typically cost $100 to repair. If a one-year lease is signed on a $900 per month apartment, what amount would Barnel report as refundable security deposit?
A
$ 1,400
B
$ 500
C
$ 350
D
$ 320
A

Explanation:
The lessor should report the full amount of the $500 refundable security deposit as a liability until the lessor has earned a portion of it by cleaning or repairing vacated apartments. Revenue is not recognized until it has been earned. Deferred revenue is reported as a liability.

844
Q

Included in Lee Corp.’s liability account balances at December 31, year 3, were the following:
14% note payable issued October 1, year 3, maturing September 30, year 4 $125,000
16% note payable issued April 1, year 1, payable in six equal installments of $50,000 beginning April 1, year 2 $200,000Lee’s December 31, year 3 financial statements were issued on March 31, year 4. On January 15, year 4, the entire $200,000 balance of the 16% note was refinanced by issuance of a long-term obligation payable in a lump sum. In addition, on March 10, year 4, Lee consummated a noncancelable agreement with the lender to refinance the 14%, $125,000 note on a long-term basis, on readily determinable terms that have not yet been implemented. Both parties are financially capable of honoring the agreement, and there have been no violations of the agreement’s provisions. On the December 31, year 3 balance sheet, the amount of the notes payable that Lee should classify as short-term obligations is
A
$175,000
B
$125,000
C
$ 50,000
D
$0

A

Explanation:
The entire $200,000 balance of the 16% note is properly excluded from short-term obligations because before the balance sheet was issued, Lee refinanced the note by issuance of a long-term obligation. The $125,000, 14% note is also properly excluded from short-term obligations because before the balance sheet was issued, Lee entered into a financing agreement that clearly permits Lee to refinance the short-term obligation on a long-term basis on terms that are readily determinable, and all of the following conditions are met: (1) the agreement is noncancelable as to all parties and extends beyond one year; (2) at the balance sheet date and at the date of its issuance, Witt is not in violation of the agreement; and (3) the lender is financially capable of honoring the agreement.

845
Q

On October 1, year 1, Gold Co. borrowed $900,000 to be repaid in three equal, annual installments. The note payable bears interest at 5% annually. Gold paid the first installment of $300,000 plus interest on September 30, year 2. What amount should Gold report as a current liability on December 31, year 2?

A
$330,000
B
$307,500
C
$303,750
D
$300,000
A

Explanation:
The correct answer is (B).

Current liabilities are obligations owed by an entity that are reasonably expected to require the use of existing resources during the entity’s operating cycle or, generally, within one year.

On December 31, year 2, Gold would be liable for the second annual installment of $300,000 within one year. Gold would also be responsible for 3 months’ interest (from October 1 through December 31).

The interest is calculated as follows: $600,000 balance × 5% interest × 3/12 time period = $7,500.

The total current liability reported on December 31, year 2, would be $307,500.

846
Q

Pane Co. had the following borrowings on its books at the end of the current year:

$100,000, 12% interest rate, borrowed five years ago on September 30; interest payable March 31 and September 30.
$75,000, 10% interest rate, borrowed two years ago on July 1; interest paid April 1, July 1, October 1, and January 1.
$200,000, non-interest bearing note, borrowed July 1 of the current year, due January 2 of next year; proceeds $178,000.
What amount should Pane report as interest payable in its December 31 balance sheet?

A
$4,875
B
$6,750
C
$26,875
D
$41,500
A

Explanation:
The correct answer is (A).

Pane should report interest payable as $4,875 on December 31st.

Accrued interest payable at 12% on $100,000 for 3 months (3/12) = $3,000.
Accrued interest payable at 10% on $75,000 for 3 months (3/12) = $1,875.
There will be no interest payable on $200,000 as it is a non-interest bearing note.
Total Interest Payable: $3,000 + $,1875 = $4,875.

847
Q
In its current year financial statements, Cris Co. reported interest expense of $85,000 in its income statement and cash paid for interest of $68,000 in its cash flow statement. There was no prepaid interest or interest capitalization either at the beginning or end of the current year. Accrued interest at December 31 of the prior year was $15,000. What amount should Cris report as accrued interest payable in its current year December 31 balance sheet?
A
$ 2,000
B
$15,000
C
$17,000
D
$32,000
A

Explanation:
Interest expense for the current year $ 85,000
Cash paid for interest in the current year (68,000)
Increase in accrued interest payable in the year $ 17,000
Accrued interest payable, 12/31 of prior year 15,000
Accrued interest payable, 12/31 of current year $ 32,000

848
Q
Herr Inc. has a fiscal year ending April 30. On May 1, of the previous year, Herr borrowed $10,000,000 at 15% to finance construction of its own building. Repayments of the loan are to commence the month following completion of the building. During the current year ended April 30, expenditures for the partially completed structure totaled $6,000,000. These expenditures were incurred evenly throughout the year. Interest earned on the unexpended portion of the loan amounted to $400,000 for the year. How much should be shown as capitalized interest on Herr's financial statements at April 30?
A
$0
B
$50,000
C
$450,000
D
$1,100,000
A

Explanation:
The amount of interest to be capitalized is determined by applying an interest rate to the average amount of accumulated expenditures for the asset during the period. The interest rate to be used is the rate on new borrowings which are specifically associated with the acquisition of the new asset, or a weighted average of the interest rates on all debt outstanding during the period. The amount of interest thus determined is not reduced or in any way offset by interest income earned during the construction period.

Average expenditure during year ($6,000,000 / 2) $3,000,000
Interest (capitalization) rate × 0.15
Capitalized interest $ 450,000

849
Q
On July 1, Ran County issued realty tax assessments for its fiscal year ended next June 30. On September 1, Day Co. purchased a warehouse in Ran County. The purchase price was reduced by a credit for accrued realty taxes. Day did not record the entire year's real estate tax obligation, but instead records tax expenses at the end of each month by adjusting prepaid real estate taxes or real estate taxes payable, as appropriate. On November 1, Day paid the first of two equal installments of $12,000 for realty taxes. What amount of this payment should Day record as a debit to real estate taxes payable?
A
$ 4,000
B
$ 8,000
C
$10,000
D
$12,000
A

Explanation:
On September 1, Day would have collected two months of real estate taxes from the seller and credited the amount to Real Estate Taxes Payable. At the ends of September and October, Day would have recorded one month of real estate taxes each month by a credit to Real Estate Taxes Payable. When the payment was made for six months of real estate taxes, on November 1, the $12,000 payment would be for the four months prior to that date that would have already been accrued and for the two months that follow the payment date. Therefore, $8,000 (i.e., $12,000 x 4/6) of the payment should be recorded as a decrease in Real Estate Taxes Payable and $4,000 (i.e., $12,000 x 2/6) of the payment should be recorded as an increase in Prepaid Real Estate Taxes.

850
Q
Regal Department Store sells gift certificates, redeemable for store merchandise, that expire one year after their issuance. Regal has the following information pertaining to its gift certificates sales and redemptions:
Unredeemed at 12/31 of prior year	$ 75,000
Current year sales	250,000
Redemptions of prior year sales	25,000
Redemptions of current year sales	175,000Regal's experience indicates that 10% of gift certificates sold will not be redeemed. In its current year December 31 balance sheet, what amount should Regal report as unearned revenue?
A
$125,000
B
$112,500
C
$100,000
D
$ 50,000
A

Explanation:
Of the $250,000 of gift certificates sold in the current year, only $225,000 [i.e., $250,000 × (100% - 10%)] are expected to be redeemed. Since $175,000 of the gift certificates sold in the current year were redeemed in the current year, $50,000 should be reported as unearned revenue at 12/31 of the current year. (At 12/31 of the current year there is no liability for unredeemed gift certificates sold in the prior year because the certificates expire one year after their issuance.).

851
Q
Wall Co. sells a product under a two-year warranty. The estimated cost of warranty repairs is 2% of net sales. During Wall's first two years in business, it made the following sales and incurred the following warranty repair costs:
Year 1	
Total sales	$250,000
Total repair costs incurred	4,500
Year 2	
Total sales	$300,000
Total repair costs incurred	5,000What amount should Wall report as warranty expense for year 2?
A
$1,000
B
$5,000
C
$5,900
D
$6,000
A

Explanation:
A warranty or guarantee is a promise made by the seller to the buyer to make good certain deficiencies in the product during a specified period of time after the sale. Obligations related to product warranties and product defects are an example of loss contingencies. An estimated loss from a loss contingency shall be accrued by a charge to income if both of the following conditions are met: 1) information available prior to the issuance of the financial statements indicates that it is probable that an asset had been impaired or a liability had been incurred at the date of the financial statements, and 2) the amount of the loss can be reasonably estimated. It is probable that repairs are required and the 2% of net sales is a reasonable estimate so there is a charge to income through warranty expense. Proper accrual of the warranty expense for year 2 would be 2% of the net sales from year 2 (0.02 × $300,000 = $6,000).

852
Q
Ross Co. pays all salaried employees on a Monday for the five-day workweek ended the previous Friday. The last payroll recorded for the year ended December 31, Year 2, was for the week ended December 25, Year 2. The payroll for the week ended January 1, Year 3, included regular weekly salaries of $80,000 and vacation pay of $25,000 for vacation time earned in year 2 not taken by December 31, Year 2. Ross had accrued a liability of $20,000 for vacation pay at December 31, Year 1. In its December 31, Year 2, balance sheet, what amount should Ross report as accrued salary and vacation pay?
A
$64,000
B
$68,000
C
$69,000
D
$89,000
A

Explanation:
Ross should report as accrued salary and vacation pay the sum of the payroll for the week ended 1/1, year 3 that pertains to year 2 and the entire amount of the vacation pay earned in year 2 but not taken by 12/31, year 2.

Accrued salary, 12/31 year 2 ($80,000 × 4/5) $64,000
Accrued vacation time earned in year 2 25,000
Accrued salary and vacation time, 12/31 year 2 $89,000
Four days of the 5-day workweek ended 1/1 year 3 pertain to year 2.

853
Q

A note payable was issued in payment for services received. The services had a fair value less than the face amount of the note payable. The note payable has no stated interest rate. How should the note payable be presented in the statement of financial position?

A
At the face amount.
B
At the face amount with a separate deferred asset for the discount calculated at the imputed interest rate.
C
At the face amount with a separate deferred credit for the discount calculated at the imputed interest rate.
D
At the face amount minus a discount calculated at the imputed interest rate

A

Explanation:
The correct answer is (D)

If the note is a non-interest bearing or the interest rate is below market, the value of the note should be determined by imputing the market rate of interest and determining the value of the note by using the Effective Interest method. The note payable has no stated interest rate, and the services provided have a fair value less than the face amount of the note payable. Therefore, the note payable should be presented, in the statement of financial position, at the face amount minus a discount calculated at the imputed rate.

Expenses for Services XXX
Discount XXX
Notes Payable XXX
Next, the difference between what is recorded, and the face value is considered a discount. And the applicable interest rate is called the imputed rate. This is the rate at which the present value of the face amount of the note will be equal to the amount at which it is originally recorded. As a result, the note payable is recorded at its face amount minus a discount calculated at the imputed interest rate.

854
Q
As of December 15 of the current year, Aviator had dividends in arrears of $200,000 on its cumulative preferred stock. Dividends for this year of $100,000 have not yet been declared. The board of directors plan to declare cash dividends on its preferred and common stock on January 16 of next year. Aviator paid an annual bonus to its CEO based on the company's annual profits. The bonus for this year was $50,000, and it will be paid on February 10 of next year. What amount should Aviator report as current liabilities on its current year balance sheet at December 31?
A
$50,000
B
$150,000
C
$200,000
D
$350,000
A

Explanation:
When declared, cash and property dividends represent legal obligations due within one year and are reported as current liabilities. Stock dividends and undeclared dividends on cumulative preferred stock are not reported as liabilities. The dividends for the year have not yet been declared, and are not reported as liabilities. Cumulative preferred stock dividends in arrears must be disclosed in the notes to the statements, and are not reported as liabilities. The only amount that should be reported as a current liability is the bonus.

855
Q
Baker Co. sells consumer products that are packaged in boxes. Baker offered an unbreakable glass in exchange for two box tops and $1 as a promotion during the current year. The cost of the glass was $2.00. Baker estimated at the end of the year that it would be probable that 50% of the box tops will be redeemed. Baker sold 100,000 boxes of the product during the current year and 40,000 box tops were redeemed during the year for the glasses. What amount should Baker accrue as an estimated liability at the end of the current year, related to the redemption of box tops?
A
$ 0
B
$ 5,000
C
$20,000
D
$25,000
A

Explanation:
In order to increase sales and promote certain products, companies may offer premiums to those customers who return box tops, coupons, labels, wrappers, etc., as proof of purchase. The cost of these premiums represents an expense that should be matched against revenue from the sales benefited. At the end of the accounting period, an expense account should be debited and a liability account credited for the cost of outstanding premiums expected to be redeemed in subsequent periods. There were 50,000 box tops expected to be redeemed (100,000 sold x 50% probable redemption). As 40,000 box tops were redeemed during the year, an estimated 10,000 box tops will be redeemed in the future. The premium required two box tops and $1 per redemption. An estimated 10,000 box tops equates to an estimated 5,000 unbreakable glasses (10,000 / 2). The glasses cost $2 each, or $10,000 total ($2 x 5,000). The company also estimated receiving $5,000 ($1 x 5,000) in the redemption. The estimated liability at the end of the current year related to the premium is $5,000 ($10,000 glasses cost - $5,000 received with box tops).

856
Q
At December 31 of the current year, Taos Co. estimates that its employees have earned vacation pay of $100,000. Employees will receive their vacation pay in next year. Should Taos accrue a liability at December 31 if the rights to this compensation accumulated over time or if the rights are vested?
Accumulated	Vested
A	Yes	No
B	No	No
C	Yes	Yes
D	No	Yes
A

Explanation:
An employer must accrue a liability for employees’ rights to receive vacation pay benefits if the four conditions are met. Three of these conditions are met in the question data. These are: (1) the obligation is attributable to employees’ services already rendered, (2) payment of the compensation is probable, and (3) the amount can be reasonably estimated (i.e., the employees have earned vacation pay of $100,000 that they will receive in the next year). The fourth condition necessary for accrual is that the employees’ rights to receive the vacation pay benefits accumulate or vest.

857
Q

Hill Corp. began production of a new product. During the first calendar year, 1,000 units of the product were sold for $1,200 per unit. Each unit had a two-year warranty. Based on warranty costs for similar products, Hill estimates that warranty costs will average $100 per unit. Hill incurred $12,000 in warranty costs during the first year and $22,000 in warranty costs during the second year. The company uses the expense warranty accrual method. What should be the balance in the estimated liability under warranties account at the end of the first calendar year?

A
$66,000
B
$88,000
C
$100,000
D
$112,000
A

Explanation:
The correct answer is (B).

A liability for warranty costs is recognized when the related revenue is recognized (i.e., on the day the product is sold). Even if the warranty covers a period longer than the period in which the product is sold, the entire liability for the expected warranty costs must be recognized on the day the product is sold.

Beginning of Year 1:

Warranty liability recognized = $100,000 (1,000 units × $100 estimated warranty cost per unit)

Warranty Expense $100,000
Estimated warranty liability $100,000
End of year 1:

Actual payments for warranty costs reduce the amount of warranty liability recognized.

Estimated warranty liability $12,000
Cash $12,000
Warranty liability = $88,000 ($100,000 - $12,000 actual warranty costs incurred during the first year).

The balance in the estimated liability under the warranties account at the end of the first calendar year will be $88,000.

858
Q

Ryan Co. sells major household appliance service contracts for cash. The service contracts are for a one-year, two-year, or three-year period. Cash receipts from contracts are credited to unearned service contract revenues. This account had a balance of $720,000 at December 31, year 1, before year-end adjustment. Service contract costs are charged as incurred to the service contract expense account, which had a balance of $180,000 at December 31, year 1.

Outstanding service contracts at December 31, year 1 expire as follows:

During year 2	-	$150,000
During year 3	-	225,000
During year 4	-	100,000What amounts should be reported as unearned service contract revenues in Ryan's December 31, year 1 balance sheet?
A
$540,000
B
$475,000
C
$295,000
D
$245,000
A

Explanation:
The requirement is the amount to be reported for unearned service contract revenues at December 31, Year 1. This is simply the sum of the outstanding service contracts at December 31, year 1, that expire during years 2, 3, and 4 ($150,000 + $225,000 + $100,000 = $475,000). The balance of the service contract expense account ($180,000) does not affect the amount reported as unearned service contract revenues.

859
Q
Lime Co.'s payroll for the month ended January 31 is summarized as follows:
Total wages	$10,000
Federal income tax withheld	1,200All wages paid were subject to FICA. FICA tax rates were 7% each for employee and employer. Lime remits payroll taxes on the 15th of the following month. In its financial statements for the month ended January 31, what amounts should Lime report as total payroll tax liability and as payroll tax expense?
Liability	Expense
A	$1,200	$1,400
B	$1,900	$1,400
C	$1,900	$700
D	$2,600	$700
A

Explanation:
Liabilities include social security taxes and federal taxes withheld in addition to the employer portion of social security accrued. Federal income taxes withheld and the employee’s share of social security should not be classified as payroll expenses by the employer.

Liability Expense
FIT Withheld $1,200
FICA Employee Portion ($10,000 @ 7%) 700
FICA Employer Portion ($10,000 @ 7%) 700 $700
Totals $2,600 $700

860
Q

Hemple Co. maintains escrow accounts for various mortgage companies. Hemple collects the receipts and pays the bills on behalf of the customers. Hemple holds the escrow monies in interest-bearing accounts. They charge a 10% maintenance fee to the customers based on interest earned. Hemple reported the following account data:
Escrow liability beginning of year $ 500,000
Escrow receipts during the year 1,200,000
Real estate taxes paid during the year 1,450,000
Interest earned during the year 40,000What amount represents the escrow liability balance on Hemple’s books?
A
$290,000
B
$286,000
C
$214,000
D
$210,000

A

Explanation:
The escrow liability on Hemple’s books is as follows:

Escrow liability beginning of year $ 500,000
Escrow receipts during the year 1,200,000
Real estate taxes paid during the year (1,450,000)
Interest earned during the year 40,000
Maintenance fee ($40,000 * 10%) (4,000)
Escrow liability end of year $ 286,000

861
Q
Rabb Co. records its purchases at gross amounts but wishes to change to recording purchases net of purchase discounts. Discounts available on purchases recorded from last October to this September 30 totaled $2,000. Of this amount, $200 is still available in the accounts payable balance. The balances in Rabb's accounts as of and for the current year ended September 30 before conversion are:
Purchases	$100,000
Purchase discounts taken	800
Accounts payable	30,000What is Rabb's current year accounts payable balance as of September 30 after the conversion?
A
$29,800
B
$29,200
C
$28,800
D
$28,200
A

Explanation:
After the conversion, the accounts payable balance will be reported net of purchase discounts. The gross balance of the accounts payable of $30,000 is reduced by the $200 of discounts still available in the accounts payable balance to arrive at the net amount of $29,800.

862
Q

Kew Co.’s accounts payable balance at December 31 of the prior year was $2,200,000 before considering the following data:
Goods shipped to Kew F.O.B. shipping point on December 22 of the prior year were lost in transit. The invoice cost of $40,000 was not recorded by Kew. On January 7 of the current year, Kew filed a $40,000 claim against the common carrier.
On December 27 of the prior year, a vendor authorized Kew to return, for full credit, goods shipped and billed at $70,000 on December 3 of the prior year. The returned goods were shipped by Kew on December 28 of the prior year. A $70,000 credit memo was received and recorded by Kew on January 5,of the current year.
Goods shipped to Kew F.O.B. destination on December 20 of the prior year, were received on January 6 of the current year. The invoice cost was $50,000.
What amount should Kew report as accounts payable in its prior year December 31 balance sheet?
A
$2,170,000
B
$2,180,000
C
$2,230,000
D
$2,280,000

A

Explanation:
The cost of goods should be included in accounts payable when legal title passes from the seller to the purchaser. Kew should include the $40,000 cost of the goods lost in transit in accounts payable at 12/31 of the prior year. The goods were shipped F.O.B. shipping point, and the title of these goods passed to Kew, and the related liability was incurred, when the vendor delivered the goods to the common carrier on 12/22 of the prior year. Since the vendor authorized Kew to return goods for full credit before year-end, and Kew shipped the goods before year-end, the prior year 12/31 accounts payable balance should be reduced by $70,000, the cost of the goods returned. The $50,000 cost of the goods that were shipped from a vendor F.O.B. destination on 12/20 of the prior year should not be included in accounts payable at 12/31 of the prior year. Title of these goods did not pass to Kew, and the related liability was not incurred, until Kew received the goods on 1/6 of the current year. Therefore, accounts payable should be reported at $2,170,000 ($2,200,000 + $40,000 - $70,000).

863
Q
\_\_\_\_\_\_\_\_\_\_ is a promise made by the seller to the buyer to make good certain deficiencies in the product during a specified period of time after the sale.
A
Deferred revenue
B
An advanced payment
C
A warranty
D
A derivative instrument
A

Explanation:
A warranty or guarantee is a promise made by the seller to the buyer to make good certain deficiencies in the product during a specified period of time after the sale.

864
Q

On January 1, Y1, an entity enters into a non-cancellable contract to transfer a product to a customer on March 31, Y1. The contract requires the customer to pay consideration of $1,000 in advance on January 31, Y1. The customer pays the consideration on March 1, Y1. The entity transfers the product on March 31, Y1. When does the entity record a contract liability?

A
January 1, Y1
B
January 31, Y1
C
March 1, Y1
D
March 31, Y1
A

Explanation:
The amount of consideration is due on January 31, Y1 which is when the entity recognizes a receivable and a liability because it has an unconditional right to consideration as this is a non-cancellable contract.

Option (A) is incorrect because on the date of entering into the contract a liability is not recorded, liability is recorded only when the consideration is due and a receivable is recorded on January 31, Y1.

Option (C) is incorrect because the entity does not have to wait till the actual receipt of the consideration. As this is a non-cancellable contract a receivable and a liability can be recorded at the date the consideration is due.

Option (D) is incorrect because the entity transfers the product on this date as the liability would be cancelled and revenue would be recognized on satisfaction of the performance obligation

865
Q

On May 1 of the current year, Marno County issued property tax assessments for the fiscal year ending the following June 30. The first of two equal installments was due on November 1 of this year. On September 1, Dyur Co. purchased a four-year-old factory in Marno subject to an allowance for accrued taxes. Dyur did not record the entire year’s property tax obligation, but instead records tax expenses at the end of each month by adjusting prepaid property taxes or property taxes payable, as appropriate. The recording of the November 1 payment by Dyur should have been allocated between an increase in prepaid property taxes and a decrease in property taxes payable in which of the following percentages?
Percentage Allocated to
Increase in prepaid property taxes Decrease in property taxes payable
A 66 2/3% 33 1/3%
B 0% 100%
C 50% 50%
D 33 1/3% 66 2/3%

A

Explanation:
On September 1, Dyur Co. would have credited two months of taxes from the seller to Property Taxes Payable. At the end of September and the end of October, Dyur would have recorded one month of property taxes each month by a credit to Property Taxes Payable. When the payment was made for six months of taxes on November 1, the payment would be for the four months prior to that date that have already been accrued and for the two months that follow the payment date which should be recorded as Prepaid Property Taxes. Therefore, two-thirds of the payment should be allocated to a decrease in property taxes payable and one-third of the payment should be recorded as an increase in prepaid property taxes.

866
Q
A company finances the purchase of equipment with a $500,000 five-year note payable. The note has an interest rate of 12% and a monthly payment of $11,122. After two payments have been made, what amount should the company report as the note payable balance in its December 31 balance sheet?
A
$477,756
B
$487,695
C
$487,756
D
$490,061
A

Explanation:
A 12% annual interest rate would equate to 1% interest each month. Of the first $11,122 monthly payment, $5000 ($500,000 note balance × 1%) would be interest expense and the remaining $6,122 would reduce the note balance down to $493,878. The second $11,122 monthly payment would consist of $4,939 (rounded) in interest expense ($493,878 note balance × 1%) and the remaining $6,183 which would reduce the note balance down to $487,695.

867
Q
Under state law, Acme may pay 3% of eligible gross wages or it may reimburse the state directly for actual unemployment claims. Acme believes that actual unemployment claims will be 2% of eligible gross wages and has chosen to reimburse the state. Eligible gross wages are defined as the first $10,000 of gross wages paid to each employee. Acme had five employees, each of whom earned $20,000 during the year. In its December 31, balance sheet, what amount should Acme report as accrued liability for unemployment claims?
A
$1,000
B
$1,500
C
$2,000
D
$3,000
A

Explanation:
2% of $10,000 x 5 employees = $1,000. Acme has chosen to reimburse the state for actual claims instead of using the 3% of eligible gross wages rate. The liability is computed using eligible gross wages, not total wages, for each employee. The actual unemployment claim is accrued, not the state rate times actual wages.

868
Q
House Publishers offered a contest in which the winner would receive $1,000,000, payable over 20 years. On December 31, year 3, House announced the winner of the contest and signed a note payable to the winner for $1,000,000, payable in $50,000 installments every January 2. Also on December 31, year 3, House purchased an annuity for $418,250 to provide the $950,000 prize monies remaining after the first $50,000 installment, which was paid on January 2, year 4.
In its year 3 income statement, what should House report as contest prize expense?
A
$0
B
$418,250
C
$468,250
D
$1,000,000
A

Explanation:
On December 31, year 3, House announced the winner of the contest and signed a note payable to the winner.

First installment of note payable, due 1/2, year 4 $ 50,000
Cost of annuity purchased on 12/31, year 3 to provide prize moneys remaining after first installment 418,250
Contest prize expense for year 3 $468,250

869
Q

On June 1, year 2, Archer, Inc. issued a purchase order to Cotton Co. for a new copier machine. The machine requires one month to produce and is shipped f.o.b. destination on July 1, year 2, and is received by Archer on July 15, year 2. Cotton issues a sales invoice dated July 2, year 2, for the machine. As of what date should Archer record a liability for the machine?

A
June 1, Year 2
B
July 1, Year 2
C
July 2, Year 2
D
July 15, Year 2
A

Explanation:
The correct answer is (D).

When Sales /Purchases are made on F.O.B. (free on board) destination terms, the title/ownership of the goods passes from the seller to the buyer when the goods reach the destination. When the goods are in transit the title remains with the seller. The copier machine even though shipped on July 1, year 2, reaches Archer only on July 15, year 2. The legal title is transferred to Archer on July 15. Liability is recognized once the legal title is transferred, on July 15, year 2.

870
Q

Black Co. requires advance payments with special orders for machinery constructed to customer specifications. These advances are nonrefundable. Information for the current year is as follows:
Customer advances—prior year balance 12/31 $118,000
Advances received with orders in the current year 184,000
Advances applied to orders shipped in the current year 164,000
Advances applicable to orders canceled in the current year 50,000In Black’s current year December 31 balance sheet, what amount should be reported as a current liability for advances from customer?
A
$0
B
$88,000
C
$138,000
D
$148,000

A

Explanation:
Advanced payments received from customers are liabilities until the transaction is completed (or canceled). Black’s balance is computed as follows: $118,000 + 184,000 - 164,000 - 50,000 = $88,000.

871
Q
After three profitable years, Dodd Co. decided to offer a bonus to its branch manager, Cone, of 25% of income over $100,000 earned by his branch. For year 2, income for Cone's branch was $160,000 before income taxes and Cone's bonus. Cone's bonus is computed on income in excess of $100,000 after deducting the bonus, but before deducting taxes. What is Cone's bonus for year 2?
A
$12,000
B
$15,000
C
$25,000
D
$32,000
A

Explanation:
The amount of the bonus is determined by solving the proper equation that describes the terms of the bonus agreement. Using B to represent the bonus, B equals 25% of the total of income before taxes less $100,000 less B. The answer is computed as follows:

B = 0.25 × ($160,000 - $100,000 - B); setup
B = 0.25 × ($60,000 - B); reduced $ figure
B = $15,000 - .25B; simplified right-side
1.25B = $15,000; added .25B to both sides
B = $12,000; divided both sides by 1.25
872
Q

An entity would not be in compliance with authoritative guidance on long-term liabilities if it did which of the following?
A
Recognized an asset retirement obligation as a liability at fair value in the period in which it was incurred, if subject to reasonable estimation
B
Classified as a long-term liability the repayment of short-term obligations after the balance sheet date, and subsequently refinanced them as long-term obligations before the balance sheet was issued
C
Discounted its asset retirement obligation liability, and recognized accretion expense using the credit-adjusted risk-free interest rate in effect at initial recognition
D
None of the above

A

Explanation:
Short-term obligations repaid after the balance sheet date and subsequently refinanced before the issuance of the balance sheet must be classified as current liabilities as of the balance sheet date, because current assets were used for the repayment.

873
Q
Ivy Co. operates a retail store. All items are sold subject to a 6% state sales tax, which Ivy collects and records as sales revenue. Ivy files quarterly sales tax returns when due, by the 20th day following the end of the sales quarter. However, in accordance with state requirements, Ivy remits sales tax collected by the 20th day of the month following any month such collections exceed $500. Ivy takes these payments as credits on the quarterly sales tax return. The sales taxes paid by Ivy are charged against sales revenue. Following is a monthly summary appearing in Ivy's first quarter sales revenue account:
Debit	Credit
January	--	$10,600
February	$600	7,420
March	   --   	 8,480 
$600	$26,500In its March 31, balance sheet, what amount should Ivy report as sales taxes payable?
A
$ 600
B
$ 900
C
$1,500
D
$1,590
A

Explanation:
Ivy Co. should report sales taxes based on February and March sales as sales taxes payable in its March 31 balance sheet. The taxes based on January sales were paid in February because they exceeded $500 ($10,600 sales / 1.06 = sales of $10,000; $10,600 - $10,000 = $600 in sales taxes). This is the $600 debit in February.

(a) Total credits to sales revenue account (b) Sales without sales taxes (a/1.06) (a-b) Sales taxes
February $7,420 $7,000 $420
March $8,480 $8,000 $480
Total sales taxes payable, March 31 $900

874
Q

Which of the following statements characterizes convertible debt?

A
The holder of the debt must be repaid with shares of the issuer’s stock.
B
No value is assigned to the conversion feature when convertible debt is issued.
C
The transaction should be recorded as the issuance of stock.
D
The issuer’s stock price is less than market value when the debt is converted.

A

Explanation:
Convertible debt securities are those debt securities which are convertible into common stock of the issuer or an affiliated company at a specified price at the option of the holder and which are sold at a price or have a value at issuance not significantly in excess of the face amount. The terms of such securities generally include (1) an interest rate which is lower than the issuer could establish for nonconvertible debt, (2) an initial conversion price which is greater than the market value of the common stock at time of issuance, and (3) a conversion price which does not decrease except pursuant to antidilution provisions. No proceeds from the debt issue are to be assigned to the conversion feature (even though the convertible bonds may sell for substantially more than similar nonconvertible bonds). The reason for no allocation to equity is that the debt cannot be separated from the conversion feature, as would be the case with detachable stock warrants. The holder of the debt need not be repaid with shares of the issuer’s stock and the transaction should not be recorded as the issuance of stock.

Option (A) is incorrect because the issuer may repay in cash or convert to shares at the option of the bondholder.
Option (C) is incorrect because the transaction is recorded as debt not as issuance of stock.
Option (D) is incorrect because bondholder is unlikely to convert the bonds in to stock when the stock price is below the market value of the bond.

875
Q

Ina Co. had the following beginning and ending balances in its prepaid expense and accrued liabilities accounts for the current year:

Prepaid expenses Accrued liabilities

Beginning balance $ 5,000 $ 8,000
Ending balance 10,000 20,000
Debits to operating expenses totaled $100,000. What amount did Ina pay for operating expenses during the current year?

A
$83,000
B
$93,000
C
$107,000
D
$117,000
A
Explanation:
Prepaid expenses (change)	5,000	 
Operating expenses (given)	100,000	 
Accrued liabilities (change)	 	12,000
Cash (plug)	 	93,000
876
Q

Kent Co., a division of National Realty, Inc., maintains escrow accounts and pays real estate taxes for National’s mortgage customers. Escrow funds are kept in interest-bearing accounts. Interest, less a 10% service fee, is credited to the mortgagee’s account and used to reduce future escrow payments. Additional information follows:
Escrow accounts liability, 1/1 $ 700,000
Escrow payments received during the year 1,580,000
Real estate taxes paid during the year 1,720,000
Interest on escrow funds during the year 50,000What amount should Kent report as escrow accounts liability in its December 31 balance sheet?
A
$510,000
B
$515,000
C
$605,000
D
$610,000

A

Explanation:
Escrow liability, 1/1 $ 700,000
Add: Escrow payments received during year 1,580,000
Add: Interest on escrow funds during year 50,000
Less: 10% service fee ($50,000 × 10%) (5,000)
Less: Real estate taxes paid (1,720,000)
Escrow liability, 12/31 $ 605,000

877
Q

During the current year, Casual Wear Co. had total retail sales of $800,000 and collected a 5% state sales tax on all sales. At the end of the prior year, Casual Wear had $4,500 in sales taxes that had not been remitted to the state authorities. During the current year, Casual Wear remitted $39,500 in state sales tax. What amount should be recorded in Casual Wear’s current-year financial statements?

A
$5,000 in sales tax payable
B
$39,500 in sales tax expense
C
$40,000 in sales tax revenue
D
$840,000 in sales revenue
A

Explanation:
The correct answer is (A).

Beginning Sales Tax Payable = $4,500.
Sales Tax Expense = $800,000 x 5% = $40,000.
Sales Tax Paid = $39,500.
Sales Tax Payable = Beginning Sales Tax Payable + Sales Tax Expense - Sales Tax Paid = $40,000 + $4,500 - $39,500 = $5,000.
With $39,500 in sales taxes paid during the current year, of which $4,500 was for prior year taxes, Casual Wear Co. will have $5,000 in sales tax payable be recorded in Casual Wear’s current-year financial statements.

878
Q
Hudson Hotel collects 15% in city sales taxes on room rentals, in addition to a $2 per room, per night, occupancy tax. Sales taxes for each month are due at the end of the following month, and occupancy taxes are due 15 days after the end of each calendar quarter. On January 3, Year 4, Hudson paid its November year 3 sales taxes and its fourth quarter year 3 occupancy taxes. Additional information pertaining to Hudson's operations is:
Year 3	Room rentals	Room nights
October	$100,000	1,100
November	$110,000	1,200
December	$150,000	1,800What amounts should Hudson report as sales taxes payable and occupancy taxes payable in its December 31, Year 3, balance sheet?
Sales taxes	Occupancy taxes
A	$39,000	$6,000
B	$39,000	$8,200
C	$54,000	$6,000
D	$54,000	$8,200
A

Explanation:
In calculating the sales tax payable, October rentals are not included because the tax was paid at the end of November. In calculating the occupancy tax payable, October room nights are included because the tax is not paid until 15 days after the end of the quarter.

November	$110,000
December	150,000
$260,000
Rate of tax	× 15%
$39,000
October	1,100
November	1,200
December	1,800
4,100
Rate of tax	× $2
$8,200
879
Q
In December, Mill Co. began including one coupon in each package of candy that it sells and offering a toy in exchange for 50 cents and five coupons. The toys cost Mill 80 cents each. Eventually 60% of the coupons will be redeemed. During December, Mill sold 110,000 packages of candy and no coupons were redeemed. In its December 31 balance sheet, what amount should Mill report as estimated liability for coupons?
A
$ 3,960
B
$10,560
C
$19,800
D
$52,800
A

Explanation:
Cost of toys to Mill Co., each $ 0.80
Less: Cash to be received with redemption (.50)
Net cost to Mill Co. for each toy $ 0.30
Total packages of candy sold in December $110,000
Times: Anticipated redemption rate × 60%
Total coupons anticipated to be redeemed 66,000
Divided by: Number of coupons required for each toy / 5
Anticipated number of toys needed 13,200
Times: Net cost to Mill Co. for each toy (above) × 0.30
Total estimated coupons liability, 12/31 $ 3,960

880
Q

On January 1, year 1, a company’s new CEO was awarded a $200,000 bonus that would be paid out in two $100,000 installments in years 3 and 4 of employment, contingent on employment through the year ended December 31, year 2. What amount should the company expense for this bonus for years 2 and 3?

A
Nil for year 2 and $100,000 for year 3
B
$100,000 for year 2 and nil for year 3
C
$100,000 for year 2 and $100,000 for year 3
D
$200,000 for year 2 and nil for year 3
A

Explanation:
The correct answer is (B).

The $200,000 bonus will be considered deferred compensation because it is contingent upon successful completion of two years of employment from Jan 1 Year 1 to Dec 31 Year 2. As the bonus will not be paid unless the CEO continues his employment for two years, the cost of the benefits should be associated with the service period required, which is years 1 and 2.

Year 1 expense = $100,000
Year 2 expense = $100,000
$100,000 of expense will be recorded each in years 1 and 2 and accrued as a liability. When the actual payment occurs in years 3 and 4, there is no effect on the income statement. Therefore, Year 2 will show an expense of $100,000 for the service period, and Year 3 will show $0 because it is not a service period.

(A) is incorrect because year 2 should be allocated with bonus expense of $100,000 and not in year 3.

(C) is incorrect because year 3 bonus expense should be nil.

(D) is incorrect because $200,000 is entirely expensed in year 2, instead of allocating between year 1 and year 2.

881
Q

Strand Inc., provides an incentive compensation plan under which its president receives a bonus equal to 10% of the corporation’s income in excess of
$200,000 before income tax but after deduction of the bonus. If income before income tax and the bonus is $640,000 and the tax rate is 21%, the amount of the
bonus would be

A
$40,000
B
$44,000
C
$58,180
D
$64,000
A

Explanation:
The correct answer is (A).

Bonus = 10% of Corporation’s Income - $200,000 - Bonus

Bonus = 0.10 ($640,000 - $200,000 - Bonus)

Bonus = 0.10 ($440,000 – Bonus)

Bonus = $44,000 – 0.10 Bonus

1.10 Bonus = $44,000

Bonus = $44,000/1.10

Bonus = $40,000

882
Q
Pine Corp. is required to contribute to an employee stock ownership plan (ESOP), 10% of its income after deduction for this contribution but before income tax. Pine's income before charges for the contribution and income tax was $75,000. The income tax rate is 30%. What amount should be accrued as a contribution to the ESOP?
A
$7,500
B
$6,818
C
$5,250
D
$4,773
A
xplanation:
C = 0.10 ($75,000 - C)
C = $7,500 - 0.10C
1.10	C = $7,500
C = $6,818
883
Q
Case Cereal Co. frequently distributes coupons to promote new products. On October 1,Case mailed 1,000,000 coupons for $.45 off each box of cereal purchased. Case expects 120,000 of these coupons to be redeemed before the December 31 expiration date. It takes 30 days from the redemption date for Case to receive the coupons from the retailers. Case reimburses the retailers an additional $.05 for each coupon redeemed. As of December 31, Case had paid retailers $25,000 related to these coupons and had 50,000 coupons on hand that had not been processed for payment. What amount should Case report as a liability for coupons in its December 31 balance sheet?
A
$35,000
B
$29,000
C
$25,000
D $22,500
A

Explanation:
The coupon liability at 12/31 is not reduced by the 50,000 coupons on hand because the coupons had not been processed for payment at 12/31.

Coupons expected to be redeemed 120,000
Times: Payment for redeemed coupon
($0.45 + $0.05) x $0.50
Estimated total coupon liability $60,000
Less: Payments to retailers as of 12/31 25,000
Remaining coupon liability, 12/31 $35,000

884
Q
Cado Co.'s payroll for the month ended January 31 is summarized as follows:
Total wages	$100,000
Amount of wages subject to payroll taxes:	
FICA	80,000
Unemployment	20,000
Payroll tax rates:	
FICA for employer and employee	7% each
Unemployment	3%In its January 31 balance sheet, what amount should Cado accrue as its share of payroll taxes?
A
$6,200
B
$10,000
C
$11,800
D
$17,000
A

Explanation:

($80,000 × .07) + ($20,000 × .03) = ($5,600 + $600) = $6,200.

885
Q
Oak Co. offers a three-year warranty on its products. Oak previously estimated warranty costs to be 2% of sales. Due to a technological advance in production at the beginning of the current year, Oak now believes 1% of sales to be a better estimate of warranty costs. Warranty costs of $80,000 and $96,000 were reported in the two prior years. Sales for the current year were $5,000,000. What amount should be disclosed in Oak's current year financial statements as warranty expense?
A
$ 50,000
B
$ 88,000
C
$100,000
D
$138,000
A

Explanation:
As Oak believes 1% of sales to be the best estimate of warranty costs, warranty expense should be accrued accordingly. ($5,000,000 × 1% = $50,000). The technological advance applied only to current year sales; thus, in this case, the estimates of warranty costs reported in the previous two years are not changed.

886
Q

How should unearned rent that has already been paid by tenants for the next eight months of occupancy be reported in a landlord’s financial statements?

A
Current asset
B
Current liability
C
Long-term asset
D
Long-term liability
A

Explanation:
The correct answer is (B).

Prepaid rent by tenants (paid 8 months in advance) should be recorded by a landlord as a current liability. This is essentially unearned rent. Unearned rent is a liability to the landlord, not an asset. The landlord would debit cash and credit unearned revenue for all 8 months of rent. Since the rent is for eight months (less than one year), the liability would be classified as a current liability.

887
Q
Willem Co. reported the following liabilities at December 31, year 1:
Accounts payable-trade	$ 750,000
Short-term borrowings	400,000
Mortgage payable, current portion $100,000	3,500,000
Other bank loan, matures June 30, year 2	1,000,000The $1,000,000 bank loan was refinanced with a 20-year loan on January 15, year 2, with the first principal payment due January 15, year 3. Willem's audited financial statements were issued February 28, year 2. What amount should Willem report as current liabilities at December 31, year 1?
A
$850,000
B
$1,150,000
C
$1,250,000
D
$2,250,000
A

Explanation:
The term current liabilities is used principally to designate obligations whose liquidation is reasonably expected to require the use of existing resources properly classifiable as current assets, or the creation of other current liabilities. Short-term obligations are those scheduled to mature within one year or operating cycle, whichever is longer, and generally are classified as current liabilities. However, if they are to be refinanced on a long-term basis they will be appropriately classified as long-term liabilities. Exclusion from current liabilities requires two conditions be met; 1) the enterprise must intend to refinance the obligation on a long-term basis, and 2) the enterprise must have the ability to consummate the financing. A refinancing that occurs after the balance sheet date but before the issuance of the balance sheet is evidence of intent and ability. The $1,000,000 bank loan would not be reported as part of current liabilities because it was refinanced on a long-term basis. The accounts payable-trade, short-term borrowings, and current portion of mortgage payable would be reported as current liabilities ($750,000 + $100,000 + $400,000 = $1,250,000).

888
Q

At the beginning of year 1, a company hired an executive whose contract included the promise of payment of $100,000 in each of years 6, 7, and 8, if the executive is employed at the end of year 5. How should the compensation expense associated with this contract be recorded?

A
$37,500 in each of years 1 through 8
B
$60,000 in each of years 1 through 5
C
$100,000 in each of years 6 through 8
D
$300,000 when the contract is signed
A

Explanation:
In the absence of a direct association with specific revenue, expenses are recognized and recorded by attempting to allocate the expenses in a systematic and rational manner over certain periods. The $300,000 total compensation expense for years 6, 7, and 8 would be would be allocated over the period it took for the contract to be consummated, 5 years. The compensation expense associated with the contract would be $60,000 in each of years 1 through 5.

889
Q

Lind Co.’s salaries expense of $10,000 is paid every other Friday for the 10 workdays and then ending. Lind’s employees do not work on Saturdays and Sundays. The last payroll was paid on June 18. On Wednesday, June 30, the month-end balance in the salaries expense account before accruals was $14,000. What amount should Lind report as salaries expense in its income statement for the month ended June 30?

A
$24,000
B
$22,000
C
$20,000
D
$18,000
A

Explanation:
The correct answer is (B).

It is given that the company pays $10,000 for 10 workdays. The salary paid for each workday = 10,000/10 = $1,000. This is also substantiated by the fact that the salaries expense account stood at $14,000 on June 18. Given that June 18 was a Friday and that employees do not work on Saturdays and Sundays, there were only 14 workdays in June till the 18th day of the month.

From June 18 to June 30 (Wednesday), there were only 8 workdays. The salaries expense accrues to $8,000.

Therefore, the salaries expense account should reflect a total balance of expenses paid plus the expenses accrued i.e. $14,000 + $8,000 = $22,000.

890
Q
Fay Corp. pays its outside salespersons fixed monthly salaries and commissions on net sales. Sales commissions are computed and paid on a monthly basis (in the month following the month of sale), and the fixed salaries are treated as advances against commissions. However, if the fixed salaries for salespersons exceed their sales commissions earned for a month, such excess is not charged back to them. Pertinent data for the month of March for the three salespersons are as follows:
Salesperson	Fixed salary	Net sales	Commission rate
A.	$10,000	$ 200,000	4%
B.	14,000	400,000	6%
C.	  18,000  	  600,000  	6%
$42,000	$1,200,000	What amount should Fay accrue for sales commissions payable at March 31?
A
$70,000
B
$68,000
C
$28,000
D
$26,000
A

Explanation:
Salespersons receive sales commissions only to the extent that their sales commissions earned exceed their fixed salaries for that month.

Salesperson Sales commissions Fixed salary Accrued commissions
A $ 8,000 ($200,000 × 4%) $10,000 $ 0
B 24,000 ($400,000 × 6%) 14,000 10,000
C 36,000 ($600,000 × 6%) 18,000 18,000
$28,000

891
Q

Acme Co.’s accounts payable balance at December 31 was $850,000 before necessary year-end adjustments, if any, related to the following information:
At December 31, Acme has a $50,000 debit balance in its accounts payable resulting from a payment to a supplier for goods to be manufactured to Acme’s specifications.
Goods shipped f.o.b. destination on December 20 were received and recorded by Acme on January 2, the invoice cost was $45,000.
In its December 31 balance sheet, what amount should Acme report as accounts payable?
A
$850,000
B
$895,000
C
$900,000
D
$945,000

A

Explanation:
Accounts payable is a liability account incurred in obtaining goods and services from vendors in the entity’s ordinary course of business. Accounts payable should reflect the cost of those goods or services that have been appropriately included in inventory (or other asset account) or have been expensed. The $50,000 debit for goods to be manufactured should not have been posted to the accounts payable account. The $50,000 payment would be classified as a prepaid asset and not as a reduction of any liability because the goods haven’t been received yet. Adding the $50,000 to the $850,000 brings the accounts payable account to $900,000 on December 31. The $45,000 does not go into accounts payable until January 2 of the next year because the goods were shipped f.o.b. destination. Under an f.o.b. destination contract, the goods and related liability are not recorded until the goods are received.

892
Q
\_\_\_\_\_\_\_\_\_\_ are used principally to designate obligations whose liquidation is reasonably expected to require the use of existing resources properly classifiable as current assets, or the creation of other current liabilities.
A
Long-term liabilities
B
Current liabilities
C
Current assets
D
Notes receivable
A

Explanation:
The term “current liabilities” is used principally to designate obligations whose liquidation is reasonably expected to require the use of existing resources properly classifiable as current assets, or the creation of other current liabilities.

893
Q
During year 1, Gum Co. introduced a new product carrying a two-year warranty against defects. The estimated warranty costs related to dollar sales are 2% within 12 months following the sale and 4% in the second 12 months following the sale. Sales and actual warranty expenditures for the prior and current year ended December 31, are as follows:
Year	Sales	Actual warranty expenditures
1	$150,000	$2,250
2	 250,000 	 7,500 
$400,000	$9,750What amount should Gun report as estimated warranty liability in its December 31, year 2, balance sheet?
A
$ 2,500
B
$ 4,250
C
$11,250
D
$14,250
A

Explanation:
Estimated warranty liability at 12/31, year 2, should be recorded as the excess of the estimated warranty costs for year 1 and year 2 sales over the actual warranty expenditures to date.

Sales (years 1 and 2) $400,000
Estimated warranty cost percentage (2% + 4%) × 6%
Estimated warranty cost for year 1 and year 2 sales $ 24,000
Warranty expenditures to date (9,750)
Estimated warranty liability, 12/31, year 2 $ 14,250

894
Q
On March 1 of the prior year, Fine Co. borrowed $10,000 and signed a two-year note bearing interest at 12% per annum compounded annually. Interest is payable in full at maturity on February 28 of next year. What amount should Fine report as a liability for accrued interest at December 31 of the current year?
A
$0
B
$1,000
C
$1,200
D
$2,320
A

Explanation:
March 1, prior year to Feb.28, currnet year ($10,000 @ 12% X 12/12) $1,200
March 1 to Dec.31, current year [12%($10,000 + $1,200) X 10/12] 1,120
Total Accrued Interest, December 31 $2,320

895
Q

KLU Broadcast Co. entered into an agreement to exchange unsold advertising time for travel and lodging services with Hotel Co. As of June 30, travel and lodging services of $10,000 were used by KLU. However, the advertising service had not been provided. How should KLU account for travel and lodging in its June 30 financial statements?
A
Revenue and expense is recognized when the agreement is complete.
B
An asset and revenue for $10,000 is recognized.
C
An expense and liability of $10,000 is recognized.
D
Not reported.

A

Explanation:
Advanced payments received from customers and others are liabilities until the transaction is completed. The travel and lodging services that were used by KLU is a liability because use of the services indicated a payment in advance for the advertising time to be provided at a later date. Use of the services is an expense.

896
Q

On September 30, World Co. borrowed $1,000,000 on a 9% note payable. World paid the first of four quarterly payments of $264,200 when due on December 30. In its December 31, balance sheet, what amount should World report as note payable?

A
$735,800
B
$750,000
C
$758,300
D
$825,800
A

Explanation:
The correct answer is (C).

The $264,200 payment includes both principal and interest. The amount of interest is $1,000,000 (amount of note) × 0.09 (the 9% interest) ÷ 4 (payments are quarterly) = $22,500. Subtracting $22,500 from $264,200 leaves $241,700 as principal. Subtracting the $241,700 of principle from the $1,000,000 note balance results in $758,300 should be reported as note payable on the December 31 balance sheet.

897
Q

Paxton Co. signed contracts for the purchase of raw materials to be executed the following year at a firm price of $5 million. The market price of the materials dropped to $3 million on December 31. What amount should Paxton record as an estimated liability on purchase commitments as of December 31?

A
$5,000,000
B
$3,000,000
C
$2,000,000
D
$0
A

Explanation:
The correct answer is (C)

In case of a firm purchase commitment, when it is probable that a loss will occur due to the contract price being more than the market price, such loss is recognized at the time of decline in prices itself.

Paxton Co. has entered into a firm purchase commitment since it is a contract for future purchase of inventory. The market price has fallen down by $2 million than the contract price. A liability of $2 million is supposed to be recorded.

In a legal non-cancelable agreement for future purchase of Inventory, if Contract price > Market price and it is expected that loss will occur on the purchase, recognize the loss of ($3,000,000 - $5,000,000) $2,000,000 (Market Price - Contracted Price), at the time of decline in prices.

The journal entry would be:

Dr: Est. loss on purchase commitment $2,000,000

Cr: Accrued loss on purchase commitment $2,000,000

898
Q
\_\_\_\_\_\_\_\_\_\_ are expenses incurred, but not yet paid in cash.
A
Dividends payable
B
Advances
C
Accrued expenses
D
Returnable deposits
A

Explanation:

Accrued expenses are expenses incurred, but not yet paid in cash.

899
Q

A company has the following liabilities at year end:
Mortgage note payable; $16,000 due within 12 months $355,000
Short-term debt that the company is refinancing with long-term debt 175,000
Deferred tax liability arising from depreciation 25,000What amount should the company include in the current liability section of the balance sheet?
A
$0
B
$16,000
C
$41,000
D
$191,000

A

Explanation:
The term current liabilities is used primarily to designate obligations whose liquidation is reasonably expected to require the use of existing resources classified as current assets, or the creation of other current liabilities. Long-term liabilities are all obligations not expected to be liquidated by the use of existing current assets or by the creation of current liabilities. Short-term obligations are those scheduled to mature within one year or operating cycle, whichever is longer, and generally are classified as current liabilities. However, if they are to be refinanced on a long-term basis they will be appropriately classified as long-term liabilities. Income taxes payable within the next period, or operating cycle, are classified as current liabilities. A deferred tax liability is over a longer period of time and not considered a current liability. The only amount that would be reported is the $16,000 due within 12 months from the mortgage note payable.

900
Q
On December 1, Alt Department Store received 505 sweaters on consignment from Todd. Todd's cost for the sweaters was $80 each, and they were priced to sell at $100. Alt's commission on consigned goods is 10%. At December 31, 5 sweaters remained. In its December 31 balance sheet, what amount should Alt report as payable for consigned goods?
A
$49,000
B
$45,400
C
$45,000
D
$40,400
A

Explanation:
Goods held on consignment remain the property of the consignor. The consignee does not incur a liability for consigned goods until the goods are sold to a third party.

Consignment sales (500 x $100)	$50,000
Less Alt's commission on consignment sales ($50,000 x 10%)	5,000
Payable for consigned goods, 12/31	$45,000